You are on page 1of 511

www.engineeringonyourfingertips.

ooo
|ME-2019-Forenoon|

GENERAL APTITUDE

Q. No. 1 - 5 Carry One Mark Each

1. The minister avoided any mention of the issue of women’s reservation in the private sector. He
was accused of _____ the issue.
(A) belting (B) skirting (C) tying (D) collaring
Key: (B)

2. _____ I permitted him to leave, I wouldn’t have had any problem with him being
absent _____ I?
(A) Had, would (B) Have, wouldn’t
(C) Have, would (D) Had, wouldn’t
Key: (A)

3. A worker noticed that the hour hand on the factory clock had moved by 225 degrees during her
stay at the factory. For how long did she stay in the factory?
(A) 3.75 hours (B) 7.5 hours
(C) 4 hours and 15mins (D) 8.5 hours
Key: (B)

Sol: Number of hours in a clock = 12 hours


One rotation hour hand covers 360°
www.
engi
neer
ingonyour f
i
11 12 1
nger
tips.
ooo
360 degree = 12 hours
10 2
12
1 degree  hours 9 3
360 4
8
225° = ? 7 6 5
12
  225  7.5 hours
360

4. John Thomas, an ______ writer, passed away in 2018.


(A) imminent (B) prominent (C) dominant (D) eminent
Key: (D)

© All rights reserved by Thinkcell Learning Solutions Pvt. Ltd. No part of this booklet may be reproduced or utilized in any form without the written permission.

www.engineeringonyourfingertips.ooo

More Notes Join us Telegram-: http//:t.me/allexammentor


www.engineeringonyourfingertips.ooo
|ME-2019-Forenoon|

5. The sum and product of two integers are 26 and 165 respectively. The difference between these
two integers is ________.
(A) 3 (B) 6 (C) 2 (D) 4
Key: (D)

Sol: Let us take two number are a & b


Given that
a  b  26, ab  165
ab?
a  b  262  a 2  b 2  2ab  262
2

a 2  b 2  262  2ab
a  b   a 2  b 2   2ab
2

 a  b   262  2ab  2ab


2

 a  b   262  4ab   a  b   262  4  165


2 2

 a  b   16  a  b  4
2

Q. No. 6 - 10 Carry Two Marks Each

6. A person divided an amount of Rs. 100,000 into two parts and invested in two different
schemes. In one he got 10% profit and in the other he got 12%. If the profit percentages are
interchanged with these investments he would have got Rs. 120 less. Find the ratio between his
investments in the two schemes.
(A) 37:63 (B) 9:16 (C) 11:14 (D) 47:53
Key: (D)

Sol: Considering firstwww. e


scheme ng
as xi
neer i
second ngony
scheme ou
as yr
fi
nger
tips.
ooo
Given that
x  y  1,00,000  1
Assume profit of sum before interchanging percentage = z
1.1x  1.12y  z   2
After interchanging profit percentages
1.12x  1.1y  z  120   3
Solving (2) and (3)
1.12x  1.1y  z  120
1.1x  1.12y  z
  
0.02x  0.02y  120

© All rights reserved by Thinkcell Learning Solutions Pvt. Ltd. No part of this booklet may be reproduced or utilized in any form without the written permission.

www.engineeringonyourfingertips.ooo

More Notes Join us Telegram-: http//:t.me/allexammentor


www.engineeringonyourfingertips.ooo
|ME-2019-Forenoon|

120
xy   6000   4 
0.02
By solving (4) & (1)
x  y  100000
x  y  6000
2x  94000
x  47000
y  100000  47000  53000
x 47000 47
 
y 53000 53

7. Under a certain legal system, prisoners are allowed to make one statement. If their statement
turns out to be true then they are hanged. If the statement turns out to be false then they are shot.
One prisoner made a statement and the judge had no option but to set him free. Which one of
the following could be that statement?
(A) I will be shot (B) I committed the crime
(C) I did not commit the crime (D) You committed the crime
Key: (A)

8. A firm hires employees at five different skill levels P, Q, R, S, T. The shares of employment at
these skill levels of total employment in 2010 is given in the pie chart as shown. There were a
total of 600 employees in 2010 and the total employment increased by 15% from 2010 to 2016.
The total employment at skill levels P, Q and R remained unchanged during this period. If the
employment at skill level S increased by 40% from 2010 to 2016, how many employees were
there at skill level T in 2016?
www.
engi neer i
Percentagen gony
share ofour fi
skillsningerti
2010ps.
ooo

T
S 5 P
25 20

Q
R
25
25

(A) 30 (B) 72 (C) 35 (D) 60

© All rights reserved by Thinkcell Learning Solutions Pvt. Ltd. No part of this booklet may be reproduced or utilized in any form without the written permission.

www.engineeringonyourfingertips.ooo

More Notes Join us Telegram-: http//:t.me/allexammentor


www.engineeringonyourfingertips.ooo
|ME-2019-Forenoon|

Key: (D)

Sol: 2010 2016


T
5% 60
S 210 T
D 20% P
25% S
120
R Q R Q
25% 25% 150 150

In 2010:
Total number of employees = 600
Number of employees of skills
Q  R  S  25% of 600  150
Number of employees of skill P = 20% of 600 = 120
Number of employees of skill T= 5% of 600 = 30
In 2016:
Total number of employees increased by 15%
Total number of employees  1.15  600  690
As there is no change in skill level of P, Q, and R
Number of employees of skill level P = 120
Number of employees of skill level Q = 150
Number of employees of skill level R=150
Number of employees at skill level S  40% increases  1.4 150  210

Number of employees at skill level T  690  120  150  150  210   60.

9. M and N had four children P, Q, R and S. Of them, only P and R were married. They had
www.
engi
neer
ingonyour
fi
nger
tips.
ooo
children X and Y respectively. If Y is a legitimate child of W, which one of the following
statement is necessarily FALSE?
(A) M is the grandmother of Y (B) W is the wife of R
(C) W is the wife of P (D) R is the father of Y
Key: (C) M and N
Sol:
Children

P Q R W S

Wife & husband


X
Y
Child
Child
© All rights reserved by Thinkcell Learning Solutions Pvt. Ltd. No part of this booklet may be reproduced or utilized in any form without the written permission.

www.engineeringonyourfingertips.ooo

More Notes Join us Telegram-: http//:t.me/allexammentor


www.engineeringonyourfingertips.ooo
|ME-2019-Forenoon|

10. Congo was named by Europeans. Congo’s dictator Mobuto later changed the name of the
country and the river to Zaire with the objective of Africanising names of persons and spaces.
However, the name Zaire was a Portuguese alteration of Nzadi o Nzere, a local African term
meaning ‘River that swallows Rivers’. Zaire was the Portuguese name for the Congo river in the
16th and 17 centuries. Which one of the following statements can be inferred from the
paragraph above?
(A) The term Nzadi o Nzere was of Portuguese origin
(B) As a dictator Mobuto ordered the Portuguese to alter the name of the river to Zaire
(C) Mobuto’s desire to Africanise names was prevented by the Portuguese
(D) Mobuto was not entirely successful in Africanising the name of his country
Key: (D)

MECHANICAL ENGINEERING
Q. No. 1 to 25 Carry One Mark Each

1. Consider the stress-strain curve for an ideal elastic-plastic strain hardening metal as shown in
the figure. The metal was loaded in uniaxial tension starting from O. Upon loading, the stress-
strain curve passes through initial yield point at P, and then strain hardens to point Q, where the
loading was stopped. From point Q, the specimen was unloaded to point R, where the stress is
zero. If the same specimen is reloaded in tension from point R, the value of stress at which the
material yields again is _______MPa.

210 Q
www.
engi
neer
ingonyour
fi
nger
tips.
ooo
P
180
Stress (MPa)

O R Strain
Key: (210)

Sol: Strain hardening improve tensile strength, yield strength and hardness at the expense of
reduced ductility.

© All rights reserved by Thinkcell Learning Solutions Pvt. Ltd. No part of this booklet may be reproduced or utilized in any form without the written permission.

www.engineeringonyourfingertips.ooo

More Notes Join us Telegram-: http//:t.me/allexammentor


www.engineeringonyourfingertips.ooo
|ME-2019-Forenoon|

2. The length, width and thickness of a steel sample are 400 mm, 410 mm, 40 mm and 20 mm,
respectively. Its thickness needs to be uniformly reduced by 2 mm in a single pass by using
horizontal slab milling. The milling cutter (diameter: 100 mm, width: 50 mm) has 20 teeth and
rotates at 1200 rpm. The feed per tooth is 0.05 mm. The feed direction is along the length of the
sample. If the over-travel distance is the same as the approach distance, the approach distance
and time taken to complete the required machining task are
(A) 14mm, 21.4 s (B) 21 mm, 39.4 s (C) 21 mm, 28.9s (D) 14mm, 18.4 s
Key: (A)

Sol: L  400mm
b  40 mm
t  20 mm
d  2 mm
D  100 mm, Z  20, N  1200 rpm
f  0.05 mm tooth

Ca  d  D  d   2 100  2   2  98  14 mm
F  fNZ
 0.05  1200  20  1200 mm min  20 mm s
L  Ca  Co 400  14  14
tm  e   21.4s
F 20

3. As per common design practice, the three types of hydraulic turbines, in descending order of
flow rate, are
(A) Francis, Kaplan, Pleton (B) Kaplan, Francis, Pelton
(C) Pelton, Kaplan, Francis (D) Pelton, Francis, Kaplan
Key: (B) www.
engi
neer
ingonyour
fi
nger
tips.
ooo
Sol: Kaplan turbine is operating under high flow rates.
Francis turbine is operating under medium flow rates.
Pelton turbine is operating under low flow rates

4. The table presents the demand of a product. By simple three-months moving average method,
the demand-forecast of the product for the month of September is
Month Demand
January 450
February 440
March 460
April 510
May 520

© All rights reserved by Thinkcell Learning Solutions Pvt. Ltd. No part of this booklet may be reproduced or utilized in any form without the written permission.

www.engineeringonyourfingertips.ooo

More Notes Join us Telegram-: http//:t.me/allexammentor


www.engineeringonyourfingertips.ooo
|ME-2019-Forenoon|

June 495
July 475
August 560

(A) 490 (B) 536.67 (C) 510 (D) 530


Key: (C)

495  475  560


Sol: Forecast of the product of the month September =  510
3

5. The lengths of a large stock of titanium rods follow a normal distribution with a mean    of

440 mm and a standard deviation    of 1 mm. What is the percentage of rods whose lengths lie

between 438 mm and 441 mm?


(A) 86.64% (B) 68.4% (C) 99.75% (D) 81.85%
Key: (D)

Sol: Given , Mean     440mm, S.D     1 mm


The random variable ‘X’ denotes lengths of rods.
P  438  X  441  ?
X
The standard normal variable Z 

438  440 Z  2 Z  1 Z  0 Z 1 Z  2
If X  438  Z   2
1
441  440 68.26%
If X  441  Z  1
1
 P  438  X  441w
 wPw .
eg
2n 
Zi er
n1e ingonyour
fi
nger t
ips. ooo 95.44%

 P  2  Z  0  P 0  Z  1
 95.44   68.26 
  %  %   47.72  %   34.13 %
 2   2 
 P  438  X  441  81.85%

6. During a non-flow thermodynamic process (1-2) executed by a perfect gas, the heat interaction

is equal to the work interaction  Q1 2  W12  when the process is

(A) Isentropic (B) Isothermal (C) Polytropic (D) Adiabatic

© All rights reserved by Thinkcell Learning Solutions Pvt. Ltd. No part of this booklet may be reproduced or utilized in any form without the written permission.

www.engineeringonyourfingertips.ooo

More Notes Join us Telegram-: http//:t.me/allexammentor


www.engineeringonyourfingertips.ooo
|ME-2019-Forenoon|

Key: (B)
1
Sol: First law of thermodynamics for non flow (closed) system,
dQ  du  dW T  Constant
 dQ   mc dT   dW
v  1 P
2
When process is isothermal, dT = 0  Q12  W12
Shaded area shows equal amount of heat & work.
V


4
7. Evaluation of x 3 dx using a 2-equal-segment trapezoidal rule gives a value of __________.
2

Key: (63)

Sol: Using Trapezoidal rule, we have


b
h
 f  x  dx  2  y
a
0  y n   2  y1  y 2  ...  y n 1  ;

ba
where h  step size 
n
Let f  x   x 3 ; a  2; b  4; n  number of intervals  2
42
h 1
2
x 2 3 4
8 27 64
f  x   x3
 y0   y1   y2 
4
1 1
  x 3dx  8  64   2  27   72  54  63
2
2 2
4
  x 3dx  63 www.
engi
neer
ingonyour
fi
nger
tips.
ooo
2

8. The natural frequencies corresponding to the spring-mass systems I and II are I and II ,

I
respectively. The ratio is
II
k k

SYSTEM  I

© All rights reserved by Thinkcell Learning Solutions Pvt. Ltd. No part of this booklet may be reproduced or utilized in any form without the written permission.

www.engineeringonyourfingertips.ooo

More Notes Join us Telegram-: http//:t.me/allexammentor


www.engineeringonyourfingertips.ooo
|ME-2019-Forenoon|

k
SYSTEM  II
1 1
(A) (B) 4 (C) 2 (D)
2 4
Key: (A)

Sol:
k k

Since springs are in series


1 1 1 1 2
   
k eq k k k eq k
k k eq k
k eq  and I  
2 m 2m
k

www.
engi
neer
ingonyour
fi
nger
tips.
ooo
k

Since springs are in parallel k eq  k  k  k eq  2k

k eq 2k
II  
m m
k
I 2m  k  m  1

II 2k 2m 2k 2
m

9. A solid cube of side 1 m is kept at a room temperature of 32° C. The coefficient of linear
thermal expansion of the cube material is 1×10–5/°C and the bulk modulus is 200 GPa. If the

© All rights reserved by Thinkcell Learning Solutions Pvt. Ltd. No part of this booklet may be reproduced or utilized in any form without the written permission.

www.engineeringonyourfingertips.ooo

More Notes Join us Telegram-: http//:t.me/allexammentor


www.engineeringonyourfingertips.ooo
|ME-2019-Forenoon|

cube is constrained all around and heated uniformly to 42°C, then the magnitude of volumetric
(mean) stress (in MPa) induced due to heating is ________.
Key: (60)

Sol: a  1m K  200GPa
T1  32C Tf  42C
5
  1  10 v  ?
x  T  1 105  42  32   1 104
v  3 x  3  104
v
K   v  3  104  200  103  60MPa
v

10. For a hydro dynamically and thermally fully developed laminar flow through a circular pipe of
constant cross-section, the Nusselt number at constant wall heat flux (Nu q) and that at constant
wall temperature (NuT) are related as

Nu q  Nu T (B) Nu q   Nu T  (C) Nu q  Nu T (D) Nu q  Nu T


2
(A)

Key: (D)

Sol: Since average convective heat transfer coefficient h in case of constant heat flux h q is   
more that from constant wall temperature h T .  
 Nuq  NuT
For fully developed laminar flow,
Nu q  4.36, Nu T  3.66

11. A flat-faced follower is driven using a circular eccentric cam rotating at a constant angular
www.
engi
neer
ingonyour
fi
nger
tips.
ooo
velocity . At time t = 0, the vertical position of the follower is y(0) = 0, and the system is in
the configuration shown below

yt

© All rights reserved by Thinkcell Learning Solutions Pvt. Ltd. No part of this booklet may be reproduced or utilized in any form without the written permission.

www.engineeringonyourfingertips.ooo

More Notes Join us Telegram-: http//:t.me/allexammentor


www.engineeringonyourfingertips.ooo
|ME-2019-Forenoon|

The vertical position of the follower face, y(t) is given by

(A) e 1  cos 2t  (B) esin t

(C) esin 2t (D) e 1  cos t 

Key: (D)

Sol: t  0, y  0   0, 
B
A

Q
 e
P S
O

x  AB  OS  OQ  QS
 OQ  PQ cos 
 OQ  OQ cos 
y  OQ 1  cos    e 1  cos    e 1  cos  t  

12. In a casting process, a vertical channel through which molten metal and flows downward from
pouring basin to runner for reaching the mold cavity is called
(A) sprue (B) pin hole (C) riser (D) blister
Key: (A)
www.
engi
neer
ingonyour
fi
nger
tips.
ooo
13. Air of mass 1 kg, initially at 300K and 10 bar, is allowed to expand isothermally till it reaches a
pressure of 1 bar. Assuming air as an ideal gas with gas constant of 0.287 kJ/kg.K, the change in
entropy of air (in kJ/kg.K, round off to two decimal places) is __________.
Key: (0.66)

Sol: Given that for Air, initially m  1kg, T1  300K, P1  10 bar


Finally; T2  300K  Isothermal 
P2  1 bar
R  0.287 KJ kg. K
 T P 
Change in entropy, S  m C p n 2  R n 2   1
 T1 P1 

© All rights reserved by Thinkcell Learning Solutions Pvt. Ltd. No part of this booklet may be reproduced or utilized in any form without the written permission.

www.engineeringonyourfingertips.ooo

More Notes Join us Telegram-: http//:t.me/allexammentor


www.engineeringonyourfingertips.ooo
|ME-2019-Forenoon|

Since T2  T1
P2 1
S  mR n  1  0.287  n
P1 10
S  0.66084
S  0.66 kJ kg  K

14. A block of mass 10 kg rests on a horizontal floor. The acceleration due to gravity is 9.81 m/s 2.
The coefficient of static friction between the floor and the block is 0.2.

10N
10kg

A horizontal force of 10 N is applied on the block as shown in the figure. The magnitude of
force of friction (in N) on the block is _____.
Key: (10)

Sol: m  10kg,   0.2, F  10N


g  9.81 m sec2 ,
10kg 10N

F  W

F  0.2  10  9.81  19.62  10


Hence friction force = 10N.

www.
engi
neer
ingonyour
fi
nger
tips.
ooo
1 1 0 
15. Consider the matrix P  0 1 1 
0 0 1 

The number of distinct eigenvalues of P is


(A) 0 (B) 1 (C) 3 (D) 2
Key: (B)

Given 1 1 0 
P  0 1 1 
0 0 1 

Upper triangular matrix

© All rights reserved by Thinkcell Learning Solutions Pvt. Ltd. No part of this booklet may be reproduced or utilized in any form without the written permission.

www.engineeringonyourfingertips.ooo

More Notes Join us Telegram-: http//:t.me/allexammentor


www.engineeringonyourfingertips.ooo
|ME-2019-Forenoon|

 Eigen values of P are 1, 1, 1; since the Eigen values of upper triangular matrix are it’s
diagonal elements.
 Number of distinct eigen values of P = 1.

16. During a high cycle fatigue test, a metallic specimen is subjected to cyclic loading with a mean
stress of +140 MPa, and a minimum stress of –70 MPa. The R-ratio (minimum stress to
maximum stress) for this cycle loading is _____ (round off to one decimal place).
Key: (–0.2)

Sol: mean  140MPa


min
min  70MPa; ?
max
max   min
mean   140
2
max  280  70   max  350
 min 70 1
   0.2
max 350 5

17. A slender rod of length L, diameter d (L >> d) and thermal conductivity k1 is joined with
another rod of identical dimensions, but of thermal conductivity k2, to form a composite
cylindrical rod of length 2L. The heat transfer in radial direction and contact resistance are
negligible. The effective thermal conductivity of the composite rod is
2k1k 2 k1 k 2
(A) k1  k 2 (B) k1k 2 (C) (D)
k1  k 2 k1  k 2
Key: (C)

Sol: ww
Two rods of equal w. eLng
length i
n
are eer ingony
conductivity ko ur
1 andfki
n
2g er
are tips.ooo
connected.
No radial heat transfer
 Equivalent resistance per unit area Rod1 Rod2
L L k1 k2
R eq =   i 
k1 k 2
If this is single composite rod; Temp
Resistance, of composite rod per unit area
2L
R comp    ii 
k eq
From (I) and (II)

L1  L2  L

© All rights reserved by Thinkcell Learning Solutions Pvt. Ltd. No part of this booklet may be reproduced or utilized in any form without the written permission.

www.engineeringonyourfingertips.ooo

More Notes Join us Telegram-: http//:t.me/allexammentor


www.engineeringonyourfingertips.ooo
|ME-2019-Forenoon|

2L L L
 
k eq k1 k 2
2k1k 2
k eq 
k1  k 2

18. Consider an ideal vapor compression refrigeration cycle. If the throttling process is replaced by
an isentropic expansion process, keeping all the other processes unchanged, which one of the
following statements is true for the modified cycle?
(A) Coefficient of performance is the same as that of the original cycle
(B) Coefficient of performance is lower than that of the original cycle
(C) Refrigerating effect is lower than that of the original cycle
(D) Coefficient of performance is higher than that of the original cycle
Key: (D)

Sol: Figure 1: Isenthalpic expansion


Figure 2: Isentropic expansion

2 2'
3 3'

T T hc

4 1 1'
4'

S S

wFigure1
ww. engi
neer
ingonyour
fi
nger
tips.
o oo 2
Figure

Form both figures we see


h1'  h 4'  h1  h 4 (Refrigerating effect)
 COP in case (ii) is more than (i)because of more refrigerating effect.
 COP of isentropic expansion is more than isenthalpic expansion.

© All rights reserved by Thinkcell Learning Solutions Pvt. Ltd. No part of this booklet may be reproduced or utilized in any form without the written permission.

www.engineeringonyourfingertips.ooo

More Notes Join us Telegram-: http//:t.me/allexammentor


www.engineeringonyourfingertips.ooo
|ME-2019-Forenoon|

19. The position vector OP of point P(20, 10) is rotated anti-clockwise in X-Y plane by an angle
  30 such that point P occupies position Q, as shown in the figure. The coordinates (x, y) of
Q are Y
Q

P

X
O

(A) (13.40, 22.32) (B) (12.32, 18.66) (C) (22.32, 8.26) (D) (18.66, 12.32)
Key: (B)

Sol: P   20,10  ,   30o Y


Q
Q   x ', y '
 x ' cos   sin    20 
 y '   sin  cos   10 
 P
  
 3 2 1 2   20 
   X
 1 2 3 2  10  O
0.866 0.5   20  12.32 
     
 0.5 0.866  10  18.66 
 x ', y '  12.32,18.66 

20. A cylindrical rod of diameter 10 mm and length 1.0 m fixed at one end. The other end is twisted
by angle of 10° bywww. en
applying agi
neer
torque. i
Ifnthe
gon your f
maximumi
n ger
shear tips.
strain o
inoo rod is p×10 –3, then p is
the
equal to _____ (round off to two decimal places).
Key: (0.8726)

Sol: D  10mm,   10


L  1m
  P  103
L  R

1  1000  P  103  5  10 
180
50
P  0.8726
180

© All rights reserved by Thinkcell Learning Solutions Pvt. Ltd. No part of this booklet may be reproduced or utilized in any form without the written permission.

www.engineeringonyourfingertips.ooo

More Notes Join us Telegram-: http//:t.me/allexammentor


www.engineeringonyourfingertips.ooo
|ME-2019-Forenoon|

21. Which one of the following welding methods provides the highest heat flux (W/mm2)?
(A) Plasma are welding (B) Tungsten inert gas welding
(C) Oxy-acetylene gas welding (D) Laser beam welding
Key: (D)

Sol:
Welding process  W cm  2 Temp(oC)

Heat density
1. Gas welding 102  103 2500-3500
2. Shielded metal Arc 104 >6000
welding
3. Gas metal Arc 105 8000-10,000
welding
4. Plasma Arc welding 106 15000-30000
5. Electron beam 107  108 20000-30000
welding
6. Laser beam welding 109 >30,000

4 
22. Water flows through a pipe with a velocity given by V    x  y  ˆj m s, where ˆj is the unit
t 
vector in the y direction, t(>0) is in seconds, and x and y are in meters. The magnitude of total
acceleration at the point (x, y) = (1, 1) at t = 2s is ___________ m/s2.
Key: (3)

4 
Sol: V    x  y  j m sec
t  www. engi
neer
ingonyour
fi
nger
tips.
ooo
V  ui  vj  wk
Acceleration at any point x, y, z and any time‘t’ is given as
dV dV dV dV
a  x,y,z,t   u V w
dt dx dy dz

4 
Given that x  1, y  1, t  2, z  0 and u=0, V    x  y  , w  0
t 
then,
dV d  4  4
   x  y  j.j   2
dt dt  t  t
dV d  4 
   x  y  j.j  1
dx dx  t 

© All rights reserved by Thinkcell Learning Solutions Pvt. Ltd. No part of this booklet may be reproduced or utilized in any form without the written permission.

www.engineeringonyourfingertips.ooo

More Notes Join us Telegram-: http//:t.me/allexammentor


www.engineeringonyourfingertips.ooo
|ME-2019-Forenoon|

dV d  4 
   x  y  j.j  1
dy dy  t 
dV d  4 
   x  y  j.j  0
dz dz  t 
4 4 
 a 1,1,0,2  2   0 1    x  y  1   0   0 
t t 
4 4
 2  xy
t t
4 4
 2   1  1  1  2  1  1  3m sec 2
2 2

A parabola x  y with 0  x  1 is shown in the figure. The volume of the solid of rotation
2
23.
obtained by rotating the shaded area by 360° around the x-axis is

x  y2

O 1 x

 
(A) (B) (C) 2 (D) 
4 2
Key: (B)
www.
engi
neer
ingonyour
fi
nger
tips.
ooo
Sol: Volume of the solid of rotation obtained by rotating around the x – axis is given by
b
y
V  y dx
2

x a x  y2
1
V  xdx 
x 0
y 2  x 

1
 x2  
   
 2 0 2
0  x
1

© All rights reserved by Thinkcell Learning Solutions Pvt. Ltd. No part of this booklet may be reproduced or utilized in any form without the written permission.

www.engineeringonyourfingertips.ooo

More Notes Join us Telegram-: http//:t.me/allexammentor


www.engineeringonyourfingertips.ooo
|ME-2019-Forenoon|

24. A spur gear with 20° full depth teeth is transmitting 20 kW at 200 rad/s. The pitch circle
diameter of the gear is 100mm. The magnitude of the force applied on the gear in the radial
direction is
(A) 1.39 kN (B) 2.78 kN (C) 0.36 kN (D) 0.73 kN
Key: (D)

Sol:   20, P  20kW,   200rad sec.


D  100mm, FR  ?
P 20  103
T   100Nm
 200
T 100
Ft    2000N
R 50  103
FR  Ft tan   2000 tan 20  727.9N  0.73kN

 7x 2 y  0, if y  0   3 7, then the value of y(1) is


dy
25. For the equation
dx
7 7 3 3 7 3 3 3 7 7 3 7
(A) e (B) e (C) e (D) e
3 7 7 3
Key: (B)

dy
Sol: Given D.E is  7x 2 y  0, y  0   3 7
dx
The value of y (1) is ____.
dy
  7x 2  y   0  1
dx
 The equation (1) is linear D.E, Where P  7x 2 ; Q  0
7 x 3
I.F  e  e ww
2 3
P dx 7x dx
w
e. engi neer
ingonyour
fi
nger
tips.
ooo
Solution of equation (1) is
y. I.F    Q. I.F  dx  C
7x 3 7x 3
 ye 3
  0.  I.F  dx  C  ye 3
C
 7x 3
yCe 3
  2
Given y  3 7 at x  0
 2  3 7  C
3  7 x3 3
From  2  , y  e 3  y 1  e 7 3
7 7

© All rights reserved by Thinkcell Learning Solutions Pvt. Ltd. No part of this booklet may be reproduced or utilized in any form without the written permission.

www.engineeringonyourfingertips.ooo

More Notes Join us Telegram-: http//:t.me/allexammentor


www.engineeringonyourfingertips.ooo
|ME-2019-Forenoon|

Q. No. 26 to 55 Carry Two Marks Each

26. A cube of side 100 mm is placed at the bottom of an empty container on one of its faces. The
density of the material of the cube is 800 kg/ m3. Liquid of density 1000 kg/m3 is now poured
into the container. The minimum height to which the liquid needs to be poured into the
container for the cube to just lift up is _____ mm.
Key: (80)

Sol:

a  100

Given that density of cube material ecube  800kg m3


Density of liquid poured e water  1000kg m3
Weight of cube  ecube  volume of cube  g
 800  0.1 0.1 0.1 g  0.8g N
To just lift the cube, weight of cube = buoyancy force buoyancy force = weight of liquid
displaced
 eliquid  volume of liquid  g  1000  0.1 0.1 h  g  10hg
Where h = height of water poured
By equating weight of cube = buoyancy force
0.8g  10hg
0.8 www. engi
neer
ingonyour
fi
nger
tips.
ooo
h  0.08m  80 mm
10

27. A project consists of six activities. The immediate predecessor of each activity and the
estimated duration is also provided in the table below:
Activity Immediate predecessor Estimated duration (weeks)
P - 5
Q - 1
R Q 2
S P, R 4
T P 6
U S,T 3

© All rights reserved by Thinkcell Learning Solutions Pvt. Ltd. No part of this booklet may be reproduced or utilized in any form without the written permission.

www.engineeringonyourfingertips.ooo

More Notes Join us Telegram-: http//:t.me/allexammentor


www.engineeringonyourfingertips.ooo
|ME-2019-Forenoon|

If all activities other than S take the estimated amount of time, the maximum duration (in
weeks) of the activity S without delaying the completion of the project is ____________.
Key: (6)

Sol: From the given data, we can represent network flow as follows

T 6
1
P 5
U  3
1 6
1
Dummy  0 

Q 1 R  2 S 4
1 1

Considering path 1-2-5-6, time taken will be = 5+ 6+ 3= 14 weeks

Considering path 1-2-4-5-6, time taken will be=5+0+4+5=12 weeks.


Considering path 1-3-4-5-6, time taken will be = 1+2+4+3 =10 weeks
Maximum time taken is 14 weeks, so ‘2’ weeks can be delayed so that 1-2-4-5-6 path will also
take 14 weeks.
So answer is 4 weeks +2 weeks = 6 weeks
Duration can be given for activities without delay the project.

28. Consider an elastic straight beam of length L  10m, with square cross-section of side a=5
mm, and Young’s modulus E = 200 GPa. This straight beam was bent in such a way that the
two ends meet, to form a circle of mean radius R. Assuming that Euler-Bernoulli beam theory is
applicable to this bending problem, the maximum tensile bending stress in the bent beam is
___________ MPa.
www.
engi
neer
ingonyour
fi
nger
tips.
ooo

L R

Key: (100)
Ends of the beam
Sol: L  10 mts; a  5mm, E  200GPa
10
L  2R, R   5mts  5000mm.
2
a 5
y 
2 2
E 200  103  5 
 y     100MPa
R 5000 2

© All rights reserved by Thinkcell Learning Solutions Pvt. Ltd. No part of this booklet may be reproduced or utilized in any form without the written permission.

www.engineeringonyourfingertips.ooo

More Notes Join us Telegram-: http//:t.me/allexammentor


www.engineeringonyourfingertips.ooo
|ME-2019-Forenoon|

29. A truss is composed of members AB, BC, CD, AD and BD, as shown in the figure. A vertical
load of 10 kN is applied at point D. The magnitude of force (in kN) in the member BC is
________. 10kN
D

45 45
A C
B
Key: (5)

Sol: Due symmetry


10kN
RA  RC   5kN
2
Joint C
R DC
135
45

R BC 90 90

RC
5kN
R BC R DC 5
 
sin 225 sin 90 sin 45
5 sin 225
R BC   5kN  Tension 
sin 45
www.
engi
neer
ingonyour
fi
nger
tips.
ooo

30. A gas is heated in a duct as it flows over a resistance heater. Consider a 101 kW electric heating
system. The gas enters the heating section of the duct at 100 kPa and 27°C with a volume flow
rate of 15m3/s. If heat is lost from the gas in the duct to the surroundings at a rate of 51kW, the
exit temperature of the gas is
(Assume constant pressure, ideal gas, negligible change in kinetic and potential energies and
constant specific heat; Cp = 1 kJ/kg.K; R = 0.5 kJ/kg.K).
(A) 53°C (B) 32°C (C) 37°C (D) 76°C

© All rights reserved by Thinkcell Learning Solutions Pvt. Ltd. No part of this booklet may be reproduced or utilized in any form without the written permission.

www.engineeringonyourfingertips.ooo

More Notes Join us Telegram-: http//:t.me/allexammentor


www.engineeringonyourfingertips.ooo
|ME-2019-Forenoon|

Key: (B)

Sol: Q

W
Heater

Inlet conditions
P1  100kPa, T1  27C  300K
V1  V2  15m3 s
Q  51 kW
W  101 kW
P1V1
Now, mass flow rate m 
RT1
100  15
m  10 kg s
0.5  300
From 1st law of thermodynamics and steady flow energy equation
Q  m  h 2  h1   W
51  m  h 2  h1   101
m  h 2  h1   50
10  h 2  h1   50  h 2  h1  5
Cp  T2  T1   5  T2  27  5  32 C

31. ww
A harmonic function isw.engi
analyticnife
iteringon
satisfies yo
the ur fi
ngequation.
Laplace er
tips. oIfoo
u  x, y   2x 2  2y 2  4xy

is a harmonic function, then its conjugate harmonic function v  x, y  is

(A)  4xy  2y2  2x 2  constant (B) 4xy  2x 2  2y2  constant

(C) 2x 2  2y2  xy  constant (D) 4y2  4xy  constant


Key: (B)

Given, u  x, y   2x  2y  4xy is a harmonic function.


2 2
Sol:
u u
  4x  4y;  4y  4x
x y
The conjugate harmonic function v  x, y  is obtained as follows:

© All rights reserved by Thinkcell Learning Solutions Pvt. Ltd. No part of this booklet may be reproduced or utilized in any form without the written permission.

www.engineeringonyourfingertips.ooo

More Notes Join us Telegram-: http//:t.me/allexammentor


www.engineeringonyourfingertips.ooo
|ME-2019-Forenoon|

v v
dv  dx  dy  from total differential of v  x, y 
x y
 u   u 
 dv     dx    dy ,using C  R equations
 y   x 
 dv    4y  4x  dx   4x  4y  dy

Exact D.E

  dv     4y  4x  dx    4x  4y  dy
  x 2 
 v  x, y     4yx  4     4  y 2 2   C
  2 
 v  x, y   4xy  2x  2y 2  C
2

32. A uniform thin disk of mass 1 kg and radius 0.1 m is kept on a surface as shown in the figure. A
spring of stiffness k1 = 400 N/m is connected to the disk center A and another spring of stiffness
k2 = 100 N/m is connected at point B just above point A on the circumference of the disk.
Initially, both the springs are unstretched. Assume pure rolling of the disk. For small
disturbance from the equilibrium, the natural frequency of vibration of the system is ______
rad/s (round off to one decimal place).

k2
B

k1
A

www.
engi
neer
ingonyour
fi
nger
tips.
ooo
Key: (23.1)

Sol:
k2
B

k1

Give a small displacement to Disc about ‘O’

© All rights reserved by Thinkcell Learning Solutions Pvt. Ltd. No part of this booklet may be reproduced or utilized in any form without the written permission.

www.engineeringonyourfingertips.ooo

More Notes Join us Telegram-: http//:t.me/allexammentor


www.engineeringonyourfingertips.ooo
|ME-2019-Forenoon|

k 2 2r sin 
2r sin 
B
2r cos 
r
r sin
k1 rsin 
A
r cos 
I0 
O 
O'
m0  0
I0    k1r sin   r cos     k 2 2r sin   2r cos    0
mr 2 3 3
I0  IC.G.  mr 2   mr 2  mr 2   1 0.12  0.015kg m 2
2 2 2
m  1kg , r  0.1 m, k1  400 N m, k 2  100 N m
Assume sin   , cos   1
0.015    400  0.1   100  2  0.1   2  0.1  0
2

0.015  8  0
8
n   23.09  23.1 rad sec
0.015

33. In ASA system, the side cutting and end cutting edge angles of a sharp turning tool are 45° and
10°, respectively. The feed during cylindrical turning is 0.1 mm/rev. The center line average
surface roughness (in m, round off to one decimal place) of the generated surface is _______.
Key: (3.747) www.
engi
neer
ingonyour
fi
nger
tips.
ooo
Sol: Given, Cs  45, Ce  10
f  0.1 mm / rev
f
R a or CLA 
4  tan Cs  cot Ce 
0.1
  3.747  103 mm  3.747 m.
4  tan 45  cot10 

34. Consider a prismatic straight beam of length L  m, pinned at the two ends as shown in the
figure.

© All rights reserved by Thinkcell Learning Solutions Pvt. Ltd. No part of this booklet may be reproduced or utilized in any form without the written permission.

www.engineeringonyourfingertips.ooo

More Notes Join us Telegram-: http//:t.me/allexammentor


www.engineeringonyourfingertips.ooo
|ME-2019-Forenoon|

The beam has a square cross-section of side p = 6mm. The Young’s modulus E = 200 GPa, and
the coefficient of thermal expansion   3 106 K1. The minimum temperature rise required to
cause Euler buckling of the beam is ___________K.
Key: (1)

Sol: L  , T  ?
64
Area  6  6  36 mm 2 , I   108mm 4
12
E  200GPa
  3  106 K 1
2 EI 2  200  103  108
PE    21.6 MN
2
e 2  106
PE  EAT
21.6  200  103  36  3  10 6  T
T  1K.

35. The set of equations


x  y  z 1
ax  ay  3z  5
5x  3y  az  6
has infinite solutions, if a =
(A) 4 (B) –4 (C) –3 (D) 3
Key: (A)

x  y  z 1 
Sol: ax  ay  3z  5    Non  homogeneous 
5x  3y  az  6 
www.
engi
neer
ingonyour
fi
nger
tips.
ooo
1 1 1 1 
Augmented matrix, [A|B]   a a 3 5 
 5 3 a 6 

R2  R3
1 1 1 1 
5 3 a 6 
 A | B  
a a 3 5 

Applying R 2  R 2  5R1; R 3  R 3  aR1

© All rights reserved by Thinkcell Learning Solutions Pvt. Ltd. No part of this booklet may be reproduced or utilized in any form without the written permission.

www.engineeringonyourfingertips.ooo

More Notes Join us Telegram-: http//:t.me/allexammentor


www.engineeringonyourfingertips.ooo
|ME-2019-Forenoon|

1 1 1 1 
0 8 a  5 1 

0 2a 3  a 5  a 
R 3  4R 3  a R 2
1 1 1 1 
0 8 a 5 1 

0 0 a 2  a  12 20  5a 
To have infinite number of solutions,
a 2  a  12  0 & 20  5a  0
  a  4  a  3  0 a 4
 a  4  or  a  3 & a4
 a must be equal to '4' only.

36. In a UTM experiment, a sample of length 100 mm, was loaded in tension until failure. The
failure load was 40 kN. The displacement, measured using the cross-head motion, at failure,
was 15 mm. The compliance of the UTM is constant and is given by 5 × 10–8 m/N. The strain at
failure in the sample is ___________%.
Key: (13)
Sol: Sample length    100mm
Failure load  p   40 kN 40kN
Deformation at failure total  15mm
Force
 total 15
 Total strain    0.15  15%
100
1
Compliance of the UTM  5  108 m N 
www.
engi
neer
ingon your
stiffnessfi
nger
tips.
ooo
AE  1 
For axial loaded specimen, stiffness   8 
N m  permanent recoverable
 5  10 
 total
Deformation recoverable at failure load Deformation
re cov erable 2
 Recoverable strain    0.02  2%
100
Permanent strain = Total strain – Recoverable = 15 – 2 = 13%

37. A plane-strain compression (forging) of a block is shown in the figure. The strain in the z-
direction is zero. The yield strength (Sy) in uniaxial tension/compression of the material of the
block is 300 MPa and it follows the Tresca (maximum shear stress) criterion. Assume that the

© All rights reserved by Thinkcell Learning Solutions Pvt. Ltd. No part of this booklet may be reproduced or utilized in any form without the written permission.

www.engineeringonyourfingertips.ooo

More Notes Join us Telegram-: http//:t.me/allexammentor


www.engineeringonyourfingertips.ooo
|ME-2019-Forenoon|

entire block has started yielding. At a point where x  40 MPa (compressive) and xy  0, the

stress component  y is
y
Moving platen

O x

Fixed platen

(A) 260 MPa (tensile) (B) 340 MPa (compressive)


(C) 260 MPa (compressive) (D) 340 MPa (tensile)
Key: (B)

Sol:  y  300MPa,  x  40MPa  compressive  ,  xy  0,  y  ?


For plane strain
z  y
z  0  v x v  0  z  v  x   y 
E E E
 z  v  40   y 
 x   y  y  z z   x 
max  M ax  , , 
 2 2 2 

Sy  40   y  y  v   y  40  v   y  40   40 
max   Max  , , 
2  F.S  2 2 2 

40   y Sy
From the above equation Maximum will be the first one i.e., 
2 2  F.S
40   y Sywww. g e
e Sy our
or en i
n yr i
ngony f
inger
tips.
ooo
40

2 2  F.S 2 2  F.S
40   y  300 40   y  300
 y  340MPa  y  260MPa
 y  340MPa(Compressive) or  y  260MPa (Tensile)

40   y  300   y  260 MPa  Tension 

But in the forging operation  y can’t be tensile hence the answer is 340MPa (compressive).

38. Match the following sand mold casting defects with their respective causes.
Defect Cause
(P) Blow hole 1. Poor collapsibility
(Q) Misrun 2. Mold erosion

© All rights reserved by Thinkcell Learning Solutions Pvt. Ltd. No part of this booklet may be reproduced or utilized in any form without the written permission.

www.engineeringonyourfingertips.ooo

More Notes Join us Telegram-: http//:t.me/allexammentor


www.engineeringonyourfingertips.ooo
|ME-2019-Forenoon|

(R) Hot tearing 3. Poor permeability


(S) Wash 4. Insufficient fluidity
Codes:
(A) P-3, Q-4, R-2, S-1 (B) P-4, Q-3, R-1, S-2
(C) P-2, Q-4, R-1, S-3 (D) P-3, Q-4, R-1, S-2
Key: (D)

39. A steam power cycle with regeneration as shown below on the T-s diagram employs a single
open feedwater heater for efficiency improvement. The fluids mix with each other in an open
feedwater heater. The turbine is isentropic and the input (bleed) to the feedwater heater from the
turbine is at state 2 as shown in the figure. Process 3-4 occurs in the condenser. The pump work
is negligible. The input to the boiler is at state 5.
The following information is available from the steam tables:

State 1 2 3 4 5 6

Enthalpy (kJ/kg) 3350 2800 2300 175 700 1000

T
5 2
4
3

www.
engi
neer
ingonsyour
fi
nger
tips.
ooo
The mass flow rate of steam bled from the turbine as a percentage of the total mass flow rate at
the inlet to the turbine at state 1 is _______.

Key: (20)

Sol: 1
6
m2
T
5 2
m1
4
3

© All rights reserved by Thinkcell Learning Solutions Pvt. Ltd. No part of this booklet may be reproduced or utilized in any form without the written permission.

www.engineeringonyourfingertips.ooo

More Notes Join us Telegram-: http//:t.me/allexammentor


www.engineeringonyourfingertips.ooo
|ME-2019-Forenoon|

Let m 2 mass is bled at state 2 and m1 mass goes to the condenser.


m1h 4
Assuming no heat loss from the feed water heater,
m1h 4  m 2 h 2   m1  m 2  h 5  1
h 4  175 kJ kg, h 2  2800kJ kg, h 5  700 kJ kg Q0
175 m1  2800m 2   m1  m 2  700 m2 h 2
 2100 m 2  525 m1
m2 m2
 0.25   0.2. Hence 20%
m1 m1  m2
 m1  m 2  h 5
40. The rotor of turbojet engine of an aircraft has a mass 180 kg and polar moment of inertia 10
kg.m2 about the rotor axis. The rotor rotates at a constant speed of 1100 rad/s in the clockwise
direction when viewed from the front of the aircraft. The aircraft while flying at a speed of 800
km per hour takes a turn with a radius of 1.5 km to the left. The gyroscopic moment exerted by
the rotor on the aircraft structure and the direction of motion of the nose when the aircraft
turns, are
(A) 1629.6 N.m and the nose goes up
(B) 1629.6 N.m and the nose goes down
(C) 162.9 N.m and the nose goes down
(D) 162.9 N.m and the nose goes up
Key: (B)

Sol: m  180kg   1100rad sec, V  800kmph R  1.5 mts


I  10kg  m 2
5
800 
V
p   w
18ww.engi ne
0.148rad er
sec i
ngonyour
fi
nger
tips.
ooo
R 1.5  1000
C  Ip  1100  10  148.148  1629.628 Nm

a
O

Dip the nose and Raise the tail. So answer is option ‘B’.

41. The wall of a constant diameter pipe of length 1 m is heated uniformly with flux q" by
wrapping a heater coil around it. The flow at the inlet to the pipe is hydrodynamically fully

© All rights reserved by Thinkcell Learning Solutions Pvt. Ltd. No part of this booklet may be reproduced or utilized in any form without the written permission.

www.engineeringonyourfingertips.ooo

More Notes Join us Telegram-: http//:t.me/allexammentor


www.engineeringonyourfingertips.ooo
|ME-2019-Forenoon|

developed. The fluid is incompressible and the flow is assumed to be laminar and steady all
through the pipe. The bulk temperature of the fluid is equal to 0°C at the inlet and 50°C at the
exit. The wall temperatures are measured at three locations, P, Q and R, as shown in the figure.
The flow thermally develops after some distance from the inlet. The following measurements
are made:

Point P Q R
Wall Temp (°C) 50 80 90

0.2 m 0.2 m 0.2 m 0.2 m 0.2 m

P Q R
0 C 50C

Constant wall flux

Among the locations P, Q and R, the flow is thermally developed at:


(A) P and Q only (B) P, Q and R (C) R only (D) Q and R only

Key: (D)

Sol: www.
engi
neer
ingonyour
fi
nger
tips.
ooo

0 C 50C

P Q R

From heat balace


q" d  x  m.c.  TB  Tin   1
Where mc = heat capacity of fluid and
 q" d 
TB  bulk mean temperature  TB    x  Tin
 mc 
At inlet z  0, Tin  0

© All rights reserved by Thinkcell Learning Solutions Pvt. Ltd. No part of this booklet may be reproduced or utilized in any form without the written permission.

www.engineeringonyourfingertips.ooo

More Notes Join us Telegram-: http//:t.me/allexammentor


www.engineeringonyourfingertips.ooo
|ME-2019-Forenoon|

x  1m, TB  50 C
q" d
  Z  constant   50
mc
 TB  50x   ii 

Now : q" d  h*  Tw  TB  At any section 


q" d
   Tw  TB 
h
q" d
  TB  Tw
h
 q" d 
 Tw  C  TB C     constant for fully developed flow.
 h 
Tw  C  50x.
At P, Tw  50C, x  0.4 50
50 For thermally
30
 50  50  0.4  C  30 Tw developed region
At Q, Tw  80C, x  0.6 TB Tw  TB  C
 80  50  0.6  50  C is constant at Q & R
At R, Tw  90, x  0.8
90  50  0.8  50  C P Q R
 Clearly we see that Tw  TB is constant from Q
 Flow will be thermally developed between Q &R

42. At a critical point in a component, the state of stress is given as xx  100 MPa,
 yy  220 MPa, xy   yx  80 MPa and all other stress components are zero. The yield strength
of the material is 468 MPa. The factor of safety on the basis of maximum shear stress theory is
_________ (roundwww
off to .
eng
one ineer
decimal ingonyour
place). fi
nger
tips.
ooo
Key: (1.8)

Sol: xx  100MPa,  yy  220MPa,  xy   yx  80MPa,  yt  468MPa.


 yt  yt     2 1  2 
  1 , , 
F.O.S 2F.0.S  2 2 2
x  y  x  y 
2

1,2       xy
2

2  2 
100  220  100  220 
2

     80
2

2  2 
 160  100  260MPa, 60MPa
yt  260  60 260 60  468
 Max  , ,   F.O.S   1.8
2  F.O.S  2 2 2 260

© All rights reserved by Thinkcell Learning Solutions Pvt. Ltd. No part of this booklet may be reproduced or utilized in any form without the written permission.

www.engineeringonyourfingertips.ooo

More Notes Join us Telegram-: http//:t.me/allexammentor


www.engineeringonyourfingertips.ooo
|ME-2019-Forenoon|

43. A gas turbine with air as the working fluid has an isentropic efficiency of 0.70 when operating
at a pressure ratio of 3. Now, the pressure ratio of the turbine is increased to 5, while
maintaining the same inlet conditions. Assume air as a perfect gas with specific heat ratio
  1.4. If the specific work output remains the same for both the cases, the isentropic
efficiency of the turbine at the pressure ratio of 5 is _____ (round off to two decimal places).
Key: (0.51)

P1 P 1 P1
Sol:  3, l  5, rp  pressure ratio
P2 P3
Work done by turbine 1 for rp  3 P2

W1 2'  mc p  T1  T2   mc p  1 2'   T1  T2  T 2 2' P3


3'
 
 T1 
 m  c p  1 2'   T1  1 

 
3
 rp1  

  S
 1 
W1 2'   m  c p  T1   1 2' 1  1   i 
 r 
 P1 

Since W12'  W13'


   
 1   1 
13' 1  1 
 1 2' 1  1 

 rP2  
 

 r 
 P1 

Putting 1 2  0.7, rP1  3, rP2  5

13'  0.5115  13'  51.15%


w
Hence efficiencyw w. engi
0.5115 neer
ingonyour
fi
nger
tips.
ooo

44. The value of the following definite integral is ________ (round off to three decimal places)
e

x
1
n x  dx

Key: (2.097)
e
Sol: x
1
nx  dx

Let n x  t  x  e t  dx  e t dt

© All rights reserved by Thinkcell Learning Solutions Pvt. Ltd. No part of this booklet may be reproduced or utilized in any form without the written permission.

www.engineeringonyourfingertips.ooo

More Notes Join us Telegram-: http//:t.me/allexammentor


www.engineeringonyourfingertips.ooo
|ME-2019-Forenoon|

If x  1  t  0
If x  e  t  n e  1
e 1 1
  x nxdx   e t te t dt   te 2t dt
1 0 0

1
  e 2t   e2t    e2 e2   1 2 1 1 1
  t            0    e      2.097.
  2   4  0  2 4   4 2 4 4

45. Taylor’s tool life equation is given by VTn = C, where V is in m/min and T is in min. In a
turning operation, two tools X and Y are used. For tool X, n = 0.3 and C = 60 and for tool Y,
n = 0.6 and C = 90. Both the tools will have the same tool life for the cutting speed (in m/min,
round off to one decimal place) of _____.
Key: (40.5)

Sol: Tool  X Tool  Y


n  0.3 n  0.6
C  60 C  90
VTX0.3  60 VTY0.6  90
1 0.3 1 0.6
 60   90 
TX    TY   
 VX   VY 
For same toll life at breakeven  VX  VY  V 
10 6 10 6
 60   90 
Tx  TY      V  40.5m min
 Vx   VY 

46. In a four bar planar mechanism shown in the figure, AB = 5 cm, AD = 4 cm and DC = 2 cm. In

www.
the configuration shown, engAB
both i
neer
andingo
DC ny
are our fi
nger
perpendiculartipAD.
to s.
oo o bar AB rotates with an
The
angular velocity of 10 rad/s. The magnitude of angular velocity (in rad/s) of bar DC at this
instant is
B

10 rad s C

A D

(A) 25 (B) 15 (C) 10 (D) 0

© All rights reserved by Thinkcell Learning Solutions Pvt. Ltd. No part of this booklet may be reproduced or utilized in any form without the written permission.

www.engineeringonyourfingertips.ooo

More Notes Join us Telegram-: http//:t.me/allexammentor


www.engineeringonyourfingertips.ooo
|ME-2019-Forenoon|

Key: (A)
B I 23
Sol: AB = 5 cm
AD = 4 cm and DC = 2cm, 10 rad s
AB  10rad sec, DC  ? C I
34

I13   so the velocity of link


BC is zero and now VB  VC A I 24
D
rAB . AB  rCD .CD I12 I 41
5  10  2  CD
CD  25rad sec I13  

47. If one mole of H2 gas occupies a rigid container with a capacity of 1000 liters and the
temperature is raised from 27°C to 37°C, the change in pressure of the contained gas (round off
to two decimal places), assuming ideal gas behavior, is _____ Pa. (R = 8.314 J/mol.K).
Key: (83.14)

Sol: Initially T1  27C  300K


n1  1 mole
R  8.314 KJ mol  K

V1  1000 litres  1m 3
Finally, T2  37C  310K
P2  ?
From ideal Gas relation
P1V1   nR  T1
P1  8.314  300 pascal
P1  300 R
www.
engi
neer
ingonyour
fi
nger
tips.
Now since the volume of container is constant hence.
ooo
V1  V2

 R  TP   nR  TP
1 2

1 2

T2 310
P2  P1  300R 
T1 300
 P2  310R  Pascal 
 Change in pressure,  P2  P1    310  300  R  10R  10  8.314
P  83.14 pascal

© All rights reserved by Thinkcell Learning Solutions Pvt. Ltd. No part of this booklet may be reproduced or utilized in any form without the written permission.

www.engineeringonyourfingertips.ooo

More Notes Join us Telegram-: http//:t.me/allexammentor


www.engineeringonyourfingertips.ooo
|ME-2019-Forenoon|

48. Three slabs are joined together as shown in the figure. There is no thermal contact resistance at
the interfaces. The center slab experience a non-uniform internal heat generation with an
average value equal to 10000 Wm–3, while the left and right slabs have no internal heat
generation.

Left extreme
face T1  100C T2
1m 1m 1m

100 W m2 .K 100 W m2 .K
30C 30C

All slabs have thickness equal to 1 m and thermal conductivity of each slab is equal to 5 Wm–1
K–1. The two extreme faces are exposed to fluid with heat transfer coefficient 100 Wm–2K–1 and
bulk temperature 30°C as shown. The heat transfer in the slabs is assumed to be one
dimensional and steady, and all properties are constant. If the left extreme face temperature T1 is
measured to be 100°C, the right extreme faced temperature T2 is _____ °C.
Key: (60)

Sol:

Left extreme Right


face T1  100C T2 entrance face
1m 1m 1m
www.
engi
neer
ingonyour
fi
nger
tips.
ooo
100 W m .K 2 100 W m 2 .K
T1  30C T2  30C

100  30    T1  T   7 kN m
Heat flowing from left slab to left extreme face  1 2

 1  1
   
 100  h
Heat generated in the central slab = 10 kW m3

For 1m length heat generated in the central slab  10 kN m3 1m  10 kW m2

© All rights reserved by Thinkcell Learning Solutions Pvt. Ltd. No part of this booklet may be reproduced or utilized in any form without the written permission.

www.engineeringonyourfingertips.ooo

More Notes Join us Telegram-: http//:t.me/allexammentor


www.engineeringonyourfingertips.ooo
|ME-2019-Forenoon|

Out of 10 kW m2 heat generated in the central slab, 7 kW m2 will be flowing out through
left slab remaining 3kW m 2 should flow through right slab.
Applying heat flow equation at right extreme face
 
T T 
2
3000   
2

  1  
 h 
   

3000  2
T  30   T  60C
 
2
1
 
 100 

49. Five jobs (J1, J2, J3, J4 and J5) need to be processed in a factory. Each job can be assigned to any
of the five different machines (M1, M2, M3, M4 and M5). The time duration taken (in minutes)
by the machines for each of the jobs, are given in the table. However, each job is assigned to a
specific machine in such a way that the total processing time is minimum. The total processing
time is _____ minutes.

M1 M2 M3 M4 M5

J1 40 30 50 50 58

J2 269 38 60 26 38

J3 40 34 28 24 30

J4 28 40 40 32 48

J 28 32 38 22 44
w5ww.
engi
neer
ingonyour
fi
nger
tips.
ooo
Key: (146)

Sol: This problem can be solved by assignment problem

M1 M2 M3 M4 M5

J1 40 30 50 50 58

J2 26 38 60 26 38

J3 40 34 28 24 30

J4 28 40 40 32 48

J5 28 32 38 22 44

Row minimization matrix is

© All rights reserved by Thinkcell Learning Solutions Pvt. Ltd. No part of this booklet may be reproduced or utilized in any form without the written permission.

www.engineeringonyourfingertips.ooo

More Notes Join us Telegram-: http//:t.me/allexammentor


www.engineeringonyourfingertips.ooo
|ME-2019-Forenoon|

10 0 20 20 28
0 12 34 0 12
16 10 4 0 6
0 12 12 4 20
6 10 16 0 22

Column minimization matrix is


10 0 16 20 22
0 12 30 0 6
16 10 0 0 0
0 12 8 4 14
6 10 12 0 16

In the above matrix all zeros can be coved with only four lines as follows

10 0 16 20 22
0 12 30 0 6
16 10 0 0 0
0 12 8 4 14
6 10 12 0 16

The least value in the uncrossed calls is 8. It is subtracted from the uncrossed cell and added
for the intersection of the vertical line and horizontal lines
18 0 16 28 22
0 4 22 0 6
24 10 0 8 0
0 4 0 4 14
6 2 4 ww
0 w.
engi
16 neer
ingonyour
fi
nger
tips.
ooo
Since the above matrix can only be covers with ‘5’ lines the assignment can be done as
follows
M1 M2 M3 M4 M5
J1 18 0 16 28 22
J2 0 4 22 0 6
J3 24 0 0 8 0
J4 0 4 0 4 14
J5 6 2 4 0 16

 Assignments of jobs are


J1  M 2 J 2  M1 J3  M5 J 4  M3 J5  M4

30 38 28 28 22
© All rights reserved by Thinkcell Learning Solutions Pvt. Ltd. No part of this booklet may be reproduced or utilized in any form without the written permission.

www.engineeringonyourfingertips.ooo

More Notes Join us Telegram-: http//:t.me/allexammentor


www.engineeringonyourfingertips.ooo
|ME-2019-Forenoon|

 Total processing time = 30 + 38 + 28 + 28 + 22 = 146 minutes

50. In orthogonal turning of a cylindrical tube of wall thickness 5mm, the axial and the tangential
cutting forces were measured at 1259 N and 1601 N, respectively. The measured chip thickness
after machining was found to be 0.3 mm. The rake angel was 10° and the axial feed was 100
mm/min. The rotational speed of the spindle was 1000 rpm. Assuming the material to be
perfectly plastic and Merchant’s first solution, the shear strength of the martial is closest to
(A) 722 MPa (B) 875 MPa (C) 200 MPa (D) 920 MPa
Key: (A)

Sol: Ft  1259 N, FC  1601 N, t C  0.3 mm


  10, F  100 mm min , N  1000 rpm
F 100
f   0.1mm / rev
N 1000
Since it is orthogonal machining
t  f  0.1 mm
t 0.1
r   0.33
t c 0.3
r cos  0.33cos10
tan     0.348
1  r sin  1  0.33 sin10
  19.18
FS  FC cos   Ft sin 
 1601 cos 19.18  1259 sin 19.18  1098.42 N
  5  0.1
FS   bt sin   1098.42 
sin 19.18 
  721.74 MPa
www.
engi
neer
ingonyour
fi
nger
tips.
ooo
51. A single block brake with a short shoe and torque capacity of 250 N-m is shown. The
cylindrical brake drum rotates anticlockwise at 100 rpm and the coefficient of friction is 0.25.

1.5a a
P

a 4

© All rights reserved by Thinkcell Learning Solutions Pvt. Ltd. No part of this booklet may be reproduced or utilized in any form without the written permission.

www.engineeringonyourfingertips.ooo

More Notes Join us Telegram-: http//:t.me/allexammentor


www.engineeringonyourfingertips.ooo
|ME-2019-Forenoon|

The value of a, in mm (round off to one decimal place), such that the maximum actuating force
P is 2000 N, is _____.
Key: (212.5)

Sol: FBD of drum is


RN
R N  Ff

Given that Braking torque b  250 N.m


b  Ff  a
250
250  Ff  a  Ff  N
a
F 250 1000
Ff  R N  R N  f   N
 a  0.25  a
FBD of lever is

a 1.5a a

o a 4

RN
wwwR.
M 0  0  P  2.5a en gi neer ai
ngonyour
fi
nger
tips.
ooo
N  a  Ff 
4
1000 250 a
 2000   2.5a  a  
a a 4
5000a  1000  62.5
a  0.2125m  212.5 mm

0.01
52. A circular shaft having diameter 65.000.05 mm is manufactured by turning process. A 50 m
thick coating of TiN is deposited on the shaft Allowed variation in TiN film thickness is 5m.
The minimum hole diameter (in mm) to just provide clearance fit is
(A) 65.12 (B) 64.95 (C) 65.01 (D) 65.10
Key: (A)

© All rights reserved by Thinkcell Learning Solutions Pvt. Ltd. No part of this booklet may be reproduced or utilized in any form without the written permission.

www.engineeringonyourfingertips.ooo

More Notes Join us Telegram-: http//:t.me/allexammentor


www.engineeringonyourfingertips.ooo
|ME-2019-Forenoon|

Sol: Shaft  65.120.01


0.05

Coating thickness  50  5m


55m  0.055mm
or 45m  0.045 mm
Clearance Fit

t max t max
Hole
ULS

ULH

Maximum
LLH Minimum clearance Basic size
clearance
ULS
LLS

Shaft

Hole

For just clearance Fit


Minimum clearance  zero
 LLH  ULS
 ULS before electro plating = 65.01
 ULS after electroplating  65.01  2  0.055  65.12mm
www.
engi
neer
ingonyour
fi
nger
tips.
ooo

53. Two immiscible, incompressible, viscous fluids having same densities but different viscosities
are contained between two infinite horizontal parallel plates, 2 m apart as shown below. The
bottom plate is fixed and the upper plate moves to the right with a constant velocity of 3 m/s.
With the assumptions of Newtonian fluid, steady, and fully developed laminar flow with zero
pressure gradient in all directions, the momentum equations simplify to

y 1
2  21
2m
2 1m

d2u x
 0.
dy 2

© All rights reserved by Thinkcell Learning Solutions Pvt. Ltd. No part of this booklet may be reproduced or utilized in any form without the written permission.

www.engineeringonyourfingertips.ooo

More Notes Join us Telegram-: http//:t.me/allexammentor


www.engineeringonyourfingertips.ooo
|ME-2019-Forenoon|

If the dynamic viscosity of the lower fluid,  2 , is twice that of the upper fluid, 1 , then the
velocity at the interface (round off to two decimal places) is ___________ m/s.
Key: (1)

Sol:

y 1
2  21
2m
2 1m

d2u
Given that  0  1
dy 2
Integrating once, the above equation becomes
du
 C1   2 
dy
Integrating equation  2  ,
u  C1 y  C2   3
From equation (3) we can say that, velocity is linearly varying so the shear stress will be
constant at the interface of two viscous fluids
i.e., shear stress at y=1m, from fixed plate = shear stress at 1m from moving plate.
 v 0  v  vi 
2  2   1  
 1   1 
where v= velocity of moving plate, vi = velocity at interface of two fluids
v   3  vi 
21  i   1    2vi  3  vi  vi  1m / sec
 
1  1 

Then the velocity profile ill be as follows


www.
engi
neer
ingon
3m /your
sec f
inger
tips.
ooo

1 1m
2m
2 1m / sec 1m

© All rights reserved by Thinkcell Learning Solutions Pvt. Ltd. No part of this booklet may be reproduced or utilized in any form without the written permission.

www.engineeringonyourfingertips.ooo

More Notes Join us Telegram-: http//:t.me/allexammentor


www.engineeringonyourfingertips.ooo
|ME-2019-Forenoon|

54. A car having weight W is moving in the direction as shown in the figure. The centre of gravity
(CG) of the car is located at height h from the ground, midway between the front and rear
a
wheels.
Direction
of motion
CG

h W

l
Rr Rf

The distance between the front and rear wheels is . The acceleration of the car is a, and
acceleration due to gravity is g. The reactions on the front wheels (Rf) and rear wheels (Rr) are
given by
W Wh
(A) Rf  Rr    a
2 g  
W Wh W Wh
(B) Rf     a; R r    a
2 g   2 g  
W Wh
(C) Rf  Rr    a
2 g  
W Wh W Wh
(D) Rf     a; R r    a
2 g   2 g  
Key: (D)

Sol: W
a
g

h W
www.
engi
neer
ingonyour
fi
nger
tips.
ooo
R F
l

Rr Rf
M y  0

Rr  Rf  W  1
M R  0
 L W 
 W     a  h   Rf  0
 2  g 
W W h 
Rf    a
2 g L
W Wh 
Rr     a.
2 g L

© All rights reserved by Thinkcell Learning Solutions Pvt. Ltd. No part of this booklet may be reproduced or utilized in any form without the written permission.

www.engineeringonyourfingertips.ooo

More Notes Join us Telegram-: http//:t.me/allexammentor


www.engineeringonyourfingertips.ooo
|ME-2019-Forenoon|

55. The variable x takes a value between 0 and 10 with uniform probability distribution. The
variable y takes a value between 0 and 20 with uniform probability distribution. The probability
of the sum of variables (x + y) being greater than 20 is
(A) 0.33 (B) 0.50 (C) 0.25 (D) 0
Key: (C)

Sol: x  0, 10 y Required portion


y  0, 20
20
A
y  x  20

Area of 'S' at  x, y   S


0
10 20
P  x  y  20  ?
1 1
 f  x, y   
Area of S 200
 P  x  y  20  P  y  20  x 
1
 10  10
area of A 2 
    0.25
area of S 200 4
 or 
1
P  x  y  20   f  x, y  dxdy   dx dy
A A
200
1 1
   dx dy   Area of triangle
200 A 200

   10 w
10w
1 1  w. engi neer
ingonyour
fi
nger
tips.
ooo
   0.25.
200  2 

© All rights reserved by Thinkcell Learning Solutions Pvt. Ltd. No part of this booklet may be reproduced or utilized in any form without the written permission.

www.engineeringonyourfingertips.ooo

More Notes Join us Telegram-: http//:t.me/allexammentor


www.engineeringonyourfingertips.ooo
ME-GATE-2019-Afternoon

GENERAL APTITUDE

Q. No. 1 - 5 Carry One Mark Each

1. If IMHO = JNIP; IDK=JEL; and SO = TP, then IDC= ________.


(A) JDE (B) JDC (C) JCD (D) JED
Key: (D)
Sol: IMHO  JNIP IDK  JEL SO  TP
I M H O I D K S O I D C
1 1 1 1 1 1 1 1 1  1  1  1
J N I P J E L T P J E D

2. Once the team of analysts identify the problem, we ______ in a better position to comment on
the issue.
Which one of the following choices CANNOT fill the given blank?
(A) might be (B) were to be (C) are going to be (D) will be
Key: (B)

3. The product of three integers X, Y and Z is 192. Z is equal to 4 and P is equal to the average of
X and Y. What is the minimum possible value of P?
(A) 7 (B) 6 (C) 9.5 (D) 8
Key: (A)
Sol: Given X, Y, Z = 192
Z4
192
XY   48 www.
engi
neer
ingonyour
fi
nger
tips.
ooo
4
Possible values of X & Y for XY=48 are
X  48 or 1, Y  1 or 48
X  16or 3, Y  3 or 16
X  12 or 4, Y  4 or 12
X  8 or 6, Y  6 or 8
X  Y 68
min values of P   7
2 2

4. A final examination is the _____ of a series of evaluations that a student has to go through.
(A) insinuation (B) culmination (C) desperation (D) consultation
Key: (B)

© All rights reserved by Thinkcell Learning Solutions Pvt. Ltd. No part of this booklet may be reproduced or utilized in any form without the written permission.

www.engineeringonyourfingertips.ooo 1

More Notes Join us Telegram-: http//:t.me/allexammentor


www.engineeringonyourfingertips.ooo
ME-GATE-2019-Afternoon

5. Are there enough seats here? There are _____ people here than I expected.
(B) least (C) many (A) most (D) more
Key: (D)
Q. No. 6 - 10 Carry Two Marks Each

6. X is an online media provider. By offering unlimited and exclusive online content at attractive
prices for a loyalty membership, X is almost forcing its customers towards its loyalty
membership. If its loyalty membership continues to grow at its current rate, within the next
eight years more households will be watching X than cable television.
Which one of the following statements can be inferred from the above paragraph?
(A) The X is cancelling accounts of non-members
(B) Non-members prefer to watch cable television

(C) Most households that subscribe to X’s loyalty membership discontinue watching cable
television
(D) Cable television operators don’t subscribe to X’s loyalty membership
Key: (C)

7. Two pipes P and Q can fill a tank in 6 hours and 9 hours respectively, while a third pipe R can
empty the tank in 12 hours. Initially, P and R are open for 4 hours. Then P is closed and Q is
opened. After 6 more hours R is closed. The total time taken to fill the tank (in hours) is ____.
(A) 16.50 (B) 14.50 (C) 13.50 (D) 15.50
Key: (B)
Sol: P & Q can fill tank in 6 hours and 9 hours respectively
In 1 hour
w
1 ww.
engi
neer
ingonyour
fi
nger
tips.
ooo
P along can fill =
6
1
Q along can fill =
9
1
R along can fill =
12
In first four hours P and R are opened
1 1 2 1 1
4  P  4R  4   4   
6 12 3 3 3
After 4 hours P is closed and Q and R opened for 6 more hours
1 1 2 1 1
6  Q  6R  6   6    
9 12 3 2 6

© All rights reserved by Thinkcell Learning Solutions Pvt. Ltd. No part of this booklet may be reproduced or utilized in any form without the written permission.

www.engineeringonyourfingertips.ooo 2

More Notes Join us Telegram-: http//:t.me/allexammentor


www.engineeringonyourfingertips.ooo
ME-GATE-2019-Afternoon

1 1 1
In 10 hours tank filled =  
3 6 2
1
Remaining = tank
2
9
Q will take to fill remain half tank = hour  4.5 hours
2
Total time taken = 4  6  4.5  14.5hrs.

8. Mola is a digital platform for taxis in a city. It offers three types of rides – Pool, Mini and
Prime. The table below presents the number of rides for the past four months. The platform
earns one US dollar per ride. What is the percentage share of revenue contributed by Prime to
the total revenues of Mola, for the entire duration?

Month
Type January February March April

Pool 170 320 215 190

Mini 110 220 180 70

Prime 75 180 120 90

(A) 16.24 (B) 23.97 (C) 25.86 (D) 38.74


Key: (B)
Sol:
Month
Type Jan Feb Mar Apr Total no. of rides Revenu 1$ per ride
Pool 170 320ww w.
215 engi
190 neeri
ng onyour
895 fi
nger t i
ps.
895$ ooo
Mini 110 220 180 70 580 580$
Prime 75 180 120 90 465 465$
Total revenue 1940$

Percentage Share of revenue contributed by prime


Revenue of prime 465
= 100  100  23.97%
Total revenue 1940

9. Fiscal deficit was 4% of the GDP in 2015 and that increased to 5% in 2016. If the GDP
increased by 10% from 2015 to 2016, the percentage increase in the actual fiscal deficit is
(A) 37.50 (B) 25.00 (C) 35.70 (D) 10.00
Key: (A)
Sol: Let us take, In 2015, Fiscal deficit =X, GDP = y

© All rights reserved by Thinkcell Learning Solutions Pvt. Ltd. No part of this booklet may be reproduced or utilized in any form without the written permission.

www.engineeringonyourfingertips.ooo 3

More Notes Join us Telegram-: http//:t.me/allexammentor


www.engineeringonyourfingertips.ooo
ME-GATE-2019-Afternoon

Given
in 2015 x  4% of y  0.04y
in 2016 y '  increases by10%  1.10y
x '  increases to5% of y '  0.05  y '
x '  0.05  1.1y  0.055y
0.055y  0.04y
Percentage increase   100  37.5%
0.04y

10. While teaching a creative writing class in India, I was surprised at receiving stories from the
students that were all set in distant places: in the American West with cowboys and in
Manhattan penthouses with clinking ice cubes. This was, till an eminent Caribbean writer gave
the writers in the once-colonised countries the confidence to see the shabby lives around them
as worthy of being “told”.

The writer of this passage is surprised by the creative writing assignments of his students
because ______.
(A) None of the students had written stories set in India
(B) Some of the students had written about ice cubes and cowboys
(C) Some of the students had written stories set in foreign places
(D) None of the students had written about ice cubes and cowboys
Key: (A)

MECHANICAL ENGINEERING
Q. No. 1 to 25 Carry One Mark Each

1. If x is the mean ofwww


data 3,.
ex,n2gine
and er
4, i
ngo
then thenyour
mode fi
is nger
tips.
ooo
Key: (3)
Sol: Given data values are 3, x, 2 and 4
3 x  2 4 9x
 Mean  x  Mean  x 
4 4
 4x  9  x  3x  9
x 3
 Data value sare3, 3, 2, 4
Mode = 3 [most frequently repeated observation]

2. The cold forming process in which a hardened tool is pressed against a workpiece (when there
is relative motion between the tool and the workpiece) to produce a roughened surface with a
regular pattern is
(A) Chamfering (B) Roll forming (C) Knurling (D) Strip rolling
© All rights reserved by Thinkcell Learning Solutions Pvt. Ltd. No part of this booklet may be reproduced or utilized in any form without the written permission.

www.engineeringonyourfingertips.ooo 4

More Notes Join us Telegram-: http//:t.me/allexammentor


www.engineeringonyourfingertips.ooo
ME-GATE-2019-Afternoon

Key: (C)
Sol:

Knurled surface

3. The figure shows an idealized plane truss. If a horizontal force of 300N is applied at point A,
then the magnitude of the force produced in member CD is _____N.

A 300N

B C

D E
75 75
F G

Key: (0)
Sol: Joint A
255
A 300

75
30
R AB
www.
engi
neer
ingonyour
fi
nger
tips.
ooo
R AC

300 R AC R AB
 
sin 30 sin255 sin 75
300 sin 255
R AC   579.55 N  C 
sin 30
R AB  579.55N  T 

From the figure AB and BD are collinear. So, R BC  0 and AC and CE are collinear then
RCD=0.

© All rights reserved by Thinkcell Learning Solutions Pvt. Ltd. No part of this booklet may be reproduced or utilized in any form without the written permission.

www.engineeringonyourfingertips.ooo 5

More Notes Join us Telegram-: http//:t.me/allexammentor


www.engineeringonyourfingertips.ooo
ME-GATE-2019-Afternoon

4. The fluidity of molten metal of cast alloys (without any addition of fluxes) increases with
increase in
(A) viscosity (B) degree of superheat
(C) surface tension (D) freezing range
Key: (B)

5. Consider a linear elastic rectangular thin sheet of metal, subjected to uniform uniaxial tensile
stress of 100 MPa along the length direction. Assume plane stress condition in the plane normal
to the thickness. The Young’s modulus E = 200 MPa and Poisson’s ratio v  0.3 are given. The
principal strains in the plane of the sheet are
(A) (0.35, –0.15) (B) (0.5, –0.5) (C) (0.5, 0.0) (D) (0.5, –0.15)
Key: (D)
Sol: 1  100MPa,   0.3, E  200MPa, 2  0
1  100
1   2  1   0.5
E E 200
  0.3100
2  2   1  2   0.15
E E 200
1 , 2    0.5,  0.15  option  D 

6. The state of stress at a point in a component is represented by a Mohr’s circle of radius 100
MPa centered at 200 MPa on the normal stress axis. On a plane passing through the same point,
the normal stress is 260 MPa. The magnitude of the shear stress on the same plane at the same
point is ____ MPa.
Key: (80)
Sol:

www.
eDngi
neer
ingonyour
fi
nger
tips.
ooo

C
0
E

R  100MPa, C  200MPa,   260MPa,   ?


OC  200, CD  100, OE  260, DE  ?
DE  CD 2  CE 2  100 2   260  200   100 2  60 2  80MPa
2

7. A wire of circular cross-section of diameter 1.0 mm is bent into a circular are of radius 1.0 m
by application of pure bending moments at its ends. The Young’s modulus of the material of
the wire is 100 GPa. The maximum tensile stress developed in the wire is ____ MPa.
© All rights reserved by Thinkcell Learning Solutions Pvt. Ltd. No part of this booklet may be reproduced or utilized in any form without the written permission.

www.engineeringonyourfingertips.ooo 6

More Notes Join us Telegram-: http//:t.me/allexammentor


www.engineeringonyourfingertips.ooo
ME-GATE-2019-Afternoon

Key: (50)
Sol: d  1mm, R  1.0mts, E  100GPa,   ?
E  1 
 y  y   0.5mm 
R  2 
2100  10  1 
3
     50MPa
1000 2

8. A two-dimensional incompressible frictionless flow field is given by u  xi  y j. If  is the


density of the fluid, the expression for pressure gradient vector at any point in the flow field is
given as

(A) 
 xi  y j  
(B)  x 2 i  y 2 j  
(C)  xi  y j  
(D)  xi  y j 
Key: (D)

Sol: Given that, flow field V  xiˆ  yiˆ


V  uiˆ  vjˆ
Euler’s equation of motion in ‘x’ and ‘y’ directions are
u u du u 1 p
u v w  xx 
t x dy z  x
v v v v 1 p
 u  v  w  xy 
t x x z  y
Where u = x, v = -y, w = 0 xx, xy are body forces in ‘x’ and ‘y’ directions and they are equal
to zero since there is no extra forces acting on fluid.
p p
 pressure gradient in x-directoin;  pressure gradient in y-directoin
x y

  w.    f
 x    x  ww x  e
 n
yg
i
ne e
 xr
i
n
 g0on0you1r pi
nger
tips.
ooo
 
t x  y   x
1 p p
0x0   x
 x x
   1 p
  y  x  y   y  y   0  0 
t x y  y
1 p p
y   y
 y y

Pressure gradient vector 


p ˆ p ˆ
x
i  j  xiˆ  yjˆ   xiˆ  yjˆ
y
 
So answer is option ‘D’.

© All rights reserved by Thinkcell Learning Solutions Pvt. Ltd. No part of this booklet may be reproduced or utilized in any form without the written permission.

www.engineeringonyourfingertips.ooo 7

More Notes Join us Telegram-: http//:t.me/allexammentor


www.engineeringonyourfingertips.ooo
ME-GATE-2019-Afternoon

9. One-dimensional steady state heat conduction takes place through a solid whose cross-sectional
area varies linearly in the direction of heat transfer. Assume there is no heat generation in the
solid and the thermal conductivity of the material is constant and independent of temperature.
The temperature distribution in the solid is
(A) Logarithmic (B) Quadratic (C) Linear (D) Exponential
Key: (A)
Sol: 1-D steady state with no heat generation
d  dT 
 k A 0
dx  dx 
dT
 kA  C1
dx
A  Cx  B  linear variation 
C1
2 2
  dT   dx
1 1
k  Cx  B 
C1  Cx 2  B 
T2  T1  n 
K  Cx1  B 
 Temperature distribution in the solid is logarithmic

10. Endurance limit of a beam subjected to pure bending decreases with


(A) decrease in the surface roughness and increase in the size of the beam
(B) increase in the surface roughness and decrease in the size of the beam
(C) increase in the surface roughness and increase in the size of the beam
(D) decrease in the surface roughness and decrease in the size of the beam
Key: (C)
Sol: e  k a .k b .k c . k d .  'e
 'e  endurance strength,
www. n
e i
egn eer ingendurance
corrected onyour f
inger t
strength i
ps.
ooo
K a  size factor, K b  surface factor, K c  load factor, K d  temperature factor
With the increase in surface roughness, size factor etc the endurance strength drops.

11. Which one of the following modifications of the simple ideal Rankine cycle increases the
thermal efficiency and reduces the moisture content of the steam at the turbine outlet?
(A) Decreasing the condenser pressure
(B) Increasing the boiler pressure
(C) Decreasing the boiler pressure
(D) Increasing the turbine inlet temperature

© All rights reserved by Thinkcell Learning Solutions Pvt. Ltd. No part of this booklet may be reproduced or utilized in any form without the written permission.

www.engineeringonyourfingertips.ooo 8

More Notes Join us Telegram-: http//:t.me/allexammentor


www.engineeringonyourfingertips.ooo
ME-GATE-2019-Afternoon

Key: (D)
Sol:
T
1

4
3 2'
2

S
Increasing the inlet temperature to the turbine improves of quality of steam at the outlet of the
turbine.
This is the reason we use reheat cycle.

12. A rigid triangular body, PQR, with sides of equal length of 1 unit moves on a flat plane. At the
instant shown, edge QR is parallel to the x-axis, and the body moves such that velocities of
points P and R are V P and VR, in the x and y directions, respectively. The magnitude of the
angular velocity of the body is
y
P
VP
x

VR

Q R
www.
engi
neer
ingonyour
fi
nger
tips.
ooo
(A) VR 3 (B) VP 3 (C) 2VP (D) 2VR
Key: (D)
Sol: Locating the I-centre A
Vp
AI  AC sin 60
CI  AC cos 60 VR
2
Vp   AI      Vp
3 60
VR   CI      2VR  Option  D  C
I

© All rights reserved by Thinkcell Learning Solutions Pvt. Ltd. No part of this booklet may be reproduced or utilized in any form without the written permission.

www.engineeringonyourfingertips.ooo 9

More Notes Join us Telegram-: http//:t.me/allexammentor


www.engineeringonyourfingertips.ooo
ME-GATE-2019-Afternoon

13. For a simple compressible system, v, s, p and T are specific volume, specific entropy, pressure
 v 
and temperature, respectively. As per Maxwell’s relation,   is equal to
 s  P

 T   T   s   p 
(A)   (B)    (C)   (D)  
 p  s  p p  T  P  v T
Key: (A)

 V   T 
Sol:    
 S  P  P S
dh  Tds  Vdp
dx  Mdy  Ndz
Maxwell’s relation can be obtained by using
 M   N   T   V 
       
 Z  y  y z  P s  S  p

14. The most common limit gage used for inspecting the hole diameter is
(A) Snap gage (B) Plug gage (C) Ring gage (D) Master gage
Key: (B)

15. The directional derivative of the function f  x, y   x 2  y 2 along a line directed from (0, 0) to
(1, 1), evaluated at the point x = 1, y = 1 is
(A) 4 2 (B) 2 (C) 2 2 (D) 2
Key: (C)
Sol: Given, Scalar valued function f  x,y   x 2  y 2

Line directed fromw ww


(0, 0).
en
to gi
(1, ne
1) iser
1i
n
g
0 o
ˆi n
y
1o
u
0r
f
ˆji
nger
tips.
ooo

∴ Directional derivative of f  x,y  at (1, 1) in the direction of ˆi  ˆj is given by

 ˆi  ˆj
i.e.,  DD of f 1, 1 in  
ˆi  ˆj direction   f  .
1,1 ˆi  ˆj
... 1

 2
 f  ˆi
x
 x  y2   ˆj y  x 2  y2 
 f  ˆi  2x   ˆj  2y    f 1,1  2iˆ  2jˆ

 ˆi  ˆj 
∴ From (1), D.D of f = 2iˆ  2ˆj .   ˆi  ˆj
22
1 1
2 2

4
2
2 2

© All rights reserved by Thinkcell Learning Solutions Pvt. Ltd. No part of this booklet may be reproduced or utilized in any form without the written permission.

www.engineeringonyourfingertips.ooo 10

More Notes Join us Telegram-: http//:t.me/allexammentor


www.engineeringonyourfingertips.ooo
ME-GATE-2019-Afternoon

16. Water enters a circular pipe of length L = 5.0 m and diameter D = 0.20 m with Reynolds
number ReD  500. The velocity profile at the inlet of the pipe is uniform while it is parabolic at
the exit. The Reynolds number at the exit of the pipe is _______.
Key: (500)
Sol:

D  0.20m

L  5.0m

Given that Reynold’s number(ReD) = 500


We know that Reynold’s number for flow through pipes is,
Vavg D
Re D 

  density of fluid, Vavg  average velocity of fluid
D  diameter of pipe,   dynamic vis cosity of fluid
4Q
Vavg  , where Q  discharge
D 2
 4Q   4Q 
 2 
D  
D  D 
 Re D    
 
So Reynold’s number is a function of density, discharge, diameter and dynamic viscosity.
Since density, diameter and dynamic viscosities are same at entrance and exit and discharge is
same hence the Reynold’s number is also same at entrance and exit.

www.
engi
neer
ingonyour
fi
nger
tips.
ooo
17. In matrix equation [A]{X} = {R},
4 8 4  2  32 
 A   8 16 4 ,X  1  and R  16 .
   

 4 4 15  4  64 
   
One of the eigenvalues of matrix [A] is
(A) 8 (B) 16 (C) 15 (D) 4
Key: (B)
4 8 4  2  32 
     
Sol: Given  A   8 16 4  , x  1  and R  16 
   
 4 4 15  4  64 

© All rights reserved by Thinkcell Learning Solutions Pvt. Ltd. No part of this booklet may be reproduced or utilized in any form without the written permission.

www.engineeringonyourfingertips.ooo 11

More Notes Join us Telegram-: http//:t.me/allexammentor


www.engineeringonyourfingertips.ooo
ME-GATE-2019-Afternoon

4 8 4 2  32  2
     
Clearly,  A  x  R  8 16 4 

1   16   16 1 
 4 4 15       
4  64  4 
∴ The above equation is of the form AX  X; where '  ' isan eigen value
∴ One of the Eigen values of matrix must be '16'.

18. Sphere 1 with a diameter of 0.1 m is completely enclosed by another sphere 2 of diameter 0.4
m. The view factor F12 is
(A) 0.0625 (B) 0.5 (C) 1.0 (D) 0.25

Key: (C)

Sol: d1  0.1 m
2
d 2  0.4 m
1
F11  0
Summation rule
F11  F1 2  1
 F1 2  1  F11  0 

19. In an electrical discharge machining process, the breakdown voltage across inter electrode gap
(IEG) is 200V and the capacitance of the RC circuit is 50F. The energy (in J) released per
spark across the IEG is ____

Key: (1)
Sol: IEG = 200
www.
engi
neer
ingonyour
fi
nger
tips.
ooo
C  50F
Energy  e   ?
1
E  CV 2
2
E 1J

20. A thin vertical flat plate of height L, and infinite width perpendicular to the plane of the figure,
is losing heat to the surroundings by natural convection. The temperature of the plate and the
surroundings, and the properties of the surrounding fluid, are constant. The relationship
between the average Nusselt and Rayleigh numbers is given as Nu  KRa1 4 , where K is a
constant. The length scales for Nusselt and Rayleigh numbers are the height of the plate. The
height of the plate is increased to 16L keeping all other factors constant

© All rights reserved by Thinkcell Learning Solutions Pvt. Ltd. No part of this booklet may be reproduced or utilized in any form without the written permission.

www.engineeringonyourfingertips.ooo 12

More Notes Join us Telegram-: http//:t.me/allexammentor


www.engineeringonyourfingertips.ooo
ME-GATE-2019-Afternoon

Vertical Plate

L
g  9.8 m s 2

If the average heat transfer coefficient for the first plate is h1 and that for the second plate is h 2 ,
the value of the ratio h1 h 2 is___________.
Key: (2)

Nu  K  R a 
14
Sol:
 N u   Ra  Rayleigh number R a  G r Pr
14

 Nu   Gr.Pr 
14
As Prandtl number can be considered as a property. It remains constant

 N u   Gr 
14

14
hL  gTL3 
    hL  L3 4
K  v2 
3
L3 4 1
h  h  L4  h  L1 4
L
14
www.
engi
neer
ingonyour
fi
nger
tips.
ooo
h L 
 2  1 
h1  L 2 
14
h L 
14
 16L 
  24 
14 h1
and 1   2     2
h 2  L1   L  h2

21. A spur gear has pitch circle diameter D and number of teeth T. The circular pitch of the gear is
D 2 D D T
(A) (B) (C) (D)
T T T D
Key: (C)
D
Sol: Circular pitch = m 
T

© All rights reserved by Thinkcell Learning Solutions Pvt. Ltd. No part of this booklet may be reproduced or utilized in any form without the written permission.

www.engineeringonyourfingertips.ooo 13

More Notes Join us Telegram-: http//:t.me/allexammentor


www.engineeringonyourfingertips.ooo
ME-GATE-2019-Afternoon

22. An analytic function f(z) of complex variable z = x + iy may be written as f(z) = u(x, y) + iv(x,
y). Then u(x, y) and v(x, y) must satisfy
u v u v u v u v
(A)  and  (B)   and 
x y y x x y y x
u v u v u v u v
(C)   and  (D)  and 
x y y x x y y x
Key: (D)
Sol: Given an analytic function f  z   u  x,y   iv  x,y 

If f  z   u  iv is analytic then u, v must satisfy Cauchy –Riemann equations i.e,

u v u v
 & 
x y y x

23. Hardenability of steel is a measure of


(A) the ability to retain its hardness when it is heated to elevated temperatures
(B) the ability to harden when it is cold worked
(C) the depth to which required hardening is obtained when it is austenitized and then
quenched
(D) the maximum hardness that can be obtained when it is austenitized and then quenched
Key: (C)

24. The transformation matrix for mirroring a point in x-y plane about the line y = x is given by
0 1   0 1  1 0  1 0 
(A) 1 0  (B)   (C)   (D)  
   1 0   0 1 0 1
Key: (A)
www.
engi
neer
ingonyour
fi
nger
tips.
ooo

dy
25. The differential equation  4y  5 is valid in the domain 0    1 with y  0   2.25.
dx
The solution of the differential equation is
(A) y  e4x  1.25 (B) y  e4x  1.25

(C) y  e4x  5 (D) y  e4x  5


Key: (A)
dy
Sol: Given differential equation  4y  5, 0  x  1 with y  0   2.25
dx
Clearly the above differential equation is linear, where P=4, Q=5

 I.Fe
4dx
 e4x

© All rights reserved by Thinkcell Learning Solutions Pvt. Ltd. No part of this booklet may be reproduced or utilized in any form without the written permission.

www.engineeringonyourfingertips.ooo 14

More Notes Join us Telegram-: http//:t.me/allexammentor


www.engineeringonyourfingertips.ooo
ME-GATE-2019-Afternoon

Solution of equation (1) is


e 4x
y  e 4x    5.e 4x .dx  C  ye 4x  5 C ...  2 
4
Given y=2.25 at x = 0
5
(2)  2.25 1   C  C  2.25  1.25  1  C  1
4
∴ From (2), the required solution is
5 4x
ye 4x  e 1
4
5
y   e 4x
4
 y  e 4x  1.25

Q. No. 26 to 55 Carry Two Marks Each

26. In an orthogonal machining with a single point cutting tool of rake angle 10°, the uncut chip
thickness and the chip thickness are 0.125 mm and 0.22 mm, respectively. Using Merchant’s
first solution for the condition of minimum cutting force, the coefficient of friction at the chip-
tool interface is _____ (round off to two decimal places).

Key: (0.74)

Sol:   10, t1  0.125mm t 2  0.22mm


cos 10 0.98
tan   
 sin10 1.76  0.1736
0.22
0.125 www. engi neer ingonyour
fi
nger
tips.
ooo
tan   0.6177    31.706
For minimum cutting force

2       90
2
2  31.706    10  90
  36.59
tan     0.7423    0.74

© All rights reserved by Thinkcell Learning Solutions Pvt. Ltd. No part of this booklet may be reproduced or utilized in any form without the written permission.

www.engineeringonyourfingertips.ooo 15

More Notes Join us Telegram-: http//:t.me/allexammentor


www.engineeringonyourfingertips.ooo
ME-GATE-2019-Afternoon

27. Given a vector u 


1
3
 
 y3 i  x 3 j  z3 k and n as the unit normal vector to the surface of the

hemisphere  x 2  y2  z 2  1; z  0  , the value of integral     u  .n ds evaluated on the curved

surface of the hemisphere S is


  
(A) (B) (C)  (D) 
2 3 2
Key: (A)
Sol: Method –I:
x 2  y2 z 2 1
Given u 
1
3

 y3ˆi  x 3ˆi  z 3 kˆ and 
n̂  Unit normal vector to the surface of the hemi-sphere

x 2  y2  z 2  1; z  0.
Using Stoke's theorem, we have
 y3 x3 z3  C
    u  . nds
s
ˆ   u . dr     dx  dy  dz 
 3 c
3 3  c 0,1
3 3
y x
  dx  dy  z  0 on 'c'
c
3 3 R r
 
M N

   x 2  y 2  dx dy  Using Green 's theorem   1,0 1,0


R
2 1
  r
2
r dr d  Using polar co  ordination x 2  y 2  r 2 , J  r 
 0 r  0
1  0,1
2
1  2
 r4  1 2 
    r 3dr  d     d   0 
 0 
 0  r  0  4 0 4 2

Method –II: www.


engi
neer
ingonyour
fi
nger
tips.
ooo
Given, u 
1
3

 y3ˆi  x 3ˆj  z 3 kˆ 
ˆi ˆj kˆ
  
u   kˆ  x 2  y 2 
x y z
y3 x3 z3

3 3 3

 n̂ 


 xiˆ  yjˆ  zkˆ    ˆi  2x   ˆj  2y   kˆ  2z  
  4x 2  4y 2  4z 2  4 1  2
ˆ   z  x
    u  .nds
2
 y 2  ds ... 1

© All rights reserved by Thinkcell Learning Solutions Pvt. Ltd. No part of this booklet may be reproduced or utilized in any form without the written permission.

www.engineeringonyourfingertips.ooo 16

More Notes Join us Telegram-: http//:t.me/allexammentor


www.engineeringonyourfingertips.ooo
ME-GATE-2019-Afternoon


Let us parameterize ‘S’ in spherical coordinates, with colatitude 0    and longitude
2
0    2 ;
x  sin  cos  ; y sin  sin ; z  cos ; ds  sin  dd
/ 2 2
     u  .nds
ˆ    cos  sin
2
 cos 2   sin 2  sin 2   sin  dd
 0  0
/ 2 2
   cos . sin  dd   sin 2   cos 2   1
3

 0  0
/ 2 / 2
  3  1 
cos  sin 3   0 .d   2 
2
   cos  sin . d  2 
3
;
 0  0  4 2 
 / 2
 m  1 m  3)....    n  1 n  3 ..... 
  sin n x. cos n xdx   
k
 0  m  n  m  n  2  ...... 
 Where k   / 2; if n & m both are even; k  1; otherwise.

2
 2 
4 2

     u  .nˆ ds  2

28. The derivative of f  x   cos  x  can be estimated using the approximation

f x  h  f x  h
f ' x   . The percentage error is calculated as
2h
 Exact value  Aprroximate value 
   100. The percentage error in the derivative of
 Exact value 
f  x  at x   6 radian, choosing h  0.1 radian, is
(A) >5% www(B)
.en>gineer
0.1% ing
and ony
<1% our
(C) f
inger
<0.1% t
ips.
oo o >1 % and <5%
(D)
Key: (B)
Sol: Given f  x   cos x

 f '  x    sin x
   1
 f '  x  x  /6   sin       0.5 
6 2
∴ Exact value of the derivative = 0.5 ... 1

Approximate value of derivative

© All rights reserved by Thinkcell Learning Solutions Pvt. Ltd. No part of this booklet may be reproduced or utilized in any form without the written permission.

www.engineeringonyourfingertips.ooo 17

More Notes Join us Telegram-: http//:t.me/allexammentor


www.engineeringonyourfingertips.ooo
ME-GATE-2019-Afternoon

f x  h  f x  h cos  x  h   cos  x  h 
f ' x    f ' x  
2h 2h
   
cos   0.1  cos   0.1
 6  6  ; since h  0.1
 f '  
6 2  0.1
         
cos  6  .cos  0.1  sin  6  .sin  0.1   cos  6  cos  0.1  sin  6  sin  0.1 
         
 
2  0.1

2sin   .sin  0.1
6   0.5 sin  0.1
    0.4992
2  0.1  0.1

 f '     0.4992  Approximate value  ... 2 
6

 0.5   0.4992 
Percentage error     100  0.16% ie.  0.1% and  1%
 0.5 

29. Two masses A and B having mass m a and m b , respectively, lying in the plane of the figure
shown, are rigidly attached to a shaft which revolves about an axis through O perpendicular to
the plane of the figure. The radii of rotation of the masses m a and m b are ra and rb , respectively.
The angle between lines OA and OB is 90°. If ma = 10 kg, mb  20 kg ra  200 mm and
rb  400mm, then the balance mass to be placed at a radius of 200 mm is ____ kg (round off to
two decimal places)

mb B

www.
engi
neer
ingonyour
fi
nger
tips.
ooo
rb

90 ma
ra
A
O

Key: (41.231)
Sol: Method-1:
m a  10kg, m b  20kg, ra  200mm,  0.2mts,
rb  400mm  0.4mts, rr  200mm  0.2mts.

 mr resultant   mr a   mr b  10  0.2    20  0.4   8.24


2 2 2 2

8.24
m resultant   41.231 kg
0.2

© All rights reserved by Thinkcell Learning Solutions Pvt. Ltd. No part of this booklet may be reproduced or utilized in any form without the written permission.

www.engineeringonyourfingertips.ooo 18

More Notes Join us Telegram-: http//:t.me/allexammentor


www.engineeringonyourfingertips.ooo
ME-GATE-2019-Afternoon

Method-2:
m a  10kg ra  200mm B
mb
m b  20kg rb  400mm
at r  200 mm m  ? for balancing
Fx  0 rb
m a ra  m b rb cos  90   mr cos   0 90
ra ma
m a ra  0  mr cos   0
A
Fy  0 O
m b rb  mr sin   0
sin  m b rb

cos  m a ra
20  0.4
tan  
10  0.2
1
  tan 4    75.9640
mr
m  b b  41.23kg
r sin 

30. A through hole is drilled in an aluminum alloy plate of 15 mm thickness with a drill bit of
diameter 10 mm, at a feed of 0.25 mm/rev and a spindle speed of 1200 rpm. If the specific
energy required for cutting this material is 0.7 N  m mm3 , the power required for drilling is
___ W (round off to two decimal places).
Key: (274.889)
Sol: t  15mm
d  10mm
f  0.25mm rev
N  1200rpm www.
engi
neer
ingonyour
fi
nger
tips.
ooo
Specific energy required = 0.7 Nm mm 2
Power = ?
 2 N
Power  Df   SER   274.889 W
4 60

31. A horizontal cantilever beam of circular cross-section, length 1.0 m and flexural rigidity
EI  200 N  m 2 is subjected to an applied moment MA  1.0 N-m at the free end as shown in
the figure.
MA

1.0m
The magnitude of the vertical deflection of the free end is ____ mm (round off to one decimal
place)

© All rights reserved by Thinkcell Learning Solutions Pvt. Ltd. No part of this booklet may be reproduced or utilized in any form without the written permission.

www.engineeringonyourfingertips.ooo 19

More Notes Join us Telegram-: http//:t.me/allexammentor


www.engineeringonyourfingertips.ooo
ME-GATE-2019-Afternoon

Key: (2.5)
Sol:
A
MA
L

EI  200N  m 2 , L  1.0mts, M A  1.0N  mts


moment of Area underB.M.D ML L ML2
   
EI EI 2 2EI
1 12
  2.5mm
2  200

32. Consider two concentric circular cylinders of different materials M and N in contact with each
other at r = b, as shown below. The interface at r = b is frictionless. The composite cylinder
system is subjected to internal pressure P. Let  u M
r ,u   and  rr,   denote the radial and
M M M

tangential displacement and stress components, respectively, in material M. Similarly

u N
r , u N  and  rrN , 
N
 denote the radial and tangential displacement and stress components,
respectively, in material N. The boundary condition that need to be satisfied at the frictionless
interface between the two cylinders are:

r
N
M
www.
engi
neer
i our
ngony fi
nger
tips.
ooo
b

r  u r and rr  rr and u   u  and   


uM N M N M N M N
(A)

(B) u M  u N and 


m
 
N
only

(C) M
rr  rr and    only
N M N

r  u r and rr  rr only


uM N M N
(D)

© All rights reserved by Thinkcell Learning Solutions Pvt. Ltd. No part of this booklet may be reproduced or utilized in any form without the written permission.

www.engineeringonyourfingertips.ooo 20

More Notes Join us Telegram-: http//:t.me/allexammentor


www.engineeringonyourfingertips.ooo
ME-GATE-2019-Afternoon

Key: (D)
r
Sol:

N
M

At the radial location r=b, there will be a contact pressure and element at interface of cylinder
M and N the radial displacement will be same

r  u r , RR  RR
uM N M N

u M  u N , 
M
 
N

33. A slender uniform rigid bar of mass m is hinged at O and supported by two springs, with
stiffnesses 3k and k, and a damper with damping coefficient c, as shown in the figure. For the
system to be critically damped, the ratio c / km should be

3k c k

L 2

O
www
L.e
4ngi
neer
ingonyour
fi
nger
tips.
ooo
L

(A) 4 7 (B) 4 (C) 2 7 (D) 2


Key: (A)
2
mL2 mL2  L  7mL
2
Sol: IC  , I0  IC  mr 2   m  
12 12 4 48
Taking moment about 0, Mo  0

 3L  3L   L  L  L  L
I0   k      3k      C       0
 4  4   4  4  4 4
2 2 2
7mL CL 12kL
  0
48 16 16
7m C 12k
   0
48 16 16

© All rights reserved by Thinkcell Learning Solutions Pvt. Ltd. No part of this booklet may be reproduced or utilized in any form without the written permission.

www.engineeringonyourfingertips.ooo 21

More Notes Join us Telegram-: http//:t.me/allexammentor


www.engineeringonyourfingertips.ooo
ME-GATE-2019-Afternoon

Critical damping, the roots of above differential equation are real and equal so we can use
b2  4ac  0
7m C 12k
a , b , c
48 16 16
(Critical damping)C  16  7mk
2

C  4 7mk   C
mk

4 7

34. An air standard Otto cycle has thermal efficiency of 0.5 and the mean effective pressure of the
cycle is 1000 kPa. For air, assume specific heat ratio   1.4 and specific gas constant
R = 0.287 kJ/kg.K. If the pressure and temperature at the beginning of the compression stroke
are 100 kPa and 300 K, respectively, then the specific net work output of the cycle is ___ kJ/kg
(round off to two decimal places).
Key: (708.6)

Sol: OHO  0.5


Pm  1000KPa P1  100KPa
  1.4 T1  300K
R  0.287 KJ kgk
1
 1
 rc 
1

1
0.5  1 
 rc 
1.4 1

1
1  1  0.4
 0.5  rc     5.65
 rc   0.5 
0.4

P1V1  mRT www.


engi
neer
ingonyour
fi
nger
tips.
ooo
V1 RT1

m P1
0.287  300
V1   0.861 m 3 kg
100
V1
 rc 
V2
V1 0.861
 V2  
rc 5.65
 V2  0.1524 m3 kg swept volume
Vs  V1  V2
w.D
Pm   W.D  Pm   V1  V2 
V1  V2
 1000   0.681  0.1524   708.6KJ kg

© All rights reserved by Thinkcell Learning Solutions Pvt. Ltd. No part of this booklet may be reproduced or utilized in any form without the written permission.

www.engineeringonyourfingertips.ooo 22

More Notes Join us Telegram-: http//:t.me/allexammentor


www.engineeringonyourfingertips.ooo
ME-GATE-2019-Afternoon

35. An idealized centrifugal pump (blade outer radius of 50mm) consumes 2kW power while
running at 3000 rpm. The entry of the liquid into the pump is axial and exit from the pump is
radial with respect to impeller. If the losses are neglected, then the mass flow rate of the liquid
through the pump is ________kg/s (round off to two decimal places).
Key: (8.1057)
Sol: From the given data, since the inlet flow is axial, whirl component at inlet is zero VW1  0

And also because of radial discharge whirl component exit is VW2  u 2

Power input to the pump is P=2kW and N=3000rpm and d2  2r2  100mm  0.1mm

  0.1 3000
 VW2  u 2   5m/sec
60
Power input to the pump 'P' = m VW2 u 2  VW1 u1  
where m  mass flow rate of liquid in kg/sec
2000  m((5)(5)) ( Vw1  0)  m  8.1057kg / sec

36. A ball of mass 3 kg moving with a velocity of 4 m/s undergoes a perfectly-clastic direct-central
impact with a stationary ball of mass m. After the impact is over, the kinetic energy of the 3 kg
ball is 6J. The possible value (s) of m is/are
(A) 6 kg only (B) 1kg, 9 kg (C) 1 kg, 6 kg (D) 1 kg only
Key: (B)
Sol: m  3kg, V1  4m / sec, m2  m u 2  0 , kE1  65, e  1
Conservation of Momentum
m1u1  m 2 u 2  m1v1  m 2 v 2
 3  4    m2  0   3v1  mv2
3v1  mv 2  12 www.
engi
n...
ee 1r
i
ngonyour
fi
nger
tips.
ooo
v 2  v1
e  1  v 2  v1  4 ...  2 
u1  u 2
Conservation of Energy
 KE1 i   KE 2 i   KE1 f   KE 2 f
1 2 1
  3  4   0  6  mv 2  mv 2  36
2 2

2  2
From (1) and (2)
3  v 2  4   mv 2  12
 24 
3v 2  mv 2  24  v 2   
 3 m 
2
 24 
m   36
 3 m 
© All rights reserved by Thinkcell Learning Solutions Pvt. Ltd. No part of this booklet may be reproduced or utilized in any form without the written permission.

www.engineeringonyourfingertips.ooo 23

More Notes Join us Telegram-: http//:t.me/allexammentor


www.engineeringonyourfingertips.ooo
ME-GATE-2019-Afternoon

 576m   9  m 2  6m  36
36m2  360m  324  0  m 2  10m  9  0
 m  9  m  1  0  m  1,9kg

37. The annual demand of valves per year in a company is 10,000 units. The current order quantity
is 400 valves per order. The holding cost is Rs. 24 per valve per year and the ordering cost is
Rs. 400 per order. If the current order quantity is changed to Economic order quantity, then the
saving in the total cost of inventory per year will be Rs____ (round of to two decimal places).
Key: (943.60)
Sol: D  1000 units
Q  400 Values / order
C n  24 rs value / year
C0  400 Rs order
2DCo
EOQ 
Cn
2  10000  400

24
Q  EOQ  577.35
TIC  Q   2DCo Cn  2 10000  400  24  13856
D Q
TIC  Q   C0  Cn
Q 2
10000 400
  400  24
400 2
 10000  4800  14800
saving  14800  13856.41  Rs.943.60
www.
engi
neer
ingonyour
fi
nger
tips.
ooo
38. Water flowing at the rate of 1 kg/s through a system is heated using an electric heater such that
the specific enthalpy of the water increases by 2.50 kJ/kg and the specific entropy increases by
0.007 kJ/kg K. The power input to the electric heater is 2.50 kW. There is no other work or heat
interaction between the system and the surroundings, Assuming an ambient temperature of 300
K, the irreversibility rate of the system is _______kW (round off to two decimal places).
Key: (2.1)

Sol: m  1kg s
h 2  h1  2.5 kJ kg , S2  S1  0.007 kj kg k
w  2.5kW, T0  300K
irreversibility  I   T0  S   300  0.007  2.1 kJ kg
 I  m  2.1  1  2.1  2.1 kW

© All rights reserved by Thinkcell Learning Solutions Pvt. Ltd. No part of this booklet may be reproduced or utilized in any form without the written permission.

www.engineeringonyourfingertips.ooo 24

More Notes Join us Telegram-: http//:t.me/allexammentor


www.engineeringonyourfingertips.ooo
ME-GATE-2019-Afternoon

39. The activities of a project, their duration and the precedence relationship are given in the table.
For example, in a precedence relationship “X <Y, Z” means that X is predecessor of activities
Y and Z. The time to complete the activities along the critical path is _______weeks,

Activity Duration (weeks) Precedence Relationship


A 5 A<B, C,D
B 7 B<E,F,G
C 10 C<I
D 6 D <G
E 3 E<H
F 9 F<I
G 7 G<I
H 4 H<I
I 2 ……

(A) 21 (B) 23 (C) 17 (D) 25


Key: (B)
Sol: C 0

E  3 H  4
A  5 B7 I  2

D 6 F 9

G 7

ACI  5  10  2  17days
www. engi neer i
ngonyour
fi
nger
tips.
ooo
ABEHI  5  7  3  4  2  21days
ABFI  5  7  9  2  23 days
ABGI  5  7  7  2  21days
Time required = 23 days

40. A differential equation is given as


d2 y dy
x2 2
 2x  2y  4.
dx dx
The solution of the differential equation in terms of arbitrary constants C1 and C2 is
C1
(A) y  C1x 2  C2 x  4 (B) y   C2 x  4
x2
C1
(C) y  C2 x  2 (D) y  C1x 2  C2 x  2
x2

© All rights reserved by Thinkcell Learning Solutions Pvt. Ltd. No part of this booklet may be reproduced or utilized in any form without the written permission.

www.engineeringonyourfingertips.ooo 25

More Notes Join us Telegram-: http//:t.me/allexammentor


www.engineeringonyourfingertips.ooo
ME-GATE-2019-Afternoon

Key: (D)
Sol: Given differential equation
d2 y dy
x2 2
 2x  2y  4 ...Cauchy Euler LinearD.E
dx dx
  x D  2xD  2  y  4
2 2
... 1
d
ConsiderxD  ; x 2 D 2      1 where   & x  ez
dz
 From 1 , 
    1  2  2  y  4
 2    2  2  y  4
 2  3  2  y  4 ...  2 
A.E m 2  3m  2  0   m  2  m  1  0
 m  2, 1  real &distinct
 y C  C1e 2z  C2 e z ...  3
1
 yp  4.e0z
  3  2
2

substitute   0; then
1
yp  4  yp  2
2
 Complete solution is y  y C  y p
 y  C1e2z  C2 ez  2  y  C1x 2  C2 x  2  x  e z 

41. Water flows through two different pipes A and B of the same circular cross-section but at
different flow rates. The length of pipe A is 1.0 m and that of pipe B is 2.0 m. The flow in both

ww
the pipes is laminar wfully
and .
eng i
neer ing
developed. Ifony
the our fing
frictional er t
headip s.
loss ooo the length of the pipes
across
is same, the ratio of volume flow rate QB QA is (round off to two decimal places).
Key: (0.5)
Sol: Given that, diameter of pipe ‘A’ (dA) = diameter of pipe ‘B’ (dB)
Length of pipe ‘A’ is  A   1m
Length of pipe ‘B’ is  B   2m
32v
Frictional head loss in laminar flow is h f 
gd 2

It is mentioned in the question as  h f A   h f B

© All rights reserved by Thinkcell Learning Solutions Pvt. Ltd. No part of this booklet may be reproduced or utilized in any form without the written permission.

www.engineeringonyourfingertips.ooo 26

More Notes Join us Telegram-: http//:t.me/allexammentor


www.engineeringonyourfingertips.ooo
ME-GATE-2019-Afternoon

32VA 32VB
 A
 B
 dA  dB 
gd 2A gd B2
VA A  VB B

VA V 1
VA 1  VB  2   2 B 
VB VA 2
 2
Flow rate pipe ‘A’ is QA  VA dA
4
 2
Flow rate in pipe ‘B’ is Q B  VB dB
4
 2
 VB  
dB 
QB
  4   VB  1  0.5

QA
 VA   d 2A  VA 2
4 

42. A prismatic, straight, elastic, cantilever beam is subjected to a linearly distributed transverse
load as shown below. If the beam length is L, Young’s modulus E, and are moment of inertia I,
the magnitude of the maximum deflection is

y
q

L
x

qL4 qL4 qL4 qL4


(A) (B) (C) (D)
10EI 15EI 60EI 30EI
Key: (D) www.
engi
neer
ingonyour
fi
nger
tips.
ooo
Sol: y
q

L
x
B.M.D L

qL2
6 3rd degree curve

© All rights reserved by Thinkcell Learning Solutions Pvt. Ltd. No part of this booklet may be reproduced or utilized in any form without the written permission.

www.engineeringonyourfingertips.ooo 27

More Notes Join us Telegram-: http//:t.me/allexammentor


www.engineeringonyourfingertips.ooo
ME-GATE-2019-Afternoon

1 qx x qx 3
Mxx  x  
2 L 3 6L
1 L
M A   q  L  (at A, x  L)
2 3
Moment of area under BMD about free end
Deflection 
EI
 1 
 Area of spandral  n  1 bh 
  n 3
centriod of spandral about apex  x   n  1 b 
 n  2 
Ax  1   qL2   3 1  L qL4
    . L .   
EI  3  1   6   3  2  EI 30EI

43. A four bar mechanism is shown in the figure. The link numbers are mentioned near the links,
input link 2 is rotating anticlockwise with a constant angular speed 2 . Length of different
links are:
B
O 2 O 4  O 2 A  L, 3
AB  Q 4 B  2L A

2 4
4
2
O2 O4

The magnitude of the angular speed of the output link 4 is 4 at the instant when link 2 makes
4
an angle of 90° with O2 O4 as shown. The ratio is ______(round off to two decimal
2
places). www.
engi
neer
ingonyour
fi
nger
tips.
ooo

Key: (0.788)
B
Sol: O2 A  O2O4  L I 34
60
AB  O 4 B  2L

A I 23
60
75
45 2L

60 75

15 90
12 45 I 41

I 24 I12 O4
© All rights reserved by Thinkcell Learning Solutions Pvt. Ltd. No part of this booklet may be reproduced or utilized in any form without the written permission.

www.engineeringonyourfingertips.ooo 28

More Notes Join us Telegram-: http//:t.me/allexammentor


www.engineeringonyourfingertips.ooo
ME-GATE-2019-Afternoon

I 24 I12 2  I 24 I 414
I 24 I12  L tan 75  from  e I 24 I12 I 23 
I 24 I 41  L  L tan 75
4 I 24 I12 L tan 75
   0.788
2 I24 I 41 L  Ltan 750

44. A gas tungsten are welding operation is performed using a current of 250 A and an arc voltage
of 20 V at a welding speed of 5 mm/s. Assuming that the arc efficiency is 70%the net heat input
per unit length of the weld will be______ kJ/mm (round off to one decimal place).
Key: (0.7)
Sol: Given that, V = 20V, I = 250A
Welding speed = 5 mm/sec
Arc efficiency = 70% = 0.7
Power given to the welding operation  VI   20  250   5000 watts  5000 Joules sec

Since arc efficiency is 70%, net heat input will be  0.7  5000  3500 Joules sec

net heat input


Net heat input per unit length of the weld 
welding speed
3500  J / sec 
  700 J mm  0.7 kJ mm
50  mm / sec 

45. Three sets of parallel plate LM, NR and PQ are given in Figures 1, 2 and 3. The view factor FIJ
is defined as the fraction of radiation leaving plate I that is intercepted by plate J. Assume that
the values of FLM and FNR are 0.8 and 0.4 respectively. The value of FPQ (round off to one
www. engi
decimal place) is________. neer
ingonyour
fi
nger
tips.
ooo

1m 1m 1m
N P
1m 1m 1m
M
R
Q
1m 1m 1m 1m 1m 1m

Figure1 Figure 2 Figure 3

Key: (0.6)

Sol: From the figures we can say that view factor between o parallel 1m plates  0.4  FNR
View factor between one 1m plate to another two equally inclined

© All rights reserved by Thinkcell Learning Solutions Pvt. Ltd. No part of this booklet may be reproduced or utilized in any form without the written permission.

www.engineeringonyourfingertips.ooo 29

More Notes Join us Telegram-: http//:t.me/allexammentor


www.engineeringonyourfingertips.ooo
ME-GATE-2019-Afternoon

plates  FLM  FNR  0.8  0.4  0.4


View factor between one 1m 4 plate to and this equally inclined.
0.4
Plate   0.2
2
 For figure ‘3’ view factor form one 1m plate to another parallel plate and another equally
inclined plate is
FPQ  0.4  0.2  0.6

46. A uniform disc with radius r and a mass of m kg is mounted centrally on a horizontal axle of
negligible mass and length of 1.5r. P

P Q

RP RQ

The disc spins counter-clockwise about the axle with angular speed , when viewed from the
www.
engi
neer
ingonyour
fi
nger
tips.
ooo
right-hand side bearing Q, a The axle processes about a vertical axis at p   / 10 in the

clockwise direction when viewed from above. Let R p and R Q (positive upwards) be the
resultant reaction forces due to the mass and the gyroscopic effect, at bearings P and Q,
respectively. Assuming 2 r  300m / s2 and g  10m s 2 , the ratio of the larger to the smaller
bearing reaction force (considering appropriate signs) is________
Key: (-3)
Sol: RP

15mr
RQ
© All rights reserved by Thinkcell Learning Solutions Pvt. Ltd. No part of this booklet may be reproduced or utilized in any form without the written permission.

www.engineeringonyourfingertips.ooo 30

More Notes Join us Telegram-: http//:t.me/allexammentor


www.engineeringonyourfingertips.ooo
ME-GATE-2019-Afternoon


p  , 2 r  300m/sec 2 , g  10m/sec 2 ,  1.5r
10
Gyroscope couple,C  Ip
 mr 2 
C   .p
 2 
 mr  r mr 300
2
  .    15mr
 2  10 2 10

mg 10m
Reaction due to weight, R P  R Q    5m (due to symmetry)
2 2
15MR
Reaction due to gyroscopic couple, R Q   10m, R P  10m
1.5R
Now net reaction, R Q  10m  5m  15m
R P  5m  10m  5m
RQ 15m
  3
RP 5m

47. The figure shows a heat engine (HE) working between two reservoirs. The amount of heat
 Q 2  rejected by the heat engine is drawn by a heat pump (HP). The heat pump receives the

entire work output (W) of the heat engine. If temperatures, T1  T3  T2 , then the relation

between the efficiency   of the heat engine and the coefficient and the coefficient of
performance (COP) of the heat pump is

www.
engi
neer
ingonyour
fi
nger
tips.
ooo
T1 T3

Q1 Q3
W
HE HP

Q2 Q2

T2

(A) COP   (B) COP  1  1 (C) COP  1 (D) COP  1  
Key: (C)

© All rights reserved by Thinkcell Learning Solutions Pvt. Ltd. No part of this booklet may be reproduced or utilized in any form without the written permission.

www.engineeringonyourfingertips.ooo 31

More Notes Join us Telegram-: http//:t.me/allexammentor


www.engineeringonyourfingertips.ooo
ME-GATE-2019-Afternoon

Sol:

T1 T3

Q1 Q3

W
HE HP

Q2 Q2

T2

T1  T3  T2  Given 
Q3
COPH.P 
W
W  Q1  Q 2  Q 2  Q1  W
W Q
 or 1  2  W   Q1
Q1 Q1
Also Q3  Q 2  W
Q2  W
COPH.P 
W
Q Q W Q
 2 1  1 1  1 11
W W W
1
 COP  

48. The aerodynamic drag on a sports car depends on its shape. The car has a drag coefficient of
0.1 with the windows and the roof closed. With the windows and the roof open, the drag
coefficient becomes 0.8. The car travels at 44 km/h with the windows and roof closed. For the
www.
engi
neer
ingonyour
fi
nger
tips.
ooo
same amount of power needed to overcome the aerodynamic drag, the speed of the car with the
windows and roof open (round off to two decimal places), is ____ km/h. (The density of air and
the frontal area may be assumed to be constant.)
Key: (22)
Sol: Co-efficient of drag with doors and roof closed  C d C  0.1

Co-efficient of drag with doors and roof open  Cd O  0.8

Velocity of car with doors and roof closed VC  44km/sec

Velocity of car with doors and roof open VO  ?


Power needed in overcoming aerodynamic drag with doors and roof closed =
Power needed in overcoming aerodynamic with doors and roof open

© All rights reserved by Thinkcell Learning Solutions Pvt. Ltd. No part of this booklet may be reproduced or utilized in any form without the written permission.

www.engineeringonyourfingertips.ooo 32

More Notes Join us Telegram-: http//:t.me/allexammentor


www.engineeringonyourfingertips.ooo
ME-GATE-2019-Afternoon

 Drag force  velocity closed   Drag force  velocity open


1  1 
   Cd C  eA  VC2  VC     Cd O  eA  VO2  VO 
2  2 
 0.1  44  0.8  V0
3 3

V0  22 km / hr

49. The binary phase diagram of metals P and Q is shown in the figure. An alloy X containing 60%
P and 40% Q (by weight) is cooled from liquid to solid state. The fractions of solid and liquid
(in weight percent) at 1250°C, respectively, will be

Liquidus
1500 X
Liquid

1250
Temperature Solidus
 C 
Solid
1000

100% P 20 32 40 60 68 80 100%Q

Concentration of Q
 in weight % 

(A) 22.2% and 77.8% (B) 68.0% and 32.0%


www.
(C) 32.0% and 68.0% engi
neer
ingonyour
(D) f
inge
77.8% rt
i
and ps.ooo
22.2%
Key: (A)

co  c 40  32
Sol: ms    22.2%
cs  c 68  32
m  77.8%

© All rights reserved by Thinkcell Learning Solutions Pvt. Ltd. No part of this booklet may be reproduced or utilized in any form without the written permission.

www.engineeringonyourfingertips.ooo 33

More Notes Join us Telegram-: http//:t.me/allexammentor


www.engineeringonyourfingertips.ooo
ME-GATE-2019-Afternoon

50. The crank of a slider-crank mechanism rotates counter clockwise (CCW) with a constant
angular velocity , as sown. Assume the length of the crank to be r.


1

2 1

Using exact analysis. The acceleration of the slider in the y-direction, at the instant shown,
where the crank is parallel to x-axis, is given by
(A) 22 r (B) 22 r (C) 2 r (D) 2 r
Key: (C)
Sol:
O
 A

Velocity of OA  V OA  r

Velocity of piston B
www.
engi

neer
ingonyour
fi
nger
tips.
ooo
Since both the vectors are parallel VOA  VB and VAB  0
r
Radial acceleration of OA a OA  r2   
t
Tangential acceleration of AB a AB

t
a 0B

aB

a O
r
a 0A

© All rights reserved by Thinkcell Learning Solutions Pvt. Ltd. No part of this booklet may be reproduced or utilized in any form without the written permission.

www.engineeringonyourfingertips.ooo 34

More Notes Join us Telegram-: http//:t.me/allexammentor


www.engineeringonyourfingertips.ooo
ME-GATE-2019-Afternoon

aB
tan 45  r
a OA
a B  r2 tan 45  r2

51. The probability that a part manufactured by a company will be defective is 0.05. If such parts
are selected randomly and inspected, then the probability that at least two parts will be
defective is ____ (round off to two decimal places).
Key: (0.17)
Sol: Given, the probability of manufactured part will be defective i.e,
P  D   0.05  q  1  0.05  0.95

Number of trials = n= 15 (15 parts are selected)


The R.V X → denote the number of defective parts
 P  X  2  ?
 P  X  2  1  P  x  2   1  P  X  0   P  X  1

 
 1  15Co .p O q15  15C1 p1q14 , Using Binomial distribution

 1  q 15

 15pq14  1   0.95  15  0.05  0.95 
15 14

 1  0.46  0.37  1  0.83  0.17

52. The figure shows a pouring arrangement for casting of a metal block. Frictional losses are
negligible. The acceleration due to gravity is 9.81 m / s2 . The time (in s, round off to two
decimal places) to fill up the mold cavity  of size 40 cm  30 cm  15 cm  is_____

Atmospheric pressure
www.
engi
neer
ingonyour
fi
nger
tips.
ooo
1

Filing height 2

20 cm Diameter 2cm

3
30 cm

Open to atmosphere

Volume to be casted
15 cm
 mold cavity 

40 cm

© All rights reserved by Thinkcell Learning Solutions Pvt. Ltd. No part of this booklet may be reproduced or utilized in any form without the written permission.

www.engineeringonyourfingertips.ooo 35

More Notes Join us Telegram-: http//:t.me/allexammentor


www.engineeringonyourfingertips.ooo
ME-GATE-2019-Afternoon

Key: (28.94)

Sol: Vg  2gh t  2  9.81 0.2


Vg  1.98m s
30  40  15 1200  4  15
t f1  
   4  198
  2   198
2

4
t f 1  28.94 sec.

53. Hot and cold fluids enter a parallel flow double tube heat exchanger at 100 °C and 15 °C,
respectively. The heat capacity rates of hot and cold fluids are Ch  200 W k and
Cc  1200 W K, respectively. If the outlet temperature of the cold fluid is 45°C, the log mean
temperature difference (LMTD) of the heat exchanger is ___________ K (round of to two
decimal places).
Key: (57.71)
T
Sol:
t h1  100C

t h 2  82C
t c2  45C

t c1  15C

Ch  2000 W K
Cc  1200 W K
www.
engi
neer
ingonyour
fi
nger
tips.
ooo
Heat lost by hot fluid = heat gained by cold fluid
 
Ch t h1  t h 2  Cc t c2  t c1  
 
2000 100  t h 2  1200  45  15   t h 2  82C
where 1  t h1  t c1  100  15  85o C
2  t h 2  t c2  82  45  37 o C

1  2
LMTD  m  
n  1 2 
85  37
  57.71o C
n  85 37 
 LMTD  m   57.71K

© All rights reserved by Thinkcell Learning Solutions Pvt. Ltd. No part of this booklet may be reproduced or utilized in any form without the written permission.

www.engineeringonyourfingertips.ooo 36

More Notes Join us Telegram-: http//:t.me/allexammentor


www.engineeringonyourfingertips.ooo
ME-GATE-2019-Afternoon

54. The thickness of a sheet is reduced by rolling (without any change in width) using 600 mm
diameter rolls. Neglect elastic deflection of the rolls and assume that the coefficient of friction
at the roll-workpiece interface is 0.05. The sheet enters the rotating rolls unaided. If the initial
sheet thickness is 2 mm, the minimum possible final thickness that can be produced by this
process in a single pass is _____ mm (round of to two decimal places).
Key: (1.25)
Sol: D  600mm
R  300mm;   0.05
h i  2mm
hf  ?
h i  h f  2 R
h i  h f  0.052  300
h i  h f  0.75
h f  2  0.75
h f  1.25mm

55. A short shoe external drum brake is shown in the figure. The diameter of the brake drum is 500
mm. The dimensions a = 1000 mm, b = 500 mm and c = 200 mm. The coefficient of friction
between the drum and the shoe is 0.35. The force applied on the lever F = 100 N as shown in
the figure. The drum is rotating anti-clockwise. The braking torque on the drum is ____ N-m
(round off to two decimal places).

F
a

www.
engi
neer
ingonyour
fi
nger
tips.
ooo
c
Brake
shoe

b

Key: (20.34)
Sol: F

R N
c
H
RN

b
© All rights reserved by Thinkcell Learning Solutions Pvt. Ltd. No part of this booklet may be reproduced or utilized in any form without the written permission.

www.engineeringonyourfingertips.ooo 37

More Notes Join us Telegram-: http//:t.me/allexammentor


www.engineeringonyourfingertips.ooo
ME-GATE-2019-Afternoon

D  500mm, a  1000mm, b  500mm


C  200mm,   0.35, F  100N

M H  0
 F  a    R N  c    R N  b   0
100  1000    0.35  R N  200   R N  500   0
R N  232.55N
Braking torque  TB  .R N .R
500
 0.35  232.55  55   103  20.34N
2

www.
engi
neer
ingonyour
fi
nger
tips.
ooo

© All rights reserved by Thinkcell Learning Solutions Pvt. Ltd. No part of this booklet may be reproduced or utilized in any form without the written permission.

www.engineeringonyourfingertips.ooo 38

More Notes Join us Telegram-: http//:t.me/allexammentor


www.engineeringonyourfingertips.ooo
ME-GATE 2018

Section-I: General Ability

1. Seven machines take 7 minutes to make 7 identical toys. At the same rate, how many minutes
would it take for 100 machines to make 100 toys?
(A) 1 (B) 7 (C) 100 (D) 700
Key: (B)
7  7 100  T2
Sol: 
7 100
 T2  7 minutes

2. "Her ______ should not be confused with miserliness; she is ever willing to assist those in need."
The word that best fills the blank in the above sentence is:
(A) cleanliness (B) punctuality (C) frugality (D) greatness
Key: (C)
Sol: The sentence explains that the person spoken of is not miserly, since she is quite prepared to be
generous. So for the sentence to make sense, the word filling the blank has to be something that is
consistent with generosity and yet might, by those without a full understanding of her behaviour,
be mistaken for miserliness. The words “frugality” and thrift fulfill this requirement and yield two
sentences that are alike in meaning. Thus the correct answer is frugality.

3. A number consists of two digits. The sum of the digits is 9. If 45 is subtracted from the number, its
digits are interchanged. What is the number?
(A) 63 (B) 72 (C) 81 (D) 90
Key: (B)
Sol: www.
Let number be „xy‟ engi
neer
ingonyour
finger
tips.
ooo
Given x  y  9 ... i 
& xy  45  yx

 10x  y   45  10y  x
 9x  9y  45
xy5 ... ii 

solving  i  &  ii   x  7, y  2
Required number  72

© All rights reserved by Thinkcell Learning Solutions Pvt. Ltd. No part of this booklet may be reproduced or utilized in any form without the written permission.

www.engineeringonyourfingertips.ooo 2

More Notes Join us Telegram-: http//:t.me/allexammentor


www.engineeringonyourfingertips.ooo
ME-GATE 2018

4. Going by the ______ that many hands make light work, the school _______ involved all the
students in the task."
(A) principle, principal (B) principal, principle
(C) principle, principle (D) principal, principal
Key: (A)

5. A rectangle becomes a square when its length and breadth are reduced by 10 m and 5 m,
respectively. During this process, the rectangle loses 650 m2 of area. What is the area of the original
rectangle in square meters?
(A) 1125 (B) 2250 (C) 2924 (D) 4500
Key: (B)
Sol:

x 5 x

x  10 x

Area of rectangle = (x+10)(x+5) = x2+15x+50 …(1)


Area of square = x2 … (2)
By the given condition,
15x  50  650
 15x  600  x  40.
 Area of rectangle   40   15  40  50  2250( from(1))
2

6. ww
Given that a and b are w. eng
integers ine
and a+er
a2i
bn
3
gisonyo
odd, urfi
whichn ge
one r
oftips
the .ooo statements is correct?
following
(A) a and b are both odd (B) a and bare both even
(C) a is even and b is odd (D) a is odd and b is even
Key: (D)
Sol: By verifying options
Let us consider a=1, b=2
a  a 2 b3  1  12  23  9 i.e.,odd
Only this condition is valid.

© All rights reserved by Thinkcell Learning Solutions Pvt. Ltd. No part of this booklet may be reproduced or utilized in any form without the written permission.

www.engineeringonyourfingertips.ooo 3

More Notes Join us Telegram-: http//:t.me/allexammentor


www.engineeringonyourfingertips.ooo
ME-GATE 2018

7. Consider the following three statements:


(i) Some roses are red
(ii) All red flowers fade quickly.
(iii) Some roses fade quickly
Which of the following statements can be logically inferred from the above statements?
(A) If (i) is true and (ii) is false, then (iii) is false.
(B) If (i) is true and (ii) is false, then (iii) is true.
(C) If (i) and (ii) are true, then (iii) is true.
(D) If (i) and (ii) are false, then (iii) is false.
Key: (C)
Sol:
Fadedquickly
Roses
Red

8. Which of the following functions describe the graph shown in the below figure.

y
3
2
1
-3 -2 -1 0 1 2 3x
-1
-2

www.
engi
neer
ingonyour
finger
tips.
ooo
-3

(A) y  x  1  2 (B) y  x  1  1 (C) y  x  1  1 (D) y  x  1  1


Key: (B)
Sol: Given graph passing through X-axis at x= –2, 2
From option (B), y=0 at x= –2, 2
Alternate Method:
Step 1
y x
0

© All rights reserved by Thinkcell Learning Solutions Pvt. Ltd. No part of this booklet may be reproduced or utilized in any form without the written permission.

www.engineeringonyourfingertips.ooo 4

More Notes Join us Telegram-: http//:t.me/allexammentor


www.engineeringonyourfingertips.ooo
ME-GATE 2018

Step  2 
y 1  x
 or  1
y  x 1

Step  3 1
y  x 1

Step  4 
y 1  x 1
0
 or 
y  x 1 1

9. From the time the front of a train enters a platform, it takes 25 seconds for the back of the train to
leave the platform, while travelling at a constant speed of 54 km/h. At the same speed, it takes 14
seconds to pass a man running at 9 km/h in the same directions as the train. What is the length of
the train and that of the platform in meters, respectively?
(A) 210 and 140 (B) 162.5 and 187.5
(C) 245 and 130 (D) 175 and 200
Key: (D)
Sol: Let Train length  x
Platform length  y

x  y 25   54 
5
  375
 18 
  ww
 w.engi
neer
ingonyour
finger
tips.
ooo
Distance  Time  Speed

x
also,  14
5
 54  9 
18

Relative speed
 x  175 and y  200

10. For integers a, b and c, what would be the minimum and maximum values respectively of a + b + c
if log a  lob b  log c  0?

(A) –3 and 3 (B) –1 and 1 (C) –1 and 3 (D) 1 and 3

© All rights reserved by Thinkcell Learning Solutions Pvt. Ltd. No part of this booklet may be reproduced or utilized in any form without the written permission.

www.engineeringonyourfingertips.ooo 5

More Notes Join us Telegram-: http//:t.me/allexammentor


www.engineeringonyourfingertips.ooo
ME-GATE 2018

Key: (A)
Sol: Given log a  log b  log c  0
 log e a b c  0
 a b c  1  a  1, b  1,c  1 ( a,b,c are integers)
Minimum value  1  1  1  3
Maximum value  1  1  1  3

Section-I: Mechanical Engineering

1. An ideal gas undergoes a process from state 1 (T 1 = 300 K, p1 = 100 kPa) to state 2 (T2 = 600K,
p2 = 500 kPa). The specific heats of the ideal gas are: cp = 1 kJ/kg-K and cv = 0.7 kJ/kgK. The
change in specific entropy of the ideal gas from state 1 to state 2 (in kJ/kg-K) is _____ (correct to
two decimal places).
Key: (0.21)
Sol: Given that,
T1  300K, P1  100kPa
T2  600K , P2  500kPa
Cp  1kJ/kg-k
Cv  0.7kJ/kg-k,
R  Cp  Cv  1  0.7  0.3kJ/kg-k
Changein Entropy  s 2  s1  per unit mass
T  P 
 Cp n  2   R n  2 
 T1   P1 
 600   500 
1 n   0.3 w
n ww. engi
neer
ingonyour
finger
tips.
ooo
 300   100 
s 2  s1  0.21kJ/kg-K

2. In a linearly hardening plastic material, the true stress beyond initial yielding
(A) increases linearly with the true strain
(B) decreases linearly with the true strain
(C) first increases linearly and then decreases linearly with the true strain
(D) remain constant
Key: (A)

© All rights reserved by Thinkcell Learning Solutions Pvt. Ltd. No part of this booklet may be reproduced or utilized in any form without the written permission.

www.engineeringonyourfingertips.ooo 6

More Notes Join us Telegram-: http//:t.me/allexammentor


www.engineeringonyourfingertips.ooo
ME-GATE 2018

3. Using the Taylor's tool life equation with exponent n = 0.5, if the cutting speed is reduced by 50%,
the ratio of new tool life to original tool life is
(A) 4 (B) 2 (C) 1 (D) 0.5
Key: (A)
Sol: VT n  C
V1T1n  V2 T2n
1
T2  V1  n
 
T1  V2 
1
T2  V1  0.5
   V2  0.5V1 
T1  0.5V1 
T2  4T1

4. The height (in mm) for a 125 mm sine bar to measure a taper of 27o32' on a flat work piece is ____
(correct to three decimal places).
Key: (57.77)
Sol:   2732'  27.53
h
sin  

h  sin    25sin 27.53  57.77mm

5. A six-faced fair dice is rolled five times. The probability (in %) of obtaining "ONE" at least four
times is
(A) 33.3 (B) 3.33 (C) 0.33 (D) 0.0033
Key: (C) www.
engi
neer
ingonyour
finger
tips.
ooo
1
Sol: n  5, p  obtaining 'ONE ' 
6
1 5
 q 1 
6 6
p  x  4  ?
By Binomial distribution,
p  x  4   p  x  4   p  x  5
4 5 4 5 5 5
1 5 1 5
 5 C 4      5 C5    
6 6 6 6
26 26
 5  5 100  in%  0.33
6 6

© All rights reserved by Thinkcell Learning Solutions Pvt. Ltd. No part of this booklet may be reproduced or utilized in any form without the written permission.

www.engineeringonyourfingertips.ooo 7

More Notes Join us Telegram-: http//:t.me/allexammentor


www.engineeringonyourfingertips.ooo
ME-GATE 2018

6. The time series forecasting method that gives equal weightage to each of the m most recent
observations is
(A) Moving average method (B) Exponential smoothing with linear trend
(C) Triple Exponential smoothing (D) Kalman Filter
Key (A)
Sol ∵ in Moving Average Method
Dn  Dn 1  Dn  2  ...
Fn 1 
n
1
So, weightage of all past data is
n
i.e., equal weightage provided to all past data.

 4 1 1
7. The rank of the matrix  1 1 1 is
 7 3 1 

(A) 1 (B) 2 (C) 3 (D) 4


Key: (B)

 4 1 1
Sol: Let A   1 1 1
 7 3 1 
A  4  1  3  1 1 7    3  7   0
4 1
consider a 2  2 sub matrix,  4  1 5  0
1 1
 Rank of A  2 www.
engi
neer
ingonyour
finger
tips.
ooo

8. Interpolator in a CNC machine


(A) controls spindle speed (B) coordinates axes movements
(C) operates tool changer (D) commands canned cycle
Key: (B)

9. Four red balls, four green balls and four blue balls are put in a box. Three balls are pulled out of the box
at random one after another without replacement. The probability that all the three balls are red is
(A) 1/72 (B) 1/55 (C) 1/36 (D) 1/27

© All rights reserved by Thinkcell Learning Solutions Pvt. Ltd. No part of this booklet may be reproduced or utilized in any form without the written permission.

www.engineeringonyourfingertips.ooo 8

More Notes Join us Telegram-: http//:t.me/allexammentor


www.engineeringonyourfingertips.ooo
ME-GATE 2018

Key: (B)
Sol: 4-Red; 4-Green; 4-Blue
4 3 2 1
Required probability    
12 11 10 55

10. A grinding ratio of 200 implies that the


(A) grinding wheel wears 200 times the volume of the material removed.
(B) grinding wheel wears 0.005 times the volume of the material removed
(C) aspect ratio of abrasive particles used in the grinding wheel is 200
(D) ratio of volume of abrasive particle to that of grinding wheel is 200
Key: (B)
MRR
Sol: Grinding ratio 
wheel wear
1
200 
wheel wear
1
wheel wear   0.005
200

11. Which one of the following statements is correct for a superheated vapour?
(A) Its pressure is less than the saturation pressure at a given temperature.
(B) Its temperature is less than the saturation temperature at a given pressure.
(C) Its volume is less than the volume of the saturated vapour at a given temperature.
(D) Its enthalpy is less than enthalpy of the saturated vapour at a given pressure.
Key: (A)
Sol: At a given temperature the pressure of superheated vapour is less than the saturation pressure.
www.
engi
neer
ingonyour
finger
tips.
ooo
For example let us take100oC, the saturation pressure corresponding to this temperature is
1.01325bar. If the saturation pressure decreases saturation temperature decreases and vice-versa.
From this we can easily say that if the superheated temperature has to be 100oC then saturation
temperature should be less than 100oC, therefore corresponding saturation pressure will be less.
Hence we can say that for same temperature the pressure of superheated vapour is less than the
saturation pressure.

12. If 1 and 3 are the algebraically largest and smallest principal stresses respectively, the value of
the maximum shear stress is

1  3 1  3  1  3 1  3
(A) (B) (C) (D)
2 2 2 2

© All rights reserved by Thinkcell Learning Solutions Pvt. Ltd. No part of this booklet may be reproduced or utilized in any form without the written permission.

www.engineeringonyourfingertips.ooo 9

More Notes Join us Telegram-: http//:t.me/allexammentor


www.engineeringonyourfingertips.ooo
ME-GATE 2018

Key: (B)
Sol: Diameter of Mohr circle = 1  3 and

1  3
 max  Radius 
2

13. The number of atoms per unit cell and the number of slip systems, respectively, for a face-centered
cubic (FCC) crystal are
(A) 3, 3 (B) 3, 12 (C) 4, 12 (D) 4, 48
Key: (C)

14. A four bar mechanism is made up of links of length 100, 200, 300 and 350 mm. If the 350 mm link
is fixed, the number of links that can rotate fully is ______.
Key: (1) 200
Sol: s  p  q
100  350  200  300 100 300
Hence it is crank rocker
(100) link will rotate -1.
350
So, link whose length is 100 mm will rotate fully. So, only one link will rotate fully.

15. For an Oldham coupling used between two shafts, which among the following statements are
correct?
I. Torsional load is transferred along shaft axis.
II. A velocity ratio of 1:2 between shafts is obtained without using gears.
www.
engi
neer
ingonyour
finger
tips.
ooo
III. Bending load is transferred transverse to shaft axis.
IV. Rotation is transferred along shaft axis.
(A) I and III (B) I and IV (C) II and III (D) II and IV
Key: (B)

16. A flat plate of width L = 1 m is pushed down with a velocity U = 0.01 m/s towards a wall resulting
in the drainage of the fluid between the plate and the wall as shown in the figure.
L
U Plate
u avg d  0.1m u avg

Wall
© All rights reserved by Thinkcell Learning Solutions Pvt. Ltd. No part of this booklet may be reproduced or utilized in any form without the written permission.

www.engineeringonyourfingertips.ooo 10

More Notes Join us Telegram-: http//:t.me/allexammentor


www.engineeringonyourfingertips.ooo
ME-GATE 2018

Assume two-dimensional incompressible flow and that the plate remains parallel to the wall. The
average velocity, uavg of the fluid (in m/s) draining out at the instant shown in the figure is ___
(correct to three decimal places).
Key: (0.05)
Sol: Let b = length of plate perpendicular to paper
Volume drain out per sec = L.b.U
So, L.b.U  2b.u avg .d
L.U 1  0.01
u avg  
2d 2  0.1
u avg  0.05m/s

17. According to the Mean Value Theorem, for a continuous function f(x) in the interval [a, b], there
b
exists a value  in this interval such that  f (x)dx 
a

(A) f    (b a) (B) f (b) (  a) (C) f (a)(b  ) (D) 0

Key: (A)
y
Sol: By using mean value theorem for definite integrals,
b
f  a, b  s.t  f  x  dx   b  a  f    f x
a d
f   c
 in between a & b s.t

Area of rectangle abcd = Area under the curve f(x) with x-axis
x
b
a  b
i.e.,  b  a  f      f  x  dx
www.
engi
neer
ingonyour
finger
tips.
ooo
a

18. A steel column of rectangular section (15 mm × 10 mm) and length 1.5 m is simply supported at
both ends. Assuming modulus of elasticity, E = 200GPa for steel, the critical axial load (in kN) is
___ (correct to two decimal places)
Key: (1.097)
Sol: e  1500mm, b  15mm,d  10mm, E  200GPa
2 EI 15103
PE  2
; I
e 12
15103
2  200 103 
 12  1.097kN
1500 
2

© All rights reserved by Thinkcell Learning Solutions Pvt. Ltd. No part of this booklet may be reproduced or utilized in any form without the written permission.

www.engineeringonyourfingertips.ooo 11

More Notes Join us Telegram-: http//:t.me/allexammentor


www.engineeringonyourfingertips.ooo
ME-GATE 2018

19. If the wire diameter of a compressive helical spring is increased by 2%, the change in spring
stiffness (in %) is ___ (correct to two decimal places.)
Key: (8.243)
Sol: d 2  1.02d1
Gd 4
k  k  d4
8WD3
4
k1  d1 
   k 2  1.082
k 2  1.02d1 
k  k1  1.082  1 
% change in stiffness  2   100%  8.243%
k1  1 

20. For a two-dimensional incompressible flow field given by u  A xiˆ  yjˆ , where A > 0,  
which one of the following statements is FALSE?
(A) It satisfies continuity equation
(B) It is unidirectional when x  0 and y  ,
(C) Its streamlines are given by x = y.
(D) It is irrotational
Key: (C)
Sol: ∵ Equation of streamline
dy v  y
 
dx u x
dy dx
 y    x  ny  nx  nc
xy  C. So, option C is false.
www.
engi
neer
ingonyour
finger
tips.
ooo
21. The equation of motion for a spring-mass system excited by a harmonic force is
mx + Kx = Fcos(  t),
where M is the mass, K is the spring stiffness, F is the force amplitude and  is the angular
frequency of excitation. Resonance occurs when  is equal to
M 1 K K K
(A) (B) (C) 2 (D)
K 2 M M M
Key: (D)

Sol: Resonance will occur when, r  1.


K
i.e., when   n 
M

© All rights reserved by Thinkcell Learning Solutions Pvt. Ltd. No part of this booklet may be reproduced or utilized in any form without the written permission.

www.engineeringonyourfingertips.ooo 12

More Notes Join us Telegram-: http//:t.me/allexammentor


www.engineeringonyourfingertips.ooo
ME-GATE 2018

22. A bar of uniform cross section and weighing 100 N is held horizontally using two massless and
inextensible strings S1 and S2 as shown in the figure.

Rigid support

T1=? T2  ?
S1 S2 Bar

L/2 L/2
The tensions in the strings are
(A) T1 = 100 N and T2 = 0 N (B) T1 = 0 N and T2 = 100 N
(C) T1 = 75 N and T2 = 25 N (D) T1 = 25 N and T2 = 75 N
Key: (B)
Sol: M A  0
T1 T2  ?
L L
T2  100 
2 2
T2  100 L/2 L/2
T1  T2  100 T1 T2
T1  0N, T2  100N

A
L/2
100N

23. F (z) is a function of the complex variable z = x + i y given by

wwwF(z)
.eng ineer i
ngonyour
= iz + k Re(z) + i Im(z)
f
inger
tips.
ooo
For what value of k will F (z) satisfy the Cauchy-Riemann equations?
(A) 0 (B) 1 (C) –1 (D) y
Key: (B)
Sol: F  z   iz  k Re  z   iI m  z 
 i  x  iy   kx  iy
 ix  y  kx  iy
  kx  y   i  x  y 
 u  iv  say 

u v u v
Applying C-R equations, i.e.,  &   k 1
x y y x

© All rights reserved by Thinkcell Learning Solutions Pvt. Ltd. No part of this booklet may be reproduced or utilized in any form without the written permission.

www.engineeringonyourfingertips.ooo 13

More Notes Join us Telegram-: http//:t.me/allexammentor


www.engineeringonyourfingertips.ooo
ME-GATE 2018

24. The type of weld represented by the shaded region in the figure is

(A) groove (B) spot (C) fillet (D) plug


Key: (C)

25. For a Pelton wheel with a given water jet velocity, the maximum output power from the Pelton
wheel is obtained when the ratio of the bucket speed to the water jet speed is _____ (correct to two
decimal places).
Key: (0.5)
Sol: ∵ Efficiency of Pelton wheel is maximum,
u
when  0.5
v
Where u = velocity of blade
v = velocity of jet (Absolute) at inlet.

26. Let X1, X2 be two independent normal random variables with means μ1, μ2 and standard deviations
1 ,  2 respectively. Consider Y = X1 – X2; μ1 = μ2 = 1, 1  1, 2  2, Then,

(A) Y is normal distributed with mean 0 and variance 1


(B) Y is normally distributed with mean 0 and variance 5
www.
engi
neer
ingonyour
finger
tips.
ooo
(C) Y has mean 0 and variance 5, but is NOT normally distributed
(D) Y has mean 0 and variance 1, but is NOT normally distributed
Key: (B)
Sol: Sum or difference of normal random variables is again a normal random variable
Mean of ' y'  E  y   E  x1  x 2   E  x1   E  x 2 
 1  2
 1 1  0
Variance of ' y'  v  y   v  x1  x 2   v  x1   v  x 2 
12  22
1  4  5

© All rights reserved by Thinkcell Learning Solutions Pvt. Ltd. No part of this booklet may be reproduced or utilized in any form without the written permission.

www.engineeringonyourfingertips.ooo 14

More Notes Join us Telegram-: http//:t.me/allexammentor


www.engineeringonyourfingertips.ooo
ME-GATE 2018

27. A slider crank mechanism is shown in the figure. At some instant, the crank angle is 45° and a force
of 40 N is acting towards the left on the slider. The length of the crank is 30 mm and the connecting
rod is 70 mm. Ignoring the effect of gravity, friction and inertial forces, the magnitude of the
crankshaft torque (in Nm) needed to keep the mechanism in equilibrium is ___ (correct to two
decimal places).

40 N

Key: (1.117)
Sol: Fp  40N
30sin 45  70sin  30 70
sin   0.303   17.6 o

45 40 N
Fp
Torque  sin       r
cos 
40  sin 17.6  45 
  0.03  1.117N  mts
cos17.6

28. The value of integral

 r.n dS
S

over the closed surface S bounding a volume V, where r  xiˆ  y j  zkˆ is the position vector and n
is the normal to the surface S, is
(A) V (B) 2 V (C) 3 V (D) 4 V
Key: (C)
www.
engi
neer
ingonyour
finger
tips.
ooo
Sol: By Gauss divergence theorem,

 r.n ds   div r dv
s v

 r  xiˆ  yj  zkˆ 
  3dv  3V  
v  div r 111  3

29. A plane slab of thickness L and thermal conductivity k is heated with a fluid on one side (P), and
the other side (Q) is maintained at a constant temperature, TQ of 25°C, as shown in the figure. The
fluid is at 45°C and the surface heat transfer coefficient, h, is 10 W/m2K. The steady state
temperature, TP (in °C) of the side which is exposed to the fluid is ____ (correct to two decimal
places).

© All rights reserved by Thinkcell Learning Solutions Pvt. Ltd. No part of this booklet may be reproduced or utilized in any form without the written permission.

www.engineeringonyourfingertips.ooo 15

More Notes Join us Telegram-: http//:t.me/allexammentor


www.engineeringonyourfingertips.ooo
ME-GATE 2018

TP

h  10 W / m2 K TQ  25C
T  45C
k  2.5W / mK

L  20cm

Key: (33.89)
Sol: For the steady state heat transfer,
T  Tp TP  TQ
Q 
 1   L  TP
   
 hA   kA 
45  Tp T  25
Q  P h  10 W / m2 K
 1   0.2  TQ  25C
    T  45C
k  2.5W / mk
 10  A   2.5 A 
2.5  Tp  25 
10  45  Tp  
0.2 L  20cm
  90  2Tp    2.5  Tp  62.5
 Tp  33.89C, So answer is33.89

30. F(s) is the Laplace transform of the function


f (t)  2t 2 e  t www.
engi
neer
ingonyour
finger
tips.
ooo
F(1) is ____ (correct to two decimal places).
Key: (0.5)

Sol: L  2t 2 e t   2L  t 2 e  t 
2  2! 2 
 L  t   3  3 & L e f  t   F  s  a   
 at
 2 2

 s  1  
3
s s
4
  Fs
 s  1
3

4
 F 1   0.5
8

© All rights reserved by Thinkcell Learning Solutions Pvt. Ltd. No part of this booklet may be reproduced or utilized in any form without the written permission.

www.engineeringonyourfingertips.ooo 16

More Notes Join us Telegram-: http//:t.me/allexammentor


www.engineeringonyourfingertips.ooo
ME-GATE 2018

31. The true stress (  ) - true strain (  ) diagram of a strain
(MPa)
hardening material is shown in figure. First, there is A
500
loading up to point A, i.e. up to stress of 500 MPa and
strain of 0.5. Then from point A, there is unloading up to
point B, i.e., to stress of 100 MPa, Given that the
100 B
Young's modulus E = 200GPa, the natural strain at point
B (εB) ___ (correct to two decimal places). 
B 0.5

Exp: (0.498) 
(MPa)
AC A
Sol: E  slope  500
BC
400 106
200 109   BC  2 103  0.002
BC
B
So, B  0.5  0.002  0.498 100 C

B 0.5 

32. The percentage scrap in a sheet metal blanking operation of a continuous strip of sheet metal as
shown in the figure is___ (correct to two decimal places).

D/5 D/5 D/5


D/5 D/5
Feed D D D

D/5 D/5 D/5

www.
engi
neer
ingonyour
finger
tips.
ooo
Key: (53.27)
Sol: Area of rectangle A B
6D 7D 42 2
ABCD =   D  1.68D2
5 5 25
D 7D
D  2 
and 5 5
 2
Area of blank = D  0.785D2
4 D C
1.68D  0.785D
2 2
So, % scrap = 100  53.27%
1.68D2 D 6D
D 
5 5

© All rights reserved by Thinkcell Learning Solutions Pvt. Ltd. No part of this booklet may be reproduced or utilized in any form without the written permission.

www.engineeringonyourfingertips.ooo 17

More Notes Join us Telegram-: http//:t.me/allexammentor


www.engineeringonyourfingertips.ooo
ME-GATE 2018

33. An epicyclic gear train is shown in the figure below. The number of teeth on the gears A, B and D
are 20, 30 and 20 respectively. Gear C has 80 teeth on the inner surface and 100 teeth on the outer
surface. If the carrier arm AB is fixed and the sun gear A rotates at 300 rpm in the clockwise
direction, then the rpm of D in the clockwise direction is
(A) 240 C
B
(B) –240
A D
(C) 375

(D) –375
Key: (C)
Sol: N A  300rpm; TA  20; TB  30; Tcin  80
Tcext  100
N A TA 300  20
NB    200
TB 30
N B TB 200  30
NC    75
TC 80
N C TC 75100
ND    375 rpm
TD 20

34. The minimum value of 3x + 5y such that:


3x + 5y  15
4x + 9y  8
13x + 2y  2
x  0, y  0 www.
engi
neer
ingonyour
finger
tips.
ooo
is _______.
Key: (0)
Sol: ∵ All main constraints are of  type and

Non-negativity constraints are of  type.

So, feasible region contain origin (0, 0) as corner point.


So, at origin (0, 0)
zmin  3 0  5 0  0

© All rights reserved by Thinkcell Learning Solutions Pvt. Ltd. No part of this booklet may be reproduced or utilized in any form without the written permission.

www.engineeringonyourfingertips.ooo 18

More Notes Join us Telegram-: http//:t.me/allexammentor


www.engineeringonyourfingertips.ooo
ME-GATE 2018

35. Block P of mass 2 kg slides down the surface and has a speed 20 m/s at the lowest point, Q, where
the local radius of curvature is 2 m as shown in the figure. Assuming g = 10 m/s2, the normal force
(in N) at Q is ____ (correct to two decimal places).
P

Q
Key: (420)
mv2
Sol: R Q  mg 
R
 2  202 
  2 10      20  400  420N
 2 

36. A solid block of 2.0 kg mass slides steadily at a velocity V along a vertical wall as shown in the
figure below. A thin oil film of thickness h = 0.15 mm provides lubrication between the block and
the wall. The surface area of the face of the block in contact with the oil film is 0.04 m2. The
velocity distribution within the oil film gap is linear as shown in the figure. Take dynamic viscosity
of oil as 7 × 10–3 Pa-s and acceleration due to gravity as 10 m/s2. Neglect weight of the oil. The
terminal velocity V (in m/s) of the block is ___ (correct to one decimal place).

h  0.15mm

m kg
m=2.0 2.0kg
www.
engi
neer
ingonyour
fvi
nger
tips.
ooo
V

A  0.04m2
Impermeable
Key: (10.714) wall
Sol: F  .A
 du  V
F     .A  F  . .A
 dy  h
V
 2 10  7 103   0.04
0.15 103
 V  10.714m/s

© All rights reserved by Thinkcell Learning Solutions Pvt. Ltd. No part of this booklet may be reproduced or utilized in any form without the written permission.

www.engineeringonyourfingertips.ooo 19

More Notes Join us Telegram-: http//:t.me/allexammentor


www.engineeringonyourfingertips.ooo
ME-GATE 2018

37. A machine of mass m = 200 kg is supported on two mounts, each of stiffness k = 10 kN/m. The
machine is subjected to an external force (in N) F(t) = 50 cos5t. Assuming only vertical translatory
motion, the magnitude of the dynamic force (in N) transmitted from each mount to the ground is
___ (correct to two decimal places).
F(t)

k k

Key: (33.33)
Sol: k  10kN/m, m=200kg
F=50cos5t,   5rad/sec
F=50
20000
n   10rad / sec
200
0

1   2r 
2
F
 T 
F
1  r    2r 
2 2 2

FT 1
 2
50 5
1  
 10 
FT 1 200
  FT   66.66
50  3 3
 
4
www. engi neer
ingonyour
finger
tips.
ooo
66.66
Forceon each mount   33.33
2

38. A bar is compressed to half of its original length. The magnitude of true strain produced in the
deformed bar is ____ (correct to two decimal places).
Key: (0.693)
 
Sol:   n  f    n0.5
 i
  0.693
But magnitude of true strain = 0.693

© All rights reserved by Thinkcell Learning Solutions Pvt. Ltd. No part of this booklet may be reproduced or utilized in any form without the written permission.

www.engineeringonyourfingertips.ooo 20

More Notes Join us Telegram-: http//:t.me/allexammentor


www.engineeringonyourfingertips.ooo
ME-GATE 2018

39. A carpenter glues a pair of cylindrical wooden logs by bonding their end faces at an angle of  =
30° as shown in the figure.

Log1 Log 2
Interface  axis
4MPa 4MPa

  30
The glue used at the interface fails if
Criterion 1: the maximum normal stress exceeds 2.5MPa.
Criterion 2: the maximum normal stress exceeds 1.5MPa.
Assume that the interface fails before the logs fail. When a uniform tensile stress of 4 MPa is
applied, the interface
(A) fails only because of criterion 1
(B) fails only because of criterion 2
(C) fails because of both criterion 1 and 2
(D) does not fail
Key: (C)
P
Sol:   30o     4MPa
A
P
  cos 2 
A
 4cos 2 30  3MPa  2.5MPa
P 4
  sin 2    sin 601.732 1.5MPa
2A 2

ww1w
Fails by both Criterion .en
and 2gi
neer
ingonyour
finger
tips.
ooo

40. In a Lagrangian system, the position of a fluid particle in a flow is described as x = x0e-kt and
Y = y0ekt where t is the time while x0, y0, and k are constants. The flow is
(A) unsteady and one-dimensional (B) steady and two-dimensional
(C) steady and one-dimensional (D) unsteady and two-dimensional
Key: (B)
dx
Sol: u
dt
u  kx o e kt  u  kx
dy
&v   v  ky
dt

© All rights reserved by Thinkcell Learning Solutions Pvt. Ltd. No part of this booklet may be reproduced or utilized in any form without the written permission.

www.engineeringonyourfingertips.ooo 21

More Notes Join us Telegram-: http//:t.me/allexammentor


www.engineeringonyourfingertips.ooo
ME-GATE 2018

So, V  kxi  kyj


i.e,V  f  t 
So, steady flow & V  f  x, y . So, 2D flow.

41. Processing times (including setup times) and due dates for six jobs waiting to be processed at a
work centre are given in the table. The average tardiness (in days) using shortest processing time
rule is ___ (correct to two decimal places).

Job Processing time (days) Due date (days)

A 3 8

B 7 16

C 4 4

D 9 18

E 5 17

F 13 19

Key: (6.33)
,

Sol: By SPT rule

Job Sequence P.T Machining D.D Tardiness

In Out

A 3 0 3 8 0
www.
engi
neer
ingonyour
finger
tips.
ooo
C 4 3 7 4 3

E 5 7 12 17 0

B 7 12 19 16 3

D 9 19 28 18 10

F 13 28 41 19 22

Total Tardiness = 38
38
Average Tardiness   6.33days
6

© All rights reserved by Thinkcell Learning Solutions Pvt. Ltd. No part of this booklet may be reproduced or utilized in any form without the written permission.

www.engineeringonyourfingertips.ooo 22

More Notes Join us Telegram-: http//:t.me/allexammentor


www.engineeringonyourfingertips.ooo
ME-GATE 2018

42. Steam flows through a nozzle at mass flow rate of m = 0.1 kg/s with a heat loss of 5 kW. The
enthalpies at inlet and exit are 2500 kJ/kg and 2350 kJ/kg, respectively. Assuming negligible
velocity at inlet (C1  0), the velocity (C2) of steam (in m/s) at the nozzle exit is ___ (correct to two
decimal places)
Q  5kW

h1  2500 kJ / kg h 2  2350kJ / kg
m  0.1kg / s
C1  0 C2

Key: (447.21)
Sol: m  0.1kg/sec, Q  5kW  Heat Rejected 
h1  2500kJ/kg, h 2  2350kJ/kg
C1  0m/sec,C2  ?
Q  5kW

h1  2500 kJ / kg h 2  2350kJ / kg
m  0.1kg / s
C1  0 C2  ?

www.
engi
neer
ingonyour
finger
tips.
ooo
Applying steady flow energy equation

 C2   C2 
m  h1  1   Q  m  h 2  2   W
 2000   2000 
W  0, since no work done, C1  0
 C2 
 0.1 2500   5  0.1 2350  2 
 2000 
 C 
2
C2
245  0.1 2350  2   2450  2350  2
 2000  2000
 C2  447.21m/sec

© All rights reserved by Thinkcell Learning Solutions Pvt. Ltd. No part of this booklet may be reproduced or utilized in any form without the written permission.

www.engineeringonyourfingertips.ooo 23

More Notes Join us Telegram-: http//:t.me/allexammentor


www.engineeringonyourfingertips.ooo
ME-GATE 2018

43. A point mass is shot vertically up from ground level with a velocity of 4 m/s at time, t = 0. It loses
20% of its impact velocity after each collision with the ground. Assuming that the acceleration due
to gravity is 10 m/s2 and that air resistance is negligible, the mass stops bouncing and comes to
complete rest on the ground after a total time (in seconds) of
(A) 1 (B) 2 (C) 4 (D) 
Key: (C)
Sol: V1  4m / sec, V2  0.8 4  3.2m / sec

2V1
V1  V  t1 
g
2V2
V2  0.8V  t 2 
g
2V
V3  0.64V  t 3  3
g
T  t1  t 2  ...................
 2V 2V 2V 
  1  2  3  .......
 g g g 
2
  4 1 0.8  0.64  .....
10
2
  4 1 0.8   0.8  ..... 
2

10  
1 0.8
 0.8   4  4sec
1  0.8 0.2

44. A tank of volume 0.05 m3 contains a mixture of saturated water and saturated steam at 200°C. The
mass of the liquid present is 8 kg. The entropy (in kJ/kgK) of the mixture is _____ (correct to two
decimal places) www. engi neer ingonyour fi
nger t ips.ooo
Property data for saturated steam and water are:
At 200°c, Psat = 1.5538 MPa
vf = 0.001157 m3/kg, vg = 0.12736 m3/kg
sfg = 4.1014 kJ/kgK, sf = 2.3309 kJ/kgK
Key: (2.49)
Sol: V  0.05m3 ,Tsat  200C, m L  8kg
At 200C, Psat  1.5538MPa
vf  0.001157m3 /kg, v g  0.12736m 3 /kg
sfg  4.1014kJ/kgK, s f  2.3309kJ/kgK

© All rights reserved by Thinkcell Learning Solutions Pvt. Ltd. No part of this booklet may be reproduced or utilized in any form without the written permission.

www.engineeringonyourfingertips.ooo 24

More Notes Join us Telegram-: http//:t.me/allexammentor


www.engineeringonyourfingertips.ooo
ME-GATE 2018

Given container has both water vapor & liquid


i.e., it is in wet-vapour region,
In that case entropy of mixture is s  sf  xsfg

mv
Where x=dryness fraction=
m v  mL

vf  liquid   m L vf   8  0.001157 
vf  liquid   0.01
v vapour  v  vf  liquid   0.05  0.01  0.04m3
v vapour 0.04
mv    0.31kg
vg 0.12736
0.31
x  0.04
 0.31  8
 s  2.3309  0.04  4.1014   2.49 kJ/kgK

45. A tank open at the top with a water level of 1 m, as shown in the figure, has a hole at a height of 0.5
m. A free jet leaves horizontally from the smooth hole. The distance X (in m) where the jet strikes
the floor is

Free Jet
1m

0.5m

www.
engi
neer
ingonyour
finger
tips.
ooo
x

(A) 0.5 (B) 1.0 (C) 2.0 (D) 4.0


Key: (B)

Sol: u x  2  9.81 0.5  3.13


uy  0
1 1
sy  u y t  a y t 2  t 
2 g
1 1
s x  u x  a x t 2  s x  3.13 (here a x  0)
2 g
s x  1m

© All rights reserved by Thinkcell Learning Solutions Pvt. Ltd. No part of this booklet may be reproduced or utilized in any form without the written permission.

www.engineeringonyourfingertips.ooo 25

More Notes Join us Telegram-: http//:t.me/allexammentor


www.engineeringonyourfingertips.ooo
ME-GATE 2018

46. A self-aligning ball bearing has a basic dynamic load rating (C10, for 106 revolutions) of 35 kN. If
the equivalent radial load on the bearing is 45kN, the expected life (in 106 revolutions) is
(A) below 0.5 (B) 0.5 to 0.8 (C) 0.8 to 1.0 (D) above 1.0
Key: (A)
Sol: Pe  45kN; C  35kN
3
 C   35 
3

L10        0.47  0.5


 Pe   45 

47. A simply supported beam of width 100 mm, height 200 mm and length 4 m is carrying a uniformly
distributed load of intensity 10kN/m. The maximum bending stress (in MPa) in the beam is __
(correct to one decimal place)
10kN / m

4m

Key: (30)
M wL2 10  42
Sol:  y, M    20kN-m
I 8 8
100  2003
I=  66.67 106
12
200
y  100;
2
20 106  200 
    30MPa
66.67 106  2 
www.
engi
neer
ingonyour
finger
tips.
ooo
300
48. The schematic of an external drum rotating 200
clockwise engaging with a short shoe is shown in
the figure. The shoe is mounted at point Y on a rigid
lever XYZ hinged at point X. A force F = 100 N is X
applied at the free end of the lever as shown. Given 300
F
that the coefficient of friction between the shoe and Y
the drum is 0.3, the braking torque (in Nm) applied Z
on the drum is ____ (correct to two decimal places). 100

Key: (8.18)
Sol: M x  0 (All dimensions are in mm)

© All rights reserved by Thinkcell Learning Solutions Pvt. Ltd. No part of this booklet may be reproduced or utilized in any form without the written permission.

www.engineeringonyourfingertips.ooo 26

More Notes Join us Telegram-: http//:t.me/allexammentor


www.engineeringonyourfingertips.ooo
ME-GATE 2018

 R N  200    R N  300    F 300   0


 R N  200    0.3 R N  300   100  300   0
RN 
100 300   272.72 N
 200   0.3 300  
T  R N  Radius  8.18 N  mts

49. The state of stress at a point, for a body in plane stress, is shown in the figure below. If the
minimum principal stress is 10 kPa, then the normal stress y (in kPa) is
y

xy  50 kPa

x  100kPa

(A) 9.45 (B) 18.88 (C) 37.78 (D) 75.50


Key: (C)

 x  y   x  y 
2

Sol: 2       Txy
2

 2   2 

100   y   100  y 
2

10       50
2

 2   2 
2
 100   y    100   y 
2

10    w  ww. eng ine er i


ngonyour fi
ngert
ips.
ooo
2
50
  2   2 
 100   y   100  y   100   y 
2 2

100  2 10         2500


 2   2   2 
4 100  y
100  1000  10 y   2500
4
90 y  2500 1000  100  y  37.77 MPa

50. An electrochemical machining (ECM) is to be used to cut a through hole into a 12 mm thick
aluminium plate. The hole has a rectangular cross-section, 10 mm × 30 mm. The ECM operation
will be accomplished in 2 minutes, with efficiency of 90%. Assuming specific removal rate for
aluminium as 3.44 × 10–2 mm3/ (A s), the current (in A) required is ____ (correct to two decimal
places).

© All rights reserved by Thinkcell Learning Solutions Pvt. Ltd. No part of this booklet may be reproduced or utilized in any form without the written permission.

www.engineeringonyourfingertips.ooo 27

More Notes Join us Telegram-: http//:t.me/allexammentor


www.engineeringonyourfingertips.ooo
ME-GATE 2018

Key: (968.99)
Sol: Volume to be removed = 10  30 12  3600 mm 3
3600
Energy required =  116279 As
3.44 102  0.9
116279
Current   968.99A
120

51. An engine working on air standard Otto cycle is supplied with air at 0.1 MPa and 35 oC. The
compression ratio is 8. The heat supplied is 500kJ/kg. Property data for air: cp = 1.005 kJ/kgK, cv =
0.718 kJ/kg K, R = 0.287 kJ/kg K. The maximum temperature (in K) of the cycle is _____ (correct
to one decimal place).
Key: (1403.98)
Sol: P  0.1MPa,T1  35C, rc  8

Qsup plied  500kJ/kg 3 3


VC
SC
Cp  1.005kJ/kgK, 2 4
T P
CV  0.718kJ/kgK, VC
4 2
1 SC
R  0.287kJ/kgK 1
S
T1  35  273  308K V
1
T2  V1 
   rc 
T1  V2 
1.4 1
 T2   3088  707.60K
Qsup plied  Cv  T3  T2 
500   0.718 T3  w ww.
707.60 engi
neer
ingonyour
finger
tips.
ooo
T3  1403.98K

52. An orthogonal cutting operation is being carried out in which uncut thickness is 0.010 mm, cutting
speed is 130 m/min, rake angle is 15° and width of cut is 6 mm. It is observed that the chip
thickness is 0.015 mm, the cutting force is 60 N and the thrust force is 25 N. The ratio of friction
energy to total energy is ____ (correct to two decimal places).
Key: (0.4406)
Sol: t1  0.01mm; v  130m / min;   15
b  6mm; t c  0.015mm,
cutting force  FH  60N
Thrust force  FV  25N

© All rights reserved by Thinkcell Learning Solutions Pvt. Ltd. No part of this booklet may be reproduced or utilized in any form without the written permission.

www.engineeringonyourfingertips.ooo 28

More Notes Join us Telegram-: http//:t.me/allexammentor


www.engineeringonyourfingertips.ooo
ME-GATE 2018

R  602  252  65N


FH
F  R sin   39.66N

F 25
tan       v   22.61 Fv

FH 60 R F

  37.61
Friction energy F t1 N
 .  0.4406
Totalenergy FH t c

53. An explicit forward Euler method is used to numerically integrate the differential equation
dy
y
dt
using a time step of 0.1. With the initial condition y (0) = 1, t he value of y (1) computed by this
method is ___ (correct to two decimal places).
Key: (2.59)
Sol: By Euler method,
y n 1  y n  nf  t n , y n 
where given f  t, y   y
 y n 1  y n  hy n
 y n 1  1  h  y n
y1  y  0.1  1  0.1 1  1.1  y 0  1
y 2  y  0.2   1.11.1
y3  y  0.3  1.1 1.11.1
:
y10  y 1  1.1
10
2.59
www.
engi
neer
ingonyour
finger
tips.
ooo

54. The maximum reduction in cross-sectional area per pass (R) of a cold wire drawing process is
R = 1 – e–(n + 1)
where n represents the strain hardening coefficient. For the case of a perfectly plastic material, R is
(A) 0.865 (B) 0.826 (C) 0.777 (D) 0.632
Key: (D)
Sol: ∵ for perfectly plastic material, n=0
So, R  1  e1  R  0.632

© All rights reserved by Thinkcell Learning Solutions Pvt. Ltd. No part of this booklet may be reproduced or utilized in any form without the written permission.

www.engineeringonyourfingertips.ooo 29

More Notes Join us Telegram-: http//:t.me/allexammentor


www.engineeringonyourfingertips.ooo
ME-GATE 2018

55. A sprinkler shown in the figure rotates about its hinge point in a horizontal plane due to water flow
discharged through its two exit nozzles.

Q/2 Q/2

10 cm 20 cm

The total flow rate Q through the sprinkler is 1 litre/sec and the cross-sectional area of each exit
nozzle is 1 cm2. Assuming equal flow rate through both arms and a frictionless hinge, the steady
state angular speed of rotation (rad/s) of the sprinkler is ________ (correct to two decimal places).
Key: (10)
Q 1103
Sol: u1  u 2    5m/s u1 u2
2A 2 104
Now Absolute velocity
v1  u1  r1 1  2
v2  u 2  r2  0.1  r1 r2  0.2m
external Torque  0 
m1 v1 r1  m2 v2 r2  0 ( m1  m2 )
   10rad / s

www.
engi
neer
ingonyour
finger
tips.
ooo

© All rights reserved by Thinkcell Learning Solutions Pvt. Ltd. No part of this booklet may be reproduced or utilized in any form without the written permission.

www.engineeringonyourfingertips.ooo 30

More Notes Join us Telegram-: http//:t.me/allexammentor


www.engineeringonyourfingertips.ooo
ME-GATE 2018

Section-I: General Ability

1. The perimeters of a circle, a square and an equilateral triangle are equal. Which one of the
following statements is true?
(A) The circle has the largest area (B) The square has the largest area
(C) The equilateral triangle has the largest area. (D) All the three shapes have the same area.
Key: (A)
Sol: Let, side of equilateral triangle as 'a'.
Then perimeter of equilateral triangle = 3a.
We know that, perimeter of circle = 2r
Perimeter of square = 4x; [where x is side of square]
3a
Given, 3a  2r 3a  4x  x 
3a 4
r 3a
2  sideof square 
3a 4
 Radiusof circle  r  3a
2  x
4

 9a 2  9a 2
 Area of circle  r 2   2   0.716a 2
 4  4
3a 3a 9a 2
∴ Area of square = x  x    0.563a 2
4 4 16
3a 2
∴ Area of equilateral triangle =  0.433a 2
4
∴ The Circle has the largest area.

2. Find the missing group of letters in the following series:


BC, FGH, LMNO,w________
ww. engi neer ingonyour fi
nger
tips.
ooo
(A) UVWXY (B) TUVWX (C) STUVW (D) RSTUV
Key: (B)
3 4 5
Sol: BC FGH LMNO TUVWX

3. “The judge‟s standing in the legal community, though shaken by false allegations of wrongdoing,
remained _______.”
The world that best fills the blank in the above sentence is
(A) Undiminished (B) damaged (C) illegal (D) uncertain
Key: (A)

© All rights reserved by Thinkcell Learning Solutions Pvt. Ltd. No part of this booklet may be reproduced or utilized in any form without the written permission.

www.engineeringonyourfingertips.ooo 2

More Notes Join us Telegram-: http//:t.me/allexammentor


www.engineeringonyourfingertips.ooo
ME-GATE 2018

1 1 1
4. The value of the expression   is ______
1  log u vw 1  log v wu 1  log w uv
(A) –1 (B) 0 (C) 1 (D) 3
Key: (C)
1 1 1
Sol:  
log vw log wu log uv
1 1 1
log u log v log w
log u log v log w
  
log u  log vw log v  log wu log w  log uv
log u log v log w log  uvw 
    1
log  uvw  log  uvw  log  uvw  log  uvw 

5. “The dress ______ her so well that they all immediately ______ her on her appearance.”
The words that best fill the blanks in the above sentence are
(A) complemented, complemented (B) complimented, complemented
(C) complimented, complimented (D) complemented, complimented
Key: (D)

6. Forty students watched films A, B and C over a week. Each student watched either only one film
or all three. Thirteen students watched film A, sixteen students watched film B and nineteen
students watched film C. How many students watched all three films?
(A) 0 (B) 2 (C) 4 (D) 8
Key: (C)
Sol: Given, Total no. of students who watched films A, B and C over a week = n  s   40.
Also given that, each student watched either only one film or all three.
  
i.e.,n A  B  C  n B  C  A  n  C  A  B  0; 
Where  A, B, C are
ww w.engi neer
complementatry ingo
events ny
of A,our
B,Cfinger ti
ps
respectively 
.ooo
Given, n  A   13;n  B  16;n  C  19
Assume that no. of students who watch all three films = n  A  B  C  x


Given, n A  B  C  13  x  n S  40

n  B  C  A   16  x
A 13 B16
n  C  B  A   19  x 13  x 0 16  x
x
n  s   13  x  16  x  19  x  x 0 0

 40  48  2x  2x  48  40 19  x
8
 2x  8  x   4 C 19
2
∴ No. of students who watched all the three films = x=4
© All rights reserved by Thinkcell Learning Solutions Pvt. Ltd. No part of this booklet may be reproduced or utilized in any form without the written permission.

www.engineeringonyourfingertips.ooo 3

More Notes Join us Telegram-: http//:t.me/allexammentor


www.engineeringonyourfingertips.ooo
ME-GATE 2018

7. A house has a number which needs to be identified. The following three statements are given that
can help in identifying the house number.
i. If the house number is a multiple of 3, then it is a number from 50 to 59.
ii. If the house number is NOT a multiple of 4, then it is a number from 60 to 69.
iii. If the house number is NOT a multiple of 6, then it is a number from 70 to 79.
What is the house number?
(A) 54 (B) 65 (C) 66 (D) 76
Key: (D)
Sol: From Statement-i, we have
if house no. is multiple of 3, then house no. 50,59
From Statement-ii, we have
if house no. is not a multiple of 4, then house no. 60,69
From Statement-iii, we have
if house no. is not a multiple of 6, then house no 70,79
∴ The required house number, is 76 among {54, 65, 66, 76};
Since (i) 76 is not multiple of 3, so house no. 50,59
(ii) 76 is not multiple of 6, so house no. 70, 79
(iii) 76 is multiple of 4, so house no. 60, 69
∴ The required house no. is 76.

8. A contract is to be completed in 52 days and 125 identical robots were employed, each operational
for 7 hours a day. After 39 days, five-seventh of the work was completed. How many additional
robots would be required to complete the work on time, if each robot is now operational for 8
hours a day?
(A) 50 (B) 89 (C) 146 (D) 175
Key: (0)
Sol: 1Robot 1hr 1w ww.
work engi neer ingonyour fi
nger tips. ooo
7 hrs  7 work
 125 Robots  7 hrs / day  125 7
125Robots  7 hrs / day  52days 125  7 52  45,500  Total work
Given, After 39 days, 5/7th of work was completed.
5
i.e., 45,500   32,500.  Actually,in 39days 34125work has to becompleted 
7
Remaining work  45,500  32,500  13,000 work
After 39 days, each robot is working 8hrs/day (given)
13days  8hrs/day  125 robots can work 13,000 work.
i.e.Addtional robots not required, if each robot work8hrs/dayon13days.

© All rights reserved by Thinkcell Learning Solutions Pvt. Ltd. No part of this booklet may be reproduced or utilized in any form without the written permission.

www.engineeringonyourfingertips.ooo 4

More Notes Join us Telegram-: http//:t.me/allexammentor


www.engineeringonyourfingertips.ooo
ME-GATE 2018

9. An unbiased coin is tossed six times in a row and four different such trials are conducted. One trial
implies six tosses of the coin. If H stands for head and T stands for tail, the following are the
observations from the four trials:
(1) HTHTHT (2) TTHHHT (3) HTTHHT (4) HHHT __ __.
Which statement describing the last two coin tosses of the fourth trial has the highest probability
of being correct?
(A) Two T will occur (B) One H and one T will occur
(C) Two H will occur (D) One H will be followed by one T
Key: (B)
Sol: In this, we are talking about 4th trial, ie., nothing but one trial
One trial → Tossing a coin six times (or) six coins tossed at a time
6

1  P  H T H T H T          
1 1 1 1 1 1 1
2 2 2 2 2 2 2
6

 2   P T T H H H T   
1

2

 3  P  H T T H H T   
1

2
 4  H H H T _ _

Option(A) : Option(B) : Option(C) : Option(D) :


HHHTTT HHHTHT HHHTHH HHHTTH
 Required probability  or   Required probability  Required probability
6 6 6
1 HHHTTH 1 1
     
2 www.
en
 ginee
Required ringonyour
probability f
inger
tips
2.
o
 oo 2
6 6 6
1 1 1
      2 
2 2 2

10. A wire would enclose an area of 1936 m2, if it is bent into a square. The wire is cut into two
pieces. The longer piece is thrice as long as the shorter piece. The long and the short pieces are
bent into a square and a circle, respectively. Which of the following choices is closest to the sum
of the areas enclosed by the two pieces in square meters?
(A) 1096 (B) 1111 (C) 1243 (D) 2486
Key: (C)
Given that, a wire bent over square has area of 1936m2
Let us assume that, the side of square as 'x'.
Then x 2 1936  x  44meters.

© All rights reserved by Thinkcell Learning Solutions Pvt. Ltd. No part of this booklet may be reproduced or utilized in any form without the written permission.

www.engineeringonyourfingertips.ooo 5

More Notes Join us Telegram-: http//:t.me/allexammentor


www.engineeringonyourfingertips.ooo
ME-GATE 2018

∴ Length of wire= perimeter of square = 4x 176m


Again given a  b  176 such that a  3b
 3b  b  17b  4b 17b
 b  44  a  132  a  3b 
Given 'a' is bent over square and 'b' bent over circle.
132
i.e,. 4x '  a  132  x '   33  x '  length of thesideof square 
4
 x '  33
Area of square  33 33 1089 ...1
 2r  b  'b'bent over circle
44
r   b  44  r  7
2
 Area of circle   72 154   ... 2 
 From 1  &  2   sumof area 's 1089 154 1243m 2

Section-II: Mechanical Engineering

1. The Fourier cosine series for an even function f(x) is given by



f  x   a 0   a n cos  nx 
n 1

The value of the coefficient a2 for the function f  x   cos  x  in 0,  is


2

(A) –0.5 (B) 0.0 (C) 0.5 (D) 1.0


Key: (C)
Exp: We have, Fourier expression for even function:

cos nx
f  x   0   a nwww. e;nifgine iseringon you rfinge rtip
tos.
o ooperiod 2L]
a
f(x) defined over the range –L L. [i.e.,
2 n 1 L
1  cos 2x
But given f  x   cos 2 x 
2
1 1 a
  cos 2x  0  a 2 .cos 2x
2 2 2
 a 0  1; a 2  1 ; a1  0
2

2. Select the correct statement for 50% reaction stage in a steam turbine.
(A) The rotor blade is symmetric.
(B) The stator blade is symmetric.
(C) The absolute inlet flow angle is equal to absolute exit flow angle.
(D) The absolute exit flow angle is equal to inlet angle of rotor blade.

© All rights reserved by Thinkcell Learning Solutions Pvt. Ltd. No part of this booklet may be reproduced or utilized in any form without the written permission.

www.engineeringonyourfingertips.ooo 6

More Notes Join us Telegram-: http//:t.me/allexammentor


www.engineeringonyourfingertips.ooo
ME-GATE 2018

Key: (D)
Exp: For 50% reaction turbine   ,   

3. During solidification of a pure molten metal, the grains in the casting near the mould wall are
(A) coarse and randomly oriented (B) fine and randomly oriented
(C) fine and ordered (D) coarse and ordered
Key: (B)
Exp: Cooling at wall is faster so, grain size is fine & randomly oriented

4. The peak wavelength of radiation emitted by a black body at a temperature of 2000 K is 1.45 m.
If the peak wavelength of emitted radiation changes to 2.90 m, then the temperature (in K) of the
black body is
(A) 500 (B) 1000 (C) 4000 (D) 8000
Key: (B)
Exp: 1  Tmax 1   2  Tmax 2
1.45  2000  2.90   Tmax 2
  Tmax 2  1000K

5. Metal removal in electric discharge machining takes place through


(A) ion displacement (B) melting and vaporization
(C) corrosive reaction (D) plastic shear
Key: (B)
Exp: In EDM, electric spark is used to melt the metal & vaporization of metal takes place

6. The preferred option for holding an odd-shaped work piece in a centre lathe is
(A) Live and dead centres (B) three jaw chuck
(C) lathe dog www. engi neer i
ngonyo(D) ur finge
four r
jawtip s.ooo
chuck
Key: (D)

7. The arrival of customers over fixed time intervals in a bank follow a Poisson distribution with an
average of 30 customers / hour. The probability that the time between successive customer arrival
is between 1 and 3 minutes is _____ (Correct to two decimal places).
Key: (0.3834)
Exp:   30 / hr
  0.5 / min
 P t   1  et
P 1  1  e0.51  0.3934
P  3  1  e0.53  0.7768
So, P 1  T  3  0.7768  0.3934  0.3834

© All rights reserved by Thinkcell Learning Solutions Pvt. Ltd. No part of this booklet may be reproduced or utilized in any form without the written permission.

www.engineeringonyourfingertips.ooo 7

More Notes Join us Telegram-: http//:t.me/allexammentor


www.engineeringonyourfingertips.ooo
ME-GATE 2018

8. For an ideal gas with constant properties undergoing a quasi-static process, which one of the
following represents the change of entropy  s  from state 1 to 2?
T  P  T  V 
(A) s  Cp ln  2   R ln  2  (B) s  Cv ln  2   Cp ln  2 
 T1   P1   T1   V1 
T  P  T   V1 
(C) s  Cp ln  2   Cv ln  2  (D) s  Cv ln  2   R ln  V 
 T1   P1   T1   2
Key: (A)
Exp: For an ideal gas undergoing a quasi static process,
T  P 
Change in entropy, s  Cp n  2   R n  2 
 T1   P1 
Proof:- dQ  dU  pdV  CV dT  pdV
dQ dT P
s   CV  dV
T T T
dT R
 CV  dV  PV  RT   'v' is specific volume
T V
On Integration,
T  V 
s2  s1  CV n  2   R n  2  ... i 
 T1   V1 
We know that
P1V1 P2 V2

T1 T2
V2 P1 T2
   ... 2 
V1 P2 T1
Substituting (2) in (1)
T  P T 
s 2  s1  CV n  2   R n  1  2 
 1w
T  ww. e n
Pg
2 in Te
1 e
r
ingonyourf
inger
tips.
ooo
T  P  T 
 CV n  2   R n  1   R n  2 
 T1   P2   T1 
T  P 
 n  2   Cv  R   R  1 
 T1   P2 
T  P   Cp  C v  R 
 s 2  s1  Cp n  2   R n  2   
 T1   P1  Cp  Cv  R 

© All rights reserved by Thinkcell Learning Solutions Pvt. Ltd. No part of this booklet may be reproduced or utilized in any form without the written permission.

www.engineeringonyourfingertips.ooo 8

More Notes Join us Telegram-: http//:t.me/allexammentor


www.engineeringonyourfingertips.ooo
ME-GATE 2018

9. In a single degree of freedom under damped spring-mass-damper system


as shown in the figure, an additional damper is added in parallel such that M
the system still remains underdamped. Which one of the following
statements is ALWAYS true?
(A) Transmissibility will increase K C
(B) Transmissibility will decrease
(C) Time period of free oscillations will increase.
(D) Time period of free oscillations will decrease.
Key: (C)
Exp: d  1  2
If additionaldamper used, then  increases and d decrease
2
Td 
d
so, Td increases

10. The divergence of the vector field u  ex  cos y ˆi  sin y ˆj is


(B) e cos y  e sin y
x x x x
(A) 0 (C) 2e cos y (D) 2e sin y
Key: (C)
Exp: We have; Div u  .u
  ˆ  ˆ   x
  ˆi  j  k  . e cos yiˆ  e x sin yjˆ 
  x y z 
 x 
 e cos y   e x sin y 
x y
 e x cos y  e x cos y  2e x cos y
 Div u  2e x cos y
11. ww
Match the following w. eng
products inee
with r
theingony
suitable our fi
nger
manufacturing t ips. ooo
process
Product Manufacturing Process
P Toothpaste tube 1 Centrifugal casting
Q Metallic pipes 2 Blow moulding
R Plastic bottles 3 Rolling
S Threaded bolts 4 Impact extrusion

(A) P-4, Q-3, R-1, S-2 (B) P-2, Q-1, R-3, S-4
(C) P-4, Q-1, R-2, S-3 (D) P-1, Q-3, R-4, S-2
Key: (C)

12. A hollow circular shaft of inner radius 10mm, outer radius 20mm and length 1m is to be used as a
torsional spring. If the shear modulus of the material of the shaft is 150 GPa, the torsional stiffness
of the shaft (in kN-m/rad) is _______ (correct to two decimal places).

© All rights reserved by Thinkcell Learning Solutions Pvt. Ltd. No part of this booklet may be reproduced or utilized in any form without the written permission.

www.engineeringonyourfingertips.ooo 9

More Notes Join us Telegram-: http//:t.me/allexammentor


www.engineeringonyourfingertips.ooo
ME-GATE 2018

Key: (35.34)
Exp: Ri =10mm. Di=20mm
R0=20mm, D0=40mm
L=1mts =1000mm
G=150GPa

150  103   404  204 
T GJ 32
 
 1000 
 35.34 kN  m / rad
dy
13. If y is the solution of the differential equation y3  x 3  0, y  0   1, the value of y(-1) is
dx
(A) -2 (B) -1 (C) 0 (D) 1
Key: (C)
Exp: Given D.E
dy
y3  x3  0
dx
 y3dy  x 3dx  0  var iable  separable D.E
  y3dy   x 3dx  c
y4 x 4
   c  1
4 4
Given y  0   1; i.e., y  1 at x  0
1
From 1 ; c 
4
x 4 y4 1
 From 1 ;  
4 4 4
 x 4  y4  1
 y4  1w
 xw
4
w.
engi
neer
ingonyour
finger
tips.
ooo
 y  4 1  x4
 y  1  4 1   1  0
4

14. An engine operates on the reversible cycle 650


as shown in the figure. p
The work output from the engine (in kJ/cycle)  kPa 
is ________ (correct to two decimal places).
400

2 2.5
V  m3 

© All rights reserved by Thinkcell Learning Solutions Pvt. Ltd. No part of this booklet may be reproduced or utilized in any form without the written permission.

www.engineeringonyourfingertips.ooo 10

More Notes Join us Telegram-: http//:t.me/allexammentor


www.engineeringonyourfingertips.ooo
ME-GATE 2018

Key: (62.5)
1
Exp: Work done = Area   0.50  250 650
2
 62.5kN  m

400

m3
2 2.5

15. Pre-tensioning of a bolted joint is used to


(A) strain harden the bolt head (B) decrease stiffness of the bolted joint
(C) increase stiffness of the bolted joint (D) prevent yielding of the thread root
Key: (C)
Exp: Pre-tensioning increases the stiffness of the bolts.

16. A ball is dropped from rest from a height of 1 m in a frictionless tube as shown in the figure. If the
tube profile is approximated by two straight lines (ignoring the curved portion), the total distance
travelled (in m) by the ball is ______ (correct to two decimal places).

1.0 m

45

Key: (2.414)
Exp: Ball will start moving from point B to point A & then reach at point C.
www.
Since ignoring friction,
engi neer ingonyour finger t ips. ooo
i.e, potential energy at point B and C will be same.

B C
 
B C

1.0 m
1m 1m

45
A D A

1
 1  AC, AC 
sin 45
 1  2  2.414mts

© All rights reserved by Thinkcell Learning Solutions Pvt. Ltd. No part of this booklet may be reproduced or utilized in any form without the written permission.

www.engineeringonyourfingertips.ooo 11

More Notes Join us Telegram-: http//:t.me/allexammentor


www.engineeringonyourfingertips.ooo
ME-GATE 2018

17. The viscous laminar flow of air over a flat plate results in the formation of a boundary layer. The
boundary layer thickness at the end of the plate of length L is  L . When the plate length is
increased to twice its original length. The percentage change in laminar boundary layer thickness
at the end of the plate (with respect to L ) is _______ (correct to two decimal places).
Key: (41.42)
Exp: For laminar boundary layer
Where   x
 =Boundary Layer thickness of location „x‟ from leading edge
2 2L
So, 
1 L
2  1.41421
2  1
So, % change   100
1
1.41421  1
  100  41.42%
1

18. The minimum axial compressive load, P, required to initiate buckling for a pinned-pinned slender
column with bending stiffness EI and length L is
2 EI 2 EI 32 EI 42 EI
(A) P  (B) P  (C) P  (D) P 
4L2 L2 4L2 L2
Key: (B)
2 EI
Exp: PE 
L2e
for both ends pinned,
Le  L
2 EI
PE  www.
engi
neer
ingonyour
finger
tips.
ooo
L2

19. Consider a function u which depends on position x and time t. The partial differential equation
u  2 u

t x 2
is known as the
(A) Wave equation (B) Heat equation
(B) Laplace‟s equation (D) Elasticity equation
Key: (B)
du  2 u
Exp: Clearly;  is known as the heat equation
dt x 2

© All rights reserved by Thinkcell Learning Solutions Pvt. Ltd. No part of this booklet may be reproduced or utilized in any form without the written permission.

www.engineeringonyourfingertips.ooo 12

More Notes Join us Telegram-: http//:t.me/allexammentor


www.engineeringonyourfingertips.ooo
ME-GATE 2018

20. Feed rate in slab milling operation is equal to


(A) rotation per minute (rpm)
(B) product of rpm and number of teeth in the cutter
(C) product of rpm, feed per tooth and number of teeth in the cutter
(D) product of rpm, feed per tooth and number of teeth in contact
Key: (C)
Exp: feed rate in milling,
fm  ft  Z  N
f t  feed per tooth
Z  No. of teeth
N  rpm

21. Denoting L as liquid and M as solid in a phase-diagram with the subscripts representing different
phases, a eutectoid reaction is described by
(A) M1  M 2  M3 (B) L1  M1  M2
(C) L1  M1  M2 (D) M1  M 2  M3
Key: (A)
Exp: At Eutectoid point one solid converts into the another solid
M1  M 2  M3
M = Solid
i.e.     Fe3C

1 2 3
22. If A  0 4 5  then det(A-1) is _______ (correct to two decimal places).
0 0 1 
Key: (0.25)
Exp: Given, www.
engi
neer
ingonyour
finger
tips.
ooo
1 2 3
A  0 4 5
0 0 1
Clearly; A is upper triangular matrix

det  A1   A1 


1
( From the properties of determinant)
A
1 1
 A 1    0.25
1 4  1 4
( The determinant of upper triangular matrix is the product of diagonal elements).

© All rights reserved by Thinkcell Learning Solutions Pvt. Ltd. No part of this booklet may be reproduced or utilized in any form without the written permission.

www.engineeringonyourfingertips.ooo 13

More Notes Join us Telegram-: http//:t.me/allexammentor


www.engineeringonyourfingertips.ooo
ME-GATE 2018

23. A local tyre distributor expects to sell approximately 9600 steel belted radial tyres next year.
Annual carrying cost is Rs. 16 per tyre and ordering cost is Rs. 75. The economic order quantity
of the tyres is
(A) 64 (B) 212 (C) 300 (D) 1200
Key: (C)
Exp: D = 9600units/year
C = Rs.16/unit/year
F = Rs. 75/order
2DF
EOQ 
C
2  9600  75

16
 300 units

24. Fatigue life of a material for a fully reversed loading condition is estimated from
a  1100N 0.15
Where a is the stress amplitude in MPa and N is the failure life in cycles. The maximum
allowable stress amplitude (in MPa) for a life of 1×105cycles under the same loading condition is
______ (correct to two decimal places).
Key: (195.61)
Exp: For completely reversed loading,
max  a
a  1100N 0.15
a  1100  1 105 
0.15

 195.61MPa

www.
engi
neer
ingonyour
finger
tips.
ooo
25. A frictionless gear train is shown in the figure. The leftmost 12-teeth gear is given a torque of
100N-m. The output torque from the 60-teeth gear on the right in N-m is
48 Teeth
12 Teeth
T  100 N  m

60 Teeth
12 Teeth
(A) 5 (B) 20 (C) 500 (D) 2000

© All rights reserved by Thinkcell Learning Solutions Pvt. Ltd. No part of this booklet may be reproduced or utilized in any form without the written permission.

www.engineeringonyourfingertips.ooo 14

More Notes Join us Telegram-: http//:t.me/allexammentor


www.engineeringonyourfingertips.ooo
ME-GATE 2018

Key: (D)
T1 T2
Exp: 
Z1 Z 2
100  48
T2   400N  m
12
T2  T3  400N  m
T3 T4

Z3 Z4
400 T4

12 60
400  60
T4   2000N  m
12

26. A circular hole of 25 mm diameter and depth of 20mm is machined by EDM process. The material
removal rate (in mm3/min) is expressed as
4×104IT-1.23
Where I=300 A and the melting point of the material, T=1600°C. The time (in minutes) for
machining this hole is _________ (correct to two decimal places)
Key: (7.143)
Exp: MRR  4  104 IT –1.23
 4  104  300  16001.23
 1374.4mm3 / min
 2
Volume to be removed  DL
4

 252  20  9817.4mm3

4
9817.4
Time required  www. e ngi neer
7.143min ingonyour
finger
tips.
ooo
1374.4

27. Following data correspond to an orthogonal turning of a 100mm diameter rod on a lathe. Rake
angle: +15°; Uncut chip thickness: 0.5 mm; nominal chip thickness after the cut:1.25 mm. The
shear angle (in degrees) for this process is ________ (correct to two decimal places).
Key: (23.31)
Exp: d=100mm
  15, t1  0.5mm
t c  1.25mm
t1
r  0.4
tc

© All rights reserved by Thinkcell Learning Solutions Pvt. Ltd. No part of this booklet may be reproduced or utilized in any form without the written permission.

www.engineeringonyourfingertips.ooo 15

More Notes Join us Telegram-: http//:t.me/allexammentor


www.engineeringonyourfingertips.ooo
ME-GATE 2018

r cos  0.4  cos15


tan    tan  
1  r sin  1  0.4  sin15
tan   0.4309   23.31

28. A rigid rod of length 1 m is resting at an angle   45 as shown in the figure. The end P is
dragged with a velocity of U=5 m/s to the right. At the instant shown, the magnitude of the
velocity V (in m/s) of point Q as it moves along the wall without losing contact is
y

U  5m / s
  45 P
x

(A) 5 (B) 6 (C) 8 (D) 10


Key: (A)
Exp: VP   IP  
VQ   IQ  
Q cos 45
IP  IQ  cos 45 I

VP  VQ  5m/sec
sin 45

U  5m / s
45
P

29. The true stress (in MPa) versus true stain relationship for a metal is given by
  10200.4
ww
The cross-sectional w.
area aten gi
the nee
start ofr
iangon
test you
(when r f
thein ger
stress tips
and .ooo
strain values are equal to zero) is
100mm2. The cross – sectional area at the time of necking (in mm2) is _________ (correct to two
decimal places)
Key: (67.032)
Exp:   1020 0.4
  K n Ai  100mm 2
at necking n  0.4
   Ai 
 n    n A 
f

 i   f
A  100
0.4  n  i   e0.4 
 Af  Af
100
Af  0.4
 A f  67.032mm2
e

© All rights reserved by Thinkcell Learning Solutions Pvt. Ltd. No part of this booklet may be reproduced or utilized in any form without the written permission.

www.engineeringonyourfingertips.ooo 16

More Notes Join us Telegram-: http//:t.me/allexammentor


www.engineeringonyourfingertips.ooo
ME-GATE 2018

30. A bar of circular cross section is clamped at ends P and Q as shown in the figure. A torsional
moment T=150 Nm is applied at a distance of 100mm from end P. The torsional reactions (T P,TQ)
in Nm at the ends P and Q respectively are
P T
Q

100 200

(All dimensions are in mm)

(A) (50, 100) (B) (75, 75) (C) (100, 50) (D) (120, 30)
Key: (C)
Exp: T  150N  m
T
TP  TQ  150N  m
1  2
TP 1 TQ 2 1  2

GJ GJ
TP 100   TQ  200 
100 200

TP  2TQ  TQ  0.5TP
TP  TQ  150
TP  0.5TP  150
TP  100N  m, TQ  50N  m
 T ,T   100,50
P Q

www.
engi
neer
ingonyour
finger
tips.
ooo
31. Air flows at the rate of 1.5m3/s through a horizontal pipe with a gradually reducing cross-section
as shown in the figure. The two cross-sections of the pipe have diameters of 400mm and 200 mm.
Take the air density as 1.2 kg/m3 and assume inviscid incompressible flow. The change in
pressure (P2-P1) (in kPa) between sections 1 and 2 is
200mm
(1)
Air Flow 400mm (2)

1.5 m3 / s

(A) –1.28 (B) 2.56 (C) -2.13 (D) 1.28

© All rights reserved by Thinkcell Learning Solutions Pvt. Ltd. No part of this booklet may be reproduced or utilized in any form without the written permission.

www.engineeringonyourfingertips.ooo 17

More Notes Join us Telegram-: http//:t.me/allexammentor


www.engineeringonyourfingertips.ooo
ME-GATE 2018

Key: (A)
Exp: Q=1.5m2/s
Apply Bernoulli‟s equation b/w (1) & (2)
P1 V12 P V2 Q 1.5  4
  Z1  2  2  Z2 V ; V1   11.936m / s
 2g  2g   0.4 
2
A1
P1  P2 V22  V12 1.5  4
 V2   47.746m / s
g    0.2 
2
2g
 47.7462  11.9362 
So, P1  P2  1.2   
 2 
P1  P2  1.28kPa
or P2  P1  1.28kPa

32. A frictionless circular piston of area 10-2 m2 and mass g  10m / s 2


patm
100kg sinks into a cylindrical container of the same area
filled with water of density 1000 kg/m3 as shown in the 100kg
figure. The container has a hole of area 10-3 m2 at the
bottom that is open to the atmosphere. Assuming there is
no leakage from the edges of the piston and considering 0.5m

water to be incompressible, the magnitude of the piston


velocity (in m/s) at the instant shown is ______ (correct
to three decimal places).
Key: (1.456)
Exp: A1V1  A 2 V2
102 V1  103 V2
V2  10V1
1
100  10
P1  N/m 2

102 www.engi
neer
ingonyour
finger
tips.ooo
P2  0 0.5m
Apply Bernoulli‟s equation between (1) & (2)
P1 V12 P V2 2
  Z1  2  2  Z2
 2g  2g
105 V12 100V12
  0.5  0  0
103  10 2  10 2  10
99V12
 10  0.5
20
V1  1.456m/s

33. A 0.2 m thick infinite black plate having a thermal conductivity of 3.96 W/m-K is exposed to two
infinite black surfaces at 300 K and 400 K as shown in the figure. At steady state, the surface

© All rights reserved by Thinkcell Learning Solutions Pvt. Ltd. No part of this booklet may be reproduced or utilized in any form without the written permission.

www.engineeringonyourfingertips.ooo 18

More Notes Join us Telegram-: http//:t.me/allexammentor


www.engineeringonyourfingertips.ooo
ME-GATE 2018

temperature of the plate facing the cold side is 350K. The value of Stefan-Boltzmann constant,  ,
is 5.67×10-8 W/m2 K4. Assuming 1-D heat conduction, the magnitude of heat flux through the
plate (in W/m2) is _______ (correct to two decimal places).
0.2m

400K
vacuum vacuum
300K

0.2m

Key: (391.58)
Exp: 1-D study state & Black Surfaces    1
Heat flux(q)    T14  T24 
Vacuum Vacuum

 5.67  108 3504  3004 
 391.58 W
m2
T2  300K T1  350K 400K

34. A steel wire is drawn from an initial diameter (di) of 10 mm to a final diameter (df) of 7.5mm. The
half cone angle    of the die is 5° and the coefficient of friction    between the die and the
wire is 0.1. The average of the initial and final yield stress   Y  avg  is 350 MPa. The equation
for drawing stress f ,(in MPa) is given as:

 ww 1 w. e  ngi
dneeri
ngonyour
finger
tips.
ooo 2  cot 
f 
f   Y avg 1   1    
  cot     d i  
The drawing stress (in MPa) required to carry out this operation is _______ (correct to two
decimal places).
Key: (316.24)
Exp: di  10mm,   5, y  350MPa
d f  7.5mm   0.1
1    dt  
2  cot 

f    y  1   1    
avg
  cot     d i  
   7.5  
20.1cot5
 1
 350 1   1      316.24MPa
 0.1cot 5    10  

© All rights reserved by Thinkcell Learning Solutions Pvt. Ltd. No part of this booklet may be reproduced or utilized in any form without the written permission.

www.engineeringonyourfingertips.ooo 19

More Notes Join us Telegram-: http//:t.me/allexammentor


www.engineeringonyourfingertips.ooo
ME-GATE 2018

35. The arc lengths of a directed graph of a project are as shown in the figure. The shortest path length
from node 1 to node 6 is ____________
4
2 4
2
2
2
1 1 1 6

4 4
3
Key: (7) 3 5
Exp: Shortest Path is
(1) – (2) – (5) – (4) – (6)
& length of shortest path = 2+2+1+2 = 7

36. The problem of maximizing z=x1-x2 subject to constraints x1  x 2  10, x1  0, x 2  0 and


x 2  5 has
(A) no solution (B) one solution
(C) two solutions (D) more than two solutions
Key: (B)
Exp: slope of objective function is not equal to slope of any one constraint so, unique optional
solution

37. Ambient air is at a pressure of 100kPa, dry bulb temperature of 30°C and 60% relative humidity.
The saturation pressure of water at 300 C is 4.24 kPa. The specific humidity of air (in g/kg of dry
air) is ___________ (correct to two decimal places).
Key: (16.24)
Exp: Pt  100kPa,   60%
t db  30C, Pvs  4.424kPa
www. engi
neer
ingonyour
finger
tips.
ooo
Pv
  Pv   Pvs
Pvs
 0.6  4.24  2.544kPa
Pv
Specific humidity  w   0.622
Pt  Pv
2.544
 0.622 
100  2.544
 0.016236 kgw.v
kgd.a
 16.24 gmw.v
kgd.a

© All rights reserved by Thinkcell Learning Solutions Pvt. Ltd. No part of this booklet may be reproduced or utilized in any form without the written permission.

www.engineeringonyourfingertips.ooo 20

More Notes Join us Telegram-: http//:t.me/allexammentor


www.engineeringonyourfingertips.ooo
ME-GATE 2018

38. A standard vapor compression refrigeration cycle operating with a condensing temperature of
35°C and an evaporating temperature of -10°C develops 15 kW of cooling. The p-h diagram
shows the enthalpies at various states. If the isentropic efficiency of the compressor is 0.75, the
magnitude of compressor power (in kW) is _____ (correct to two decimal places).

35C
650

p
 kPa 
10C
400

250 400 475


h  kJ / kg 

Key: (10)
Exp: R .E  15k W h1  400kJ / kg
isent C  0.75 h 2  475kJ / kg
h 4  250kJ / kg 3 35C 2
650
h1  h 4 400  250 150
C.O.P    2
h 2  h1 475  400 75 p
 kPa 
R.E 10C
But C.O.P  400 1
Win 4

15
Win   7.5kW 250 400 475
2
Isotropic efficiency of compressor is given, actual compressor power required
W 7.5
 in   10kW
isent C 0.75 www. engi neer ingonyour finger tips. ooo

39. For sand-casting a steel rectangular plate with dimensions 80mm×120mm×20mm, a cylindrical
riser has to be designed. The height of the riser is equal to its diameter. The total solidification
time for the casting is 2 minutes. In Chvorinov‟s law for the estimation of the total solidification
time, exponent is to be taken as 2. For a solidification time of 3 minutes in the riser, the diameter
(in mm) of the riser is _______ (correct to two decimal places).
Key: (51.84)

Exp: Given, t c  2min A r  d 2  d 2
2
3
t r  3min A r  d 2
2

© All rights reserved by Thinkcell Learning Solutions Pvt. Ltd. No part of this booklet may be reproduced or utilized in any form without the written permission.

www.engineeringonyourfingertips.ooo 21

More Notes Join us Telegram-: http//:t.me/allexammentor


www.engineeringonyourfingertips.ooo
ME-GATE 2018

Vc  80  120  20  192000mm3
 3
Vr  d
4
Ac  2  80 120   120  20    20  80    27200mm 2
2
t c  Vc A r 
 
t r  A c Vr 
2
 3 2
d 
2  192000 2
 
3  27200  3 
d
 4 
2
2  42.352 
  d  51.84mm
3  d 

40. A welding operation is being performed with voltage =30V and current =100A. The cross
sectional area of the weld bead is 20mm2. The work-piece and filler are of titanium for which the
specific energy of melting is 14 J/mm3. Assuming a thermal efficiency of the welding process
70% the welding speed (in mm/s) is ______ (correct to two decimal places).
Key: (7.5)
Exp: Hm .A.V  .V.I
14×20×V=0.7×30×100
V=7.5m/s

41. For a position vector r  xiˆ  yjˆ  zkˆ the norm of the vector can be defined as | r | x 2  y2  z2 .
Given a function   ln | r |, its gradient  is
r r r
(A) r (B) (C) (D)
|r| r.r | r |3
www.
engi
neer
ingonyour
finger
tips.
ooo
Key: (C)
Exp: Given r  xiˆ  yjˆ  zkˆ

| r | x 2  y 2  z 2
Also given;   n | r |
 n  x 2  y 2  z 2 

n  x 2  y2  z 2 
1

2
gradient of   
 1 
 î  n  x 2  y2  z 2 
x  2 

© All rights reserved by Thinkcell Learning Solutions Pvt. Ltd. No part of this booklet may be reproduced or utilized in any form without the written permission.

www.engineeringonyourfingertips.ooo 22

More Notes Join us Telegram-: http//:t.me/allexammentor


www.engineeringonyourfingertips.ooo
ME-GATE 2018

1 1  
 î  2 2x
2  x  y2  z 2 
 x 
 î  2 2 
x  y  z 
2

 x  ˆ y  ˆ z 
 ˆi  2 2 
 j 2 2 
k 2 2
x  y  z  x  y  z  x  y  z 
2 2 2

xiˆ  yjˆ  zkˆ r


 2 
x y z 2 2
r.r

42. A bar is subjected to a combination of a steady load of 60kN and a load fluctuating between -
10kN and 90 kN. The corrected endurance limit of the bar is 150 MPa, the yield strength of the
material is 480 MPa and the ultimate strength of the material is 600MPa. The bar cross-section is
square with side a. If the factor of safety is 2, the value of a (in mm), according to the modified
Goodman‟s criterion, is _____ (correct to two decimal places).
Key: (31.62)
Exp: Pstatic  60kN, yt  480MPa
Pmax  90kN, ut  600MPa
Pmin  10kN, e  150MPa,F.S  2
Considering Static varying load
Pmax  90  60  150kN
Pmin  60  10  50kN
150  50
Pmean   100kN
2
150  50
PV   50kN
2
P
mean  mean www. engi
neer
ingonyour
finger
tips.
ooo
a2
P
 v  V2
a
Good man equation
mean  v 1
 
ut e F.S
100  103 50 103
a2 2 1
  a 
600 150 2
2
a =1000
a=31.62mm

© All rights reserved by Thinkcell Learning Solutions Pvt. Ltd. No part of this booklet may be reproduced or utilized in any form without the written permission.

www.engineeringonyourfingertips.ooo 23

More Notes Join us Telegram-: http//:t.me/allexammentor


www.engineeringonyourfingertips.ooo
ME-GATE 2018

43. A bimetallic cylindrical bar of cross sectional area 1 m2 is made by bonding steel (Young‟s
modulus =210 GPa) and Aluminium (Young‟s modulus =70GPa) as shown in the figure. To
maintain tensile axial strain of magnitude 10-6 in Steel bar and compressive axial strain of
magnitude 10-6 in Aluminium bar, the magnitude of the required force P (in kN) along the
indicated direction is
L/2 L/2

clamped end P clamped end


Steel Aluminium
perfectly bonded interface

(A) 70 (B) 140 (C) 210 (D) 280


Key: (D)
Exp: al st  106
st al
Pst Pal

Ast .Est A al .E al
210
Pst  Pal  Pst  3Pal
70
st  3al
st st st
 210  103 106  0.21MPa
al  70  103 106  0.07
P   st  al  A
  0.21  0.07  106  280kN
www.
engi
neer
ingonyour
finger
tips.
ooo
44. A vehicle powered by a spark ignition engine follows air standard Otto cycle    1.4 . The
engine generates 70kW while consuming 10.3kg/hr of fuel. The calorific value of fuel is
44,000kJ/kg. The compression ratio is _____ (correct to two decimal places).
Key: (7.61)
Exp: γ = 1.4, W=70kW
mf = 10.3kg/h
C.V=44000KJ/kg
rc =?
For Otto cycle
1
 1
 rc 
1

© All rights reserved by Thinkcell Learning Solutions Pvt. Ltd. No part of this booklet may be reproduced or utilized in any form without the written permission.

www.engineeringonyourfingertips.ooo 24

More Notes Join us Telegram-: http//:t.me/allexammentor


www.engineeringonyourfingertips.ooo
ME-GATE 2018

W.D
But  
H.S
W.D 1
 1
 rc 
1
H.S
70 1
1
 rc 
1.4 1
10.3
 44000
3600
rc  7.61

45. Steam in the condenser of a thermal power plant is to be condensed at a temperature of 300 C with
cooling water which enters the tubes of the condenser at 14°C and exits at 22°C. The total surface
area of the tubes is 50 m2, and the overall heat transfer coefficient is 2000 W/m2 K. The heat
transfer (in MW) to the condenser is ______ (correct to two decimal places).
Key: (1.15) T
Exp: t h1  t h2  30C

t c2  22C

t c1  14C
L

1  t h1  t c1  30  14  16C A  50m
2

  t  t  30  22  8C U  2000 W
2 h2 c2
m2 K
1  2
LMTD  m  
 1 
nw ww.engi
neer
ingonyour
finger
tips.
ooo
 2
16  8
  11.54C
 16 
n 
8
 Heat Transfer  Q   UAm
 2000  50  11.54  1154156W  1.15MW

46. Air is held inside a non insulated cylinder using a piston (mass M=25kg and area A=100 cm2) and
stoppers (of negligible area), as shown in the figure. The initial pressure Pi and temperature Ti of
air inside the cylinder are 200kPa and 400°C, respectively. The ambient pressure P and
temperature T are 100kPa and 27°C, respectively. The temperature of the air inside the cylinder
(°C) at which the piston will begin to move is ________ (correct to two decimal places).

© All rights reserved by Thinkcell Learning Solutions Pvt. Ltd. No part of this booklet may be reproduced or utilized in any form without the written permission.

www.engineeringonyourfingertips.ooo 25

More Notes Join us Telegram-: http//:t.me/allexammentor


www.engineeringonyourfingertips.ooo
ME-GATE 2018

P  100kPa
T  27C

PISTON M  25kg
A  100cm2
AIR
g  10m / s 2
Pi  200kPa
Ti  400C

Key: (147.63)
Exp: P  100kPa
T  27C

Piston M  25kg
A  100cm 2
stops
Air
g  10m / s 2
Pi  200kPa
Ti  400C

Total pressure on the piston = P  pressure due to weight of piston


 mg 
 P   
 A 
 25  10  103 
 100     125kPa
 100  104 
www. engi neer ingonyour fi
nger
tips.
ooo
Internal pressure of air =200kPa
The pressure at which the piston can move is 125kPa
Let T2 the temperature corresponding to P2=125KPa
PV P V
 From 1 1  2 2
T1 T2
200 125
  volume of air constant 
673 T2
T2  420.625K
 147.625C  147.63C

47. In a cam-follower, the follower rises by h as the cam rotates by  (radians) at constant angular
velocity  (radians/s). The follower is uniformly accelerating during the first half of the rise

© All rights reserved by Thinkcell Learning Solutions Pvt. Ltd. No part of this booklet may be reproduced or utilized in any form without the written permission.

www.engineeringonyourfingertips.ooo 26

More Notes Join us Telegram-: http//:t.me/allexammentor


www.engineeringonyourfingertips.ooo
ME-GATE 2018

period and it is uniformly decelerating in the latter half of the rise period. Assuming that the
magnitudes of the acceleration and deceleration are same, the maximum velocity of the follower is
4h 2h
(A) (B) h (C) (D) 2h
 
Key: (C)
Exp: S=h,   
 
t 
 

V  u  at  V  a  h
2
h h t 
1  
S  ut  at 2 2 2 2 2
2
2
h 1 t
 a  
2 2 2
  
2
h 1
 a  
2 2  2 
42 h
a
2
 42 h 
V a  2 
2  2
2h
V 

48. A thin-walled cylindrical can with rigid end caps has a mean radius R=100 mm and a wall
thickness of t=5 mm. The can is pressurized and an additional tensile stress of 50MPa is imposed
along the axial direction as shown in the figure. Assume that the state of stress in the wall is
uniform along its length. If the magnitudes of axial and circumferential components of stress in
wthe
the can are equal, wwpressure
.engi n(in
eer ingo
MPa) nyou
inside r
thefinge
can isr
t i
ps. ooo
__________(correct to two decimal
places).

end cap t end cap

Pr essurized can
50MPa 50MPa
R

Key: (5)
Exp: t=5mm
R=100m  D  200mm
Additional   50MPa

© All rights reserved by Thinkcell Learning Solutions Pvt. Ltd. No part of this booklet may be reproduced or utilized in any form without the written permission.

www.engineeringonyourfingertips.ooo 27

More Notes Join us Telegram-: http//:t.me/allexammentor


www.engineeringonyourfingertips.ooo
ME-GATE 2018

 t  L  50
PD PD
  50
2t 4t
PD
 50
4t
50  4  5
P  5MPa
200

49. In a rigid body in plane motion, the point R is accelerating with respect to point P at 10180
m/s2. If the instantaneous acceleration of point Q is zero, the acceleration (in m/s2) of point R is

20
y

16 R
Q x

(A) 8233 (B) 10255 (C) 10217 (D) 8217


Key: (D)
Exp: a PR  a R  a P , PR  20
10   PR   2

10 1
2  
20 2
a QR   QR  2

2
 1 
 16     8m / sec
2

 2
www. engi
neer
ingonyour
finger
tips.
ooo
120
PQR  sin  
20
1
 
  sin 0.6  36.86.
180  36.86  216.86  217
8217

50. A force of 100N is applied to the centre of a circular disc, of


mass 10 kg and radius 1m, resting on a floor as shown in the 100N
figure. If the disc rolls without slipping on the floor, the linear
acceleration (in m/s2) of the centre of the disc is ______
(correct to two decimal places).

© All rights reserved by Thinkcell Learning Solutions Pvt. Ltd. No part of this booklet may be reproduced or utilized in any form without the written permission.

www.engineeringonyourfingertips.ooo 28

More Notes Join us Telegram-: http//:t.me/allexammentor


www.engineeringonyourfingertips.ooo
ME-GATE 2018

Key: (6.66)
Exp: r =1m
Fx  ma  100  F   ma
100  F   10  a
100  10a   F 100N
Torque, I  F  r
1 2   
 2 mr    100  10a  1
1 F
2

 2  10  1    100  10a

5  100  10  15  100


  6.66rad / sec2
a  r  6.66 1  6.66m / sec2

51. A test is conducted on a one-fifth scale model of a Francis turbine under a head of 2m and
volumetric flow rate of 1m3/s at 450 rpm. Take the water density and the acceleration due to
gravity as 103 kg/m3 and 10 m/s2, respectively. Assume no losses both in model and prototype
turbines. The power (in MW) of a full sized turbine while working under a head of 30 m is
___________ (correct to two decimal places).
Key: (29.025)
D1 Dm 1
Exp:  
D2 Dp 5
(1)  2
H1  2m H2  30m
Q1  1m /s, N1  450rpm P2  ?
3

H  N 2 D2
H 2  N 2   D2 
2 ww
2w.engi
neer
ingonyour
finger
tips.
ooo
 
H1  N1   D1 
2
30  N 2  N
   52  2  0.774
2  N1  N1
3
Q2  N 2  D 2 

Q1  N1  
 D1 
Q2
 0.774  53
1
Q2  96.75  m3 / s
P2  gQ2 H2  1000 10  96.75  30  29.025MW

© All rights reserved by Thinkcell Learning Solutions Pvt. Ltd. No part of this booklet may be reproduced or utilized in any form without the written permission.

www.engineeringonyourfingertips.ooo 29

More Notes Join us Telegram-: http//:t.me/allexammentor


www.engineeringonyourfingertips.ooo
ME-GATE 2018

52. Given the ordinary differential equation


d2 y dy
  6y  0
dx 2 dx
dy
With y  0   0 and  0   1, the value of y(1) is _____ (correct to two decimal places).
dx
Key: (1.47)
Exp: Given D.E
d2 y dy
  6y  0
dx 2 dx
  D2  D  6  y  0
The Auxiliary equation is m 2  m  6  0
 m 2  3m  2m  6  0
 m  m  3  2  m  3  0
  m  3  m  2   0
 m  2, 3.  Roots are real and distinct 
The solution is
y  C1e 3x  C2 e 2x ...  i 
given y  0   0
from  i   0  C1  C2 ...  ii 
dy
from  i    3C1e3x  2C2 e2x
dx
dy
  1 at x  0
dx
1  3C1  2C2   iii 
 1  3C1  2  C1   From  ii  
 C1   1  Cw 2  5  C 2  C1 
5
w1w.engi neer ingonyour
finger
tips.
ooo
1 3x 1 2x
 The required solution is y  e  e
5 5
1 1
 y 1  e3  e 2
5 5
 y 1   e2  e3 
1
5
 
 y 1  1.47

© All rights reserved by Thinkcell Learning Solutions Pvt. Ltd. No part of this booklet may be reproduced or utilized in any form without the written permission.

www.engineeringonyourfingertips.ooo 30

More Notes Join us Telegram-: http//:t.me/allexammentor


www.engineeringonyourfingertips.ooo
ME-GATE 2018

53. Let z be a complex variable. For a counter-clockwise integration around a unit circle C centred at
origin,
1
C 5z  4 dz  Ai,
the value of A is
(A) 2/5 (B) 1/2 (C) 2 (D) 4/5
Key: (A)
Exp: Singular point is z = 4/5, which lies inside the unit circle C: |z|=1
By Cauchy‟s integral formula:

C 5z  4 dz  C 5 z  4 dz  5  2i 1
1 1 1

5  
2
 i ... i 
5
1
But given  dz  Ai
C 5z  4

2
 i  Ai  from  i  
5
A2
5

54. Let X1 and X2 be two independent exponentially distributed random variables with means 0.5 and
0.25, respectively. Then Y=min (X1, X2) is
(A) exponentially distributed with mean 1/6
(B) exponentially distributed with mean 2
(C) normally distributed with mean 3/4
(D) normally distributed with mean 1/6
Key: (A)
www.
engi
neer
ingonyour
finger
tips.
ooo
Exp: We know that, if X1 and X2 are independent and exponential R. V‟s with parameters 1 and 2
then X = min(X1, X2) is exponential R.V with parameter   1   2 .

Given E  X1   0.5 E  X 2   0.25


1 1
 parameter of X1   X1    X2  4
0.5 0.25
  X1  2
 parameter of Y   X1   X2  2  4  6
1
Mean of Y  E  Y  
6

© All rights reserved by Thinkcell Learning Solutions Pvt. Ltd. No part of this booklet may be reproduced or utilized in any form without the written permission.

www.engineeringonyourfingertips.ooo 31

More Notes Join us Telegram-: http//:t.me/allexammentor


www.engineeringonyourfingertips.ooo
ME-GATE 2018

55. Taylor‟s tool life equation is used to estimate the life of a batch of identical HSS twist drills by
drilling through holes at constant feed in 20mm thick mild steel plates. In test 1, a drill lasted 300
holes at 150 rpm while in test 2, another drill lasted 200 holes at 300 rpm. The maximum number
of holes that can be made by another drill from the above batch at 200 rpm is _______ (correct to
two decimal places).
Key: (254)
(1)  2
Exp: T1  300holes T2  200holes
N1  150rpm N 2  300rpm
At N3=200rpm, T3=?
VT n  C  V1T1n  V2 T2n
n n
N1  T2  150  200 
    
N 2  T1  300  300 
n
2
0.5     0.67n  0.5
3
log 0.67  n  n  1.73
0.5

V1T1n  V3T3n
1/n 1/1.73
 T3   N1   T3   150 
 T    N    300    200 
 1  3  
T3  254holes

www.
engi
neer
ingonyour
finger
tips.
ooo

© All rights reserved by Thinkcell Learning Solutions Pvt. Ltd. No part of this booklet may be reproduced or utilized in any form without the written permission.

www.engineeringonyourfingertips.ooo 32

More Notes Join us Telegram-: http//:t.me/allexammentor


www.engineeringonyourfingertips.ooo
|ME| GATE-2017-PAPER-I www.gateforum.com

Mechanical Engineering
Q. No. 1 to 25 Carry One Mark Each

1. A motor driving a solid circular steel shaft transmits 40kW of power at 500 rpm. If the
diameter of the shaft is 40 mm, the maximum shear stress in the shaft is ________MPa.
Key: 60 to 61
Exp: P=40kW N=500rpm
16T
D= 40mm T=
d 3
P  60,000 40  60,000
T   763.94N  m
2N 2  500
16  763.94  103
  60.79MPa
   40 
3

2. Consider the following partial differential equation for u(x,y) with the constant c > 1 :
u u
c 0
y x
Solution of this equation is
(A) u  x, y   f  x  cy  (B) u  x, y   f  x  cy 
(C) u  x, y   f  cx  y  (D) u  x, y   f  cx  y 
Key: (B)
u u
Exp: Given c 0
y x
u
 f '  x  cy 
x
u
 c f '  x  cy 
x
u u www.
engi
neer
ingonyour
fi
nger
tips.
ooo
 0
x y
u  x, y   f  x  cy 

3. The following figure shows the velocity- time plot for a particle traveling along a straight line.
The distance covered by the particle from t = 0 to t= 5 s is __________m.

4
3
Velocity (m / s)
2
1

1 2 3 4 5 6
Time(s)

 ICP–Intensive Classroom Program  eGATE-Live Internet Based Classes DLP  TarGATE-All India Test Series
www.engineeringonyourfingertips.ooo
Leaders in GATE Preparations  65+ Centers across India
© All rights reserved by Gateforum Educational Services Pvt. Ltd. No part of this booklet may be reproduced or utilized in any form without the written permission.

More Notes Join us Telegram-: http//:t.me/allexammentor


www.engineeringonyourfingertips.ooo
|ME| GATE-2017-PAPER-I www.gateforum.com

Key: 10 to 10
Exp: Area under V- T wave
S  a1  a 2  a 3  a 4  a 5
1    1  4    4  2  
  11   1 1     1     2
2    2    2  
 0.5  1  2.5  6  10m

4. The damping ratio for a viscously damped spring mass system, governed by the relationship
d2 x dx
m 2 C  kx  F  t  , is given by
dt dt
c c c c
(A) (B) (C) (D)
mk 2 km km 2mk
Key: (B)
dx 2 dx c c
Exp: m 2
 c  k x  F t ;  
dt dt cc 2 km

d2 y
5. The differential equation  16y  0 for y  x  with the two boundary conditions
dx 2
dy dy
 1 and  1 has
dx x 0 dx x  
2

(A) no solution (B) exactly two solutions


(C) exactly one solution (D) infinitely many solutions
Key: (A)
d2 y dy dy
Exp:  16y  0 1  1
dx 2 dx x 0 dx x

2

 m 2  16  0
m  0  4i www. engi neer i
ngonyour
fi
nger
tips.
ooo
yc  c1cos 4x  c 2 sin 4x and y p  0
 y  c1 cos 4x  c 2 sin 4x
y'  x   4c1 sin 4x  4c2 cos 4x
1
y'  0   1  0  4c2  1  c2 
4
 1
y '    1  0  4c 2  1  c 2 
2 4
1 1
c 2  and both not possible
4 4
Hence there is no solution

6. Metric thread of 0.8 mm pitch is to be cut on a lathe. Pitch of the lead screw is 1.5 mm. If the
spindle rotates at 1500 rpm, the speed of rotation of the lead screw (rpm) will be _________
Key: 800 to 800
 ICP–Intensive Classroom Program  eGATE-Live Internet Based Classes DLP  TarGATE-All India Test Series
www.engineeringonyourfingertips.ooo
Leaders in GATE Preparations  65+ Centers across India
© All rights reserved by Gateforum Educational Services Pvt. Ltd. No part of this booklet may be reproduced or utilized in any form without the written permission.

More Notes Join us Telegram-: http//:t.me/allexammentor


www.engineeringonyourfingertips.ooo
|ME| GATE-2017-PAPER-I www.gateforum.com

1500  0.8
Exp: Speed of rotation of lead screw   800rpm
1.5

7. The molar specific heat at constant volume of an ideal gas is equal to 2.5 times the universal
gas constant (8.314 J/mol.K). When the temperature increases by 100K, the change in molar
specific enthalpy is _______________ J/mol.
Key: 2908 to 2911
Exp: C v  2.5R v where  R v  8.314 J mol.K 
T 100K
H  ?
H  Cp T
Cp  C v  R v
[Cp  3.5R v ]
So, H  3.5  8.314 100
[H  2909.9 J mol]

8. A particle of unit mass is moving on a plane. Its trajectory, in polar coordinates, is given by
r(t) = t2,   t   t, where t is time. The kinetic energy of the particle at time t = 2 is
(A) 4 (B) 12 (C) 16 (D) 24
Key: (C)
Exp: r = t 2 ;θ = t
1
K.E = mv 2  ? at t  2 sec
2
 m 1kg

V  r(t) ˆ  dr rˆ  t 2 1(t)
ˆ  2trˆ
dt
ˆ  2t(r)
V  t 2 (t) ˆ
www.
engi
neer
ingonyour
fi
nger
tips.
ooo
at t  2s
ˆ  4(r)
V  4(t) ˆ
V  16  16  32
1 1
K.E.  mv 2  1 32  16
2 2

9. The Poisson‟s ratio for a perfectly incompressible linear elastic material is


(A) 1 (B) 0.5 (C) 0 (D) infinity
Key: (B)

10. A heat pump absorbs 10 kW of heat from outside environment at 250 K while absorbing 15
kW of work. It delivers the heat to a room that must be kept warm at 300K. The Coefficient of
Performance (COP) of the heat pump is ___________.
Key: 1.66 to 1.70
 ICP–Intensive Classroom Program  eGATE-Live Internet Based Classes DLP  TarGATE-All India Test Series
www.engineeringonyourfingertips.ooo
Leaders in GATE Preparations  65+ Centers across India
© All rights reserved by Gateforum Educational Services Pvt. Ltd. No part of this booklet may be reproduced or utilized in any form without the written permission.

More Notes Join us Telegram-: http//:t.me/allexammentor


www.engineeringonyourfingertips.ooo
|ME| GATE-2017-PAPER-I www.gateforum.com

Exp:
300K

25kW  QH QH 25
COP    1.66
Wi/ p 15
H.P 15kW

10kW

250K

11. Which one of the following is NOT a rotating machine?


(A) Centrifugal pump (B) Gear pump
(C) Jet pump (D) Vane pump
Key: (C)
Exp: In the given options all the pumps have rotating machine elements except Jet pump.

12. Consider the schematic of a riveted lap joint subjected to tensile load F, as shown below. Let d
be the diameter of the rivets, and Sf be the maximum permissible tensile stress in the plates.
What should be the minimum value for the thickness of the plates to guard against tensile
failure of the plates? Assume the plates to be identical.

F F F 2F
(A) (B) (C) (D)
Sf  W  2d  Sf W Sf  W  d  Sf W
Key: (A)
F F F
Exp: Sf   Sf  ; t
A  Www2d
tw .
e ngi
neer i
Sn won
fg y ur
2do fi
nger
tips.
ooo
13. Water (density =1000 kg/m3) at ambient temperature flows through a horizontal pipe of
uniform cross section at the rate of 1 kg/s. If the pressure drop across the pipe is 100 kPa, the
minimum power required to pump the water across the pipe, in watts, is _______
Key: 100 to 100
Exp: given,
w 1000 kg m3

m 1kg s
P 10kPa

So, minimum power require  m gh f
  P 
 mg 
 g 
1  100 1000
 100Watts
1000

 ICP–Intensive Classroom Program  eGATE-Live Internet Based Classes DLP  TarGATE-All India Test Series
www.engineeringonyourfingertips.ooo
Leaders in GATE Preparations  65+ Centers across India
© All rights reserved by Gateforum Educational Services Pvt. Ltd. No part of this booklet may be reproduced or utilized in any form without the written permission.

More Notes Join us Telegram-: http//:t.me/allexammentor


www.engineeringonyourfingertips.ooo
|ME| GATE-2017-PAPER-I www.gateforum.com

14. For steady flow of a viscous incompressible fluid through a circular pipe of constant diameter,
the average velocity in the fully developed region is constant. Which one of the following
statements about the average velocity in the developing region is TRUE?
(A) It increases until the flow is fully developed.
(B) It is constant and is equal to the average velocity in the fully developed region.
(C) It decreases until the flow is fully developed.
(D) It is constant but always lower than the average velocity in the fully developed region.
Key: (B)
Exp: The average velocity in pipe flow always be same either for developing flow or fully
developed flow.

15. Cylindrical pins of diameter 150.020 mm are being produced on a machine. Statistical quality
control tests show a mean of 14.995 mm and standard deviation of 0.004mm. The process
capability index Cp is
(A) 0.833 (B) 1.667 (C) 3.333 (D) 3.750
Key: (B)
USL- LSL 15.02  14.98
Exp: Cp    1.666
6σ 6  0.004

2 0 1 
16. The product of Eigen values of the matrix P is P   4 3 3
0 2 1
(A) -6 (B) 2 (C) 6 (D) -2
Key: (B)
2 0 1 
Exp: P   4 3 3
0 2 1
We know that, product of eigen values of P = determinant of P
www.
engi
neer
ingonyour
fi
nger
tips.
ooo
 2  3  6   0  18   6  8  2

17. Match the processes with their characteristics.


Process Characteristics
P : Electrical Discharge machining 1. No residual stress
Q : Ultrasonic machining 2. Machining of electrically conductive
materials
R : Chemical machining 3. Machining of glass
S : Ion Beam Machining 4. Nano-machining
(A) P – 2, Q – 3, R – 1, S – 4 (B) P – 3, Q – 2, R – 1, S – 4
(C) P – 3, Q – 2, R – 4, S – 1 (D) P – 2, Q – 4, R – 3, S – 1
Key: (A)

 ICP–Intensive Classroom Program  eGATE-Live Internet Based Classes DLP  TarGATE-All India Test Series
www.engineeringonyourfingertips.ooo
Leaders in GATE Preparations  65+ Centers across India
© All rights reserved by Gateforum Educational Services Pvt. Ltd. No part of this booklet may be reproduced or utilized in any form without the written permission.

More Notes Join us Telegram-: http//:t.me/allexammentor


www.engineeringonyourfingertips.ooo
|ME| GATE-2017-PAPER-I www.gateforum.com

x 3  sin  x 
18. The Value of limx 0 is
x
(A) 0 (B) 3 (C) 1 (D) -1
Key: (D)

Exp: t
x 0
x 3  sin x
x x 0
 
 t x 2   t
 x 0 x 

sin x 
  0  1  1

19. In an arc welding process, welding speed is doubled. Assuming all other process parameters to
be constant, the cross sectional area of the weld bead will
(A) Increase by 25% (B) Increase by 50% (C) Reduce by 25% (D) Reduce by 50%
Key: (D)
Exp:  V.I  H m A.V
1
A
V
A 2 V1 V
 
A1 V2 2V
A1
A2 
2
By doubling welding speed, Area reduces by 50%

20. A six-face fair dice is rolled a large number of times. The mean value of the outcomes is
______.
Key: 3.5 to 3.5
Exp: The Probabilities corresponding to the outcomes are given below:

Face 1 2 3 4 5 6
Probability 1/6 1/6 1/6 1/6 1/6 1/6

mean  E  x    x.P  x 
1 1  6   2  16 ww w. engi neer ingonyour
3 1   4  1   5  1   6  1 
6 6 6 6
fi
nger
tips.
ooo

1 21
 1  2  3  4  5  6   3.5
6 6


21. Consider the two dimensional velocity field given by V  5  a1x  b1y  i   4  a 2 x  b2 y  j,
wher a1 , b1 , a 2 and b2 are constants. Which one of the following conditions needs to be
satisfied for the flow to be incompressible?
(A) a1  b1  0 (B) a1  b 2  0 (C) a 2  b2  0 (D) a 2  b1  0
Key: (B)

Exp: Given V   5  a1 x  b1 y  i   4  a 2 x  b 2 y  j
 u V
For Incompressible V  0; i.e.,  0
x y
a1  b 2  0

 ICP–Intensive Classroom Program  eGATE-Live Internet Based Classes DLP  TarGATE-All India Test Series
www.engineeringonyourfingertips.ooo
Leaders in GATE Preparations  65+ Centers across India
© All rights reserved by Gateforum Educational Services Pvt. Ltd. No part of this booklet may be reproduced or utilized in any form without the written permission.

More Notes Join us Telegram-: http//:t.me/allexammentor


www.engineeringonyourfingertips.ooo
|ME| GATE-2017-PAPER-I www.gateforum.com

22. Consider a beam with circular cross-section of diameter d. The ratio of the second moment of
area about the neutral axis to the section modulus of the area is.
d d
(A) (B) (C) d (D) d
2 2
Key: (A)
 4
d
I I 64 d
Exp: Z y  
y Z 
d3 2
32

23. Saturated steam at 100°C condenses on the outside of a tube. Cold fluid enters the tube at 20°
C and exists at 50°C. The value of the Log Mean Temperature Difference (LMTD) is
________°C.
Key: 63.5 to 64 S
o  i 80  50
Exp: LMTD=  100C 100C 
n  o i  n  80 50  i  50C  
LMTD  63.82C 50C  o  80C
20C 

24. In a metal forming operation when the material has just started yielding, the principal stresses
are 1  180 MPa, 2  100 MPa, 3  0. Following Von Mises criterion, the yield stress is
________ MPa.
Key: 245 to 246
Exp: As per Von-Mises criteria
 1  2    1  3    5  1   2m2
2 2 2

180  100    100  0    0  180   22m


2 2 2

m  245.76MPa
www.
engi
neer
ingonyour
fi
nger
tips.
ooo
25. In the engineering stress-strain curve for mild steel, the Ultimate Tensile Strength (UTS) refers
to
(A) Yield stress (B) Proportional limit
(C) Maximum stress (D) Fracture stress.
Key: (C)

Q. No. 26 to 55 Carry Two Marks Each

 u   u 
26. A parametric curve defined by x  cos   , y  sin   in the range 0  u  1 is rotated
 2   2 
about the X – axis by 360 degrees. Area of the surface generated is.

(A) (B)  (C) 2  (D) 4 
2
Key: (C)
 ICP–Intensive Classroom Program  eGATE-Live Internet Based Classes DLP  TarGATE-All India Test Series
www.engineeringonyourfingertips.ooo
Leaders in GATE Preparations  65+ Centers across India
© All rights reserved by Gateforum Educational Services Pvt. Ltd. No part of this booklet may be reproduced or utilized in any form without the written permission.

More Notes Join us Telegram-: http//:t.me/allexammentor


www.engineeringonyourfingertips.ooo
|ME| GATE-2017-PAPER-I www.gateforum.com

 u   u 
Exp: Given x  cos   , y  sin   0    1
 2   2 
dx   u 
 sin  
d 2  2 
dy  u
 cos
dx 2 2
We know that surface area when the curve revolved about X- axis of a parametric curve is
2 2
 u     u      u  
2 2
 dx   dy 
1 1
 2 y      du  2 sin    sin      cos    du
0  du   du  0  2   2  2   2  2 
u 
1

1
u  
2
u  1cos 2 
1
2  
 2 sin du  2   sin dx   
2
    cos  cos 0 
2

2 4 20 2    2 
0
2 
 0
 2 cos 0  1  2

27. Assume that the surface roughness profile is triangular as shown schematically in the figure. If
the peak to valley height is 20 m, The central line average surface roughness Ra (in m ) is

(A) 5 (B) 6.67 (B) 10 (D) 20


Key: (A)
R max 20
Exp: Ra    5m
4 4

28. A thin uniform rigid bar of length L and mass M is hinged at point O, located at a distance of
L
from one of its ends. The bar is further supported using springs, each of stiffness k, located
3
www. engi neer ingoM nyour f
inger tips.ooo
at the two ends. A particle of mass m  is fixed at one end of the bar, as shown in the
4
figure. For small rotations of the bar about O, the natural frequency of the systems is.

5k 5k 3k 3k
(A) (B) (C) (D)
M 2M 2M M
Key: (B)
Exp: Max moment of inertia of Rod.

 ICP–Intensive Classroom Program  eGATE-Live Internet Based Classes DLP  TarGATE-All India Test Series
www.engineeringonyourfingertips.ooo
Leaders in GATE Preparations  65+ Centers across India
© All rights reserved by Gateforum Educational Services Pvt. Ltd. No part of this booklet may be reproduced or utilized in any form without the written permission.

More Notes Join us Telegram-: http//:t.me/allexammentor


www.engineeringonyourfingertips.ooo
|ME| GATE-2017-PAPER-I www.gateforum.com

I0  Ic  mr 2 L
2 L
ML2  2L L  m
I0   M   3
12  3 2
ML2 ML2 ML2 0
I0    K K
12 36 9
Mass moment of inertia of particular mass
2
M  2L  ML2
I particular    
4  3  9
ML2 ML2 2(ML2 )
ITotal   
9 9 9
 M0  0
 L  L   2L  2L  
K      K      I  0
 3  3   3  3 
5L2
 2ML 2
   5L 2
 .k
9 5k
   K    0  n  
 9   9  2ML2 2M
9

29. A point mass of 100 kg is dropped onto a massless elastic bar (cross-sectional area = 100 mm2,
length = 1m, Young‟s moduls = 100 GPa) from a height H of 10mm as shown (Figure is not to
scale). If g = 10m/s2, the maximum compression of the elastic bar is _______ mm.
m  100kg
H  10mm

L  1m
Key: 1.50 to 1.52
www.
engi
neer
ingonyour
fi
nger
tips.
ooo
Exp: Given that
m  100kg, g  10m sec 2 , E  100GPa
H  10mm, L  1m  100mm,
A  100mm2
From the given figure, we can say that this is case of Impact loading,
We know that, stress due to Impact load is
2
P  P   2PhE 
I.L      
A  A   AL 
P  mg  100  10  1000N

 1000   2  1000  10  1  10 
2 5
1000
I.L    
    151.7745 N mm
2

100  100   100  1000 


I.L  L 151.7745  1000
Compression       1.5177 mm
E 1  105

 ICP–Intensive Classroom Program  eGATE-Live Internet Based Classes DLP  TarGATE-All India Test Series
www.engineeringonyourfingertips.ooo
Leaders in GATE Preparations  65+ Centers across India
© All rights reserved by Gateforum Educational Services Pvt. Ltd. No part of this booklet may be reproduced or utilized in any form without the written permission.

More Notes Join us Telegram-: http//:t.me/allexammentor


www.engineeringonyourfingertips.ooo
|ME| GATE-2017-PAPER-I www.gateforum.com

30. One kg of an ideal gas (gas constant, R = 400 J/kg.K; specific heat at constant volume,
c  1000J/kg.K) at 1 bar, and 300 K is contained in a sealed rigid cylinder. During an
adiabatic process, 100kJ of work is done on the system by a stirrer. The increase in entropy of
the system is _________ J/K.
Key: 286 to 288
Exp: Given that m  1kg, R  400 kJ kgK, CV  1000 J kgK
P1  1 bar, T1  300K
Since the gas is contained in a sealed rigid cylinder, and given that adiabatic process is done to
the system, means no heat is transferred from/to the system, Q = 0
And we know from first law of thermo dynamics, Adiabatic wall, Q  0
Q  du  W
0  mCV  T2  T1   100  103  du  mC V  T2  T1  
mCV  T2  T1   100  103 still Q work
given  100 kJ
1  1000  T2  300   100  103
100  103
T2   300  400K
1000
We know that the first law of thermodynamics can be written as
Tds  mC V dT  pdV
dT P  mRT 
ds  mC V  dv  PV  mRT  P  
T T  V 
dT mRT dT dV
ds  mC V  dV  mC V  mR
T rT T V
T  V 
Entropy increase  S2  S1   mCV n  2   mR n  2 
 T1   V1 
Since the above process is a constant volume process that is V2  V1
 400 
 S2  S1   1  1000   n    287.6821 J K
www. engi n 300
eer ingonyour f
inger
tips.
ooo

31. For an inline slider-crank mechanism, the lengths of the crank and connecting rod are 3m and
4m, respectively. At the instant when the connecting rod is perpendicular to the crank, if the
velocity of the slider is 1m/s, the magnitude of angular velocity (upto 3 decimal points
accuracy) of the crank is _________ radian/s.
Key: 0.26 to 0.27
Exp: Given that velocity of slider
VB  1m / sec A
Length of crank (OA) = 3m 4mm
Length of connecting rod (AB) = 4m 3mm
From the configuration diagram  B
A O 5mm
sin      53.130o C
5

 ICP–Intensive Classroom Program  eGATE-Live Internet Based Classes DLP  TarGATE-All India Test Series
www.engineeringonyourfingertips.ooo
Leaders in GATE Preparations  65+ Centers across India
© All rights reserved by Gateforum Educational Services Pvt. Ltd. No part of this booklet may be reproduced or utilized in any form without the written permission.

10

More Notes Join us Telegram-: http//:t.me/allexammentor


www.engineeringonyourfingertips.ooo
|ME| GATE-2017-PAPER-I www.gateforum.com

The velocity diagram for the above configuration diagramO,c


is 1m sec
Oa b
 From velocity diagram sin 53.13o 
Ob 53.130o
 Oa  1 sin 53.130o  0.8
 VA  Oa  OA  OA  0.8
a
0.8
Angular velocity of crank  OA    0.267 rad sec
3

32. In an epicyclic gear train, shown in the figure, the outer


ring gear is fixed, while the sun gear rotates
counterclockwise at 100rpm. Let the number of teeth on
the sun, planet and outer gears to be 50, 25, and 100,
respectively. The ratio of magnitudes of angular velocity
of the planet gear to the angular velocity of the carrier
arm is _________.

Key: 3 to 3
Exp: TS = 50
TP=25
TR=100

Arm
S.No Condition of motion Gear S Gear P Gear R
Gear A
Arm is fixed gear S Ts Ts
1 0 +1  
with +1 revolution TP TR
Arm is fixed Gear S Ts Ts
2 0 +x -x x
with +x revolution TP TR
Arm with + y
3 y y y y
revolution
Ts T
4 Total y s x
y x+y yx
www.
engi
neer
ingonyour
fi
nger
ti
Tp
Ps.
ooo TR

NS  x  y  100 ...(1)
50
NR  y  x0
100
y  0.5x ...  2 
x  0.5x  100
100
x  66.66rpm
1.5
y  33.34 rpm
 50 
N P  33.34   66.66    99.99rpm
 25 
NP 99.99
  3  approx 
N arm 33.32

 ICP–Intensive Classroom Program  eGATE-Live Internet Based Classes DLP  TarGATE-All India Test Series
www.engineeringonyourfingertips.ooo
Leaders in GATE Preparations  65+ Centers across India
© All rights reserved by Gateforum Educational Services Pvt. Ltd. No part of this booklet may be reproduced or utilized in any form without the written permission.

11

More Notes Join us Telegram-: http//:t.me/allexammentor


www.engineeringonyourfingertips.ooo
|ME| GATE-2017-PAPER-I www.gateforum.com

33. Moist air is treated as an ideal gas mixture of water vapor and dry air (molecular weight of air
= 28.84 and molecular weight of water = 18). At a location, the total pressure is 100 kPa, the
temperature is 30°C and the relative humidity is 55%. Given that the saturation pressure of
water at 30°C is 4246 Pa, the mass of water vapor per kg of dry air is _____________ grams.
Key: 14.7 to 15.1
Exp: Ma  28.84
M w 18
P 100kPa, T  30C, RH  55%,
Ps  4246Pa
Pv P
  0.55  v  Pv  2335.3Pa
Ps 4246
 P   2335.3 
 0.622  v   0.622   
 P  Pv   100000  2335.3 
 0.01487 kg of vapour kg of D.A
14.87 gm of vapour kg of D.A

34. Following data refers to the jobs (P, Q, R, S) which have arrived at a machine for scheduling.
The shortest possible average flow time is ___________ days.

Job Processing Time (days)


P 15
Q 9
R 22
S 12

Key: 31 (not matching with IIT key)


Exp: For shortest avg. flow time SPT rule is used
Jobw ww. engi
Sequence neer ingo
Processing nyouIn
Time r fi
nger
Out t
ips.
Flowo oo
Time
Q 9 0 9 9
S 12 9 21 21
P 15 21 36 36
R 22 36 58 58
9  21  36  58
Min Avg. Flow time   31 days
4

35. Two models, P and Q, of a product earn profits of Rs. 100 and Rs. 80 per piece, respectively.
Production times for P and Q are 5 hours and 3 hours, respectively, while the total production
time available is 150 hours. For a total batch size of 40, to maximize profit, the number of
units of P to be produced is ____________.
Key: 15 to 15
Exp: Let x1  No. of units of P
x 2  No. of units of Q

 ICP–Intensive Classroom Program  eGATE-Live Internet Based Classes DLP  TarGATE-All India Test Series
www.engineeringonyourfingertips.ooo
Leaders in GATE Preparations  65+ Centers across India
© All rights reserved by Gateforum Educational Services Pvt. Ltd. No part of this booklet may be reproduced or utilized in any form without the written permission.

12

More Notes Join us Telegram-: http//:t.me/allexammentor


www.engineeringonyourfingertips.ooo
|ME| GATE-2017-PAPER-I www.gateforum.com
x2
max. z  100 x1  80x 2  0,50 
5x1  3x 2  150
x1  x 2  40  0, 40  15, 25 
Z 0,40  Rs.3200
Z15,225  Rs.3500  max.profit  0,0 
x1
 30,0   40,0 
So, for maximum profit, No. of units of P produced is 15 units.

36. Circular arc on a part profile is being machined on a vertical CNC milling machine. CNC part
program using metric units with absolute dimensions is listed below:
--------------------------------
N60 G01 X 30 Y 55 Z – 5 F 50
N70 G02 X 50 Y 35 R 20
N80 G01 Z 5
--------------------------------
The coordinates of the centre of the circular arc are :
(A) (30, 55) (B) (50, 55) (C) (50, 35) (D) (30, 35)
Key: (D)
Exp: y

 30,56 

20

 50,35 
 30,35 

x
20

Centre of circular arc is (30, 35)


37. Two black surfaces, AB and BC, of lengths 5m and 6m,
respectively, are oriented as shown. Both surfaces extend
wwthird
infinitely into the w. en gi neer ing
dimension. onyo
Given ur
that f in ger
view tips.
factor ooo
F12=0.5, T1=800K, T2=600K, Tsurrounding=300K and Stefan
Boltzmann constant,   5.67 108 W /  m2 K 4  , the heat
transfer rate from Surface 2 to the surrounding environment is
____________ kW.
Key: 13.7 to 13.9
Exp: Given that two black surfaces „AB‟ and „BC‟

Length of AB = 5m, BC = 6 m A
And temperature of
Surface „1‟  TBC  T1   800o K Surface '1'
Surroundings'3'
Temperature of surface „2‟  TAB  T2  600 K  o

Temperature of surroundings  T3   300o K


B C

 ICP–Intensive Classroom Program  eGATE-Live Internet Based Classes DLP  TarGATE-All India Test Series
www.engineeringonyourfingertips.ooo
Leaders in GATE Preparations  65+ Centers across India
© All rights reserved by Gateforum Educational Services Pvt. Ltd. No part of this booklet may be reproduced or utilized in any form without the written permission.

13

More Notes Join us Telegram-: http//:t.me/allexammentor


www.engineeringonyourfingertips.ooo
|ME| GATE-2017-PAPER-I www.gateforum.com

  5.67  108 W / m 2 K 4
F12  0.5
W.K.T. F11  F22  0, sin ce they areflat surfaces
F21  F22  F23  1
A1F12  A 2 F21
  6  0.5    5  F21  Assume unit width for surfaces 
3
 F21   0.6
5
0.6  0  F23  1  F23  1  0.6  0.4
Using resistance concept we can draw as follows
Since surfaces are black and area of surrounding is large we can write  1 
E b1  J1 , E b2  J 2 , E b3  J 3  E b2  J 2   A 2 F23   E b3  J 3 
E  E b3   T2  T3 
4 4

Q 23  b2  2 3
 1  1
  A 2 F23 1 1
 A 2 F23 
 5.67  108   6004  300 4   5  0.4
A2 F21 A1F13

 13.778kW / metre
1
 1 0 1  E b1  J1
 2 2 

38. Consider the matrix P   0 1 0 .
 1 0 1 

 2 2 
Which one of the following statements about P is INCORRECT?
(A) Determinant of P is equal to 1.
(B) P is orthogonal.
(C) Inverse of P is equal to its transpose.
(D) All Eigen values of P are real numbers
www.
engi
neer
ingonyour
fi
nger
tips.
ooo
Key: ( D)
 1 0 1 
 2 2 

Exp: P  0 1 0 
 1 0 1 
 
 2 2 
1  1  1  1  1 1
P    0  0      1
2 2  2 2 2 2
 1 0 1  1 0 1 
 2    
 2 2 2  1 0 0 
P.P T  0 1 0  0 1 0   0 1 0 
  
1  0 0 1 
 1 0 1   1 0
 2 2   2 2 

 ICP–Intensive Classroom Program  eGATE-Live Internet Based Classes DLP  TarGATE-All India Test Series
www.engineeringonyourfingertips.ooo
Leaders in GATE Preparations  65+ Centers across India
© All rights reserved by Gateforum Educational Services Pvt. Ltd. No part of this booklet may be reproduced or utilized in any form without the written permission.

14

More Notes Join us Telegram-: http//:t.me/allexammentor


www.engineeringonyourfingertips.ooo
|ME| GATE-2017-PAPER-I www.gateforum.com

P is an orthogonal matrix
(A) Is correct  Inverse of P is its transpose only
(B) and (C) both are correct
(D) is incorrect

39. The Pressure ratio across a gas turbine (for air, specific heat at constant pressure,
c p  1040J / kg.K and ratio of specific heats,   1.4) is 10. If the inlet temperature to the
turbine is 1200K and the isentropic efficiency is 0.9, the gas temperature at turbine exit is
______ K.
Key: 675 to 684
Exp: Cp  1040 J kg.K , r 1.4
P2 P1  10, T3 1200K
is  0.9
T
P2
 Isentropic Expansion,
r 1 3 P1
 P3  r  T3  1200 2
      10  
0.4/1.4

 P4   T4  T4
41
T4  621.5K 4
1
T3  T 1
S
iso  4

T3  T4
So, T41  T3  iso  T3  T4   1200  09 1200  621.5 
T4  679.38K

40. An initially stress-free massless elastic beam of length L and circular cross-section with
diameter d (d << L) is held fixed between two walls as shown. The beam material has Young‟s
modulus E and coefficient of thermal expansion  .

www.
engi
neer
ingonyour
fi
nger
tips.
ooo

If the beam is slowly and uniformly heated, the temperature rise required to cause the beam to
buckle is proportional to
(A) d (B) d2 (C) d3 (D) d4
Key: (B)
EI
Exp: P  EA  T  
 2e
 1  I
T    
  A
 4 d
d L
 1 64
T   
   d2
4
T  d 2
 ICP–Intensive Classroom Program  eGATE-Live Internet Based Classes DLP  TarGATE-All India Test Series
www.engineeringonyourfingertips.ooo
Leaders in GATE Preparations  65+ Centers across India
© All rights reserved by Gateforum Educational Services Pvt. Ltd. No part of this booklet may be reproduced or utilized in any form without the written permission.

15

More Notes Join us Telegram-: http//:t.me/allexammentor


www.engineeringonyourfingertips.ooo
|ME| GATE-2017-PAPER-I www.gateforum.com

 
41. For the vector V  2yzi  3xzj  4xyk,
 the value of .   V is ____________
 
Key: 0 to 0

Exp: V  2yzi  3xzj  4xyk
we know that .   V   0 for any vector V

42. A 10 mm deep cylindrical cup with diameter of 15mm is drawn from a circular blank.
Neglecting the variation in the sheet thickness, the diameter (upto 2 decimal points accuracy)
of the blank is _________ mm.
Key: 28.71 to 28.73

Exp: D  d2  4dh  152  4 10 15  28.72 mm

43. A machine element has an ultimate strength  u  of 600 N/mm2, and endurance limit   en  of
250 N/mm2. The fatigue curve for the element on log-log plot is shown below. If the element
is to be designed for a finite of 10000 cycles, the maximum amplitude of a completely
reversed operating stress is _________ N/mm2.

Key: 370 to 390


A (0.8 u )
Exp: u  600MPa

en  250MPa B
e
N  10000 cycle
www.
engi
neer
ingonyour
fi
nger
tips.
ooo
log  0.8u   log  250  log  0.8 u   log   

36 34
log  480   log  250  log  480   log    103 104 106

3 1
3log  480   log  480   log  250  2log  480   log  250 
log     
3 3
max  386.19MPa

44. A sprue in a sand mould has a top diameter of 20mm and height of 200mm. The velocity of
the molten metal at the entry of the sprue is 0.5m/s. Assume acceleration due to gravity as 9.8
m/s2 and neglect all losses. If the mould is well ventilated, the velocity (upto 3 decimal points
accuracy) of the molten metal at the bottom of the sprue is ________ m/s.

Key: 2.04 to 2.07

 ICP–Intensive Classroom Program  eGATE-Live Internet Based Classes DLP  TarGATE-All India Test Series
www.engineeringonyourfingertips.ooo
Leaders in GATE Preparations  65+ Centers across India
© All rights reserved by Gateforum Educational Services Pvt. Ltd. No part of this booklet may be reproduced or utilized in any form without the written permission.

16

More Notes Join us Telegram-: http//:t.me/allexammentor


www.engineeringonyourfingertips.ooo
|ME| GATE-2017-PAPER-I www.gateforum.com

Exp: Apply Bernoulli‟s between (1) and (2)


P1 V12 P V2 1
  z1  2  2  z 2
 2g  2g
0.52 V22
 0.2  200 mm
2g 2g
V2  2.042 m s
2

45. Air contains 79% N2 and 21% O2 on a molar basis. Methane (CH4) is burned with 50% excess
air than required stoichiometrically. Assuming complete combustion of methane, the molar
percentage of N2 in the products is ________________
Key: 73 to 74
 79  79
Exp: Stoichiometric reaction CH4  2. O2  N2   2H2O  CO2  2   N2
 21  21
50% excess air
 79  79
CH 4  3. O 2  N 2   2H 2O  CO 2  3   N 2  O 2
 21  21
3  79
%N 2  21 100  73.83%
79
2 11 3
21

46. P(0,3), Q(0.5, 4), and R (1,5) are three points on the curve defined by f(x). Numerical
integration is carried out using both Trapezoidal rule and Simpson‟s rule within limits x = 0
and x =1 for the curve. The difference between the two results will be.
(A) 0 (B) 0.25 (C) 0.5 (D) 1
Key: (A)
Exp: Let x 0 0.5 1
www.
engi
neer
ingonyour
fi
nger
tips.
ooo
y 3 4 5
Trapezoidial rule
1
0.5 0.5
 f  x  dx 
0
2 
 3  5   2  4   
2
 16  4

Simpsons rule
1
0.5 0.5
 f  x  dx 
0
3
 3  5   0  4  4   
3
 24  4

Difference  0

47. Heat is generated uniformly in a long solid cylindrical rod (diameter = 10mm) at the rate of
4×107 W/m3. The thermal conductivity of the rod material is 25W/m.K. Under steady state
conditions, the temperature difference between the centre and the surface of the rod is
_________ °C.
Key: 10 to 10

 ICP–Intensive Classroom Program  eGATE-Live Internet Based Classes DLP  TarGATE-All India Test Series
www.engineeringonyourfingertips.ooo
Leaders in GATE Preparations  65+ Centers across India
© All rights reserved by Gateforum Educational Services Pvt. Ltd. No part of this booklet may be reproduced or utilized in any form without the written permission.

17

More Notes Join us Telegram-: http//:t.me/allexammentor


www.engineeringonyourfingertips.ooo
|ME| GATE-2017-PAPER-I www.gateforum.com

Exp: Given that, heat is generated uniformly i.e., g  4 107 W/ m3

Diameter at the rod (d) = 10 mm


Thermal conductivity of the rod (K) = 25 W/mK

dT To
0
dr
Temperatureprofile
TS T3
g  4 107 W / m3
K  25W / m3K
d  10mm

W.K.T for steady state, with internal heat generation, the conduction equation will be,
1 d  dT  g
 r.    0
r dr  dr  k
d  dT  gr
 r  0
dr  dr  K
After Integration once
dT gr 2 dT gr  c
r.   C1    1  (1)
dr 2K dr 2K r
After second Integration
 2
gr
Tr   C1n  r   C 2  (2)
4K
dT
Substituting boundary condition of  0at r  0in eq(1)
dr
That gives C1  0
 2
gr www. engi neer ingonyour f
inger
tips.
ooo
 Tr   C 2 , substitute at r =0, T(r) = To
4k
 To   o   C2  C2  To
 2
gr
T  r    To
4K
10
Substitute r  ro   5mm  0.005m andT  r   TS
2
The above equation will become
 2
gr
TS  o  T o
4K
4  107  0.0052
 To  Ts   10o C
4  25

 ICP–Intensive Classroom Program  eGATE-Live Internet Based Classes DLP  TarGATE-All India Test Series
www.engineeringonyourfingertips.ooo
Leaders in GATE Preparations  65+ Centers across India
© All rights reserved by Gateforum Educational Services Pvt. Ltd. No part of this booklet may be reproduced or utilized in any form without the written permission.

18

More Notes Join us Telegram-: http//:t.me/allexammentor


www.engineeringonyourfingertips.ooo
|ME| GATE-2017-PAPER-I www.gateforum.com

48. Two disks A and B with identical mass (m) and radius (R) are initially at rest. They roll down
from the top of identical inclined planes without slipping. Disk A has all of its mass
concentrated at the rim, while Disk B has its mass uniformly distributed. At the bottom of the
plane, the ratio of velocity of the center of disk A to the velocity of the center of disk B is.
3 3
(A) (B) (C) 1 (D) 2
4 2
Key: (A)
Exp: IA  MR 2
MR 2
IB  A,B
2
m

P.E   K.E T  K.E R A   K.E T  K.E R B
1 1 1 1 H
 MVA2  IA2  MVB2  IB2
2 2 2 2
1 1 1 1 MR 2 2
 MVA2  MR 2 2A  MVB2  B
2 2 2 2 2
VB2
VA2  VA2  VB2 
2
3 V 3
2VA2  VB2  A 
2 VB 4

49. A block of length 200mm is machined by a slab milling cutter 34mm in diameter. The depth of
cut and table feed are set at 2mm and 18mm/minute, respectively. Considering the approach
and the over travel of the cutter to be same, the minimum estimated machining time per pass is
_____________ minutes.
Key: 12 to 12
2  Dd  d 2  L 2  (34  2)  2  200
2
Exp: Milling Time   12
f 18
Where, D  Dia of cutter  mm  , d  depth of cut  mm 
www
L  length f  feed .
en
mm gin
min eer
ingonyour
fi
nger
tips.
ooo

50. A horizontal bar, fixed at one end (x = 0), has a length of 1 m, and cross-sectional area of
100mm2. Its elastic modulus varies along its length as given by E(x) = 100e -x GPa, Where x is
the length coordinate (in m) along the axis of the bar. An axial tensile load of 10 kN is applied
at the free end (x=1). The axial displacement of the free end is _______ mm.
Key: 1.70 to 1.72
Exp: Given that

P  10kN  10  103 N, A  100mm 2 Stripof lengthdx


x
100  e  10 9
x
E  x   100e x Gpa   105  e x dx
106
P  10kN
P  dx 
Change in the length of small strip   x0 x 1
AE x
L  1mm

 ICP–Intensive Classroom Program  eGATE-Live Internet Based Classes DLP  TarGATE-All India Test Series
www.engineeringonyourfingertips.ooo
Leaders in GATE Preparations  65+ Centers across India
© All rights reserved by Gateforum Educational Services Pvt. Ltd. No part of this booklet may be reproduced or utilized in any form without the written permission.

19

More Notes Join us Telegram-: http//:t.me/allexammentor


www.engineeringonyourfingertips.ooo
|ME| GATE-2017-PAPER-I www.gateforum.com

L
Pdx
Total change in the length of the bar     
0
AE x
L L L 1
P dx P dx P P P
 x
 5  x
 5 
e x dx  5 
e x dx  e1  1
5 
0
A 100e A  10 0 e A  10 0 A 10 0 A 10
10  103
  2.7183  1 1000  1.7183mm
100  105

51. Consider steady flow of an incompressible fluid through two long and straight pipes of
diameters d1 and d2 arranged in series. Both pipes are of equal length and the flow is turbulent
in both pipes. The friction factor for turbulent flow though pipes is of the form, f  K(Re) n
where K and n are known positive constants and Re is the Reynolds number. Neglecting minor
 P 
losses, the ratio of the frictional pressure drop in pipe 1 to that in pipe 2,  1  , is given by
 P2 
 5 n  5  3 n   5 n 
d  d  d  d 
(A)  2  (B)  2  (C)  2  (D)  2 
 d1   d1   d1   d1 
Key: (A)
 f L Q2  12d 52g  f1  d 2   Re1   d 2 
5 n 5
P1  64 
Exp:   1 15  2 
    n   ......(1)  f  
P2  12d1 g  f 2 L2 Q  f 2  d1   Re2   d1   Re 

Vd   4Q 
Re  & Q  AV  d 2 V   V  2 
 4  d 
4Q d 4Q
So, Re  2  
d  d
d1 d2
5 L1  L2
1 P d n  d 
Re  From(1) 1  1 n  2 
d P2 d 2  d1 
5 n
P1  d 2  L1 L2
 
P2  d1 
www.
engi
neer
ingonyour
fi
nger
tips.
ooo
52. The velocity profile inside the boundary layer for flow over a flat plate is given as
u  y
 sin   , where U is the free stream velocity and  is the local boundary layer
U 2 
*
thickness. If  * is the local displacement thickness, the value of is

2 2 2
(A) (B) 1  (C) 1+ (D) 0
  
Key: (B)
U  y 
Exp:  sin  
U  2 
displacement thickness
  y 
*   1  U U  dy  1  sin  dy
0
 2 

 ICP–Intensive Classroom Program  eGATE-Live Internet Based Classes DLP  TarGATE-All India Test Series
www.engineeringonyourfingertips.ooo
Leaders in GATE Preparations  65+ Centers across India
© All rights reserved by Gateforum Educational Services Pvt. Ltd. No part of this booklet may be reproduced or utilized in any form without the written permission.

20

More Notes Join us Telegram-: http//:t.me/allexammentor


www.engineeringonyourfingertips.ooo
|ME| GATE-2017-PAPER-I www.gateforum.com


cos  y 2   0
  
*

  2 
2
*    cos  2  cos 0

 * 2 
     
 
   1  2  
*


53. For a steady flow, the velocity field is V   x 2  3y  i   2xy  j. The magnitude of the
acceleration of a particle at (1, -1) is
(A) 2 (B) 1 (C) 2 5 (D) 0
Key: (C)

Exp: V   x 2  3y  i   2xy  j
U   x 2 2 3y
U   x  3y
V  2xy
V  2xy
U U
a x  U U V U   x 2 2 3y   2x    2xy  3  2x 3  6xy  6xy  2x 3 3
ax  U  x V y    x  3y   2x    2xy  3  2x  6xy  6xy  2x
3

x y
V V
a y  U V V V   x 2 2 3y   2y    2xy  2x   2x 2 y2  6y 2 2 4x 2 y2  2x 2 y2  6y 2 2
a y  Ux  Vy    x  3y   2y    2xy  2x   2x y  6y  4x y  2x y  6y
x y
but x  1, y  1
but x  1, y  1
ax  2
ax  2 2
a y  2 1 2 1  6  1 2 6  2  4
2

a y  2 1  1  6  1  6  2  4


a  a 2x  a 2y  4  16  20
a  a 2x  a 2y  4  16  20
a2 5
a2 5

www.
engi
neer
ingonyour
fi
nger
tips.
ooo

54. Two cutting tools with tool life equations given below are being compared:
Tool 1: VT0.1=150
Tool 2: VT0.3=300
Where V is cutting speed in m/minute and T is tool life in minutes. The breakeven cutting
speed beyond which Tool 2 will have a higher tool life is ________ m/minute.
Key: 105 to 107
Exp: At Breakeven point
T1  T2
1 0.1 1 0.3
 150   300 
   
 V   V 
V  106.121 m min

 ICP–Intensive Classroom Program  eGATE-Live Internet Based Classes DLP  TarGATE-All India Test Series
www.engineeringonyourfingertips.ooo
Leaders in GATE Preparations  65+ Centers across India
© All rights reserved by Gateforum Educational Services Pvt. Ltd. No part of this booklet may be reproduced or utilized in any form without the written permission.

21

More Notes Join us Telegram-: http//:t.me/allexammentor


www.engineeringonyourfingertips.ooo
|ME| GATE-2017-PAPER-I www.gateforum.com

55. A rectangular region in a solid is in a state of plane strain. The (x,y) coordinates of the corners
of the under deformed rectangle are given by P(0,0), Q (4,0), S (0,3). The rectangle is
subjected to uniform strains,  xx  0.001,  yy  0.002,  xy  0.003. The deformed length of the
elongated diagonal, up to three decimal places, is _________ units.
Key: 5.013 to 5.015
Exp: Given that

 xx  0.001,  yy  0.002
rxy  0.003

Length of the diagonal  PR   42  32  5m


y
 yy
S  0,3 R  4,3 

5mm  xx
 xx rxy
x
P  0,0  Q  4,0 
3  yy
tan 1  
4
To find the diagonal (PR) strain, the direction of the plane angle from the +ve x-axis will from
„R‟ towards „P‟
  ?
Where   180  tan 1  3 / 4   216.87o
 xx   yy  xx   yy rxy
 216.87o   cos 2 
sin 2
2 2 2
0.001  0.002 0.001  0.002
sin  2  216.87 o 
0.003
  cos  2  216.87  
2 2 2
 0.0015  1.4  104  1.44  103
www.
engi
neer
ingonyour
fi
nger
tips.
ooo
 2.8  103
Elongation of the diagonal = 216.87  5  2.8 103 •5
  0.014
Defined length of diagonal  5  0.014  5.014

 ICP–Intensive Classroom Program  eGATE-Live Internet Based Classes DLP  TarGATE-All India Test Series
www.engineeringonyourfingertips.ooo
Leaders in GATE Preparations  65+ Centers across India
© All rights reserved by Gateforum Educational Services Pvt. Ltd. No part of this booklet may be reproduced or utilized in any form without the written permission.

22

More Notes Join us Telegram-: http//:t.me/allexammentor


www.engineeringonyourfingertips.ooo
|ME| GATE-2017-PAPER-I www.gateforum.com

General Aptitude
Q. No. 1 - 5 Carry One Mark Each
1. A right – angled cone (with base radius 5cm and height 12cm), as shown in the figure below,
is rolled on the ground keeping the point P fixed until the point Q (at the base of the cone, as
shown) touches the ground again.

By what angle (in radians) about P does the cone travel?


5 5 24 10
(A) (B) (C) (D)
12 24 5 13
Key: (D)
Exp: L  52  122 13cm 
L
Circumference of base circle = length of arc QR
R P
(2r)  R (R  Slant height of the Cone  13 cm)
(2  5) 13
10 R

13
2. In a company with 100 employees, 45 earn Rs. 20,000 per month, 25 earn Rs. 30,000, 20 earn
Rs. 40,000,8 earn Rs. 60,000, and 2 earn Rs. 150,000. The median of the salaries is
(A) Rs. 20,000 (B) Rs.30,000 (C) Rs. 32,300 (D) Rs. 40,000
Key: (B)
Exp: All the values put either in ascending or descending order first.
Now number of observations equal to 100 [even]
The median of these values = Avg of two middle most observations.
50 th observation  51st observation 30,000  30,000
   30,000
ww2 w. engi neer i
ngonyo 2ur finger ti
ps. ooo
3. As the two speakers became increasingly agitated, the debate became __________.
(A) lukewarm (B) poetic (C) forgiving (D) heated
Key: (D)

4. P,Q, and R talk about S‟s car collection. P states that S has at least 3 cars. Q believes that S has
less than 3 cars. R indicates that to his knowledge, S has at least one Car. Only one of P, Q and
R is right the number cars owned by S is.
(A) 0 (B) 1 (C) 3 (D) Cannot be determined
Key: (A)
Exp: P States that S has atleast 3 cars, i.e.,  3
Q believes that S has less than 3 cars, i.e.,  3
R indicates that S has atleast one car  1
P‟s and Q‟s statements are exactly opposite in nature and R‟s statement is proportional to P‟s
statement.

 ICP–Intensive Classroom Program  eGATE-Live Internet Based Classes DLP  TarGATE-All India Test Series
www.engineeringonyourfingertips.ooo
Leaders in GATE Preparations  65+ Centers across India
© All rights reserved by Gateforum Educational Services Pvt. Ltd. No part of this booklet may be reproduced or utilized in any form without the written permission.

23

More Notes Join us Telegram-: http//:t.me/allexammentor


www.engineeringonyourfingertips.ooo
|ME| GATE-2017-PAPER-I www.gateforum.com

From the given data, only one person statement is right as it mean that two persons statements
are wrong, i.e., P and R wrong when S has zero cars.

5. He was one of my best __________ and I felt his loss _________.


(A) friend, keenly (B) friends, keen (C) friend, keener (D) friends, keenly
Key: (D)
Q. No. 6- 10 Carry Two Marks Each

6. Two very famous sportsmen Mark and Steve happened to be brothers, and played for country K.
Mark teased James, an opponent from country E, “There is no way you are good enough to play
for your country.‟‟ James replied, “Maybe not, but at least I am the best player in my own family.”
Which one of the following can be inferred from this conversation?
(A) Mark was known to play better than James
(B) Steve was known to play better than Mark
(C) James and Steve were good friends
(D) James played better than Steve
Key: (B)

7. “Here, throughout the early 1820s, Stuart continued to fight his losing battle to allow his
sepoys to wear their caste-marks and their own choice of facial hair on parade, being again
reprimanded by the commander-in-chief. His retort that „A stronger instance than this of
European prejudice with relation to this country has never come under my observations‟ had
no effect on his superiors.”
According to this paragraph, which of the statements below is most accurate?
(A) Stuart‟s commander – in chief was moved by this demonstration of his prejudice.
(B) The Europeans were accommodating of the sepoys‟ desire to wear their caste – marks.
(C) Stuart‟s losing battle‟ refers to his inability to succeed in enabling sepoys to wear caste-
marks.
(D) The commander– in – Chief was exempt from the European preiudice that dictated how
the sepoys were to dress.
www.
engi
neer
ingonyour
fi
nger
tips.
ooo
Key: (C)

8. The growth of bacteria (lactobacillus) in milk leads to curd formation. A minimum bacterial
population density of 0.8(in suitable units) is needed to form curd. In the graph below, the
population density of lactobacillus in 1 litre of milk is plotted as a function of time, at two
different temperatures, 25°C and 37°C.

Consider the following statements based on the data shown above:


 ICP–Intensive Classroom Program  eGATE-Live Internet Based Classes DLP  TarGATE-All India Test Series
www.engineeringonyourfingertips.ooo
Leaders in GATE Preparations  65+ Centers across India
© All rights reserved by Gateforum Educational Services Pvt. Ltd. No part of this booklet may be reproduced or utilized in any form without the written permission.

24

More Notes Join us Telegram-: http//:t.me/allexammentor


www.engineeringonyourfingertips.ooo
|ME| GATE-2017-PAPER-I www.gateforum.com

(i) The growth in bacterial population stops earlier at 37°C as compared to 25°C
(ii) The time taken for curd formation at 25°C is twice the time taken at 37°C
Which one of the following options is correct?
(A) Only i (B) only ii (C) Both i and ii (D) Neither i nor ii
Key: (A)
Exp: From the graph, Statement (i) is correct,
The time taken for curd formation @25o C= 120 min,
the time taken for curd formation @ 37o C= 80 min, hence (ii) is incorrect.

9. Let S1 be the plane figure consisting of the points (x,y) given by the inequalities x  1  2 and
y  2  3. Let S2 be the plane figure given by the inequalities x  y  2, y 1, and x  3 Let
S be the union of S1 and S2. The area of S is.
(A) 26 (B) 28 (C) 32 (D) 34
Key: (C)

10. What is the sum of the missing digits in the subtraction problem below?
5_ _ _ _
4 8 _ 8 9
1111
(A) 8 (B) 10 (C) 11 (D) Cannot be determined
Key: (D)

www.
engi
neer
ingonyour
fi
nger
tips.
ooo

 ICP–Intensive Classroom Program  eGATE-Live Internet Based Classes DLP  TarGATE-All India Test Series
www.engineeringonyourfingertips.ooo
Leaders in GATE Preparations  65+ Centers across India
© All rights reserved by Gateforum Educational Services Pvt. Ltd. No part of this booklet may be reproduced or utilized in any form without the written permission.

25

More Notes Join us Telegram-: http//:t.me/allexammentor


www.engineeringonyourfingertips.ooo
|ME| GATE-2017-PAPER-II www.gateforum.com

Mechanical Engineering
Q. No. 1 to 25 Carry One Mark Each

1. A mass m of a perfect gas at pressure p1 and volume V1 undergoes an isothermal process. The
final pressure is p2 and volume is V2. The work done on the system is considered positive. If R
is the gas constant and T is the temperature, then the work done in the process is
V2 p1 V2 p2
(A) p1V1 ln (B) p1V1 ln (C) RT ln (D) mRT ln
V1 p2 V1 p1
Key: (B)
Exp: Isothermal work done, W   pdV
For isothermal, pV= C
p1V1  pV  C  constant 
V2
dV
so, W  p1V1 
V1
V
V  p 
W  p1V1 n  2   p1V1 n  1  ( p1V1  p 2 V2 )
 V1   p2 

2. Which one of the following statements is TRUE for the ultrasonic machining (USM) process?
(A) In USM, the tool vibrates at subsonic frequency.
(B) USM does not employ magnetostrictive transducer.
(C) USM is an excellent process for machining ductile materials.
(D) USM often uses a slurry comprising abrasive-particles and water.
Key: (D)

3. The standard deviation of linear dimensions P and Q are 3 m and 4 m, respectively. When
assembled, the standard deviation (in m ) of the resulting linear dimension (P+Q) is ________
Key: 5 to 5 www.
engi
neer
ingonyour
fi
nger
tips.
ooo
Exp: Given that
Standard deviate of P is 3 m  Variance of P is 9 m
Standard deviation of Q is 4 m  Variance of Q is 16 m
Variance of P + Q = Var (P+Q) = Variance P + Variance Q
= 9 + 16= 25
 Standard deviation of P + Q = + Variance = + 25 = 5

4. The emissive power of a blackbody is P. If its absolute temperature is doubled, the emissive
power becomes.
(A) 2P (B) 4P (C) 8P (D) 16P
Key: (D)
Exp: Emissive power of black body  E b   T 4

Given  E b 1  P  T 4

 ICP–Intensive Classroom Program  eGATE-Live Internet Based Classes DLP  TarGATE-All India Test Series
www.engineeringonyourfingertips.ooo
Leaders in GATE Preparations  65+ Centers across India
© All rights reserved by Gateforum Educational Services Pvt. Ltd. No part of this booklet may be reproduced or utilized in any form without the written permission.

More Notes Join us Telegram-: http//:t.me/allexammentor


www.engineeringonyourfingertips.ooo
|ME| GATE-2017-PAPER-II www.gateforum.com

now T  2T
 Eb 2    2T   16T 4   E b 2  16P
4

5. The state of stress at a point is x   y  z  xz  zx   yz  zy  0 and  xy   yx  50MPa .


The maximum normal stress (in MPa) at that point is _________
Key: 49.9 to 50.1

 0,  
Exp: It is a pure torsion case in 2D
xy   yx  50MPa
xy

 for pure Torsion, 


xy   yx  1
0,   xy
1
So maximum Normal stress 1  50MPa

6. The determinant of a 2×2 matrix is 50. If one eigenvalue of the matrix is 10, the other
eigenvalue is ___________
Key: 5 to 5
Exp: Given that det of 2×2 Matrix is 50 and are Eigen Value is 10.
 Other Eigen value is 5  det  product of eigenvalues 

7. Which one of the following statement is TRUE?


(A) Both Pelton and Francis turbines are impulse turbines.
(B) Francis turbine is a reaction turbine but Kaplan turbine is an impulse turbine.
(C) Francis turbine is an axial – flow reaction turbine.
(D) Kaplan turbine is an axial – flow reaction turbine.
Key: (D)

8. Two coins are tossed simultaneously. The probability (upto two decimal points accuracy) of
ww
getting at least one w.
head e
isngineer i
ngonyour
____________ fi nger tips. ooo
Key: 0.75 to 0.75
Exp: Total No of outcomes when two coins are tossed is 4 and sample space
S  HH, HT,TH,TT
Favorable out comes for existence of at least one head are HH, HT, TH.
3
Required probability =  0.75
4

9. A cantilever beam of length L and flexural modulus EI is subjected to a point load P at the free
end. The elastic strain energy stored in the beam due to bending (neglecting transverse shear)
P 2 L3 P 2 L3 PL3 PL3
(A) (B) (C) (D)
6EI 3EI 3EI 6EI
Key: (A)

 ICP–Intensive Classroom Program  eGATE-Live Internet Based Classes DLP  TarGATE-All India Test Series
www.engineeringonyourfingertips.ooo
Leaders in GATE Preparations  65+ Centers across India
© All rights reserved by Gateforum Educational Services Pvt. Ltd. No part of this booklet may be reproduced or utilized in any form without the written permission.

More Notes Join us Telegram-: http//:t.me/allexammentor


www.engineeringonyourfingertips.ooo
|ME| GATE-2017-PAPER-II www.gateforum.com

Exp: M x   P.x
x P
L 2 2 2
M dx P x dx
U   x

0
2EI 2EI L
x x
P 2 L3
U
6EI

10. It is desired to make a product having T-shaped cross-section from a rectangular aluminium
block. Which one of the following processes is expected to provide the highest strength of the
product?
(A) Welding (B) Casting (C) Metal Forming (D) Machining
Key: (C)

11. The heat loss from a fin is 6W. The effectiveness and efficiency of the fin are 3 and 0.75,
respectively. The heat loss (in W) from the fin, keeping the entire fin surface at base
temperature, is __________.
Key: 7.9 to 8.1
Exp: Given QLoss  6W
  3,   0.75
Q Qloss
  act 
Q max Heat loss keep entire fin as base temperature
6
Q max   8W
0.75

12. For a single server with Poisson arrival and exponential service time, the arrival rate is 12 per
hour. Which one of the following service rates will provide a steady state finite queue length?
(A) 6 per hour (B) 10 per hour (C) 12 per hour (D) 24 per hour
Key: (D)
Exp:   12 / hour
For steady state finite queue length
 www.
engi
neer
ingonyour
fi
nger
tips.
ooo
So,   24 / hour

13. For the stability of a floating body the


(A) centre of buoyancy must coincide with the centre of gravity
(B) centre of buoyancy must be above the centre of gravity
(C) centre of gravity must be above the centre of buoyancy
(D) metacentre must be above the centre of gravity
Key: (D)
Exp: Stability of floating body is measure with the help of Meta center.
Floating body to be stable, Meta center must be above C.G
I
 MG   BG ,

For stability MG > 0

 ICP–Intensive Classroom Program  eGATE-Live Internet Based Classes DLP  TarGATE-All India Test Series
www.engineeringonyourfingertips.ooo
Leaders in GATE Preparations  65+ Centers across India
© All rights reserved by Gateforum Educational Services Pvt. Ltd. No part of this booklet may be reproduced or utilized in any form without the written permission.

More Notes Join us Telegram-: http//:t.me/allexammentor


www.engineeringonyourfingertips.ooo
|ME| GATE-2017-PAPER-II www.gateforum.com

14. The divergence of the vector yi  xj __________


Key: 0 to 0

Exp: Let F   yi  xj
  
divergence of F    y    x   0
x y

15. For a loaded cantilever beam of uniform cross-section, the bending moment (in N.mm) along
the length is M(x) = 5x2+10x, where x is the distance (in mm) measured from the free end of
the beam. The magnitude of shear force (in N) in the cross-section at x =10 mm is ________.
Key: 110 to 110
d
Exp:  S.F  (B.M)
dx
S.F   5x 2  10x 
d
dx
S.f  10x  10   S.F  x 10mm  10  10   10  110N

16. A sample of 15 data is a follows: 17, 18, 17, 17, 13, 18, 5, 5, 6, 7, 8, 9, 20, 17, 3. The mode of
the data is
(A) 4 (B) 13 (C) 17 (D) 20
Key: (C)
Exp: We know that mode is the value of the data which occurred most of
17 is mode

17. If a mass of moist air contained in a closed metallic vessel is heated, then its
(A) relative humidity decreases (B) relative humidity increases
(C) specific humidity increases (D) specific humidity decreases
Key: (A)
Exp: Given that mass of moist air contained in a closed metallic vessel is heated, means its specific
humidity   is w ww. engi
constant. neer ingonyour finger tips. ooo
So, from the psychometric chart,

1
Relative humidity 2
    100%

1 2

DBT
We can say that at constant specific humidity, as temperature increases relative humidity
decreases i.e., 2  1
So, Answer is (A)

 ICP–Intensive Classroom Program  eGATE-Live Internet Based Classes DLP  TarGATE-All India Test Series
www.engineeringonyourfingertips.ooo
Leaders in GATE Preparations  65+ Centers across India
© All rights reserved by Gateforum Educational Services Pvt. Ltd. No part of this booklet may be reproduced or utilized in any form without the written permission.

More Notes Join us Telegram-: http//:t.me/allexammentor


www.engineeringonyourfingertips.ooo
|ME| GATE-2017-PAPER-II www.gateforum.com

18. In a slider-crank mechanism, the lengths of the crank and the connecting rod are 100mm and
160mm, respectively. The crank is rotating with an angular velocity of 10 radian/s counter-
clockwise. The magnitude of linear velocity (in m/s) of the piston at the instant corresponding
to the configuration shown in the figure is _____________

Key: 0.99 to 1.01


 sin 2 
Exp: V  r sin  
 n 
if   90
 sin180o 
V  r  sin 90    0.1  1 sin 90   1m sec
 2n 

19. A machine component made of a ductile material is subjected to a variable loading with
min  50 MPa and max  50 MPa. If the corrected endurance limit and the yield strength for
the material are 'e  100 MPa and  y  300MPa, the factor of safety is __________
Key: 1.99 to 2.01
Exp:  max  50MPa
 min   50MPa
 y  300MPa
e  100MPa
max  min 50  50
max   0
2 2
  min 50   50 
 v  max   50
2 2
max  v 1 www. engi neer i
ngonyour
fi
nger
tips.
ooo
 
 yt e F.S
0 50 1
 
300 100 F.S
F.S  2

20. The crystal structure of aluminium is


(A) body-centred cubic (B) face-centred cubic
(C) close-packed hexagonal (D) body-centred tetragonal
Key: (B)

21. A steel bar is held by two fixed supports as shown in the figure and is subjected to an increases
of temperature T  100C. and 200GPa, respectively, the magnitude of thermal stress (in
MPa) induced in the bar is __________.

 ICP–Intensive Classroom Program  eGATE-Live Internet Based Classes DLP  TarGATE-All India Test Series
www.engineeringonyourfingertips.ooo
Leaders in GATE Preparations  65+ Centers across India
© All rights reserved by Gateforum Educational Services Pvt. Ltd. No part of this booklet may be reproduced or utilized in any form without the written permission.

More Notes Join us Telegram-: http//:t.me/allexammentor


www.engineeringonyourfingertips.ooo
|ME| GATE-2017-PAPER-II www.gateforum.com

Key: 218 to 222


Exp: Thermal Stress,
Th  ET  200  103  11 106  100  220MPa

22. The Laplace transform of tet is


s 1 1 s
(A) (B) (C) (D)
 s  1  s  1  s  1 s 1
2 2 2

Key: (B)
 Leat f  t   F  s  a  
Lte  
1
Exp: t
;  
 s  1  where F  s   Lf  t  
2

23. Consider a laminar flow at zero incidence over a flat plate. The shear stress at the wall is
denoted by  w . The axial positions x1 and x2 on the plate are measured from the leading edge
in the direction of flow. If x2 > x1, then
(A) w x1
 w x2
 0 (B) w x1
 w x2
 0 (C) w x1
 w x2
(D) w x1
 w x2

Key: (C)
Exp:  w 1   w 2
 u   u   u 
     w   
 y  
1  y
y 0  y 0 
2  y y 0 

24. A mass m is attached to two identical springs having spring constant k as shown in the figure.
The natural frequency  of this single degree of freedom system is
2k k
(A) (B)
m m

k www.
engi
neer
4k ingonyour
fi
nger
tips.
ooo
(C) (D)
2m m

Key: (A)
Exp: Equivalent stiffness keq = k + k = 2k
k eq 2k
Natural frequency,  n    n 
m m

25. Given the atomic weight of Fe is 56 and that of C is 12, the weight percentage of carbon in
cementite (Fe3C) is _________.
Key: 6.3 to 7.0
12
Exp: Percentage of carbon by weight in cementite =  100  6.67%
56  3  12

 ICP–Intensive Classroom Program  eGATE-Live Internet Based Classes DLP  TarGATE-All India Test Series
www.engineeringonyourfingertips.ooo
Leaders in GATE Preparations  65+ Centers across India
© All rights reserved by Gateforum Educational Services Pvt. Ltd. No part of this booklet may be reproduced or utilized in any form without the written permission.

More Notes Join us Telegram-: http//:t.me/allexammentor


www.engineeringonyourfingertips.ooo
|ME| GATE-2017-PAPER-II www.gateforum.com

Q. No. 26 to 55 Carry Two Marks Each


26. In an orthogonal machining with a tool of 9° orthogonal rake angle, the uncut chip thickness is
0.2mm. The chip thickness fluctuates between 0.25 mm and 0.4 mm. The ratio of the
maximum shear angle to the minimum shear angle during machining is ___________
Key: 1.45 to 1.53
Exp:   9
t1  0.2mm
t c  0.25mm to 0.4mm
r cos  t
 tan   , where r  1
1  r sin  tc
if t c  0.25mm, r  0.8 &   42.08o
if t c  0.4mm, r  0.5 &   28.18o
max 42.08
  1.493
min 28.18

0.005
27. A cylindrical pin of 250.020
0.010 mm diameter is electroplated. Plating thickness is 2.0 mm.
Neglecting the gauge tolerance, the diameter (in mm, up to 3 decimal points accuracy) of the
GO ring gauge to inspect the plated pin is _________.
Key: 29.030 to 29.030
Exp: Diameter of GO end of ring gauge = 25.02 + 2(2.005) = 29.030mm

28. A helical compression spring made of wire of circular cross-section is subjected to a


compressive load. The maximum shear stress induced in the cross-section of the wire is 24
MPa. For the same compressive load, if both the wire diameter and the mean coil diameter are
doubled, the maximum shear stress (in MPa) induced in the cross-section of the wire is _____.
Key: 6 to 6
8P.D
Exp: max 
d 3
www.
Given, 1  24MPa engi
neer
ingonyour
fi
nger
tips.
ooo
P1  P2 , d 2  2d1 , D 2  2D1
1 D1 d 32
 2 
24 1 3
so,  . 3
2 D 2 d1 2 2
24
2  MPa
4
2  6MPa

29. In a counter-flow heat exchanger, water is heated at the rate of 1.5kg/s from 40°C to 80°C by
an oil entering at 120°C and leaving at 60°C. The specific heats of water and oil are
4.2kJ/kg.K and 2kJ/kg.K respectively. The overall heat transfer coefficient is 400 W/m2.K.
The required heat transfer surface area (in m2) is
(A) 0.104 (B) 0.022 (C) 10.4 (D) 21.84
Key: (D)
Exp: Given counter flow Heat Exchanger

 ICP–Intensive Classroom Program  eGATE-Live Internet Based Classes DLP  TarGATE-All India Test Series
www.engineeringonyourfingertips.ooo
Leaders in GATE Preparations  65+ Centers across India
© All rights reserved by Gateforum Educational Services Pvt. Ltd. No part of this booklet may be reproduced or utilized in any form without the written permission.

More Notes Join us Telegram-: http//:t.me/allexammentor


www.engineeringonyourfingertips.ooo
|ME| GATE-2017-PAPER-II www.gateforum.com


Water – m c  1.5kg / s Oil - Th i  120C
Tci  40C Th o  60C
Tco  80C C po  2kJ / kg.K
C pc  4.2KJ / kg.K
Overall Heat Transfer Coefficient = 400 W/m2K
Area (A) =?
Q  UA  LMTD  Thi  120C
 
i  40 
Tco  80C

 

mc Cpc Tco  Tci  UA  LMTD  
 Th o  60C 

 

mc Cpc Tco  Tci   o  20
A Tci  40C 
   o 
U i 
 n(i / o ) 
1.5  4.2   80  40   103
  21.83m 2  A  21.83m 2
 
40  20
400  
 n 40
 
20  

30. The rod PQ of length L = 2 m, and uniformly distributed mass of M = 10 kg, is released
from rest at the position shown in the figure. The ends slide along the frictionless faces OP and
OQ. Assume acceleration due to gravity, g = 10 m/s2. The mass moment of inertia of the rod
about its centre of mass and an axis perpendicular to the plane of the figure is (ML 2/12). At
this instant, the magnitude of angular acceleration (in radian/s2) of the rod is ____________

www.
engi
neer
ingonyour
fi
nger
tips.
ooo

Key: 7.25 to 7.75


Exp: M  10kg, g 10m sec2
0.5
ML2
Ic  I
12 1

 2
2
2 2
M  1  2M M 2M
II   M   12  2  3 1m
12  2
2M
T  I  Mg  0.5   Mg
3
2
 10  0.5    1m
3
15
   7.5rad sec 2
2
 ICP–Intensive Classroom Program  eGATE-Live Internet Based Classes DLP  TarGATE-All India Test Series
www.engineeringonyourfingertips.ooo
Leaders in GATE Preparations  65+ Centers across India
© All rights reserved by Gateforum Educational Services Pvt. Ltd. No part of this booklet may be reproduced or utilized in any form without the written permission.

More Notes Join us Telegram-: http//:t.me/allexammentor


www.engineeringonyourfingertips.ooo
|ME| GATE-2017-PAPER-II www.gateforum.com

31. A steel plate, connected to a fixed channel using three identical bolts A, B and C, carries a
load of 6kN as shown in the figure. Considering the effect of direct load and moment, the
magnitude of resultant shear force (in kN) on bolt C is.

(A) 13 (B) 15 (C) 17 (D) 30


Key; (C)
6
Exp: Pr imary shear   2kN
3
Secondaryshear 1500 kN  mm
 Pe  1500
M 2 2   0.3
 r1  r2   50  50 
2 2

R1A  R1C  C  r1  0.3  50  15kN


Resultant shear at 'C'  2  15 17 kN.
Resultant shear  15  2  17 kN.

32. The volume and temperature of air (assumed to be an ideal gas) in a closed vessel is 2.87 m3
and 300K, respectively. The gauge pressure indicated by a manometer fitted to the wall of the
vessel is 0.5bar. If the gas constant of air is R = 287 J/kg. K and the atmospheric pressure is 1
bar, the mass of air (in kg) in the vessel is
(A) 1.67 (B) 3.33 (C) 5.00 (D) 6.66
Key: (C)
Exp: V  2.87m 3 ; w Tw w.engi
 300K neer
ingonyour
fi
nger
tips.
ooo
Pgauge  0.5bar
R  287J / kg.K
Patm  1 bar
Pabs  Pg  Patm  1.5bar
PV  mRT
PV 1.5  105  2.87
So, m    5kg
RT 287  300

33. For the laminar flow of water over a sphere, the drag coefficient CF is defined as
CF  F /  U2 D2  , where F is the drag force,  is the fluid density, U is the fluid velocity and
D is the diameter of the sphere. The density of water is 1000 kg/m3. When the diameter of the
sphere is 100mm and the fluid velocity is 2m/s, the drag coefficient is 0.5. If water now flows
over another sphere of diameter 200mm under dynamically similar conditions, the drag force
(in N) on this sphere is _____________

 ICP–Intensive Classroom Program  eGATE-Live Internet Based Classes DLP  TarGATE-All India Test Series
www.engineeringonyourfingertips.ooo
Leaders in GATE Preparations  65+ Centers across India
© All rights reserved by Gateforum Educational Services Pvt. Ltd. No part of this booklet may be reproduced or utilized in any form without the written permission.

More Notes Join us Telegram-: http//:t.me/allexammentor


www.engineeringonyourfingertips.ooo
|ME| GATE-2017-PAPER-II www.gateforum.com

Key: 19.9 to 20.1


Exp: Given that the condition is dynamic similarity, and in the given condition, Inertia and viscous
force plays major role, hence Reynold‟s number should be same for both model and prototype.
 Re 1   Re 2
In the first case: U1  2m sec, D1  100 mm,   1000 kg m3
In the second case: U2  2m sec, D2  200mm,   1000 kg m3
 UD   UD 
   
  1    2
Since same water is flowing over both sphere
1   2 , 1   2
U1D1  U 2 D 2
  2 100    V2  200 
U 2  1m sec
So, Drage force in second case will be
F2  CFU22 D22   0.51000 (1)2 0.2  20N
2

34. A rod of length 20mm is stretched to make a rod of length 40 mm. Subsequently, it is
compressed to make a rod of final length 10mm. Consider the longitudinal tensile strain as
positive and compressive strain as negative. The total true longitudinal strain in the rod is
(A) –0.5 (B) –0.69 (C) –0.75 (D) –1.0
Key: (B)
Exp:  i  20mm  f  10mm
 truestrain,
   10 
True  n  f   n    0.69
 i   20 

35. www
A gear train shown in.
e ng
the ineer
figure ingonof
consists yo urfi
gears n
P,ge
Q,rti
Rps .
andooS.oGear Q and gear R are
mounted on the same shaft. All the gears are mounted on parallel shafts and the number of
teeth of P, Q, R and S are 24, 45, 30 and 80, respectively. Gear P is rotating at 400 rpm. The
speed (in rpm) of the gear S is _________.

Key: 120 to 120


Exp: TP  24, TQ  45, TR  30, TS  80
N P  400rpm
NP .TP  NQ TQ

 ICP–Intensive Classroom Program  eGATE-Live Internet Based Classes DLP  TarGATE-All India Test Series
www.engineeringonyourfingertips.ooo
Leaders in GATE Preparations  65+ Centers across India
© All rights reserved by Gateforum Educational Services Pvt. Ltd. No part of this booklet may be reproduced or utilized in any form without the written permission.

10

More Notes Join us Telegram-: http//:t.me/allexammentor


www.engineeringonyourfingertips.ooo
|ME| GATE-2017-PAPER-II www.gateforum.com

400  24
NQ   213.33rpm
45
N Q TQ  NSTS
213.33  45
NS   120rpm
80

36. In the Rankine cycle for a steam power plant the turbine entry and exit enthalpies are 2803 kJ/kg
and 1800 kJ/kg, respectively. The enthalpies of water at pump entry and exit are 121 kJ/kg and 124
kJ/kg, respectively. The specific steam consumption (in kg/k W.h) of the cycle is ______
Key: 3.5 to 3.7
Exp: Given that, T
h1  2803 kJ / kg, h 2  1800kJ / kg 1
h 3  121kJ / kg, h 4  124kJ / kg
4
WT  h1  h 2 3
2
 2803  1800  1003kJ / kg
Wp  h 4  h 3
S
 124  121  3kJ / kg
Wnet  WT  WP  1000kJ / kg
3600
Specific Steam consumption   3.6kg / kWh
Wnet

37. A calorically perfect gas (specific heat at constant pressure 1000 J/kg.K) enters and leaves a gas
turbine with the same velocity. The temperatures of the gas at turbine entry and exit are 1100 K
and 400 K. respectively. The power produced is 4.6 MW and heat escapes at the rate of 300 kJ/s
through the turbine casing. The mass flow rate of the gas (in kg/s) through the turbine is.
(A) 6.14 (B) 7.00 (C) 7.50 (D) 8.00
Key: (B)
Exp: Given that,
Cp  1000J / kgKwww. engi
neer
ingonyour
fi
nger
tips.
ooo
T1  1100K, P  4.6MW P
T2  400K, Q L  300kJ / s
 
E in  E out
V12 V2
h1   gz1  h 2  2  gz 2  Q L  P
2 2
1 2
 V1  V2 , Z1  Z2
So, h1  h 2  Q L  P QL

m Cp  T1  T2   Q L  P

m
 300 10    4.6 10 
3 6

1000  1100  400 



m  7kg / sec

 ICP–Intensive Classroom Program  eGATE-Live Internet Based Classes DLP  TarGATE-All India Test Series
www.engineeringonyourfingertips.ooo
Leaders in GATE Preparations  65+ Centers across India
© All rights reserved by Gateforum Educational Services Pvt. Ltd. No part of this booklet may be reproduced or utilized in any form without the written permission.

11

More Notes Join us Telegram-: http//:t.me/allexammentor


www.engineeringonyourfingertips.ooo
|ME| GATE-2017-PAPER-II www.gateforum.com

38. Three masses are connected to a rotating shaft supported on bearings A and B as shown in the
figure. The system is in a space where the gravitational effect is absent. Neglect the mass of
shaft and rods connecting the masses. For m1 = 10kg, m2 = 5kg and m3 = 2.5 kg and for a shaft
angular speed of 1000 radian/s, the magnitude of the bearing reaction (in N) at location B is
_________.

Key: 0 to 0
Exp: m1  10kg, r1  0.1m
m 2  5kg , r2  0.2m
m3  2.5kg , r3  0.4m

F x  m1r1 cos 1  m 2 r2 cos 2  m3 r3 cos 3


 10  0.1 cos 0o   5  0.2  cos120o  2.5  0.4  cos 240o
 1  0.5  0.5  0
F y m1r1 sin 1  m 2 r2 sin 2  m3 r3 sin 3  0   5  0.2  sin120o   2.5  0.4sin 240o   0
R A  R B  0kN

39. A strip of 120 mm width and 8mm thickness is rolled between two 300 mm-diameter rolls to
get a strip of 120 mm width and 7.2 mm thickness. The speed of the strip at the exit is 30
m/min. There is no front or back tension. Assuming uniform roll pressure of 200 MPa in the
roll bite and 100% mechanical efficiency, the minimum total power (in kW) required to drive
the two rolls is _________.
Key: 9.4 to 9.8
Exp: Width = 120mm
Initial thickness to = 8mm
www.
engi
neer
ingonyour
fi
nger
tips.
ooo
Diameter of Roller = 300mm
Radius of Roller = 150mm
Final thickness = 7.2mm
h  t i  t f  8  7.2  0.8 mm
Power require to drive one roller
P  T.  F  L p    .A.L p . Here, A  LP  b

P  L2p .b  here, Lp  Rh

V
P    Rh  .b.  
R
30
 200  106  0.8  103  0.12   4.8 kW
60
So, power require to drive 2 roller = 2P  2  4.8 kW  9.6 kW

 ICP–Intensive Classroom Program  eGATE-Live Internet Based Classes DLP  TarGATE-All India Test Series
www.engineeringonyourfingertips.ooo
Leaders in GATE Preparations  65+ Centers across India
© All rights reserved by Gateforum Educational Services Pvt. Ltd. No part of this booklet may be reproduced or utilized in any form without the written permission.

12

More Notes Join us Telegram-: http//:t.me/allexammentor


www.engineeringonyourfingertips.ooo
|ME| GATE-2017-PAPER-II www.gateforum.com

40. A product made in two factories p and Q, is transported to two destinations, R and S. The per
unit costs of transportation (in Rupees) from factories to destinations are as per the following
matrix:
Destination
R S
Factory
P 10 7
Q 3 4

Factory P produces 7 units and factory Q produces 9 units of the product. Each destination
requires 8 units. If the north-west corner method provides the total transportation cost as X (in
Rupees) and the optimized (the minimum) total transportation cost Y (in Rupees), then (X-Y),
in Rupees, is
(A) 0 (B) 15 (C) 35 (D) 105
Key: Answer is not matched with IIT Key
Exp: R S
10 7
P 7
3 4
Q 9
8 8
By North West corner Rule
R S
10
P 7 7
Q 3 1 4 8

Total cost, x = 10×7+7×1+4×8 = Rs.105


By VAM
V1  6 V2  7
u1  0 10

7
7 7 1
u 2  3 3
8 4
1 9 1
8 8
www .
engi
neer
ingonyour
fi
nger
tips.
ooo
7  3
Total minimum Cost, y = 3×8+4×1+7×7=Rs.77
So, x–y = Rs.28

41. One kg of an ideal gas (gas constant R = 287 J/kg.K) undergoes an irreversible process from
state-1 (1 bar, 300 K) to state -2 (2 bar, 300 K). The change in specific entropy (s2 – s1) of the
gas (in J/kg. K) in the process is ___________
Key: –201 to –197
Exp: Given m  1kg, R  287J / kg.K
P1  1bar, P2  2bar
T1  300K, T2  300K
Same Temperature

S2  S1  Rn  P P   287n  21   198.93J / kgK


2
1

 ICP–Intensive Classroom Program  eGATE-Live Internet Based Classes DLP  TarGATE-All India Test Series
www.engineeringonyourfingertips.ooo Leaders in GATE Preparations  65+ Centers across India
© All rights reserved by Gateforum Educational Services Pvt. Ltd. No part of this booklet may be reproduced or utilized in any form without the written permission.

13

More Notes Join us Telegram-: http//:t.me/allexammentor


www.engineeringonyourfingertips.ooo
|ME| GATE-2017-PAPER-II www.gateforum.com

42. A 60 mm-diameter water jet strikes a plate containing a hole of 40mm diameter as shown in
the figure. Part of the jet passes through the hole horizontally, and the remaining is deflected
vertically. The density of water is 1000 kg/m3. If velocities are as indicated in the figure, the
magnitude of horizontal force (in N) required to hold the plate is _________

Key: 627 to 629


Exp: Force in X–direction = Rate of change of momentum
  Pi  Pf  x  direction

  

  m1 V1  m 2 V2  D1  6cm
  x direction D2  4cm
 V  A1V1  A 2 V2  V1  V2 
 V 2  A1  A 2 
 20m / s  V2
 1000  20  20   0.06    0.04  
2 2

4   20m / s  V1
 628.32N

43. The arrangement shown in the figure measures the


velocity V of a gas of density 1 kg/m3 flowing
through a pipe. The acceleration due to gravity is
9.81 m/s2. If the manometric fluid is water (density
1000 kg/m3) and the velocity V is 20 m/s, the
differential head h (in mm) between the two arms of
the manometer isw ww. engi neer
____________ ingonyour finger
tips.
ooo
Key: 19 to 21
Exp: Given
g  1kg / m3 , g  9.81m / s 2
m  1000kg / m3 , V  20m / s

 
V  2gH Where H  h  m  1
  
V 2
 
So,  h  m  1
2g   
  20 2 
   h 1000  1
 2  9.81 
h  0.0204m  2.04cm  h  20.4mm

 ICP–Intensive Classroom Program  eGATE-Live Internet Based Classes DLP  TarGATE-All India Test Series
www.engineeringonyourfingertips.ooo
Leaders in GATE Preparations  65+ Centers across India
© All rights reserved by Gateforum Educational Services Pvt. Ltd. No part of this booklet may be reproduced or utilized in any form without the written permission.

14

More Notes Join us Telegram-: http//:t.me/allexammentor


www.engineeringonyourfingertips.ooo
|ME| GATE-2017-PAPER-II www.gateforum.com

44. A metal ball of diameter 60mm is initially at 220 °C. The ball is suddenly cooled by an air jet
of 20°C. The heat transfer coefficient is 200 W/m2.K and 9000kg/m3, respectively. The ball
temperature (in °C) after 90 seconds will be approximately.
(A) 141 (B) 163 (C) 189 (D) 210
Key: (A)
Exp: D = 60mm = 0.06m
Ti  220C, h  200W / m 2 K,   9000kg / m3
T  20C, Cp  400J / kgK, K  400W / mK
t  90sec T?
 h.A 
 200903 
Ti  T  .t
 .VC  220  20  
 e p    e 0.039000400 
Tf  T Tf  20
Tf  141.3C

45. A single – plate clutch has a friction disc with inner and outer radii of 20mm and 40 mm,
respectively. The friction lining in the disc is made in such a way that the coefficient of
friction  varies radially as   0.01r, where r is in mm. The clutch needs to transmit a friction
torque of 18.85kN.mm. As per uniform pressure theory, the pressure (in MPa) on the disc is
_________
Key: 0.49 to 0.51
Exp: ri  20mm, ro  40mm,   0.01r dr
T  18.85kN  mm
p?
P   p2rdr r
T   p 2r 2 dr   p  0.01r  2r 2 dr   0.0628r 3 .dr  ri
40 r0
40
 r4 
T  0.0628p  r dr  0.0628p  
3

20  4  20
 404 204 
18.85  103  0.0628p   
www. en g 4i
nee4r
in gonyour
fi
nger
tips.
ooo
p  0.5MPa

46. The surface integral  F.ndS


S
over the surface S of the sphere x 2  y 2  z 2  9, where

F=(x+y) i+ (x+z) j+(y+z) k and n is the unit outward surface normal, yields ______.
Key: 225 to 227

Exp: F   x  yi   x  z J   y  z k
   
divF   x  y    x  z    y  z   1  0  1  2
x y z
By divergence theorem,
 ^ 
S
F.n dS   divFdV
V
where V is volume of given surface of sphere x 2  y2  z2  9

4  27 
  2dV  2V  2   72  226.1947
V
3

 ICP–Intensive Classroom Program  eGATE-Live Internet Based Classes DLP  TarGATE-All India Test Series
www.engineeringonyourfingertips.ooo
Leaders in GATE Preparations  65+ Centers across India
© All rights reserved by Gateforum Educational Services Pvt. Ltd. No part of this booklet may be reproduced or utilized in any form without the written permission.

15

More Notes Join us Telegram-: http//:t.me/allexammentor


www.engineeringonyourfingertips.ooo
|ME| GATE-2017-PAPER-II www.gateforum.com

47. Block 2 slides outward on link 1 at a uniform velocity of 6 m/s as shown in the figure. Link 1
is rotating at a constant angular velocity of 20 radian/s counterclockwise. The magnitude of
the total acceleration (in m/s2) of point P of the block with respect to fixed point O is
________

Key: 243 to 244


Exp: Acceleration of the block,
v  6m / sec,   20rad / sec

  r    2v
2
a  a r 2  a cr 2  2 2

r  OP,
a cr  2v  2  6  20  240m / s 2
a r  2 r   20   0.1  40
2

a r  2402  402  243.31m / sec 2

48. During the turning of a 20mm-diameter steel bar at a spindle speed of 400 rpm, a tool life of
20 minute is obtained. When the same bar is turned at 200 rpm, the tool life becomes 60
minute. Assume that Taylor‟s tool life equation is valid. When the bar is turned at 300 rpm, the
tool life (in minute) is approximately.
(A) 25 (B) 32 (C) 40 (D) 50
Key: (B)
Exp: Taylors Tool life equation,
VT n  C www.
engi
neer
ingonyour
fi
nger
tips.
ooo
 V T  V2 T2
1 1
n n

n
 T2  V1
  
 T1  V2
n
 60  400
  
 20  200
n  0.63
at N 3  300rpm, T3  ?
n
 T3  V1
  
 T1  V3
0.63
 T3  400
 20   300
 
T3  31.57 min utes
T3  32 min utes
 ICP–Intensive Classroom Program  eGATE-Live Internet Based Classes DLP  TarGATE-All India Test Series
www.engineeringonyourfingertips.ooo
Leaders in GATE Preparations  65+ Centers across India
© All rights reserved by Gateforum Educational Services Pvt. Ltd. No part of this booklet may be reproduced or utilized in any form without the written permission.

16

More Notes Join us Telegram-: http//:t.me/allexammentor


www.engineeringonyourfingertips.ooo
|ME| GATE-2017-PAPER-II www.gateforum.com

50 70 
49. Consider the matrix A =   whose eigenvectors corresponding to eigenvalues 1 and  2
70 80 
 70    80
are x1    and x 2   2 T
 . respectively. The value of x1 x 2 is_________

 1  50   70 
Key: 0 to 0
50 70 
Exp: A 
 70 80 
 70    2  80 
Eigen vectors are X1    ; X2   
 1  50   70 
Where 1 ,  2 Eigen values of A
   80 
X1T X 2  70 1  50   2   70   2  80    1  50  70
 70 
 70 2  5600  701  3500  70  1   2   9100
 70 130   9100  9100  9100  0

 sum of eigen values  1   2 


 
 Trace  50  80  130 

50. The radius of gyration of a compound pendulum about the point of suspension is 100mm. The
distance between the point of suspension and the centre of mass is 250mm. Considering the
acceleration due to gravity as 9.81 m/s2, the natural frequency (in radian/s) of the compound
pendulum is _________.
Key: 15 to 16

Exp: k  100mm  0.1m


L  250mm  0.250m
g  9.81m / sec

I  mk 2  m  0.1
2

www.
engi
neer
ingonyour
fi
nger
tips.
ooo
mgL m  9.81 0.250
Wn    15.66rad / sec
m  0.1
2
I

51. Consider the differential equation 3y ''  x   27y  x   0 with initial conditions y(0) = 0 and
y'(0)  2000. The value of y at x = 1 is ________.
Key: 93 to 95
Exp: 3y ''  x   27y  x   0, y  0   0, y '  0   2000
Auxillary equation, 3m 2  27  0  m 2  9  0  m  0  3i
y c  c1 cos3x  c 2 sin 3x and y p  0
 y c  c1 cos3x  c 2 sin 3x
y  0   0  c1  0  0  c1  0
 y  c2 sin 3x
y '  3c 2 cos3x

 ICP–Intensive Classroom Program  eGATE-Live Internet Based Classes DLP  TarGATE-All India Test Series
www.engineeringonyourfingertips.ooo
Leaders in GATE Preparations  65+ Centers across India
© All rights reserved by Gateforum Educational Services Pvt. Ltd. No part of this booklet may be reproduced or utilized in any form without the written permission.

17

More Notes Join us Telegram-: http//:t.me/allexammentor


www.engineeringonyourfingertips.ooo
|ME| GATE-2017-PAPER-II www.gateforum.com

2000
y '  0   2000  2000  3c2  c 2 
3
2000 2000
y  sin 3x, y 1  sin 3  94.08
3 3

52. If f(z) = (x2+ay2) + ibxy is a complex analytic function of z = x + iy, where i = 1, then
(A) a = –1, b= –1 (B) a = –1, b = 2 (C) a = 1, b = 2 (D) a = 2, b = 2
Key: (B)
Exp: Given f  z    x 2  ay2   i bxy is analytic

u  x 
  , 
x y y x
value u  x 2  ay 2 ,   bxy
u 
 2x  by
x x
u 
 2ay  bx
y y
Clearly for b = 2 and a= –1 above Cauchy-Riemann equations holds

53. A project starts with activity A and ends with activity F. The precedence relation and durations
of the activities are as per the following table:

Duration
Activity Immediate Predecessor
(days)
A - 4
B A 3
C A 7
D B 14
E C 4
F D,E 9

www
The minimum project .
engi nee
completion r
ing
time (inonyou
days) isr
fi
nger
______ t
ips.
ooo
Key: 30 to 30
Exp: 3 D
B
3 14
1 A 2 5 F 6
4 4 9
C, 7 4 E
Minimum project completion Time = Length of longest path = 4+3+14+9=30 Days.

54. Maximize Z = 5x1+3x2


Subject to
x1+2x2  10,
x1–x2  8,
x1, x2  0.
In the starting Simplex tableau, x1 and x2 are non-basic variables and the value of Z is zero.
The value of Z in the next Simplex tableau is _________.
 ICP–Intensive Classroom Program  eGATE-Live Internet Based Classes DLP  TarGATE-All India Test Series
www.engineeringonyourfingertips.ooo
Leaders in GATE Preparations  65+ Centers across India
© All rights reserved by Gateforum Educational Services Pvt. Ltd. No part of this booklet may be reproduced or utilized in any form without the written permission.

18

More Notes Join us Telegram-: http//:t.me/allexammentor


www.engineeringonyourfingertips.ooo
|ME| GATE-2017-PAPER-II www.gateforum.com

x2
Key: 40 to 40
  26 2 
 26 2   0,5   , 
Exp: Zmax at  ,   3 3
 3 3

10,0  x1
1st Table at  0,0   Z  0  0,0   8,0 
2nd Table at 8,0   Z  40
  0, 8 

55. The principal stresses at a point in a critical section of a machine component are
1  60MPa, 2  5MPa and 3  40 MPa. For the material of the component, the tensile
yield strength is  y  200 MPa. According to the maximum shear stress theory, the factor of
safety is
(A) 1.67 (B) 2.00 (C) 3.60 (D) 4.00
Key: (B)
1  3  w y
Exp:   1  3 
2 2 FOS
200
60   40  
FOS
FOS  2
General Aptitude
Q. No. 1 - 5 Carry One Mark Each

1. If you choose plan P, you will have to _______ plan Q, as these two are mutually _________.
(A) forgo, exclusive (B) forget, inclusive
(C) accept, exhaustive (D) adopt, intrusive
Key: (A)

2. P looks at Q while Q looks at R. P is married, R is not. The number of people in which a


married person is looking at an unmarried person is
(A) 0
www(B)
.
en gi
1
neer
ingonyour
(C) 2
fi
nger
tips.
oo o
(D) Cannot be determined
Key: (B)

3. If a and b are integers and a – b is even, which of the following must always be even?
(A) ab (B) a2 + b2 + 1 (C) a2 + b + 1 (D) ab – b
Key: (D)
Exp: According to the given relation of a–b = even, there is a possibility of odd-odd (or) even-even
is equal to even. From the options, Option (D) is correct. Since, odd × odd–odd (or) even ×
even–even → is always even number.

Alternate Method:
Let a – b =2K  a  b  2K
ab  b   b  2K  b  b  b2  2Kb  b   b2  b   2Kb  b  b  1  2Kb
Even; since 2Kb always even & b (b-1) also even, if with b is odd or even.

 ICP–Intensive Classroom Program  eGATE-Live Internet Based Classes DLP  TarGATE-All India Test Series
www.engineeringonyourfingertips.ooo
Leaders in GATE Preparations  65+ Centers across India
© All rights reserved by Gateforum Educational Services Pvt. Ltd. No part of this booklet may be reproduced or utilized in any form without the written permission.

19

More Notes Join us Telegram-: http//:t.me/allexammentor


www.engineeringonyourfingertips.ooo
|ME| GATE-2017-PAPER-II www.gateforum.com

4. A couple has 2 children. The probability that both children are boys if the older one is a boy is
(A) 1/4 (B) 1/3 (C) 1/2 (D) 1
Key: (C)
No.of Favourablecases
Exp: Probability 
Total No.of Possible cases
1
The probability that both children are boys if the order one is a boy =
2

5. The ways in which this game can be played __________ potentially infinite.
(A) is (B) is being (C) are (D) are being
Key: (C)

Q. No. 6- 10 Carry Two Marks Each

6. “If you are looking for a history of India, or for an account of the rise and fall of the British
Raj, or for the reason of the cleaving of the subcontinent into two mutually antagonistic parts
and the effects this mutilation will have in the respective sections, and ultimately on Asia, you
will not find it in these pages; for though I have spent a lifetime in the country, I lived too near
the seat of events, and was too intimately associated with the actors, to get the perspective
needed for the impartial recording of these matters.”
Which of the following closest in meaning to „cleaving‟?
(A) Deteriorating (B) Arguing (C) Departing (D) Splitting
Key: (D)

7. There are 4 women P, Q, R, S, and 5 men V, W, X, Y, Z in a group. We are required to form


pairs each consisting of one woman and one man. P is not to be paired with Z, and Y must
necessarily be paired with someone. In how many ways can 4 such pairs be formed?
(A) 74 (B) 76 (C) 78 (D) 80
Key: (C)
Exp: If P is paired with y; they Q has 4 choices
www.
engi
R ne3er
has ingonyour
choices fi
nger
tips.
ooo
S has 2 choices
Total 24 choices
(or)
If Q is paired with y; then P has 3 choices
R has 3 choices
S has 2 choices
Total 18 choices
(or)
If R is paired with y; then P has 3 choices
Q has 3 choices
S has 2 choices
Total 18 choices

 ICP–Intensive Classroom Program  eGATE-Live Internet Based Classes DLP  TarGATE-All India Test Series
www.engineeringonyourfingertips.ooo
Leaders in GATE Preparations  65+ Centers across India
© All rights reserved by Gateforum Educational Services Pvt. Ltd. No part of this booklet may be reproduced or utilized in any form without the written permission.

20

More Notes Join us Telegram-: http//:t.me/allexammentor


www.engineeringonyourfingertips.ooo
|ME| GATE-2017-PAPER-II www.gateforum.com

(or)
If S is paired with y; then P has 3 choices
Q has 3 choices
S has 2 choices
Total 18 choices
 Total number of ways = 24+18+18+18=78

8. In the graph below, the concentration of a particular pollutant in a lake is plotted over
(alternate) days of a month in winter (average temperature 10°C) and a month in summer
(average temperature 30°C).

Consider the following statements based on the data shown above:


(i) Over the given months, the difference between the maximum and the minimum pollutant
concentrations is the same in both winter and summer.
(ii) There are at last four days in the summer month such that the pollutant concentrations on
those days are within 1 ppm of the pollutant concentrations on the corresponding days in
the winter month.
Which one of the following options is correct?
(A) Only i (B) Only ii (C) Both i and ii (D) Neither i nor ii
Key: (B)
Exp: The difference between the maximum and the minimum pollutant concentrations
wwwppm
(i) in winter = 8-0=8 .
en,gi
neer
ingonyour
fi
nger
tips.
ooo
(ii) in summer = 10.5-1.5=9 ppm
 (i) is false & (ii) is correct from the graph.

9. All people in a certain island are either „Knights‟ or „Knaves‟ and each person knows every
other person‟s identity. Knights NEVER lie, and knaves ALWAYS lie.
P says “Both of us are knights”. Q says “None of us are knaves”.
Which one of the following can be logically inferred from the above?
(A) Both P and Q are knights
(B) P is a knight; Q is a knave
(C) Both P and Q are knaves
(D) The identities of P, Q cannot be determined
Key: (D)

 ICP–Intensive Classroom Program  eGATE-Live Internet Based Classes DLP  TarGATE-All India Test Series
www.engineeringonyourfingertips.ooo
Leaders in GATE Preparations  65+ Centers across India
© All rights reserved by Gateforum Educational Services Pvt. Ltd. No part of this booklet may be reproduced or utilized in any form without the written permission.

21

More Notes Join us Telegram-: http//:t.me/allexammentor


www.engineeringonyourfingertips.ooo
|ME| GATE-2017-PAPER-II www.gateforum.com

10. X bullocks and Y tractors take 8 days to plough a field. If we halve the number of bullocks and
double the number of tractors, it takes 5days to plough the same field. How many days will it
take X bullocks alone to plough the field?
(A) 30 (B) 35 (C) 40 (D) 45
Key: (A)
Exp: Given Number of days required that X bullocks and Y tractors to plough a field = 8 days 
(1)  i.e, X  Y  8D  8X  8Y  1day  Number of days required that
X
bullocks and 2y tractors
2
to plough field  5   2 
 X 5 
i.e, 2  2Y  5D  2 X  10Y  1D 
 
From (1) & (2); we have
5X 11X
8X  8Y   10Y  Y 
2 4
15X
 From 1 X  Y  8D   8D  X  30days
4

www.
engi
neer
ingonyour
fi
nger
tips.
ooo

 ICP–Intensive Classroom Program  eGATE-Live Internet Based Classes DLP  TarGATE-All India Test Series
www.engineeringonyourfingertips.ooo
Leaders in GATE Preparations  65+ Centers across India
© All rights reserved by Gateforum Educational Services Pvt. Ltd. No part of this booklet may be reproduced or utilized in any form without the written permission.

22

More Notes Join us Telegram-: http//:t.me/allexammentor


|www.engineeringonyourfingertips.ooo
ME| GATE-2016-PAPER-01 www.gateforum.com

General Aptitude
Q. No. 1 – 5 Carry One Mark Each

1. Which of the following is CORRECT with respect to grammar and usage?


Mount Everest is .
(A) the highest peak in the world
(B) highest peak in the world
(C) one of highest peak in the world
(D) one of the highest peak in the world
Key: (A)

2. The policeman asked the victim of a theft, “What did you ?”


(A) loose (B) lose (C) loss (D) louse
Key: (B)

3. Despite the new medicine‟s____________in treating diabetes, it is not __________ widely.


(A) effectiveness --- prescribed (B) availability --- used
(C) prescription -- available (D) acceptance --- proscribed
Key: (A)

4. In a huge pile of apples and oranges, both ripe and unripe mixed together, 15% are unripe fruits. Of the
unripe fruits, 45% are apples. Of the ripe ones, 66% are oranges. If the pile contains a total of 5692000
fruits, how many of them are apples?
(A) 2029198 (B) 2467482 (C) 2789080 (D) 3577422
Key: (A)
Exp: www .
engi
5692000 n eer
 Total ingo
fruits nyour
finger
tips.
ooo

15% unripe 85% ripe

853800 4838200

45% 55% 34% 66%

apples oranges apples oranges

384210 469590 1644988 3193212


Total number of apples = 384210 + 1644988 = 2029198

 ICP–Intensive Classroom Program  eGATE-Live Internet Based Classes DLP  TarGATE-All India Test Series
Leaders in GATE Preparations  65+ Centers across India

www.engineeringonyourfingertips.ooo
© All rights reserved by Gateforum Educational Services Pvt. Ltd. No part of this booklet may be reproduced or utilized in any form without the written permission.

1
11/11
1/14
7EC
More Notes Join us Telegram-: http//:t.me/allexammentor
|www.engineeringonyourfingertips.ooo
ME| GATE-2016-PAPER-01 www.gateforum.com

5. Michael lives 10 km away from where I live. Ahmed lives 5 km away and Susan lives 7 km away from
where I live. Arun is farther away than Ahmed but closer than Susan from where I live. From the
information provided here, what is one possible distance (in km) at which I live from Arun‟s place?
(A) 3.00 (B) 4.99 (C) 6.02 (D) 7.01
Key: (C)

Q. No. 6 – 10 Carry Two Marks Each

6. A person moving through a tuberculosis prone zone has a 50% probability of becoming infected.
However, only 30% of infected people develop the disease. What percentage of people moving
through a tuberculosis prone zone remains infected but does not show symptoms of disease?
(A) 15 (B) 33 (C) 35 (D) 37
Key: (C)
Exp:

0.3 develop disease

0.5 Infected
0.7
not develop
0.5 not Infected
P(a person infected but does not show symptoms)  0.50  0.70  0.35
The percentage is 35%

7. In a world filled with uncertainty, he was glad to have many good friends. He had always assisted them
in times of need and was confident that they would reciprocate. However, the events of the last week
proved him wrong.
Which of the following inference(s) is/are logically valid and can be inferred from the above passage?
(i) His friends were always asking him to help them.
(ii) He felt that when w
in w w.
need e
ofngine
help, er
his ingon
friends your
would fi
letn ger
him ti
ps.
down. ooo
(iii) He was sure that his friends would help him when in need.
(iv) His friends did not help him last week.
(A) (i) and (ii) (B) (iii) and (iv) (C) (iii) only (D) (iv) only
Key: (B)

8. Leela is older than her cousin Pavithra. Pavithra‟s brother Shiva is older than Leela. When Pavithra and
Shiva are visiting Leela, all three like to play chess. Pavithra wins more often than Leela does.
Which one of the following statements must be TRUE based on the above?
(A) When Shiva plays chess with Leela and Pavithra, he often loses.
(B) Leela is the oldest of the three.
(C) Shiva is a better chess player than Pavithra.
(D) Pavithra is the youngest of the three.
Key: (D)
 ICP–Intensive Classroom Program  eGATE-Live Internet Based Classes DLP  TarGATE-All India Test Series
Leaders in GATE Preparations  65+ Centers across India

www.engineeringonyourfingertips.ooo
© All rights reserved by Gateforum Educational Services Pvt. Ltd. No part of this booklet may be reproduced or utilized in any form without the written permission.

2
22/12
2/14
7EC
More Notes Join us Telegram-: http//:t.me/allexammentor
|www.engineeringonyourfingertips.ooo
ME| GATE-2016-PAPER-01 www.gateforum.com

1 1 1
9. If q  a  and r  b  and s  C  , the value of abc is_____ .
r s q
(A) ( r q s ) −1 (B) 0 (C) 1 (D) r+q+s
Key: (C)
1 1 1
Exp: q a  , r b  , sc 
r s q
q a  r, r b  s, s c  q
r  q a   sc   sac
a

s  r b   s ac   s abc  abc  1
b

10. P, Q, R and S are working on a project. Q can finish the task in 25 days, working alone for 12 hours
a day. R can finish the task in 50 days, working alone for 12 hours per day. Q worked 12 hours a day
but took sick leave in the beginning for two days. R worked 18 hours a day on all days. What is the ratio
of work done by Q and R after 7 days from the start of the project?
(A) 10:11 (B) 11:10 (C) 20:21 (D) 21:20
Key: (C)
1
Exp: Q's one hour work 
25  12
1
R 's one hour work 
50  12
Since Q has taken 2 days sick leave, he has worked only 5 days on the end of seventh day.
1
Work completed by Q on 7th day= (5  12)
25  12
1
Work completed by R on 7th day= (7  18)
50  12
5  12 7  18 20
Ratio of their work    20 : 21
25  12 50  12 21

www.
enMechanical
gi
neeringon Engineering
yourf
ingert i
ps.
ooo
Q. No. 1 – 25 Carry One Mark Each

1. The solution to the system of equations


2 5
   x  2 
      is
   y  30
 4 3
(A) 6, 2 (B) −6, 2 (C) −6, −2 (D) 6, −2

Key: (D)
 2 5  6   2 
Exp: By verification method;      
 4 3  2 30

 ICP–Intensive Classroom Program  eGATE-Live Internet Based Classes DLP  TarGATE-All India Test Series
Leaders in GATE Preparations  65+ Centers across India

www.engineeringonyourfingertips.ooo
© All rights reserved by Gateforum Educational Services Pvt. Ltd. No part of this booklet may be reproduced or utilized in any form without the written permission.

3
33/13
3/14
7EC
More Notes Join us Telegram-: http//:t.me/allexammentor
|www.engineeringonyourfingertips.ooo
ME| GATE-2016-PAPER-01 www.gateforum.com

2. If f(t) is a function defined for all t ≥ 0, its Laplace transform F(s) is defined as

(A)  est f (t)dt  est f (t)dt

(B)
0 0

 
(C) 0
eist f (t)dt (D)  0
eist f (t)dt
Key: (B)
Exp: Definition of Laplace transform of f  t  v t  0.

3. f(z)=u(x,y)+iv(x,y) is an analytic function of complex variable z=x+iy where i  1. If


u(x,y)=2xy, then v(x,y) may be expressed as
(A) –x2 + y 2 + constant (B) x 2 − y 2 + constant
(C) x 2 + y 2 + constant (D) − (x 2 + y 2 ) + constant
Key: (A)
Exp: Given u  x, y   2xy
By total derivative definition; we have
v dv
dv  dx  dy
x y
u u
 dx  dy  by C  R equations … (1)
y x
u u
 u  x, y   2xy   2y;  2x
x y
From (1) dv  2xdx  2ydy … (2)

 x 2   y2 
 V  2    2    constant  equation 2 is exact D.E 
 2   2
 V   x 2  y 2  constant
4. Consider a Poisson distribution for the tossing of a biased coin. The mean for this distribution is µ. The
standard deviation for this distribution is given by
(A)  www
(B) .
2engi
neer
ingony
(C) our
finger
tip(D)
s.oo1/o

Key: (A)
Exp: Given mean of a poisson distribution for the tossing of a biased coin is .
We know that Mean  Variance  

 Standard deviation  variance   .

5. Solve the equation x = 10 cos (x) using the Newton-Raphson method. The initial guess is

x   / 4 . The value of the predicted root after the first iteration, up to second decimal, is
Key: 1.56

By Newton-Raphson method; the iterative formula for finding approximate root at  n  1 iteration is
th
Exp:

f  xn 
x n 1  x n  ; where x = 0, 1, 2 …….
f  xn 
 ICP–Intensive Classroom Program  eGATE-Live Internet Based Classes DLP  TarGATE-All India Test Series
Leaders in GATE Preparations  65+ Centers across India

www.engineeringonyourfingertips.ooo
© All rights reserved by Gateforum Educational Services Pvt. Ltd. No part of this booklet may be reproduced or utilized in any form without the written permission.

4
44/14
4/14
7EC
More Notes Join us Telegram-: http//:t.me/allexammentor
|www.engineeringonyourfingertips.ooo
ME| GATE-2016-PAPER-01 www.gateforum.com

Putting n = 0; then

f  x0 
x1  x 0  … (1)
f  x0 

    10
Let f  x   x  10cos x  f  x 0   f     .
4 4 2

 f   x   1  10sin x

 10
From (1);  f   x 0   f     1 
4 2

  10 

 4 2   1.56
 x1    
4  1  10 
 2 

6. A rigid ball of weight 100 N is suspended with the help of a string. The ball is pulled by a
horizontal force F such that the string makes an angle of 30o with the vertical. The magnitude of force
F (in N) is .

Key: 57.735
Exp: Applying Lami‟s theorem
T 30
T 100 F 120
  ww.
sin90 sin 90  30 sin 180n
  w  e gi
n
30 eer
ingonyour
finger
tips.
ooo
F
150
F  57.735 N
90

100
7. A point mass M is released from rest and slides down a spherical bowl (of radius R) from a height H as
shown in the figure below. The surface of the bowl is smooth (no friction). The velocity of the mass at
the bottom of the bowl is

(A) gH (B) 2gR (C) 2gH (D) 0

 ICP–Intensive Classroom Program  eGATE-Live Internet Based Classes DLP  TarGATE-All India Test Series
Leaders in GATE Preparations  65+ Centers across India

www.engineeringonyourfingertips.ooo
© All rights reserved by Gateforum Educational Services Pvt. Ltd. No part of this booklet may be reproduced or utilized in any form without the written permission.

5
55/15
5/14
7EC
More Notes Join us Telegram-: http//:t.me/allexammentor
|www.engineeringonyourfingertips.ooo
ME| GATE-2016-PAPER-01 www.gateforum.com

Key: (C)
Exp: Since there is no friction. Therefore there will be no loss of energy of system. Hence energy
remains conserved.
P.E1+K.E1 = P.E2+K.E2
1 1
mgH  m  0   mg  0  mVb2
2

2 2
Vb  2gH

8. The cross sections of two hollow bars made of the same material are concentric circles as shown in the
figure. It is given that r3 > r1 and r4 > r2 , and that the areas of the cross-sections are the same. J1 and J2
are the torsional rigidities of the bars on the left and right, respectively. The ratio J2/J1 is

r3
r1

r2 r4

(A) > 1 (B) < 0.5 (C) =1 (D) between 0.5 and 1
Key: (A)
Exp:
r3
r1

r2 r4

ww
Smaller w.e
Ring n
1gi
neer
ingonyour
finger
tips.oo
Bigger o  2
Ring

Given A1 = A2
  r22  r12     r42  r32 

r22  r12  r42  r32 ________(1)


We know , Torsional Rigidity = J= Shear Modulus× Polar moment of Inertia
 4 4
J2 2
G
r4  r3 J2 
r42  r32 
r32  r42 r32  r42      
    2 2  2 2  2 2
J1 
G  r24  r14 J1

r2  r1 r1  r2
 r1  r2      
2
But, r4  r2 and r3  r1
J2
 1
J1

 ICP–Intensive Classroom Program  eGATE-Live Internet Based Classes DLP  TarGATE-All India Test Series
Leaders in GATE Preparations  65+ Centers across India

www.engineeringonyourfingertips.ooo
© All rights reserved by Gateforum Educational Services Pvt. Ltd. No part of this booklet may be reproduced or utilized in any form without the written permission.

6
66/16
6/14
7EC
More Notes Join us Telegram-: http//:t.me/allexammentor
|www.engineeringonyourfingertips.ooo
ME| GATE-2016-PAPER-01 www.gateforum.com

9. A cantilever beam having square cross-section of side a is subjected to an end load. If a is increased by
19%, the tip deflection decreases approximately by
(A) 19% (B) 29% (C) 41% (D) 50%
Key: (D)
p3 1 1
Exp:      4
3EI I a
1 a 24
 where, a 2  1.19a,
2 a14
1 1
  1.19   2   0.51
4

2 1.19 4
So, deflection decrease by 50%

10. A car is moving on a curved horizontal road of radius 100 m with a speed of 20 m/s. The rotating masses
of the engine have an angular speed of 100 rad/s in clockwise direction when viewed from the front of
the car. The combined moment of inertia of the rotating masses is 10 kg-m2. The magnitude of the
gyroscopic moment (in N-m) is .
Key: 200
Exp: Given: Spin velocity ( s )= 100 rad/sec
Moment of Inertia (MOI) = 10 kg-m2
V  20 m sec

Precision Angular Velocity  p  


linear speed
R R  100 m

 p
20
  0.2 rad sec
100
Gyroscopic moment = MOI s × p
ww
w
10.
e
 ngi
100 neer
0.2 i
ngonyour
finger
tips.
ooo
 200 Nm
11. A single degree of freedom spring mass system with viscous damping has a spring constant of 10
kN/m. The system is excited by a sinusoidal force of amplitude 100 N. If the damping factor (ratio) is
0.25, the amplitude of steady state oscillation at resonance is mm.
Key: 20
Exp: Given: Spring constant (k) = 10 kN/m = 10,000 N/m
Magnitude of force (F0) = 100N
Damping factor    0.25
Forcing frequency (ω) = Natural frequency (ωn)
F 100
Static deflection of spring  0  4  102 m
k 10
 10 mm

 ICP–Intensive Classroom Program  eGATE-Live Internet Based Classes DLP  TarGATE-All India Test Series
Leaders in GATE Preparations  65+ Centers across India

www.engineeringonyourfingertips.ooo
© All rights reserved by Gateforum Educational Services Pvt. Ltd. No part of this booklet may be reproduced or utilized in any form without the written permission.

7
77/17
7/14
7EC
More Notes Join us Telegram-: http//:t.me/allexammentor
|www.engineeringonyourfingertips.ooo
ME| GATE-2016-PAPER-01 www.gateforum.com

Staticdeflection
Dynamic deflection 
   2   
2

1       2 
  n    n 
10

1  1  2 2
  2  0.25  1
2

10
  20 mm
2  0.25
12. The spring constant of a helical compression spring DOES NOT depend on
(A) coil diameter
(B) material strength
(C) number of active turns
(D) wire diameter
Key: (B)
Exp: For Helical compression spring.
64 WR 3n
Deflection,  
Gd4
W W Gd 4
Stiffness, or spring constant   
 64 WR 3 n 64R 3 n
Gd 4
From the above formula we can say that spring constant depends on coil diameter (D), wire diameter (d),
No. of active turns (n) and modulus of rigidity (G) and is independent of material strength.

13. The instantaneous stream-wise velocity of a turbulent flow is given as follows:


u(x, y, z, t) = u (x, y, z)  u  ( x, y, z, t)
The time-average of the fluctuating velocity u  ( x, y, z, t) is
(A) u  / 2 (B) u / 2 (C) zero (D) u / 2
www.
engi
neer
ingonyour
finger
tips.
ooo
Key: (C) u'
Exp:
u

time  t 
Given u  u  u ' _____(1)
T
1
T 0
where u  u dt T = period after which pattern will repeat

 ICP–Intensive Classroom Program  eGATE-Live Internet Based Classes DLP  TarGATE-All India Test Series
Leaders in GATE Preparations  65+ Centers across India

www.engineeringonyourfingertips.ooo
© All rights reserved by Gateforum Educational Services Pvt. Ltd. No part of this booklet may be reproduced or utilized in any form without the written permission.

8
88/18
8/14
7EC
More Notes Join us Telegram-: http//:t.me/allexammentor
|www.engineeringonyourfingertips.ooo
ME| GATE-2016-PAPER-01 www.gateforum.com

Rewriting equation (1)


u '  u u
taking average of fluctuating component
1T 
 
T T T T
1 1 1 1
T 0 T 0 T 0 T 0
u'  u 'dt  u  u dt  u dt  u dt  u  u   dt 
T 0 
 uu
u'  0

14. For a floating body, buoyant force acts at the


(A) centroid of the floating body
(B) center of gravity of the body
(C) centroid of the fluid vertically below the body
(D) centroid of the displaced fluid
Key: (D)
15. A plastic sleeve of outer radius r0 = 1 mm covers a wire (radius r = 0.5 mm) carrying electric
current. Thermal conductivity of the plastic is 0.15 W/m-K. The heat transfer coefficient on the outer
surface of the sleeve exposed to air is 25 W/m2-K. Due to the addition of the plastic cover, the heat
transfer from the wire to the ambient will
(A) increase
(B) remain the same
(C) decrease
(D) be zero
Key: (A)
Exp: Given
r0  1mm
r  0.5mm www.
engi
neer
ingonyour
finger
tips.
ooo
K  0.15W/mK
h  25W/m2 K
0.15
Critical radius  k/h   6mm
25
So radius of wire is less than critical radius, addition of plastic sleeve will increase the heat transfer.

16. Which of the following statements are TRUE with respect to heat and work?
(i) They are boundary phenomena
(ii) They are exact differentials
(iii) They are path functions
(A) both (i) and (ii) (B) both (i) and (iii) (C) both (ii) and (iii) (D) only (iii)
Key: (B)

 ICP–Intensive Classroom Program  eGATE-Live Internet Based Classes DLP  TarGATE-All India Test Series
Leaders in GATE Preparations  65+ Centers across India

www.engineeringonyourfingertips.ooo
© All rights reserved by Gateforum Educational Services Pvt. Ltd. No part of this booklet may be reproduced or utilized in any form without the written permission.

9
99/19
9/14
7EC
More Notes Join us Telegram-: http//:t.me/allexammentor
|www.engineeringonyourfingertips.ooo
ME| GATE-2016-PAPER-01 www.gateforum.com

17. Propane (C3H8) is burned in an oxygen atmosphere with 10% deficit oxygen with respect to the
stoichiometric requirement. Assuming no hydrocarbons in the products, the volume percentage of CO
in the products is
Key: 14.286
Exp: C3H8  5(0.9)O2  5(3.76)0.9N2  aCO  bCO2  4H2O  16.92N2
Carbon balance : 3  a  b
Qxygen balance : 9  a  2b  4
6  b  4
a  1, b  2
1
100  14.286%
7
18. Consider two hydraulic turbines having identical specific speed and effective head at the inlet. If the
speed ratio (N1/N2) of the two turbines is 2, then the respective power ratio (P1/P2) is
____.
Key: 0.25
Exp: Given: Specific speed of Turbine “1” (NS1) = Specific Speed of Turbine “2” (NS2)
Effective Head at inlet of Turbine “1” (H1) = Effective head at inlet of turbine “2” (H2)
and N1/N2 = 2
N P
Specific Speed of Turbine  NS  
H5 4
NS1 = NS2
N1 P1 N 2 P2

H15 4 H52 4
2
P1  N 2   1  1
2

       0.25
P2  N1   2  4
19. www.
The INCORRECT statement engi
about neer ingon
regeneration inyourf
vapor inger
power tips.
cycle isoo o
that
(A) it increases the irreversibility by adding the liquid with higher energy content to the steam
generator
(B) heat is exchanged between the expanding fluid in the turbine and the compressed fluid before heat
addition
(C) the principle is similar to the principle of Stirling gas cycle
(D) it is practically implemented by providing feed water heaters
Key: (A)
20. The “Jominy test” is used to find
(A) Young‟s modulus (B) hardenability
(C) yield strength (D) thermal conductivity
Key: (B)

 ICP–Intensive Classroom Program  eGATE-Live Internet Based Classes DLP  TarGATE-All India Test Series
Leaders in GATE Preparations  65+ Centers across India

www.engineeringonyourfingertips.ooo
© All rights reserved by Gateforum Educational Services Pvt. Ltd. No part of this booklet may be reproduced or utilized in any form without the written permission.

10
1010/
11010
/14
More Notes Join us Telegram-: http//:t.me/allexammentor
|www.engineeringonyourfingertips.ooo
ME| GATE-2016-PAPER-01 www.gateforum.com

21. Under optimal conditions of the process the temperatures experienced by a copper work piece in fusion
welding, brazing and soldering are such that
(A) Twelding > Tsoldering> Tbrazing (B) Tsoldering > Twelding > Tbrazing
(C) Tbrazing >Twelding > Tsoldering (D) Twelding > Tbrazing > Tsoldering
Key: (D)

22. The part of a gating system which regulates the rate of pouring of molten metal is
(A) pouring basin (B) runner (C) choke (D) ingate
Key: (C)

23. The non-traditional machining process that essentially requires vacuum is


(A) electron beam machining (B) electro chemical machining
(C) electro chemical discharge machining (D) electro discharge machining
Key: (A)
Exp: Electron beam machining requires vacuum, to avoid deflection of electrons

24. In an orthogonal cutting process the tool used has rake angle of zero degree. The measured cutting force
and thrust force are 500 N and 250 N, respectively. The coefficient of friction between the tool and
the chip is
Key: 0.5
Ft
Exp: Tan      
FC
250
Tan    0  
500
Tan  1  0.5
2

25. Match the following:


www.
engi
neer
ingonyour
finger
tips.
ooo
P. Feeler gauge I. Radius of an object
Q. Fillet gauge II. Diameter within limits by comparison
R. Snap gauge III. Clearance or gap between components
S. Cylindrical plug gauge IV. Inside diameter of straight hole

(A) P–III, Q–I, R–II, S–IV (B) P–III, Q–II, R–I, S–IV
(C) P–IV, Q–II, R–I, S–III (D) P–IV, Q–I, R–II, S–III
Key: (A)
Exp: Feeler gauge: Clearance or gap between components
Fillet gauge: Radius of an object
Snap gauge: Diameter within limits by comparison
Cylindrical plug gauge: Inside diameter of straight hole.
 ICP–Intensive Classroom Program  eGATE-Live Internet Based Classes DLP  TarGATE-All India Test Series
Leaders in GATE Preparations  65+ Centers across India

www.engineeringonyourfingertips.ooo
© All rights reserved by Gateforum Educational Services Pvt. Ltd. No part of this booklet may be reproduced or utilized in any form without the written permission.

11
1111/
11111
/14
More Notes Join us Telegram-: http//:t.me/allexammentor
|www.engineeringonyourfingertips.ooo
ME| GATE-2016-PAPER-01 www.gateforum.com

Q. No. 26 – 55 carry Two Marks Each

26. Consider the function f (x)  2x 3  3x 2 i n the domain [−1, 2]. The global minimum of f(x) is
_______
Key: -5
Exp: Given that, f  x   2x3  3x 2
 f   x   0  6x 2  6x  0
 x 2  x  0  x  x  1  0
 x  0; x  1
are Stationary points.
 f   x   12x  6
f   0  6  0
 f(x) has maximum at x = 0.
f  1  12 1  6  6  0
 f(x) has minimum at x = 1.
 f 1  2  3  1  local minimum value

But f  1  2  3  5
 Global minimum of f(x) = -5

27. If y=f(x)satisfies the boundary value problem y  9y  0, y(0)  0, y( / 2)  2, , then y( / 4) is
_________
Key: -1
Exp: Given D.E is y  9y  0

  D2  9 y  0

The A.E is D2  9  0www.


engi
neer
ingonyour
finger
tips.
ooo
 D2  9  D  3i
 y  c1 cos3x  c2 sin 3x … (1)
Given
y   / 2  2
y  0  0 and
i.e; x  0, y  0 i.e; x   / 2; y  2
From(1); 0  C1 From(1); 2  0  C2 (1)
 C2   2
 From (1); y   2 sin3x

   3   1 
 y     2 sin     2    1
4  4  2

 y    1
4
 ICP–Intensive Classroom Program  eGATE-Live Internet Based Classes DLP  TarGATE-All India Test Series
Leaders in GATE Preparations  65+ Centers across India

www.engineeringonyourfingertips.ooo
© All rights reserved by Gateforum Educational Services Pvt. Ltd. No part of this booklet may be reproduced or utilized in any form without the written permission.

12
1212/
11212
/14
More Notes Join us Telegram-: http//:t.me/allexammentor
|www.engineeringonyourfingertips.ooo
ME| GATE-2016-PAPER-01 www.gateforum.com

28. The value of the integral


 sin x
  x 2  2x  2
dx

evaluated using contour integration and the residue theorem is


(A) –  sin 1 / e (B)  cos 1 / e (C) sin(1)/e (D) cos(1)/e
Key: (A)
Exp: We know that sin x is the imaginary part of eix
eiz
∴ We consider the function f  z  
z  2z  2
2

Now, the poles of f(z) are given by z2  2z  2  0


2  4  4  2  2  4
z 
2 2
2  i2
  1 i
2
But Z  1  i is the only pole (simple) lie in the upper half of the Z-plane.
eiz
∴ Resf  z   Lt z   1  i  .

z 1i z 1i   z   1 i   z   1  i 
eiz ei 1i  ei 1 ei
 Lt   
z1i z  1  i  1  i  1  i 2i 2ie

eiz  ei  ei


Thus e z2  2z  2 dz  2 i  
 2ie  e
Equating imaginary parts on both sides we get
sin z    sin 1   sin 1
z
e
2
 2z  2
dz 
e

e

29. Gauss-Seidel method is used to solve the following equations (as per the given order):
x1  2x 2  3x 3  5
www.
engi
neer
ingonyour
finger
tips.
ooo
2x1  3x 2  x 3  1
3x1  2x 2  x 3  3
Assuming initial guess as x1  x 2  x 3  0, the value of x3 after the first iteration is ____
Key: -6
Exp: x11  0  0  5 ...1

2x11  3x 21  0  1 ...(2)


3x11  2x 21  x 31  3 ...  3

∴ From equation (1) x1   5


1

 ICP–Intensive Classroom Program  eGATE-Live Internet Based Classes DLP  TarGATE-All India Test Series
Leaders in GATE Preparations  65+ Centers across India

www.engineeringonyourfingertips.ooo
© All rights reserved by Gateforum Educational Services Pvt. Ltd. No part of this booklet may be reproduced or utilized in any form without the written permission.

13
1313/
11313
/14
More Notes Join us Telegram-: http//:t.me/allexammentor
|www.engineeringonyourfingertips.ooo
ME| GATE-2016-PAPER-01 www.gateforum.com

From equation (2), 2x11  3x 21  1


 3x 21 1  2x11
 1  2  5
 3x 21  9
9
 x 21   3  x 21  3
3
From equation (3), x 31  3  3x11  2x 21
 3  3  5   2  3
 3  15  6  6
 x 31  6
∴ After the first iteration, the value of x3 is -6.
30. A block of mass m rests on an inclined plane and is attached by a string to the wall as shown in the
figure. The coefficient of static friction between the plane and the block is 0.25. The string can withstand
a maximum force of 20 N. The maximum value of the mass (m) for which the string will not break and
the block will be in static equilibrium is kg.
Take cos   0.8and sin   0.6.

Acceleration due to gravity g = 10 m/s2

Key: 5 R
www.
engi
neer
ingonyour
finger
tips.
ooo
Exp: F  R   mg cos 
T
 0.25  m  10  0.8 m F
 2m
mgsin 
For equilibrium, 

T  F  mgsin 
 20  2m  m  10  0.6  m  5kg
mg mgsin 

 ICP–Intensive Classroom Program  eGATE-Live Internet Based Classes DLP  TarGATE-All India Test Series
Leaders in GATE Preparations  65+ Centers across India

www.engineeringonyourfingertips.ooo
© All rights reserved by Gateforum Educational Services Pvt. Ltd. No part of this booklet may be reproduced or utilized in any form without the written permission.

14
1414/
11414
/14
More Notes Join us Telegram-: http//:t.me/allexammentor
|www.engineeringonyourfingertips.ooo
ME| GATE-2016-PAPER-01 www.gateforum.com

31. A two-member truss PQR is supporting a load W. The axial forces in members PQ and QR are
respectively

(A) 2W tensile and 3W compressive

(B) 3W tensile and 2W compressive

(C) 3W compressive and 2W tensile


(D) 2Wcompressive and 3W tensile
Key: (B)
Exp: F.B.D of point Q
Fx  0
FQR sin 60
 FPQ  FQR sin 60  0 ...... 1 Q
FPQ
Fy  0 30
 FQR cos60  W  0 ....  2  60
FQR cos 60
W
 FQR   FQR  2W  compressive  FQR
cos60
From equation (1) W
FPQ  2Wsin 60  0

3
 FPQ  2W   3W  Tensile 
2

32. A horizontal bar with a constant cross-section is subjected to loading as shown in the figure. The
wsections
Young‟s moduli for the ww. enAB
gi
n eer
and i
BCng on
are yo
3E ur
and fi
E,n ger tips. ooo
respectively.

For the deflection at C to be zero, the ratio P/F is


Key: 4 A B B C
Exp:

FP 3E FP F E F
F.B.D

 
Since, net deflection at C is zero
 ICP–Intensive Classroom Program  eGATE-Live Internet Based Classes DLP  TarGATE-All India Test Series
Leaders in GATE Preparations  65+ Centers across India

www.engineeringonyourfingertips.ooo
© All rights reserved by Gateforum Educational Services Pvt. Ltd. No part of this booklet may be reproduced or utilized in any form without the written permission.

15
1515/
11515
/14
More Notes Join us Telegram-: http//:t.me/allexammentor
|www.engineeringonyourfingertips.ooo
ME| GATE-2016-PAPER-01 www.gateforum.com

 AB  BC  0


 F  P    F
0
A.3E AE
FP
 F  0
3
 4F  P  0
P
 4
F
33. The figure shows cross-section of a beam subjected to bending. The area moment of inertia

(in mm4) of this cross-section about its base is _________.

Key: 1873 - 1879


34. A simply-supported beam of length 3L is subjected to the loading shown in the figure.

www.
engi
neer
ingonyour
finger
tips.
ooo

It is given that P = 1 N, L = 1 m and Young‟s modulus E = 200 GPa. The cross-section is a square with
dimension 10 mm × 10 mm. The bending stress (in Pa) at the point A located at the top surface of the
beam at a distance of 1.5L from the left end is
(Indicate compressive stress by a negative sign and tensile stress by a positive sign.)
Key: 0 P P
Exp: A L L L B
A
RA RB

Taking moment about B


MB  0  R A  3L  P  2L  PL  0  R A   P 3

 ICP–Intensive Classroom Program  eGATE-Live Internet Based Classes DLP  TarGATE-All India Test Series
Leaders in GATE Preparations  65+ Centers across India

www.engineeringonyourfingertips.ooo
© All rights reserved by Gateforum Educational Services Pvt. Ltd. No part of this booklet may be reproduced or utilized in any form without the written permission.

16
1616/
11616
/14
More Notes Join us Telegram-: http//:t.me/allexammentor
|www.engineeringonyourfingertips.ooo
ME| GATE-2016-PAPER-01 www.gateforum.com

Fy  0  R B  R A  0  R B  P 3

Taking moment about A


M A  0

R A  1.5L  0.5PL  M A  Assuming MA anticlockwise 


P
   1.5L  0.5PL  M A
3
 MA  0

M b
we know,   b  Bendingstress   0 since, M A  0
I y
35. A slider crank mechanism with crank radius 200 mm and connecting rod length 800 mm is shown. The
crank is rotating at 600 rpm in the counterclockwise direction. In the configuration shown, the crank
makes an angle of 90o with the sliding direction of the slider, and a force of 5 kN is acting on the slider.
Neglecting the inertia forces, the turning moment on the crank (in kN-m) is _____

Key: 1
Exp: T cos   5
Moment about crank shaft  M   T  r sin  90   
  T cos   r T
 5  0.2 T

 1kN m
5
Alternate method:
H
  
M  5  0.2 sin  90   sin 180 
www. eng2ineer i
ng  onyour
finger
tips.
ooo
 1kN m
36. In the gear train shown, gear 3 is carried on arm 5. Gear 3 meshes with gear 2 and gear 4. The number of
teeth on gear 2, 3, and 4 are 60, 20, and 100, respectively. If gear 2 is fixed and gear 4 rotates with an
angular velocity of 100 rpm in the counterclockwise direction, the angular speed of arm 5 (in rpm) is

(A) 166.7 counterclockwise (B) 166.7 clockwise


(C) 62.5 counterclockwise (D) 62.5 clockwise
Key: (C)

 ICP–Intensive Classroom Program  eGATE-Live Internet Based Classes DLP  TarGATE-All India Test Series
Leaders in GATE Preparations  65+ Centers across India

www.engineeringonyourfingertips.ooo
© All rights reserved by Gateforum Educational Services Pvt. Ltd. No part of this booklet may be reproduced or utilized in any form without the written permission.

17
1717/
11717
/14
More Notes Join us Telegram-: http//:t.me/allexammentor
|www.engineeringonyourfingertips.ooo
ME| GATE-2016-PAPER-01 www.gateforum.com

Exp:
Arm Gears
2 3 4
All gear locked to
60 60
Arm & 2 is given x 0 x x x
20 100
Rotations in C.C.W
Arm also given y yx 60 60
y y x y x
Rotations in C.C.W 20 100
Given: x  y 0 ...(1)
3
y  x 100 ...(2)
5
3 5  100
y  y  100  y   62.5 counter clockwise
5 8
37. A solid disc with radius a is connected to a spring at a point d above the center of the disc. The other
end of the spring is fixed to the vertical wall. The disc is free to roll without slipping on the ground. The
mass of the disc is M and the spring constant is K. The polar moment of inertia for the disc about its
centre is J  Ma 2 / 2.

The natural frequency of this system in rad/s is given by

2K(a  d)2 www.


engi
2K neer
ingonyourf
2K(ainge
d) 2 r
tips.
ooo
K(a  d)2
(A) (B) (C) (D)
3Ma 2 3M Ma 2 Ma 2
Key: (A)
Exp: Apply D'Alembert Principle:
 k  a  d    a  d    I cm  Ma 2  
 0 cm

k a  d
2

 0
 Ma 2 
  Ma 2  x
 2 
2k  a  d 
2

n 
3Ma 2 k a  d 
cm

 ICP–Intensive Classroom Program  eGATE-Live Internet Based Classes DLP  TarGATE-All India Test Series
Leaders in GATE Preparations  65+ Centers across India

www.engineeringonyourfingertips.ooo
© All rights reserved by Gateforum Educational Services Pvt. Ltd. No part of this booklet may be reproduced or utilized in any form without the written permission.

18
1818/
11818
/14
More Notes Join us Telegram-: http//:t.me/allexammentor
|www.engineeringonyourfingertips.ooo
ME| GATE-2016-PAPER-01 www.gateforum.com

38. The principal stresses at a point inside a solid object are 1  100 MPa, 2  100 MPa and 3  0
MPa. The yield strength of the material is 200 MPa. The factor of safety calculated using Tresca
(maximum shear stress) theory is nT and the factor of safety calculated using von Mises (maximum
distortional energy) theory is nV. Which one of the following relations is TRUE?

(A) n T   3 / 2 n v

(B) n T   3n v

(C) n T  n v

(D) n v   3n T

Key: (C)

  1  3   2  3   3  1  
Exp: max  max  ,  ,     50 Mpa
 2   2   2  

max 
S yt / 2
 50 
 200 / 2    2
T T
T

  1  2 2   2  3 2   3  1 2   Syt 2
   
 2   v 

 12  22  1 2   Syt / v 


2

1  2  100 & Syt  200


2
 200 
100      v  2
2

 v 
 T  v

39. An inverted U-tube manometer is used to measure the pressure difference between two pipes A and B, as
shown in the figure. w ww
Pipe A.eisng i
neer
carrying i
ng(specific
oil onyour fi
nger
gravity =t
ips.
0.8) o oo
and pipe B is carrying water. The
densities of air and water are 1.16 kg/m3 and 1000 kg/m3, respectively. The pressure difference between
pipes A and B is kPa.

Acceleration due to gravity g = 10 m/s2.

 ICP–Intensive Classroom Program  eGATE-Live Internet Based Classes DLP  TarGATE-All India Test Series
Leaders in GATE Preparations  65+ Centers across India

www.engineeringonyourfingertips.ooo
© All rights reserved by Gateforum Educational Services Pvt. Ltd. No part of this booklet may be reproduced or utilized in any form without the written permission.

19
1919/
11919
/14
More Notes Join us Telegram-: http//:t.me/allexammentor
|www.engineeringonyourfingertips.ooo
ME| GATE-2016-PAPER-01 www.gateforum.com

Key: -2.199

Exp: PA  oil gh1  air gh 2  PB water g  h1  h 2  h 3 


PA  PB  0.8 103  10  0.2 1.16  10  0.08 103 10  0.2  0.08  0.1
 1600  0.928  3800
  2199.072 Pa
PA  PB   2.199 kPa

40. Oil (kinematic viscosity, Voil  1.0 × 10−5 m2/s) flows through a pipe of 0.5 m diameter with a
velocity of 10 m/s. Water (kinematic viscosity, vw= 0.89 × 10−6 m2/s) is flowing through a model pipe
of diameter 20 mm. For satisfying the dynamic similarity, the velocity of water (in m/s) is _____ .

Key: 22.25
Prototype
Exp:
oil  1.0 105 m2 /s
V0 =10m/s oil doil  0.5m

model
 w  0.89 106 m2 /s
Vw = ? water
d w  0.02m

There is no free surface inside pipe flow, therefore only Reynold numbers are equal in both the cases.

 R e oil   R e water  
VD   VD 
  
  mod el    prototype
Vw  0.02 10  0.5
  Vw  22.25m/sec
0.89 106 1.0 105

41. A steady laminar boundary layer is formed over a flat plate as shown in the figure. The free stream
velocity of the fluid is Uo. The velocity profile at the inlet a-b is uniform, while that at a
  y   y 2 
ww
downstream location c-d isw.eng
given ine
by ue
r
i
Un g ony
0 2  o
u
r fi
n g.er
tips.
ooo
       

 bd , leaving through the horizontal section b-d to that entering


The ratio of the mass flow rate, m
through the vertical section a-b is .
Key: 0.33
 ICP–Intensive Classroom Program  eGATE-Live Internet Based Classes DLP  TarGATE-All India Test Series
Leaders in GATE Preparations  65+ Centers across India

www.engineeringonyourfingertips.ooo
© All rights reserved by Gateforum Educational Services Pvt. Ltd. No part of this booklet may be reproduced or utilized in any form without the written permission.

20
2020/
12020
/14
More Notes Join us Telegram-: http//:t.me/allexammentor
|www.engineeringonyourfingertips.ooo
ME| GATE-2016-PAPER-01 www.gateforum.com

Exp:
This is steady state process, so time derivative will be zero. Choose b-a-c-d-b as Control Volume (CV)
   
  V.n  dA  0

cs 

where C.S  stands for control surface &



n is unit vector perpendicular to Area of flow which is always taken outside of side of cross-
sectional Area.
Let fluid is incompressible, ρ= constant

               
a  d b
  V.n  dA    V.n  dA    V.n  dA    V.n  dA  0
b  a  c  d 

  
 
  y 2  y 2 
  U0 b.dy    0.n  dA   U 0 2       bdy  m
 bd  0
0 a  c        

  
 U 0 b  0  V b     m bd  0
 3
2 
 U 0 b  bV0   m bd  0
3
 1
m bd   U0 b  
3

m bd
 0.33
 U 0 b
42. A steel ball of 10 mm diameter at 1000 K is required to be cooled to 350 K by immersing it in a water
environment at 300 K. The convective heat transfer coefficient is 1000 W/m2-K. Thermal
conductivity of steel is 40 W/m-K. The time constant for the cooling process τ is 16 s. The time
required (in s) to reach the final temperature is _____
www.
engi
neer
ingonyour
finger
tips.
ooo
Key: 42.22
Exp: Given
d = 10 mm = 0.01 m
W W
t i  1000K;t  350K;t   300K; k  40 ;h  1000 2 ; th  16s
mK mK
t  t 
 e th
ti  t
 t  t   
ln    th
 ti  t 
 350  300  
ln    16
 1000  300 
  42.22s

 ICP–Intensive Classroom Program  eGATE-Live Internet Based Classes DLP  TarGATE-All India Test Series
Leaders in GATE Preparations  65+ Centers across India

www.engineeringonyourfingertips.ooo
© All rights reserved by Gateforum Educational Services Pvt. Ltd. No part of this booklet may be reproduced or utilized in any form without the written permission.

21
2121/
12121
/14
More Notes Join us Telegram-: http//:t.me/allexammentor
|www.engineeringonyourfingertips.ooo
ME| GATE-2016-PAPER-01 www.gateforum.com

43. An infinitely long furnace of 0.5 m × 0.4 m cross-section is shown in the figure below. Consider all
surfaces of the furnace to be black. The top and bottom walls are maintained at temperature T1 = T3
= 927oC while the side walls are at temperature T2 = T4 = 527 oC. The view factor, F1-2 is 0.26. The
net radiation heat loss or gain on side 1 is W/m.

Stefan-Boltzmann constant = 5.67 × 10−8 W/m2-K4

Key: 24530.688
44. A fluid (Prandtl number, Pr = 1) at 500 K flows over a flat plate of 1.5 m length, maintained at
300 K. The velocity of the fluid is 10 m/s. Assuming kinematic viscosity, ν = 30 × 10−6 m2/s, the
thermal boundary layer thickness (in mm) at 0.5 m from the leading edge is .
Key: 6.124
Exp: Pr = 1
ux 10  0.5
Re x  
 30  106
 166666.67

 1.67  105
Hydrodynamic boundary layer thickness
5x www.
5  0.5 engi
neer
ingonyour
finger
tips.
ooo
h x  
Re x 1.67  105

 6.124  103 m
If Pr = 1
h x  Tx  6.124  103 m

 6.124 mm
 Thermal boundary layer thickness = 6.124 mm.
45. For water at 25oC, dps / dTs  0.189kPa / K (ps is the saturation pressure in kPa and Ts is the saturation
temperature in K) and the specific volume of dry saturated vapour is 43.38 m3/kg. Assume that the
specific volume of liquid is negligible in comparison with that of vapour. Using the Clausius- Clapeyron
equation, an estimate of the enthalpy of evaporation of water at 25oC (in kJ/kg) is ______.
Key: 2443.24

 ICP–Intensive Classroom Program  eGATE-Live Internet Based Classes DLP  TarGATE-All India Test Series
Leaders in GATE Preparations  65+ Centers across India

www.engineeringonyourfingertips.ooo
© All rights reserved by Gateforum Educational Services Pvt. Ltd. No part of this booklet may be reproduced or utilized in any form without the written permission.

22
2222/
12222
/14
More Notes Join us Telegram-: http//:t.me/allexammentor
|www.engineeringonyourfingertips.ooo
ME| GATE-2016-PAPER-01 www.gateforum.com

dPs h fg h fg
Exp:   0.189 
dTs Ts  g   f   25  273 43.38  0 
 h fg  2443.248kJ kg

46. An ideal gas undergoes a reversible process in which the pressure varies linearly with volume. The
conditions at the start (subscript 1) and at the end (subscript 2) of the process with usual notation are:
p1  100kPa, V1  0.2m3 and p 2  200 kPa, V2  0.1m3 and the gas constant, R = 0.275 kJ/kg-K. The
magnitude of the work required for the process (in kJ) is .
Key: 15
Exp: Pressure varies linearly with volume.
P = a + bv
P1  a  bv1

 100  a  b  0.2      (1)

P2  a  bv2

 200  a  b  0.1      (2)

Solving (1) & (2)

100
100  0.1b  b   1000
0.1
b  1000

Substituting in any of the equations to get „a‟.


 100  a   1000  0.2   a  300
2 2
W   pdv    a  bv  dv
1 1
2
www. engi neer ingonvy 2
ouvr1f
2
ingerti
ps. ooo
   300  1000v  dv  300  v2  v1   1000  2 
1  2 
  0.1   0.2  
2 2

 300  0.1  0.2   1000    30   15  15kJ


 2 
 
 Magnitude of work required is 15 kJ.
47. In a steam power plant operating on an ideal Rankine cycle, superheated steam enters the turbine at 3
MPa and 350oC. The condenser pressure is 75 kPa. The thermal efficiency of the cycle is
________ percent.
Given data:
For saturated liquid, at P = 75 kPa, h f  384.39kJ / kg, vf  0.001037m3 / kg, sf  1.213kJ / kg  K
At 75 kPa, hfg = 2278.6 kJ/kg, sfg = 6.2434 kJ/kg-K

At P = 3 MPa and T = 350oC (superheated steam), H  3115.3kJ / kg, s  6.7428kJ / kg  K


 ICP–Intensive Classroom Program  eGATE-Live Internet Based Classes DLP  TarGATE-All India Test Series
Leaders in GATE Preparations  65+ Centers across India

www.engineeringonyourfingertips.ooo
© All rights reserved by Gateforum Educational Services Pvt. Ltd. No part of this booklet may be reproduced or utilized in any form without the written permission.

23
2323/
12323
/14
More Notes Join us Telegram-: http//:t.me/allexammentor
|www.engineeringonyourfingertips.ooo
ME| GATE-2016-PAPER-01 www.gateforum.com

Key: 25.99%
Exp: 1

3 2

Given:- P1  P4  3MPa, T1  350o C  350  273  623K


h1  3115.3kJ / kg, S1  6.7428kJ / kgK
P2  P3  75kPa.
h f2  h f3  384.39kJ / kg, h fg2  2278.6kJ / kg
sf2  sf3  1.213kJ / kgK, sfg2  6.2434kJ / kgK
vf3  0.001037m3 / kg
S1  S2
 S1  Sf2  x 2Sfg2
6.7428  1.213  x 2  6.2434
x 2  0.886
h 2  h f2  x 2 h fg2  384.39  0.886  2278.6
 2403.2296kJ / kg
 2403.23
Turbine work, WT  h1  h 2  3115.3  2403.23  712.07kJ / kg
www.
engi
neer
ingonyour
Pump work, Wp  vf3  P4  P3   0.001037(3000  75)
finger
tips.
ooo
 3.03k J / kg
h 4  h 3  Wp  h f3  Wp  384.39  3.03
 387.42kJ / kg
Heat supplied to boiler
 h1  h 4  3115.3  387.42
 2727.88kJ / kg
Net work done  WT  Wp  712.07  3.03  709.04kJ / kg
Net work done 709.04
Thermal efficiency      100  25.99%
Heat supplied 2727.88

 ICP–Intensive Classroom Program  eGATE-Live Internet Based Classes DLP  TarGATE-All India Test Series
Leaders in GATE Preparations  65+ Centers across India

www.engineeringonyourfingertips.ooo
© All rights reserved by Gateforum Educational Services Pvt. Ltd. No part of this booklet may be reproduced or utilized in any form without the written permission.

24
2424/
12424
/14
More Notes Join us Telegram-: http//:t.me/allexammentor
|www.engineeringonyourfingertips.ooo
ME| GATE-2016-PAPER-01 www.gateforum.com

48. A hypothetical engineering stress-strain curve shown in the figure has three straight lines PQ, QR, RS
with coordinates P(0,0), Q(0.2,100), R(0.6,140) and S(0.8,130). 'Q' is the yield point, 'R' is the UTS
point and 'S' the fracture point.

The toughness of the material (in MJ/m3) is ________ .


Key: 0.85
Exp:

R  0.6,140 
160
140 4
3
2
S  0.8,130 
120
100
Q  0.2,100 
Stress in

80 1
MPa

60
40
20
0
P  0,0 0.2 0.4 0.6 0.8 1

Strainin %

Toughness of material
Total area – [Area of 1 + Area of 2 + Area of 3 + Area of 4]
www.
engi
neer
ingonyour
finger
tips.
ooo

 140 

0.8   1
    100 
100   2  0.2
100
 40 
0.2
100
1

  40 
2
0.4
100
1
  10 
2
0.2 
100   
 1.12   0.1  0.08  0.08  0.01  0.85MJ m3

 ICP–Intensive Classroom Program  eGATE-Live Internet Based Classes DLP  TarGATE-All India Test Series
Leaders in GATE Preparations  65+ Centers across India

www.engineeringonyourfingertips.ooo
© All rights reserved by Gateforum Educational Services Pvt. Ltd. No part of this booklet may be reproduced or utilized in any form without the written permission.

25
2525/
12525
/14
More Notes Join us Telegram-: http//:t.me/allexammentor
|www.engineeringonyourfingertips.ooo
ME| GATE-2016-PAPER-01 www.gateforum.com

49. Heat is removed from a molten metal of mass 2 kg at a constant rate of 10 kW till it is completely
solidified. The cooling curve is shown in the figure.

Assuming uniform temperature throughout the volume of the metal during solidification, the latent heat
of fusion of the metal (in kJ/kg) is .
Key: (50)
Exp: Given 20sec
m  2 kg; Q  10 kcal 10s
873k
Since heat is removed at constant rate 873k
10kw is removed per second.
So, latest heat = mL  10 kW  10sec  10 sec is time requiredfor phasechange
2 L  100 kJ
L  50kJ / kg

The tool life equation for HSS tool is VT f d  Constant. The tool life (T) of 30 min is obtained
0.14 0.7 0.4
50.
using the following cutting conditions:
V = 45 m/min, f = 0.35 mm, d = 2.0 mm
If speed (V), feed (f) and depth of cut (d) are increased individually by 25%, the tool life (in min) is
(A) 0.15 www.
(B) e ngi
1.06 neer
ingonyour
(C) finger
22.50 t
ips.
ooo(D) 30.0
Key: (B)
Exp: VT 0.14 f 0.7 d 0.4  constant
V1  45m/min; f1  0.35mm; d1  2mm
T1  30min; V2  1.25V1;
f 2  1.25f1 ; d 1 1.25d1 ;
T2  ?

 45  T10.14 f10.7 d10.4  1.25  45  T20.14 1.250.7 f10.7  1.250.4 d10.4


 T10.14 1.25 1.250.7 1.250.4  T20.14
 30 
0.14

 T2   1.055 1.06
1.252.1 / 0.14

 ICP–Intensive Classroom Program  eGATE-Live Internet Based Classes DLP  TarGATE-All India Test Series
Leaders in GATE Preparations  65+ Centers across India

www.engineeringonyourfingertips.ooo
© All rights reserved by Gateforum Educational Services Pvt. Ltd. No part of this booklet may be reproduced or utilized in any form without the written permission.

26
2626/
12626
/14
More Notes Join us Telegram-: http//:t.me/allexammentor
|www.engineeringonyourfingertips.ooo
ME| GATE-2016-PAPER-01 www.gateforum.com

51. A cylindrical job with diameter of 200 mm and height of 100 mm is to be cast using modulus
method of riser design. Assume that the bottom surface of cylindrical riser does not contribute as cooling
surface. If the diameter of the riser is equal to its height, then the height of the riser (in mm) is
(A) 150 (B) 200 (C) 100 (D) 125
Key: (A)
Exp: dc  200 mm dr  h r C  Casting
h c  100 mm hr  ? R  Rises

M r  1.2 MC

V V
   1.2  
 S r  S C
 2  2
d r hr dc h c
4  1.2 4
 
d r h r  d r2 d c h c  d c2 x 2
4 4
dr 2 h r 1.2d c2 h c
 
4d r h r  d r2 4d c h c  2d c2
1.2   200   100
2
h 3r
   d v  h r 
4h 2r  h 2r 4  200  100  2  2002
hr 200  10
  1.2 
5 4  100  2  200
200  100
 h r  1.2  5   125  1.2  150
4  200
52. A 300 mm thick slab is being cold rolled using roll of 600 mm diameter. If the coefficient of
friction is 0.08, the maximum possible reduction (in mm) is .
Key: 1.92
Exp:  h max   2 R

  0.08  300
2
www.
engi
neer
ingonyour
finger
tips.
ooo
 1.92 mm

53. The figure below represents a triangle PQR with initial coordinates of the vertices as P(1,3),
Q(4,5) and R(5,3.5). The triangle is rotated in the X-Y plane about the vertex P by angle θ in clockwise
direction. If sin θ = 0.6 and cos θ = 0.8, the new coordinates of the vertex Q are

(A) (4.6, 2.8) (B) (3.2, 4.6) (C) (7.9, 5.5) (D) (5.5, 7.9)
 ICP–Intensive Classroom Program  eGATE-Live Internet Based Classes DLP  TarGATE-All India Test Series
Leaders in GATE Preparations  65+ Centers across India

www.engineeringonyourfingertips.ooo
© All rights reserved by Gateforum Educational Services Pvt. Ltd. No part of this booklet may be reproduced or utilized in any form without the written permission.

27
2727/
12727
/14
More Notes Join us Telegram-: http//:t.me/allexammentor
|www.engineeringonyourfingertips.ooo
ME| GATE-2016-PAPER-01 www.gateforum.com

Key: (A)
Exp: y

Q  4,5

P 1,3

x
0

Rotate PQ by  in clockwise direction


y Q  xo , y0 

Q'  x, y 

P 1,3, 

x
0
Rotation of point Q about point P in anticlockwise is given as

x n  x p   x o  x p  cos    yo  yp  sin  ...(1)

yn  yp   x o  x p  sin    yo  yp  cos  ...(2)

For clockwise rotation  will be   . So the desired equation will be

x n  x p   x o  x p  cos    yo  yp  sin  ...(3)

1   4  1  0.8   5  3  0.6
 1 2.4 1.2  4.6
www.
engi
neer
ingonyour
finger
tips.ooo
yn  yp   yo  yp  sin    yo  yp  cos  ...(4)

 3   4  1  0.6   5  3  0.8
 3  1.8  1.6

yn  2.8

54. The annual demand for an item is 10,000 units. The unit cost is Rs. 100 and inventory carrying charges
are 14.4% of the unit cost per annum. The cost of one procurement is Rs. 2000. The time between two
consecutive orders to meet the above demand is ______ month(s).
Key: 2

 ICP–Intensive Classroom Program  eGATE-Live Internet Based Classes DLP  TarGATE-All India Test Series
Leaders in GATE Preparations  65+ Centers across India

www.engineeringonyourfingertips.ooo
© All rights reserved by Gateforum Educational Services Pvt. Ltd. No part of this booklet may be reproduced or utilized in any form without the written permission.

28
2828/
12828
/14
More Notes Join us Telegram-: http//:t.me/allexammentor
|www.engineeringonyourfingertips.ooo
ME| GATE-2016-PAPER-01 www.gateforum.com

Exp: D= 10,000, Cu = Rs 100, Ch= 0.144×Cu, Co=Rs 2,000.


2DC0 2  10,000  2000 Q*
Q*    1666.67 units
Ch 0.144  100

Q * 1666.67
We know T    0.1667 years = 2 months
D 10,000
T

55. Maximize Z=15X1 + 20X2


subject to
12X1+ 4X2 ≥ 36
12X1 − 6X2 ≤ 24
X1, X2 ≥ 0
The above linear programming problem has
(A) infeasible solution (B) unbounded solution
(C) alternative optimum solutions (D) degenerate solution
Key: (B)
Exp: Max Z  15x1  20x 2
Subject to
12x1  4x 2  36
12x1  6x 2  24
x1 , x 2  0
Since, there is no limitation of boundary for the feasible region therefore, the LPP has unbounded
solution.

www.
engi
neer
ingonyour
finger
tips.
ooo

 ICP–Intensive Classroom Program  eGATE-Live Internet Based Classes DLP  TarGATE-All India Test Series
Leaders in GATE Preparations  65+ Centers across India

www.engineeringonyourfingertips.ooo
© All rights reserved by Gateforum Educational Services Pvt. Ltd. No part of this booklet may be reproduced or utilized in any form without the written permission.

29
2929/
12929
/14
More Notes Join us Telegram-: http//:t.me/allexammentor
|www.engineeringonyourfingertips.ooo
ME| GATE-2016-PAPER-02 www.gateforum.com

General Aptitude
Q. No. 1 – 5 Carry One Mark Each

1. The volume of a sphere of diameter 1 unit is than the volume of a cube of side 1 unit.
(A) least (B) less (C) lesser (D) low
Key: (B)

2. The unruly crowd demanded that the accused be without trial.


(A) hanged (B) hanging (C) hankering (D) hung
Key: (A)

3. Choose the statement(s) where the underlined word is used correctly:


(i) A prone is a dried plum.
(ii) He was lying prone on the floor.
(iii) People who eat a lot of fat are prone to heart disease.
(A) (i) and (iii) only (B) (iii) only (C) (i) and (ii) only (D) (ii) and (iii) only
Key: (D)

4. Fact: If it rains, then the field is wet.


Read the following statements:
(i) It rains
(ii) The field is not wet
(iii) The field is wet
(iv) It did not rain
Which one of the options given below is NOT logically possible, based on the given fact?
(A) If (iii), then (iv). (B) If (i), then (iii).
(C) If (i), then (ii). (D) If (ii), then (iv).
Key: (C)

5. A window is made up of a square portion and an equilateral triangle portion above it. The base of the
wwwwith
triangular portion coincides .
engine
the er i
uppern gon
side ofyour
the finge
square. r
Iftips
the .ooo of the window is 6 m, the
perimeter
area of the window in m2 is .
(A) 1.43 (B) 2.06 (C) 2.68 (D) 2.88

Key: (B) x
x
Exp:
3x+2x=6
5x=6
x=6/5 x
x
6 6 36
Area of square   
5 5 25
3 2 3 6 6 3 36
Area of triangle       .
4 4 5 5 4 25 x
 3  36
1     2.06
 4  25

 ICP–Intensive Classroom Program  eGATE-Live Internet Based Classes DLP  TarGATE-All India Test Series
www.engineeringonyourfingertips.ooo
Leaders in GATE Preparations  65+ Centers across India
© All rights reserved by Gateforum Educational Services Pvt. Ltd. No part of this booklet may be reproduced or utilized in any form without the written permission.

1
11/11
More Notes Join us Telegram-: http//:t.me/allexammentor 1/14
|www.engineeringonyourfingertips.ooo
ME| GATE-2016-PAPER-02 www.gateforum.com

Q. No. 6 – 10 Carry Two Marks Each

6. Students taking an exam are divided into two groups, P and Q such that each group has the same number
of students. The performance of each of the students in a test was evaluated out of 200 marks. It was
observed that the mean of group P was 105, while that of group Q was 85. The standard deviation of
group P was 25, while that of group Q was 5. Assuming that the marks were distributed on a normal
distribution, which of the following statements will have the highest probability of being TRUE?
(A) No student in group Q scored less marks than any student in group P.
(B) No student in group P scored less marks than any student in group Q.
(C) Most students of group Q scored marks in a narrower range than students in group P.
(D) The median of the marks of group P is 100.
Key: (C)
68-95-97 rule
P Q

80 125 150 75 80 90 95
      2   2       2
105 85
 
95% of students in P scores between 65 to 150
95% of students in Q score between 75 to 95.
D is not correct www. engi neer i
ngonyour fi
nger t ips. ooo
median = mean for normal distribution.
C is correct answer.
7. A smart city integrates all modes of transport, uses clean energy and promotes sustainable use of
resources. It also uses technology to ensure safety and security of the city, something which critics argue,
will lead to a surveillance state.
Which of the following can be logically inferred from the above paragraph?
(i) All smart cities encourage the formation of surveillance states.
(ii) Surveillance is an integral part of a smart city.
(iii) Sustainability and surveillance go hand in hand in a smart city.
(iv) There is a perception that smart cities promote surveillance.
(A) (i) and (iv) only (B) (ii) and (iii) only
(C) (iv) only (D) (i) only
Key: (C)

 ICP–Intensive Classroom Program  eGATE-Live Internet Based Classes DLP  TarGATE-All India Test Series
www.engineeringonyourfingertips.ooo
Leaders in GATE Preparations  65+ Centers across India
© All rights reserved by Gateforum Educational Services Pvt. Ltd. No part of this booklet may be reproduced or utilized in any form without the written permission.

2
22/12
More Notes Join us Telegram-: http//:t.me/allexammentor 2/14
|www.engineeringonyourfingertips.ooo
ME| GATE-2016-PAPER-02 www.gateforum.com

8. Find the missing sequence in the letter series.


B, FH, LNP, _ _ _ _.
(A) SUWY (B) TUVW (C) TVXZ (D)TWXZ
Key: (C)

9. The binary operation  is defined as a  + b = ab+(a+b), where a and b are any two real numbers.
The value of the identity element of this operation, defined as the number x such that a  x = a, for any a,
is .
(A) 0 (B) 1 (C) 2 (D) 10
Key: (A)
Exp: ax  a  ax   a  x   a
 x 1  a   0  x  0 is the identity element

Which of the following curves represents the y  ln  e  for x  2 ?


 sin  x  

10. 
 
Here, x represents the abscissa and y represents the ordinate.

(A)

www.
engi
neer
ingonyour
finger
tips.
ooo

(B)

 ICP–Intensive Classroom Program  eGATE-Live Internet Based Classes DLP  TarGATE-All India Test Series
www.engineeringonyourfingertips.ooo
Leaders in GATE Preparations  65+ Centers across India
© All rights reserved by Gateforum Educational Services Pvt. Ltd. No part of this booklet may be reproduced or utilized in any form without the written permission.

3
33/13
More Notes Join us Telegram-: http//:t.me/allexammentor 3/14
|www.engineeringonyourfingertips.ooo
ME| GATE-2016-PAPER-02 www.gateforum.com

(C)

(D)

Key: (C)

Mechanical Engineering
www.
engi
neer
ingonyour
finger
tips.
ooo

Q. No. 1 – 25 Carry One Mark Each

1. The condition for which the eigen values of the matrix


2 1 
A  are positive, is
1 k 
(A) k > 1/2 (B) k > −2 (C) k> 0 (D) k < −1/2
Key: (A)
Exp: By the properties of eigen values & eigen vectors, if all the principal minors of „A‟ are +Ve then all the
eigen values of „A‟ are also +Ve.
1
 A 22  0 for k 
2
1
So k 
2

 ICP–Intensive Classroom Program  eGATE-Live Internet Based Classes DLP  TarGATE-All India Test Series
www.engineeringonyourfingertips.ooo
Leaders in GATE Preparations  65+ Centers across India
© All rights reserved by Gateforum Educational Services Pvt. Ltd. No part of this booklet may be reproduced or utilized in any form without the written permission.

4
44/14
More Notes Join us Telegram-: http//:t.me/allexammentor 4/14
|www.engineeringonyourfingertips.ooo
ME| GATE-2016-PAPER-02 www.gateforum.com

2. The values of x for which the function


x 2  3x  4
f (x)  is NOT continuous are
x 2  3x  4
(A) 4 and −1 (B) 4 and 1 (C) −4 and 1 (D) −4 and −1
Key: (C)
x 2  3x  4
Exp: The function f (x)  2 is not continuous at x  4 &1; since f(x) does not exists at x=-4 &1.
x  3x  4

3. Laplace transform of cos(ωt) is


s  s 
(A) (B) 
(C) (D)
s  2
2
s  2 s  2
2
s  2
2

Key: (A)
Exp: By the L.T of standard functions

4. A function f of the complex variable z  x  iy, is given as f (x, y)  u(x, y)  iv(x, y), where
u(x, y)  2kxy and v(x, y)  x 2  y 2 . The value of k, for which the function is analytic, is _____ .
Key: -1
Exp: From C-R equation; we have
u v u v
 & 
x y y dx
 u(x, y)  2kxy v(x, y)  x 2  y 2
u u v v
  2ky  2kx   2x;  2y
x y x y
u v
 
x y
 2ky  2y
 k  1

5. Numerical integrationw wwtrapezoidal


using .engi nee ring
rule ony
gives ou
the rfi
bestn ger
result t
ips
for a.
ooovariable function, which is
single
(A) linear (B) parabolic (C) logarithmic (D) hyperbolic
Key: (A)
6. A point mass having mass M is moving with a velocity V at an angle θ to the wall as shown in the figure.
The mass undergoes a perfectly elastic collision with the smooth wall and rebounds. The total change
(final minus initial) in the momentum of the mass is

(A) 2MV cos ˆj (B) 2MVsin ˆj (C) 2MVcos ˆj (D) 2MVsin ˆj
Key: (D)
 ICP–Intensive Classroom Program  eGATE-Live Internet Based Classes DLP  TarGATE-All India Test Series
www.engineeringonyourfingertips.ooo
Leaders in GATE Preparations  65+ Centers across India
© All rights reserved by Gateforum Educational Services Pvt. Ltd. No part of this booklet may be reproduced or utilized in any form without the written permission.

5
55/15
More Notes Join us Telegram-: http//:t.me/allexammentor 5/14
|www.engineeringonyourfingertips.ooo
ME| GATE-2016-PAPER-02 www.gateforum.com

Exp: Let w is the velocity after collision

V sin j
w cos  i

V

 w

V cos  i  w sin  j
Beforecollision After collision

velocity of separation
e  1  e  1, for perfectly elastic collision
velocity of approach
 w sin ˆj  Vsin ˆj ______ (1)
Change in momentum = final momentum – initial momentum


P   w sin j  V sin j M  
 2MV sin  j  w sin j  V sin j
7. A shaft with a circular cross-section is subjected to pure twisting moment. The ratio of the
maximum shear stress to the largest principal stress is
(A) 2.0 (B) 1.0 (C) 0.5 (D) 0
Key: (B)
Exp: Mohr 's circle
xy xy

www.
engi
neer
ingon
your
finger
tips.
ooo
xy xy
xy
xy
max  xy
1  xy where, 1 is largest principal stress
max
 1
1

8. A thin cylindrical pressure vessel with closed-ends is subjected to internal pressure. The ratio of
circumferential (hoop) stress to the longitudinal stress is
(A) 0.25 (B) 0.50 (C) 1.0 (D) 2.0
Key: (D)

 ICP–Intensive Classroom Program  eGATE-Live Internet Based Classes DLP  TarGATE-All India Test Series
www.engineeringonyourfingertips.ooo
Leaders in GATE Preparations  65+ Centers across India
© All rights reserved by Gateforum Educational Services Pvt. Ltd. No part of this booklet may be reproduced or utilized in any form without the written permission.

6
66/16
More Notes Join us Telegram-: http//:t.me/allexammentor 6/14
|www.engineeringonyourfingertips.ooo
ME| GATE-2016-PAPER-02 www.gateforum.com

pd
Exp: Circumferential stress  c  
2t
pd
Longitudinal stress    
4t
where, p is internal pressure
d is internal diameter
t is thickness
pd

 c  2t  2
 pd
4t

9. The forces F1 and F2 in a brake band and the direction of rotation of the drum are as shown in the figure.
The coefficient of friction is 0.25. The angle of wrap is 3π/2 radians. It is given that R = 1 m and
F2 = 1 N. The torque (in N-m) exerted on the drum is _____ .

Key: 2.248
F2
Exp:  exp   
F1
Torque   F2  F1  R

 F2 R 
 1  exp
ww  w.
engi
neer
ingonyour
finger
tips.
ooo

  3  
 1  1  1  exp  0.25     2.248 Nm
  2 
10. A single degree of freedom mass-spring-viscous damper system with mass m, spring constant k and
viscous damping coefficient q is critically damped. The correct relation among m, k, and q is

2k k
(A) q  2km (B) q  2 km (C) q  (D) q 
m m
Key: (B)
Exp: We know that
q
n
2m
k
q  2m   2 km
m

 ICP–Intensive Classroom Program  eGATE-Live Internet Based Classes DLP  TarGATE-All India Test Series
www.engineeringonyourfingertips.ooo
Leaders in GATE Preparations  65+ Centers across India
© All rights reserved by Gateforum Educational Services Pvt. Ltd. No part of this booklet may be reproduced or utilized in any form without the written permission.

7
77/17
More Notes Join us Telegram-: http//:t.me/allexammentor 7/14
|www.engineeringonyourfingertips.ooo
ME| GATE-2016-PAPER-02 www.gateforum.com

11. A machine element XY, fixed at end X, is subjected to an axial load P, transverse load F, and a twisting
moment T at its free end Y. The most critical point from the strength point of view is

(A) a point on the circumference at location Y


(B) a point at the center at location Y
(C) a point on the circumference at location X
(D) a point at the center at location X
Key: (C)
Exp:
F.B.D
F
F T
T T
P P
P

Y MF M  F 
X

At location Y
At circumference
 Direct stress due to direct load P is 0  p A where A is cross section area.
 Due to shear load F is 1  F A
 d
 Due to torsion T is 2  16T d3 r  
 2
www.
engi
neer
ingonyour
finger
tips.
ooo
 Due to B.M F. dx is b  0
At centre of location Y
 0  P A

 1  F A

T 
 2  0 r  0  in relation 
J r
 b  0
At location X
At circumference
 0  P A

 1  F A

 ICP–Intensive Classroom Program  eGATE-Live Internet Based Classes DLP  TarGATE-All India Test Series
www.engineeringonyourfingertips.ooo
Leaders in GATE Preparations  65+ Centers across India
© All rights reserved by Gateforum Educational Services Pvt. Ltd. No part of this booklet may be reproduced or utilized in any form without the written permission.

8
88/18
More Notes Join us Telegram-: http//:t.me/allexammentor 8/14
|www.engineeringonyourfingertips.ooo
ME| GATE-2016-PAPER-02 www.gateforum.com

 d
 2  16T / d3  r  
 2
My 32M
 b   ( y  d / 2 and M  F  )
I d3
At center
 o  P / A
 1  F / A
 2  0  r  0 
 b  0  y  0 
 The most critical point is at circumference of location X.

12. For the brake shown in the figure, which one of the following is TRUE?

(A) Self energizing for clockwise rotation of the drum


(B) Self energizing for anti-clockwise rotation of the drum
(C) Self energizing for rotation in either direction of the drum
(D) Not of the self energizing type F
Key: (A) 
b
Exp: FBD of Lever
taking moment about hinge for clockwise rotation of wheel
f  N
Nb  F  Nc  0 www.
engi
neer
ingonyour
finge
cr
tips.
ooo

N
F
 F N
 b  c  N

 b  c  
If b c  self energizing
So for clockwise rotation of the drum, the brake is self energizing.

13. The volumetric flow rate (per unit depth) between two streamlines having stream functions 1 and  2 is
(A) 1  2 (B) 1 2 (C) 1 /  2 (D) 1  2
Key: (D)
Exp: Volume flow rate per unit depth between two streamlines is given by 1   2
14. Assuming constant temperature condition and air to be an ideal gas, the variation in atmospheric pressure
with height calculated from fluid statics is
(A) linear (B) exponential (C) quadratic (D) cubic
Key: (B)
 ICP–Intensive Classroom Program  eGATE-Live Internet Based Classes DLP  TarGATE-All India Test Series
www.engineeringonyourfingertips.ooo
Leaders in GATE Preparations  65+ Centers across India
© All rights reserved by Gateforum Educational Services Pvt. Ltd. No part of this booklet may be reproduced or utilized in any form without the written permission.

9
99/19
More Notes Join us Telegram-: http//:t.me/allexammentor 9/14
|www.engineeringonyourfingertips.ooo
ME| GATE-2016-PAPER-02 www.gateforum.com

15. A hollow cylinder has length L, inner radius r1, outer radius r2, and thermal conductivity k. The thermal
resistance of the cylinder for radial conduction is

ln  r2 / r1  ln  r1 / r2  2kL 2kL
(A) (B) (C) (D)
2kL 2kL ln  r2 / r1  ln  r1 / r2 
Key: (A)
Exp: For a hollow cylinder
2kL  T1  T2  k
Q
r 
ln  2  r1 r2
 r1 
T1  T2 T1  T2
 
 r2  R th
ln  
 r1 
2kL

16. Consider the radiation heat exchange inside an annulus between two very long concentric cylinders. The
radius of the outer cylinder is R0 and that of the inner cylinder is Ri. The radiation view factor of the
outer cylinder onto itself is
1/3
Ri R R  Ri
(A) 1  (B) 1 i (C) 1   i  (D) 1 
R0 R0  R0  R0

Key: (D) 2
1
Exp: F11  0
Ri
F12  1
A1 2R i L R i R0
A 2 F21  A1F12  F2 1   
A 2 2R 0 L R 0

Rw ww.
engi
neer
ingonyour
finger
tips.
ooo
 F2 2  1  F21  1  i
R0

17. The internal energy of an ideal gas is a function of


(A) temperature and pressure
(B) volume and pressure
(C) entropy and pressure
(D) temperature only
Key: (D)
Exp: According to Joule‟s law,
Internal energy of an ideal gas is a function of temperature only.

 ICP–Intensive Classroom Program  eGATE-Live Internet Based Classes DLP  TarGATE-All India Test Series
www.engineeringonyourfingertips.ooo
Leaders in GATE Preparations  65+ Centers across India
© All rights reserved by Gateforum Educational Services Pvt. Ltd. No part of this booklet may be reproduced or utilized in any form without the written permission.

10
1010/
More Notes Join us Telegram-: http//:t.me/allexammentor 11010
|www.engineeringonyourfingertips.ooo
ME| GATE-2016-PAPER-02 www.gateforum.com

18. The heat removal rate from a refrigerated space and the power input to the compressor are 7.2 kW and
1.8 kW, respectively. The coefficient of performance (COP) of the refrigerator is .
Key: 4
Q2 7.2
Exp: C.O.PR   4
W 1.8

Source

Q1

W  1.8kW
R
Q2  7.2kW

Refrigerated
Sink
space

19. Consider a simple gas turbine (Brayton) cycle and a gas turbine cycle with perfect regeneration. In both
the cycles, the pressure ratio is 6 and the ratio of the specific heats of the working medium is 1.4. The
ratio of minimum to maximum temperatures is 0.3 (with temperatures expressed in K) in the regenerative
cycle. The ratio of the thermal efficiency of the simple cycle to that of the regenerative cycle is ____ .
Key: 0.8021
Exp: Brayton cycle:-
3
rp  6
  1.4 T 2
1
brayton  1  1 4

(rp )
1 1
1 1.4 1
www.
engi
neer
ingonyour
finger
tips.
ooo
(6) 1.4
 0.4006 S
Gas Turbine cycle with perfect regeneration:-
rp  6
  1.4
  1  Cp  T5  T2   Cp  T3  T2  4
3
 T5  T3 2
T
T1
 0.3 5
T4
6
Heat supplied  Cp  T4  T3  1

 ICP–Intensive Classroom Program  eGATE-Live Internet Based Classes DLP  TarGATE-All India Test Series
www.engineeringonyourfingertips.ooo
Leaders in GATE Preparations  65+ Centers across India
© All rights reserved by Gateforum Educational Services Pvt. Ltd. No part of this booklet may be reproduced or utilized in any form without the written permission.

11
1111/
More Notes Join us Telegram-: http//:t.me/allexammentor 11111
|www.engineeringonyourfingertips.ooo
ME| GATE-2016-PAPER-02 www.gateforum.com

Work done  WT  Wc
 Cp  T4  T5   Cp  T2  T1 
 Cp  T4  T3   Cp  T2  T1 

Work done Cp  T4  T3   Cp  T2  T1 
regenerative  
Heat supplied Cp  T4  T3 

T   T2 
T1  2  1  1 
 T2  T1   1
T   1  T1  T1
1   1 
 4
T  T3   T3  T4  1  T5 
T4 1    T 
 T4   4 

 
 1 
T1  p 
 r  1  1 1.4 1
  1   rp   1  0.3(6) 1.4  0.4994
T1
1  
T4  1  1  T4
 1

  rp   
 
brayton 0.4006
  0.8021
regenerative 0.4994

20. In a single-channel queuing model, the customer arrival rate is 12 per hour and the serving rate is 24 per
hour. The expected time that a customer is in queue is minutes.
Key: 2.5
 12
Exp:   12 hr ,   24 hr      0.5
 24
Let, expected time that a customer spend in queue is w q

  0.5  0.5
LqL   1 
wq   s   0.5  60
   12
0.5
  60  2.5mins
www. engineer ingonyour
finger
tips.
ooo
12

21. In the phase diagram shown in the figure, four samples of the same composition are heated to
temperatures marked by a, b, c and d.

At which temperature will a sample get solutionized the fastest?


(A) a (B) b (C) c (D) d
Key: (C)
 ICP–Intensive Classroom Program  eGATE-Live Internet Based Classes DLP  TarGATE-All India Test Series
www.engineeringonyourfingertips.ooo
Leaders in GATE Preparations  65+ Centers across India
© All rights reserved by Gateforum Educational Services Pvt. Ltd. No part of this booklet may be reproduced or utilized in any form without the written permission.

12
1212/
More Notes Join us Telegram-: http//:t.me/allexammentor 11212
|www.engineeringonyourfingertips.ooo
ME| GATE-2016-PAPER-02 www.gateforum.com

22. The welding process which uses a blanket of fusible granular flux is
(A) tungsten inert gas welding (B) submerged arc welding
(C) electroslag welding (D) thermit welding
Key: (B)
Exp: Submerged arc welding uses a blanket of fusible granular flux

23. The value of true strain produced in compressing a cylinder to half its original length is
(A) 0.69 (B) − 0.69 (C) 0.5 (D) − 0.5
Key: (B)
Exp: Final length = L/2
Initial length = L
Lf L 1
T  ln  ln  ln  0.69
L0 2L 2

24. The following data is applicable for a turning operation. The length of job is 900 mm, diameter of job is
200 mm, feed rate is 0.25 mm/rev and optimum cutting speed is 300 m/min. The machining time (in min)
is ___.
Key: 7.539
Exp: L  900mm
d  200mm
f  0.25mm / rev
v  300m / min
t ?
v  DN m/min
1000
300 m/min =  0.2  N
www.
engi
neer
ingonyour
finger
tips.
ooo
300
N  478 RPM
 0.2
L 900
t   7.539min
fN 0.25  478

25. In an ultrasonic machining (USM) process, the material removal rate (MRR) is plotted as a function of
the feed force of the USM tool. With increasing feed force, the MRR exhibits the following behavior:
(A) increases linearly
(B) decreases linearly
(C) does not change
(D) first increases and then decreases
Key: (D)

 ICP–Intensive Classroom Program  eGATE-Live Internet Based Classes DLP  TarGATE-All India Test Series
www.engineeringonyourfingertips.ooo
Leaders in GATE Preparations  65+ Centers across India
© All rights reserved by Gateforum Educational Services Pvt. Ltd. No part of this booklet may be reproduced or utilized in any form without the written permission.

13
1313/
More Notes Join us Telegram-: http//:t.me/allexammentor 11313
|www.engineeringonyourfingertips.ooo
ME| GATE-2016-PAPER-02 www.gateforum.com

Exp: In USM,

MRR Increase
Decrease due to Crushing
of abrasives

Feed force of tool

Q. No. 26 – 55 carry Two Marks Each

26. A scalar potential  has the following gradient.   yziˆ  xzjˆ  xyk.
ˆ Consider the integral

 
 .dr on the curve r  xiˆ  yjˆ  zk.
ˆ
c

x  t

The curve C is parameterized as follows:  y  t 2 and 1  t  3.
z  3t 2

The value of the integral is______ .

Key: 726

 .dr    yziˆ  xzjˆ  xykˆ .dxiˆ  dyjˆ  dzkˆ 



Exp:
c c

  yzdx  xzdy  xydz … (1)


c

 x  t; y  t 2 ; z  3t 2
www.
engi
neer
ingonyour
finger
tips.
ooo
 dx  dt  dy  2tdt  dz  6tdt


From (1);  .dr   t 2 (3t 2 )dt  t  3t 2  2tdt  t  t 2  6tdt
c c

  3t   6t 4  6t 4 dt
3

t 1

3
 t5 
  15t dt  15    3 35  1
3
4
t 1
 5 1

 726

 ICP–Intensive Classroom Program  eGATE-Live Internet Based Classes DLP  TarGATE-All India Test Series
www.engineeringonyourfingertips.ooo
Leaders in GATE Preparations  65+ Centers across India
© All rights reserved by Gateforum Educational Services Pvt. Ltd. No part of this booklet may be reproduced or utilized in any form without the written permission.

14
1414/
More Notes Join us Telegram-: http//:t.me/allexammentor 11414
|www.engineeringonyourfingertips.ooo
ME| GATE-2016-PAPER-02 www.gateforum.com

3z  5
27. The value of  (z 1)(z 2) dz
r
along a closed path  is equal to (4i), where z=x+iy and i  1. The

correct path  is
(A) (B)

(C) (D)

Key: (B)
Exp: Since if z=1 lies inside the closed path  and z=2 lies outside of the closed path  then by cauchy‟s
formula.
3z  5
3z  5
M (z 1)(z 2) dz  n zz  12 dz
 3z  5 
 2i  
 z  2  at z 1
 2 
 2i    4i
 1 

28. The probability that a w ww.


screw engi neer
manufactured in
bygoan your
company fi
ng
is er tips.
defective isoo oThe company sells screws in
0.1.
packets containing 5 screws and gives a guarantee of replacement if one or more screws in the packet are
found to be defective. The probability that a packet would have to be replaced is .
Key: 0.40951
Exp: Probability that a packet would have to be replaced i.e., P  X  1  ? [ Let „x‟ denote the number of
defective screws]
 P  X  1  1  P  X  1
 1  P  X  0
 1  5C0 (0.1)0 (0.9)5
 1   0.9   0.40951
5

Since by the Binomial distribution when P=probability of defective screw.



29. The error in numerically computing the integral   sin x  cos x  dx using the trapezoidal rule with three
0
intervals of equal length between 0 and  is_______ .
Key: 0.178
 ICP–Intensive Classroom Program  eGATE-Live Internet Based Classes DLP  TarGATE-All India Test Series
www.engineeringonyourfingertips.ooo
Leaders in GATE Preparations  65+ Centers across India
© All rights reserved by Gateforum Educational Services Pvt. Ltd. No part of this booklet may be reproduced or utilized in any form without the written permission.

15
1515/
More Notes Join us Telegram-: http//:t.me/allexammentor 11515
|www.engineeringonyourfingertips.ooo
ME| GATE-2016-PAPER-02 www.gateforum.com

ba 0 
Exp: h   ; f (x)  sin x  cos x
n 3 3
 2
x 0 
3 3
y  f (x) 1 1.37 0.37 1
By trapezoidal rule; we have the approximate value of the integral is
 /3
0 (sin x  cos x)dx  2 1  (1)  2(1.37  0.37)
 1.822
Exact value of the integral is

(sinx  cosx)dx    cos x  sin x 0  1  (1)  2

0

Error  Exact value  Approximate value


 2  1.822  0.178

30. A mass of 2000 kg is currently being lowered at a velocity of 2 m/s from the drum as shown in the figure.
The mass moment of inertia of the drum is 150 kg-m2. On applying the brake, the mass is brought to rest
in a distance of 0.5 m. The energy absorbed by the brake (in kJ) is

Key: 14.11
Exp:
V  R www. engi neer ingonyour f inger
tips.
ooo
Vi 2.0
i    2rad/sec
R 1

Loss in kinetic energy of Drum = J  i2  f2 


1
2
1
 KE drum  150   2   300 Joule
2

Loss in kinetic energy of block =  m  vi2  vf2 


1
2
1
  2000   2   02 
2

2  
KE block  4000 Joule
Loss of Potential energy of block = mgh
 2000 9.81  0.5
PE block  9810 Joule

 ICP–Intensive Classroom Program  eGATE-Live Internet Based Classes DLP  TarGATE-All India Test Series
www.engineeringonyourfingertips.ooo
Leaders in GATE Preparations  65+ Centers across India
© All rights reserved by Gateforum Educational Services Pvt. Ltd. No part of this booklet may be reproduced or utilized in any form without the written permission.

16
1616/
More Notes Join us Telegram-: http//:t.me/allexammentor 11616
|www.engineeringonyourfingertips.ooo
ME| GATE-2016-PAPER-02 www.gateforum.com

Total energy loss  KEdrum  KE block  PE block


 300  4000  9810 14110 joule
 14.11 kJ

31. A system of particles in motion has mass center G as shown in the figure. The particle i has mass mi and
its position with respect to a fixed point O is given by the position vector ri. The position of the particle
with respect to G is given by the vector  i . The time rate of change of the angular momentum of the
system of particles about G is
(The quantity i indicates second derivative of  i with respect to time and likewise for ri ).

(A) i ri  mii (B) i i  miri (C) i ri  miri (D) i i  mii


Key: (B)
Exp: By definition of Torque
 
Torque  rPOR  Fext
where

rPOR = position vector with respect to point of rotation (POR) of the particle on which force is acting.

Fext  External force acting on ith particle

 miri
 
i  i  mi ri
For complete Rigid body
 
 i   i  mi ri
i
www.
engi
neer
ingonyour
finger
tips.
ooo
32. A rigid horizontal rod of length 2L is fixed to a circular cylinder of radius R as shown in the figure.
Vertical forces of magnitude P are applied at the two ends as shown in the figure. The shear modulus for
the cylinder is G and the Young‟s modulus is E.

The vertical deflection at point A is


(A) PL3 / R 4 G   (B) PL3 / R 4 E  (C) 2PL3 / R 4 E   (D) 4PL3 / R 4 G  
Key: (D)

 ICP–Intensive Classroom Program  eGATE-Live Internet Based Classes DLP  TarGATE-All India Test Series
www.engineeringonyourfingertips.ooo
Leaders in GATE Preparations  65+ Centers across India
© All rights reserved by Gateforum Educational Services Pvt. Ltd. No part of this booklet may be reproduced or utilized in any form without the written permission.

17
1717/
More Notes Join us Telegram-: http//:t.me/allexammentor 11717
|www.engineeringonyourfingertips.ooo
ME| GATE-2016-PAPER-02 www.gateforum.com

Exp: F.B.D
T

T  2PL

T
Because of torsion angle of twist    will be there.
T 2  32 P2 4 P2
Where    
G.J G  d 4 R 4G
Due to angle of twist, A will reach at A ' and B will reach at B'
let A 'A'' be the vertical displacement
A' B
 
x

O
A'' 
A
B'
In  A 'O A''
A 'A '' x
sin    where, x is vertical deflection
A 'O 
x
When  is very small, sin      x  

4P 3
x
R 4G
33. A simply supported beam of length 2L is subjected to a moment M at the mid-point x = 0 as shown in the
figure. The deflection in the domain 0 ≤ x ≤ L is given by
Mx
W  L  x  (x  c),
12EIL
www.
engi
neer
ingonyour
finger
tips.
ooo
where E is the Young‟s modulus, I is the area moment of inertia and c is a constant (to be determined) .

The slope at the center x = 0 is


(A) ML / (2EI) (B) ML / (3EI) (C) ML / (6EI) (D) ML / (12EI)
Key: (C)

 ICP–Intensive Classroom Program  eGATE-Live Internet Based Classes DLP  TarGATE-All India Test Series
www.engineeringonyourfingertips.ooo
Leaders in GATE Preparations  65+ Centers across India
© All rights reserved by Gateforum Educational Services Pvt. Ltd. No part of this booklet may be reproduced or utilized in any form without the written permission.

18
1818/
More Notes Join us Telegram-: http//:t.me/allexammentor 11818
|www.engineeringonyourfingertips.ooo
ME| GATE-2016-PAPER-02 www.gateforum.com

Exp: As we know that



M2dx
U
0 2EI

U
and slope at mid-span where moment is applied will be (according to Costigliano‟s theorem).
M
Mx
M 0  x  
2

Mx
M M    x  2 
2 x

 
   Mx 2 2
 Mx 
2
 1
 U     dx     M  dx  
 0  2    2   2EI x
M M M

 x  2  dx  2
M2  x 2
 2 2 2
U    2 dx    x
2EI  0 4  4 2 

M2   3   3  M 2 
     
2EI  4 2  3  3  12EI

 U  M
 slopeat  x  0     
 M  x 0 6EI
34. In the figure, the load P = 1 N, length L = 1 m, Young‟s modulus E = 70 GPa, and the cross-section of the
links is a square with dimension 10 mm × 10 mm. All joints are pin joints.

www.
engi
neer
ingonyour
finger
tips.
ooo

The stress (in Pa) in the link AB is ________ .


(Indicate compressive stress by a negative sign and tensile stress by a positive sign.)
Key: 0

 ICP–Intensive Classroom Program  eGATE-Live Internet Based Classes DLP  TarGATE-All India Test Series
www.engineeringonyourfingertips.ooo
Leaders in GATE Preparations  65+ Centers across India
© All rights reserved by Gateforum Educational Services Pvt. Ltd. No part of this booklet may be reproduced or utilized in any form without the written permission.

19
1919/
More Notes Join us Telegram-: http//:t.me/allexammentor 11919
|www.engineeringonyourfingertips.ooo
ME| GATE-2016-PAPER-02 www.gateforum.com

Exp: F.B.D of point B


FBC

45
45 B

P
FAB

Applying Lame‟s theorem


FAB P F
  BC
sin180 sin135 sin 45
 FAB  0
Stress in AB  0

35. A circular metallic rod of length 250 mm is placed between two rigid immovable walls as shown in the
figure. The rod is in perfect contact with the wall on the left side and there is a gap of 0.2 mm between
the rod and the wall on the right side. If the temperature of the rod is increased by 200 o C, the axial stress
developed in the rod is MPa.
Young‟s modulus of the material of the rod is 200 GPa and the coefficient of thermal expansion is 10−5
per oC.

Key: 240
Exp: t  200C www.
engi
neer
ingonyour
finger
tips.
ooo
  105 C
E  2 105 MPa
  0.2 mm
We know that, Axial stress gets induced in the rod when some gap „  ‟ is provided is
 t      105  200  250  0.2 
  E     2 10
5

    250 
 240 MPa

 ICP–Intensive Classroom Program  eGATE-Live Internet Based Classes DLP  TarGATE-All India Test Series
www.engineeringonyourfingertips.ooo
Leaders in GATE Preparations  65+ Centers across India
© All rights reserved by Gateforum Educational Services Pvt. Ltd. No part of this booklet may be reproduced or utilized in any form without the written permission.

20
2020/
More Notes Join us Telegram-: http//:t.me/allexammentor 12020
|www.engineeringonyourfingertips.ooo
ME| GATE-2016-PAPER-02 www.gateforum.com

36. The rod AB, of length 1 m, shown in the figure is connected to two sliders at each end through pins. The
sliders can slide along QP and QR. If the velocity VA of the slider at A is 2 m/s, the velocity of the
midpoint of the rod at this instant is m/s.

Key: 1
VA =2m/sec 2cos60  1
Exp: Given AB  2 60 2sin60  3
A
Since Rod AB is rigid, so A
Axial velocity of A & B should be same C
C 2cos60  1
VA cos60  VB cos60 60 60 B
Q B
VA  VB  2m/sec VB

C is mid point of AB 2sin60  3

A 3
A

C
 C  Vc  1

B
1 B
 3
www.
engi
neer
ingonyour
finger
tips.
ooo
Velocity corresponding to Velocity corresponding to
pure translation part Rotational part

Alternate Method:
 2   2  x 2
2 2
y a
cos120  x
30o
2 2 2 c
2
2
1  x  120o
 1    30o
2 2 2 b o
2
3
x  2 22 3
2
x
y  3,so oc will be perpendicular toab.
2
v
sin  30   c  vc 1
2

 ICP–Intensive Classroom Program  eGATE-Live Internet Based Classes DLP  TarGATE-All India Test Series
www.engineeringonyourfingertips.ooo
Leaders in GATE Preparations  65+ Centers across India
© All rights reserved by Gateforum Educational Services Pvt. Ltd. No part of this booklet may be reproduced or utilized in any form without the written permission.

21
2121/
More Notes Join us Telegram-: http//:t.me/allexammentor 12121
|www.engineeringonyourfingertips.ooo
ME| GATE-2016-PAPER-02 www.gateforum.com

37. The system shown in the figure consists of block A of mass 5 kg connected to a spring through a massless
rope passing over pulley B of radius r and mass 20 kg. The spring constant k is 1500 N/m. If there is no
slipping of the rope over the pulley, the natural frequency of the system is rad/s.

Key: 10
Exp: Displace the block “A” & Release

 k r r   
1 2
Mr  mr 2  
 0
2 
 
  k 
   0
 1 M m 
2 
 
 k  1500
n     10 rad/sec
 1Mm  10  5
2 

Alternate method:
1 1 1
Energy of system remain conserved, E  J2  mv 2  mg y  k  y    …(1)
2

2 2 2
www.
engi
neer
ingonyour
finger
tips.
ooo
Rotational Translational Gravitational P.E stored
K.E K.E K.E in spring
where,
  Static elongation of spring at equilibrium which is calculated as follows:

mg
mg  k   
k
Differentiating Eqn.(1)w.r.t time, which will be zero because E  constant
dE
0
dt
d dv
J  mv  mgv  k  y    v  0 ...(2)
dt dt
Since there is no slipping between rope & pulley
v  r

 ICP–Intensive Classroom Program  eGATE-Live Internet Based Classes DLP  TarGATE-All India Test Series
www.engineeringonyourfingertips.ooo
Leaders in GATE Preparations  65+ Centers across India
© All rights reserved by Gateforum Educational Services Pvt. Ltd. No part of this booklet may be reproduced or utilized in any form without the written permission.

22
2222/
More Notes Join us Telegram-: http//:t.me/allexammentor 12222
|www.engineeringonyourfingertips.ooo
ME| GATE-2016-PAPER-02 www.gateforum.com

1 2  v  1 dv  dv
Mr     mv  mgv  k  y    v  0
2  r  r dt  dt
M d y
2

  m  2  ky  0
 2  dt
k 1500
n   10rad/sec
M  10  5
  m
 2 

38. In a structural member under fatigue loading, the minimum and maximum stresses developed at the
critical point are 50 MPa and 150 MPa, respectively. The endurance, yield, and the ultimate strengths of
the material are 200 MPa, 300 MPa and 400 MPa, respectively. The factor of safety using modified
Goodman criterion is

3 8 12
(A) (B) (C) (D) 2
2 5 7
Key: (D)
Exp: Given

max 150MPa, min  50MPa, m  100MPa

a  50MPa,Se  200MPa,Sut  300MPa

Sut  400MPa

Equation of line 1

a  m
 1 ...(1)
Se Sut

Equation of line 2

a 1
 tan   ...(2)
m 2 www.
engi
neer
ingonyour f
inger
tips.
ooo

Solving above two equations to get co-ordinates of point P (Sm, Sa)

Sa S
 m 1
200 400

2 Sa  Sm  400 ...(3)

Sm  2Sa ...(4)

from (3) & (4)

Sa  100MPa
Sa 100
f.o.s   2
a 50

 ICP–Intensive Classroom Program  eGATE-Live Internet Based Classes DLP  TarGATE-All India Test Series
www.engineeringonyourfingertips.ooo
Leaders in GATE Preparations  65+ Centers across India
© All rights reserved by Gateforum Educational Services Pvt. Ltd. No part of this booklet may be reproduced or utilized in any form without the written permission.

23
2323/
More Notes Join us Telegram-: http//:t.me/allexammentor 12323
|www.engineeringonyourfingertips.ooo
ME| GATE-2016-PAPER-02 www.gateforum.com

39. The large vessel shown in the figure contains oil and water. A body is submerged at the interface of oil
and water such that 45 percent of its volume is in oil while the rest is in water. The density of the body is
______kg/m3.
The specific gravity of oil is 0.7 and density of water is 1000 kg/m3.
Acceleration due to gravity g = 10 m/s2.

Key: 865
Exp: Given water  1000 kg m3
oil  700 kg m3
body  ?
Voil  0.45Vbody
Vwater  0.55Vbody
oil Voil g  water Vwater g  body Vbody g
700  0.45Vbody  1000  0.55Vbody  body Vbody
body  865kg m3

40. Consider fluid flow between two infinite horizontal plates which are parallel (the gap between them being
50 mm). The top plate wwis w .engparallel
sliding i
neer in
togony
the our fi
ng
stationary er ti
ps
bottom .ooo
plate at a speed of 3 m/s. The flow
between the plates is solely due to the motion of the top plate. The force per unit area (magnitude)
required to maintain the bottom plate stationary is N/m2.
Viscosity of the fluid µ = 0.44 kg/m-s and density ρ = 888 kg/m3.
Key: 26.4
Exp: Vtop  3m / s
50 mm
  0.44 kg m  s
  888kg m3

du Vtop  Vbottom 3
   60
dy 0.05 0.05
du
J wall    0.44  60 kg m.s 2
dy
 26.4 N m2
 ICP–Intensive Classroom Program  eGATE-Live Internet Based Classes DLP  TarGATE-All India Test Series
www.engineeringonyourfingertips.ooo
Leaders in GATE Preparations  65+ Centers across India
© All rights reserved by Gateforum Educational Services Pvt. Ltd. No part of this booklet may be reproduced or utilized in any form without the written permission.

24
2424/
More Notes Join us Telegram-: http//:t.me/allexammentor 12424
|www.engineeringonyourfingertips.ooo
ME| GATE-2016-PAPER-02 www.gateforum.com

41. Consider a frictionless, massless and leak-proof plug blocking a rectangular hole of dimensions 2R  L
the bottom of an open tank as shown in the figure. The head of the plug has the shape of a semi-cylinder
of radius R. The tank is filled with a liquid of density ρ up to the tip of the plug. The gravitational
acceleration is g. Neglect the effect of the atmospheric pressure.

The force F required to hold the plug in its position is


     2
(A) 2R 2 gL 1   (B) 2R 2 gL 1   (C) R 2gL (D) R gL
 4  4 2
Key: (A)
Exp:

2R
Downward force due to water = weight of water above curved surface
  R 2 L 
 2  g  R 2 L  
  4 
 
 2gR 2 L 1   N
 4
www.
engi
neer
ingonyour
finger
tips.
ooo
Weight of plug is neglected.

42. Consider a parallel-flow heat exchanger with area Ap and a counter-flow heat exchanger with area Ac.
In both the heat exchangers, the hot stream flowing at 1 kg/s cools from 80 ℃ to 50 ℃. For the cold
stream in both the heat exchangers, the flow rate and the inlet temperature are 2 kg/s and 10 ℃,
respectively. The hot and cold streams in both the heat exchangers are of the same fluid. Also, both the
heat exchangers have the same overall heat transfer coefficient. The ratio Ac / Ap is ________ .
Key: 0.928
Exp: mh  1kg / s
mc  2kg / s
CPh  Cpc (as both are same fluids)

 
mk Cpk t k1  t k 2  mc Cpc t c2  t c1  
 
1(80  50)  2 t c2  10  t c2  25 C o

 ICP–Intensive Classroom Program  eGATE-Live Internet Based Classes DLP  TarGATE-All India Test Series
www.engineeringonyourfingertips.ooo
Leaders in GATE Preparations  65+ Centers across India
© All rights reserved by Gateforum Educational Services Pvt. Ltd. No part of this booklet may be reproduced or utilized in any form without the written permission.

25
2525/
More Notes Join us Telegram-: http//:t.me/allexammentor 12525
|www.engineeringonyourfingertips.ooo
ME| GATE-2016-PAPER-02 www.gateforum.com

Parallel flow
T
T Counter flow

t h1  80o C t h1  80o C
1  t k1  t c2
t h2  50o C t c2  25o C
t h 2  50o C
1  t k1  t c1 2  t k2  t c2 2  t k2  t c1
t c2  25o C t c2  10o C

t c1  10o C

L
L

1  70o C 1  80  25  55o C
2  25o C 2  50  10  40o C
  1  2
mp  1 2 mc 
   
ln  1  ln  1 
 2   2 
70  25 55  40
 
 70   55 
ln   ln  
 25   40 
 43.705  47.1
Q  U.Ap mp Q  U.A c .mc
 A p mp  A c .mc
A c mp 43.705
   0.928
A p mc 47.1

43. Two cylindrical shafts A and B at the same initial temperature are simultaneously placed in a furnace.
The surfaces of the shafts remain at the furnace gas temperature at all times after they are introduced into
www.
the furnace. The temperature engi nein
variation er ing
the ony
axial our fing
direction ofert
i
the ps. oo
shafts o be assumed to be negligible.
can
The data related to shafts A and B is given in the following Table.

Quantity Shaft A Shaft B


Diameter (m) 0.4 0.1
Thermal conductivity (W/m-K) 40 20
Volumetric heat capacity (J/m3-K) 2×106 2×107

The temperature at the centerline of the shaft A reaches 400℃ after two hours. The time required
(in hours) for the centerline of the shaft B to attain the temperature of 400℃ is _____ .
Key: 2.4 – 2.6

 ICP–Intensive Classroom Program  eGATE-Live Internet Based Classes DLP  TarGATE-All India Test Series
www.engineeringonyourfingertips.ooo
Leaders in GATE Preparations  65+ Centers across India
© All rights reserved by Gateforum Educational Services Pvt. Ltd. No part of this booklet may be reproduced or utilized in any form without the written permission.

26
2626/
More Notes Join us Telegram-: http//:t.me/allexammentor 12626
|www.engineeringonyourfingertips.ooo
ME| GATE-2016-PAPER-02 www.gateforum.com

44. A piston-cylinder device initially contains 0.4 m3 of air (to be treated as an ideal gas) at 100 kPa and
80oC. The air is now isothermally compressed to 0.1 m3. The work done during this process is
kJ.
(Take the sign convention such that work done on the system is negative)
Key: -55.45
Exp: V1  0.4m3
P1  100kPa
T1  80o C  353K
V2  0.1 m3
Ideal gas & process is isothermal.
V 
 W  P1V1 ln  2 
 V1 
 0.1 
 100  0.4ln  
 0.4 
 55.45kJ

45. A reversible cycle receives 40 kJ of heat from one heat source at a temperature of 127 ℃ and 37 kJ from
another heat source at 97 ℃. The heat rejected (in kJ) to the heat sink at 47 ℃ is .
Key: 64
Exp: Reversible cycle.
T1  127  273 T2  97  273
 400K  370K

Q1  40kJ Q2  37kJ

Q3

www.
engi
neeri
ngonyour
T  47  273
f
inger
tips.
ooo
3

 320K
Q
 0
T
Q1 Q2 Q3 40 37 Q3
  0   0
T1 T2 T3 400 370 320
Q3 1 320
  Q3   64kJ
320 5 5

46. A refrigerator uses R-134a as its refrigerant and operates on an ideal vapour-compression
refrigeration cycle between 0.14 MPa and 0.8 MPa. If the mass flow rate of the refrigerant is 0.05 kg/s,
the rate of heat rejection to the environment is kW.
Given data:
At P = 0.14 MPa, h = 236.04 kJ/kg, s=0.9322 kJ/kg-K
At P = 0.8 MPa, h = 272.05 kJ/kg (superheated vapour)
At P = 0.8MPa, h = 93.42 kJ/kg (saturated liquid)
Key: 8.9315
 ICP–Intensive Classroom Program  eGATE-Live Internet Based Classes DLP  TarGATE-All India Test Series
www.engineeringonyourfingertips.ooo
Leaders in GATE Preparations  65+ Centers across India
© All rights reserved by Gateforum Educational Services Pvt. Ltd. No part of this booklet may be reproduced or utilized in any form without the written permission.

27
2727/
More Notes Join us Telegram-: http//:t.me/allexammentor 12727
|www.engineeringonyourfingertips.ooo
ME| GATE-2016-PAPER-02 www.gateforum.com

Exp: Given
h1  236.04kJ / kg P 0.8MPa 3
s1  0.9322 kJ / kg 2
h 2  272.05kJ / kg
h 3  93.42kJ / kg
0.14MPa 4 1
m  0.05kg / s

  h 2  h3 
 Heat Rejection to environment  m
 0.05  272.05  93.42  h

 8.9315kW

47. The partial pressure of water vapour in a moist air sample of relative humidity 70% is 1.6 kPa, the total
pressure being 101.325 kPa. Moist air may be treated as an ideal gas mixture of water vapour and dry air.
The relation between saturation temperature (Ts in K) and saturation pressure (ps in kPa) for water is
given by ln  ps / p0  14.317  5304 / Ts , where p0  101.325 kPa. The dry bulb temperature of the moist
air sample (in ℃) is
Key: 19.89
Exp: PV  1.6 kPa
T
  70% Ps  2.2857 kPa
Po  101.325 kPa PV  1.6 kPa
P  5304
ln  s  14.317   Ps  ? TSat
d.b.t  TSat  ?
 po  Ts  2
Pv

Psat
1 d.p.t
1.6 1.6
0.7   Ps   2.2857 kPa
Ps 0.7 S

 2.2857  5304
ln   14.317 
 101.325  wwTw sat.
engi
neer
ingonyour
finger
tips.
ooo
5304
Tsat   292.898 K
 2.2857 
14.317  n  
 101.325 
d.b.t  t sat  19.89o C

48. In a binary system of A and B, a liquid of 20% A (80% B) is coexisting with a solid of 70% A (30% B).
For an overall composition having 40% A, the fraction of solid is
(A) 0.40 (B) 0.50 (C) 0.60 (D) 0.75
Key: (A)

 ICP–Intensive Classroom Program  eGATE-Live Internet Based Classes DLP  TarGATE-All India Test Series
www.engineeringonyourfingertips.ooo
Leaders in GATE Preparations  65+ Centers across India
© All rights reserved by Gateforum Educational Services Pvt. Ltd. No part of this booklet may be reproduced or utilized in any form without the written permission.

28
2828/
More Notes Join us Telegram-: http//:t.me/allexammentor 12828
|www.engineeringonyourfingertips.ooo
ME| GATE-2016-PAPER-02 www.gateforum.com

 0.2L  0.7S  0.4 20% A 70% A


 L  S 
80% B 30% B
0.2L  0.7S  0.4L  0.4S
Liquid (L) Solid (S)
0.2L  0.3S ...(1)
S
Fraction of solid 
S L  0.2L  0.7S % A


S  0.8L  0.3S % B
0.3
S S Mixture of liquid&solid
0.2
0.2
  0.40
0.5

49. Gray cast iron blocks of size 100 mm × 50 mm × 10 mm with a central spherical cavity of diameter 4 mm
are sand cast. The shrinkage allowance for the pattern is 3%. The ratio of the volume of the pattern to
volume of the casting is ______ .
Key: 1.08 – 1.10

50. The voltage-length characteristic of a direct current arc in an arc welding process is
V  100 + 40  , where l is the length of the arc in mm and V is arc voltage in volts. During a welding
operation, the arc length varies between 1 and 2 mm and the welding current is in the range 200-250 A.
Assuming a linear power source, the short circuit current is A.
Key: 424.6
Exp: V  100  40 1  1mm &  2  2mm
V1  140V I1  250A
V2  180V I2  200A
V I
 1
0CV SCC www.
engi
neer
ingonyour
finger
tips.
ooo
140 250
 1 …(a)
OCV SCC
180 200
 1 …(b)
OCV SCC
On solving equation a and b, we have
S.C.C=424.6A

51. For a certain job, the cost of metal cutting is Rs. 18C/V and the cost of tooling is Rs. 270 C/(TV), where
C is a constant, V is the cutting speed in m/min and T is the tool life in minutes. The Taylor's tool life
equation is VT0.25  150. The cutting speed (in m/min) for the minimum total cost is .
Key: 57.9
Exp: Cost of metal cutting = Rs 18 C/V
Cost of Tooling = Rs 270 C/TV
 ICP–Intensive Classroom Program  eGATE-Live Internet Based Classes DLP  TarGATE-All India Test Series
www.engineeringonyourfingertips.ooo
Leaders in GATE Preparations  65+ Centers across India
© All rights reserved by Gateforum Educational Services Pvt. Ltd. No part of this booklet may be reproduced or utilized in any form without the written permission.

29
2929/
More Notes Join us Telegram-: http//:t.me/allexammentor 12929
|www.engineeringonyourfingertips.ooo
ME| GATE-2016-PAPER-02 www.gateforum.com

C= Constant ,V = Cutting Speed , T = tool life

C = 150, VT0.25 = 150  T  150 V 


1 0.25

T  150 V 
4

C C
Total cost  18  270
V TV
18C 270CV 4
 
V 1504 V
18C 270C V3
 
V 1504
On, differentiating total cost
18C 270 C3V 2
 
V2 1504
18 1504
V4 
3  270
 V  57.914 m min

52. The surface irregularities of electrodes used in an electrochemical machining (ECM) process are 3 µm
and 6 µm as shown in the figure. If the work-piece is of pure iron and 12V DC is applied between the
electrodes, the largest feed rate is mm/min.
Conductivity of the electrolyte 0.02 ohm-1mm-1
Over-potential voltage 1.5 V
Density of iron 7860 kg/m3
Atomic weight of iron 55.85 gm
+2
Assume the iron to be dissolved as Fe and the Faraday constant to be 96500 Coulomb.

www.
engi
neer
ingonyour
finger
tips.
ooo

Key: 51.51
Exp: Given V = 12V V  1.5V

 ICP–Intensive Classroom Program  eGATE-Live Internet Based Classes DLP  TarGATE-All India Test Series
www.engineeringonyourfingertips.ooo
Leaders in GATE Preparations  65+ Centers across India
© All rights reserved by Gateforum Educational Services Pvt. Ltd. No part of this booklet may be reproduced or utilized in any form without the written permission.

30
3030/
More Notes Join us Telegram-: http//:t.me/allexammentor 13030
|www.engineeringonyourfingertips.ooo
ME| GATE-2016-PAPER-02 www.gateforum.com

1
k  0.02 ohm 1 mm 1  0.02  0.2ohm 1 cm 1
ohm101 cm
  7860 kg m3
103
 7860  gm cm3
106
  55.85gm
 7.860 gm cm3
AI 55.85I
MRR : Q :   3.68 105 cm3 sec
ZF 7.86  2  96600
Inter electrode gap given  9 m  9 104 cm
k  V  V 
current density J 
y
0.2 12  1.5
  2333.33
9 104
I  J  S.A
I  2333.33  S.A  S.A  I 2333.33
Electrode feed rate = MRR surface area cm sec
3.68 105 I  2333.33
 cm sec
I
 0.086 10  60 mm min
 51.51mm min
53. For the situation shown in the figure below the expression for H in terms of r, R and D is

www.
engi
neer
ingonyour
finger
tips.
ooo

(A) H  D  r 2  R 2 (B) H  (R  r)  (D  r)

(C) H  (R  r)  D 2  R 2 (D) H  (R  r)  2D(R  r)  D2


Key: (D)

 ICP–Intensive Classroom Program  eGATE-Live Internet Based Classes DLP  TarGATE-All India Test Series
www.engineeringonyourfingertips.ooo
Leaders in GATE Preparations  65+ Centers across India
© All rights reserved by Gateforum Educational Services Pvt. Ltd. No part of this booklet may be reproduced or utilized in any form without the written permission.

31
3131/
More Notes Join us Telegram-: http//:t.me/allexammentor 13131
|www.engineeringonyourfingertips.ooo
ME| GATE-2016-PAPER-02 www.gateforum.com

Exp:

a c
H

a  R  r, b  D   R  r  , C  a 2  b 2

 R  r   D   R  r 
2
C 
2

H  R  r  C  R  r  R  r   D2   R  r   2D  R  r 
2 2

H   R  r   2D  R  r   D2

54. A food processing company uses 25,000 kg of corn flour every year. The quantity-discount price of corn
flour is provided in the table below:

Quantity (kg) Unit price (Rs/kg)


1-749 70
750-1499 65
1500 and above 60

The order processing charges are Rs. 500/order. The handling plus carry-over charge on an annual basis
is 20% of the purchase price of the corn flour per kg. The optimal order quantity (in kg) is .
Key: 1500
Exp: D = 25000 kg, C0 = Rs 500/order, Ch = 20% of Cu
www.
engi
neer
ingonyour
finger
tips.
ooo
Qty (kg) Cu (Rs/kg) Ch (Rs/Kg/year)
1  Q1  750 70 0.2  70  14
750  Q2  1500 65 0.2  65  13
Q3  1500 60 0.2  60  12

This problem belongs to inventory model with two price break.


2DCo
Q 
Ch
 first checking for least unit price

2  25000  500
Q*3   1443.37
12
Now, 1443.37 < 1500 therefore, the company will not get the item at Rs 60/kg
Now, checking for second minimum unit price
 ICP–Intensive Classroom Program  eGATE-Live Internet Based Classes DLP  TarGATE-All India Test Series
www.engineeringonyourfingertips.ooo
Leaders in GATE Preparations  65+ Centers across India
© All rights reserved by Gateforum Educational Services Pvt. Ltd. No part of this booklet may be reproduced or utilized in any form without the written permission.

32
3232/
More Notes Join us Telegram-: http//:t.me/allexammentor 13232
|www.engineeringonyourfingertips.ooo
ME| GATE-2016-PAPER-02 www.gateforum.com

2  25000  500
Q*2   1386.75
13
Since, 1386.75 lies between 750 and 1500
Therefore, we need to find

 
Total cost Q2  25000  65 
25000
1386.75
 500 
1386.75
2
13

 Rs1643027.755

 D Q 
T.C  D  Cu  Q  C0  2  Ch 
 
25000 1500
Totalcost 1500   25000  60   500  12  Rs 1517333.33
1500 2
Since, T.C (1500) < T.C Q*2 0  
Therefore, optimal order quantity is 1500.

55. A project consists of 14 activities, A to N. The duration of these activities (in days) are shown in brackets
on the network diagram. The latest finish time (in days) for node 10 is

Key: 14
Exp:
E5
E9
L  10
www.engi
neeri
ng
4onyour finge Lr
ti
p
14s.ooo
3 10
3 E  10
8 L  12
2 3
E2 6
L2 2 E 8
3
2 L  10 2 2
4 4 9 12
1 2 11
E0 E6 4 5 E  15 E  17 E  19
L0 L6 L  15 L  17 L  19
7
2 E  10 4
L  10

5
E4
L  11
The latest finish time for node 10 is 14 days.
 ICP–Intensive Classroom Program  eGATE-Live Internet Based Classes DLP  TarGATE-All India Test Series
www.engineeringonyourfingertips.ooo
Leaders in GATE Preparations  65+ Centers across India
© All rights reserved by Gateforum Educational Services Pvt. Ltd. No part of this booklet may be reproduced or utilized in any form without the written permission.

33
3333/
More Notes Join us Telegram-: http//:t.me/allexammentor 13333
www.engineeringonyourfingertips.ooo
|ME| GATE-2016-PAPER-03 www.gateforum.com

General Aptitude
Q. No. 1 – 5 Carry One Mark Each

1. Based on the given statements, select the appropriate option with respect to grammar and
usage. Statements
(i) The height of Mr. X is 6 feet.
(ii) The height of Mr. Y is 5 feet.
(A) Mr. X is longer than Mr. Y.
(B) Mr. X is more elongated than Mr. Y.
(C) Mr. X is taller than Mr. Y.
(D) Mr. X is lengthier than Mr. Y.
Key: (C)

2. The students the teacher on teachers’ day for twenty years of dedicated teaching.
(A) facilitated (B) felicitated (C) fantasized (D) facillitated
Key: (B)

3. After India’s cricket world cup victory in 1985, Shrotria who was playing both tennis and
cricket till then, decided to concentrate only on cricket. And the rest is history.
What does the underlined phrase mean in this context?
(A) history will rest in peace (B) rest is recorded in history books
(C) rest is well known (D) rest is archaic
Key: (C)

Given  9 inches    0.25yards 


1/2 1/2
4. , which one of the following statements is TRUE?
(A) 3 inches = 0.5 yards (B) 9 inches = 1.5 yards
(C) 9 inches = 0.25 yards (D) 81 inches = 0.0625 yards
Key: (C)
www.
engi
neer
ingonyour
finger
tips.
ooo
5. S, M, E and F are working in shifts in a team to finish a project. M works with twice the
efficiency of others but for half as many days as E worked. S and M have 6 hour shifts in a
day, whereas E and F have 12 hours shifts. What is the ratio of contribution of M to
contribution of E in the project?
(A) 1:1 (B) 1:2 (C) 1:4 (D) 2:1
Key: (B)

 ICP–Intensive Classroom Program  eGATE-Live Internet Based Classes DLP  TarGATE-All India Test Series
www.engineeringonyourfingertips.ooo
Leaders in GATE Preparation  65+ Centers across India
© All rights reserved by Gateforum Educational Services Pvt. Ltd. No part of this booklet may be reproduced or utilized in any form without the written permission.

More Notes Join us Telegram-: http//:t.me/allexammentor


www.engineeringonyourfingertips.ooo
|ME| GATE-2016-PAPER-03 www.gateforum.com

Q. No. 6 – 10 Carry Two Marks Each

6. The Venn diagram shows the preference of the student population for leisure activities.

29

From the data given, the number of students who like to read books or play sports is .
(A) 44 (B) 51 (C) 79 (D) 108
Key: (D)
Exp: From Venn diagram
n(A)  no of persons reading books  13  44  12  7  76
n(B)  no of persons playing  15  44  7  17  83
n(A  B)  51
n(A  B)  n(A)  n(B)  n(A  B)  76  83  51  108

7. Social science disciplines were in existence in an amorphous form until the colonial period
when they were institutionalized. In varying degrees, they were intended to further the colonial
interest. In the time of globalization and the economic rise of postcolonial countries like India,
conventional ways of knowledge production have become obsolete.
Which of the following can be logically inferred from the above statements?
(i) Social science disciplines have become obsolete.
(ii) Social science disciplines had a pre-colonial origin.
(iii) Social science disciplines always promote colonialism.
(iv) Social scienceww w.
must e ngi nee
maintain ringonyo
disciplinary ur f inger
boundaries. tips. ooo
(A) (ii) only (B) (i) and (iii) only
(C) (ii) and (iv) only (D) (iii) and (iv) only
Key: (A)

8. Two and a quarter hours back, when seen in a mirror, the reflection of a wall clock without
number markings seemed to show 1:30. What is the actual current time shown by the clock?
(A) 8:15 (B) 11:15 (C) 12:15 (D) 12:45
Key: (D)
Exp: If reflection is seen as Actual will be

1: 30
10 : 30
Thus present time will be 10:30  2:15 12: 45

 ICP–Intensive Classroom Program  eGATE-Live Internet Based Classes DLP  TarGATE-All India Test Series
www.engineeringonyourfingertips.ooo
Leaders in GATE Preparation  65+ Centers across India
© All rights reserved by Gateforum Educational Services Pvt. Ltd. No part of this booklet may be reproduced or utilized in any form without the written permission.

More Notes Join us Telegram-: http//:t.me/allexammentor


www.engineeringonyourfingertips.ooo
|ME| GATE-2016-PAPER-03 www.gateforum.com

9. M and N start from the same location. M travels 10 km East and then 10 km North-East. N
travels 5 km South and then 4 km South-East. What is the shortest distance (in km) between
M and N at the end of their travel?
(A) 18.60 (B) 22.50 (C) 20.61 (D) 25.00
Key: (C)

10. A wire of length 340 mm is to be cut into two parts. One of the parts is to be made into a
square and the other into a rectangle where sides are in the ratio of 1:2. What is the length of
the side of the square (in mm) such that the combined area of the square and the rectangle is a
MINIMUM?
(A) 30 (B) 40 (C) 120 (D) 180

Key: (B)
Exp: x  y  340
 x 2x 
Perimeter of rectangle  2     2x
3 3 
Perimeter of square  340  2x x 2 x/ 3

340  2x
Length of square 
4 x x/ 3
 340  2x  2 2
2

Totalarea     x  f (x)
 4  9
Square Rectangle
4 2x  340
f '(x)  x  0
9 4
4 1
 x   340  2x   x  90
9 4
340  2x
Length of square   40mm
4
Mechanical Engineering
www.
en
Q.gi
ne
No. 1e
–r
inCarry
25 gony our
One fi
ngEach
Mark er
tips.
ooo

1 A real square matrix A is called skew-symmetric if


(A) AT = A
(B) AT = A-1
(C) AT = A
(D) AT = A+A-1
Key: (C)

log e (1  4 x)
2 lt is equal to
x 0 e3x  1
1 4
(A) 0 (B) (C) (D) 1
12 3

Key: (C)

 ICP–Intensive Classroom Program  eGATE-Live Internet Based Classes DLP  TarGATE-All India Test Series
www.engineeringonyourfingertips.ooo
Leaders in GATE Preparation  65+ Centers across India
© All rights reserved by Gateforum Educational Services Pvt. Ltd. No part of this booklet may be reproduced or utilized in any form without the written permission.

More Notes Join us Telegram-: http//:t.me/allexammentor


www.engineeringonyourfingertips.ooo
|ME| GATE-2016-PAPER-03 www.gateforum.com

log e (1  4x) 0


Exp: lim  
x 0 e3x  1 0
1
.4
lim 1 3x4x 
4 4

x 0 e .3 (1  4.0)e .3 3
0

3. Solutions of Laplace’s equation having continuous second-order partial derivatives are called
(A) biharmonic functions
(B) harmonic functions
(C) conjugate harmonic functions
(D) error functions
Key: (B)

4. The area (in percentage) under standard normal distribution curve of random variable Z within
limits from −3 to +3 is _____
Key: 99.74
Exp:
3  99.74%

0
1  68.4% 
2  95.45%

5. The root of the function f(x) = x3+x1 obtained after first iteration on application of Newton-
Raphson scheme using an initial guess of x0=1 is
www. engi neer i
ngonyour f i
nger t ips. ooo
(A) 0.682 (B) 0.686 (C) 0.750 (D) 1.000
Key: (C)
f  xn 
Exp: We have x n 1  x n 
f  xn 
f  x0 
For n=0, x1  x 0 
f  x0 
f (x)  x 3  x  1  f (x)  3x 2  1
given x 0  1
f  x 0   f (1)  1, f   x 0   f (1)  4
1 3
 x1  1    0.75
4 4

 ICP–Intensive Classroom Program  eGATE-Live Internet Based Classes DLP  TarGATE-All India Test Series
www.engineeringonyourfingertips.ooo
Leaders in GATE Preparation  65+ Centers across India
© All rights reserved by Gateforum Educational Services Pvt. Ltd. No part of this booklet may be reproduced or utilized in any form without the written permission.

More Notes Join us Telegram-: http//:t.me/allexammentor


www.engineeringonyourfingertips.ooo
|ME| GATE-2016-PAPER-03 www.gateforum.com

6. A force F is acting on a bent bar which is clamped at one end as shown in the figure.

The CORRECT free body diagram is

(A) (B)

(C) (D)

Key: (A)

7. The cross-sections of two solid bars made of the same material are shown in the figure. The
square cross-section has flexural (bending) rigidity I1, while the circular cross-section has
flexural rigidity w
I 2. ww.
Bothe ngi nehave
sections er
inthe
gosame
nyou r f
inger tips
cross-sectional .
ooo
area. The ratio I1/I2 is
(A) 1/ 
(B) 2 / 
(C)  / 3
(D)  / 6
Key: (C)
Exp: Flexural rigidity = EI
Both have same cross-section area

 a 2  d2
4
Where a is side of square and d is diameter of circle.
a4 2 d 4
E1 
 4 I 2
12  12  16  
 a4  d  1 ( E1  E2 because of same material)
16 I2 d 4  4 3
E2  d
64 64

 ICP–Intensive Classroom Program  eGATE-Live Internet Based Classes DLP  TarGATE-All India Test Series
www.engineeringonyourfingertips.ooo
Leaders in GATE Preparation  65+ Centers across India
© All rights reserved by Gateforum Educational Services Pvt. Ltd. No part of this booklet may be reproduced or utilized in any form without the written permission.

More Notes Join us Telegram-: http//:t.me/allexammentor


www.engineeringonyourfingertips.ooo
|ME| GATE-2016-PAPER-03 www.gateforum.com

8. The state of stress at a point on an element is shown in figure (a). The same state of stress is
shown in another coordinate system in figure (b).
The components (xx,yy,xy) are given by


(A) p / 2,  p / 2, 0 
(B) (0, 0, p)

(C)  p, p, p / 2 
(D)  0,0, p / 2 
Key: (B) a  b
Exp: We know,
 x   y   x   y 
     cos 2  xy sin 2
 2   2 
Where,  is the location of any oblique plane which making an angle  in CCW direction.
 When   45o ,   xx ,  x  p,  y  p

pp pp
  xx      cos90  0
o
 2   2 
When   45o ,  yy

pp pp
      cos90  0
o
 2   2 
When   45o ,   xy

 x   y 
We know     sin 2  xy cos 2
 2 
pp
   xy    sin 90  p xx , yy , xy is 0, 0,p
 2 

9. A rigid link PQ is w ww. eng


undergoing i
nee
plane ringoas
motion ny our
shown fing
in er
the t i
ps.
figure o
(Voo
P and VQ are non-zero). VQP
is the relative velocity of point Q with respect to point P.

Which one of the following is TRUE?


(A) VQP has components along and perpendicular to PQ
(B) VQP has only one component directed from P to Q
(C) VQP has only one component directed from Q to P
(D) VQP has only one component perpendicular to PQ
Key: (D)
 ICP–Intensive Classroom Program  eGATE-Live Internet Based Classes DLP  TarGATE-All India Test Series
www.engineeringonyourfingertips.ooo
Leaders in GATE Preparation  65+ Centers across India
© All rights reserved by Gateforum Educational Services Pvt. Ltd. No part of this booklet may be reproduced or utilized in any form without the written permission.

More Notes Join us Telegram-: http//:t.me/allexammentor


www.engineeringonyourfingertips.ooo
|ME| GATE-2016-PAPER-03 www.gateforum.com

Exp: Let VP & VQ make an angle  and  with axis of link PQ respectively.

VQ cos Q

 VQ sin 
VQ sin 
VQ VQ cos
VP sin  VP sin 

VP
 P

VP cos VP cos

Since link PQ is rigid, so the distance between P & Q will never change. Hence relative
velocity between P & Q along axial direction should be zero.
Vp cos   VQ sin 
VQ sin 
Relative velocity between P & Q
 VP sin   PQ r to PQ
VP sin 
 VQ sin  PQ r to PQ
  VP sin   VQ sin   PQ r to PQ

10. The number of degrees of freedom in a planar mechanism having n links and j simple hinge
joints is
(A) 3(n  3)  2j (B) 3(n  1)  2j (C) 3n  2j (D) 2j  3n  4
Key: (B)
Exp: DOF F  3 n  1  2j
Where, n  total number of links
j = Effective number of binary points

11. www
The static deflection of.
ean gi ne
spring er i
ng
under onyowhen
gravity, ur
fin
ag er
ti
mass
2
p
ofs1.
o oo
kg is suspended from it, is 1
mm. Assume the acceleration due to gravity g =10 m/s . The natural frequency of this spring-
mass system (in rad/s) is ______
Key: (100)
Exp: st  103 m
g 10
n    100 rad/sec
st 103

12. Which of the bearings given below SHOULD NOT be subjected to a thrust load?
(A) Deep groove ball bearing
(B) Angular contact ball bearing
(C) Cylindrical (straight) roller bearing
(D) Single row tapered roller bearing
Key: (C)

 ICP–Intensive Classroom Program  eGATE-Live Internet Based Classes DLP  TarGATE-All India Test Series
www.engineeringonyourfingertips.ooo
Leaders in GATE Preparation  65+ Centers across India
© All rights reserved by Gateforum Educational Services Pvt. Ltd. No part of this booklet may be reproduced or utilized in any form without the written permission.

More Notes Join us Telegram-: http//:t.me/allexammentor


www.engineeringonyourfingertips.ooo
|ME| GATE-2016-PAPER-03 www.gateforum.com

13. A channel of width 450 mm branches into two sub-channels having width 300 mm and 200
mm as shown in figure. If the volumetric flow rate (taking unit depth) of an incompressible
flow through the main channel is 0.9 m3/s and the velocity in the sub-channel of width 200
mm is 3 m/s, the velocity in the sub-channel of width 300 mm is m/s.
Assume both inlet and outlet to be at the same elevation.

Key: 1
Exp: Apply Mass conservation and taking incompressibility
We have
300mm V2  ?
A1V1  A 2 V2  A3 V3 Given A1V1  0.9m3 /s
450mm
0.9  0.3V2  0.2  3
2

0.9  0.6  0.3V2 1


200mm
 V2  1m / s 3
0.9m /s 3

V3  3m/s

14. For a certain two-dimensional incompressible flow, velocity field is given by 2xyiˆ  y2ˆj . The
streamlines for this flow are given by the family of curves
(A) x 2 y 2  constant (B) xy 2  constant
www.
engi
neer
ingonyour
finger
tips.
ooo
(C) 2xy  y 2 = constant (D) xy  constant
Key: (B)
Exp: v  2xyiˆ  y 2 ˆj
 
u v
y x
2xy dy  d
On integrating
 = xy 2  f  x 

  y2  f '  x      y2 
x
 y2  f '  x   y2
f '  x   0  f  x   constant
  =xy 2  constant

 ICP–Intensive Classroom Program  eGATE-Live Internet Based Classes DLP  TarGATE-All India Test Series
www.engineeringonyourfingertips.ooo
Leaders in GATE Preparation  65+ Centers across India
© All rights reserved by Gateforum Educational Services Pvt. Ltd. No part of this booklet may be reproduced or utilized in any form without the written permission.

More Notes Join us Telegram-: http//:t.me/allexammentor


www.engineeringonyourfingertips.ooo
|ME| GATE-2016-PAPER-03 www.gateforum.com

15. Steady one-dimensional heat conduction takes place across the faces 1 and 3 of a composite
slab consisting of slabs A and B in perfect contact as shown in the figure, where kA , kB denote
the respective thermal conductivities. Using the data as given in the figure, the interface
temperature T2 (in °C) is .

Key: 67.5
T1  T2 T2  T3
Exp: 
LA LB
kA  A kB  A
130  T2 T2  30

0.1 0.3
20 100
390  3T2  5T2  150
8T2  540
T2  67.5o C
 Interface temperature  67.5o C

16. Grashof number signifies the ratio of


(A) inertia force to viscous force
(B) buoyancy force to viscous force
(C) buoyancy force to inertia force
(D) inertia force to surface tension force
Key: (B) www. engi neer ingonyour finger
tips.
ooo
Inertia force  Buoyant force
Exp: Grashof number (Gr) 
(Viscous force) 2

17. The INCORRECT statement about the characteristics of critical point of a pure substance is
that
(A) there is no constant temperature vaporization process
(B) it has point of inflection with zero slope
(C) the ice directly converts from solid phase to vapor phase
(D) saturated liquid and saturated vapor states are identical
Key: (C)
Exp: The process of conversion from solid phase to vapour phase is called sublimation and this
does not happen at critical point. All the other statements are true at Critical point.

 ICP–Intensive Classroom Program  eGATE-Live Internet Based Classes DLP  TarGATE-All India Test Series
www.engineeringonyourfingertips.ooo
Leaders in GATE Preparation  65+ Centers across India
© All rights reserved by Gateforum Educational Services Pvt. Ltd. No part of this booklet may be reproduced or utilized in any form without the written permission.

More Notes Join us Telegram-: http//:t.me/allexammentor


www.engineeringonyourfingertips.ooo
|ME| GATE-2016-PAPER-03 www.gateforum.com

18. For a heat exchanger, ∆Tmax is the maximum temperature difference and ∆Tmin is the minimum
temperature difference between the two fluids. LMTD is the log mean temperature difference.
Cmin and Cmax are the minimum and the maximum heat capacity rates. The maximum possible
heat transfer (Qmax) between the two fluids is
(A) Cmin LMTD (B) Cmin ∆Tmax (C) Cmax ∆Tmax (D) Cmax ∆Tmin
Key: (B)
Exp: In a heat exchanger, maximum possible heat transfer will be.
Qmax  Cmin Tmax

19. The blade and fluid velocities for an axial turbine are as shown in the figure.

The magnitude of absolute velocity at entry is 300 m/s at an angle of 65 to the axial direction,
while the magnitude of the absolute velocity at exit is 150 m/s. The exit velocity vector has a
component in the downward direction. Given that the axial (horizontal) velocity is the same at
entry and exit, the specific work (in kJ/kg) is _____
Key: 52.80
Exp: Given:   90  65  25o.
Let ‘1’ & ‘2’ denotes inlet & outlet of vane.
Vane velocity (u) = u1  u 2  150m / s.
Inlet:
Velocity of whirling Vw1  V1 cos  
www.
engi
neer ing
300cos25 onyour
271.89m /f
si
nger
tips.
ooo
 
Velocity of flow Vf1  V1 sin 
 300sin 25  126.7854m / s
 
Absolute velocity of Inlet = Vw1  Vf1
It is given that horizontal velocity is same at entry & exit.
So,
Vf1  Vf 2
126.7859  V2 cos 

126.7854 
  cos 1    32.3o
 150 
Vw2  V2 sin(32.3)  80.16m / s

 ICP–Intensive Classroom Program  eGATE-Live Internet Based Classes DLP  TarGATE-All India Test Series
www.engineeringonyourfingertips.ooo
Leaders in GATE Preparation  65+ Centers across India
© All rights reserved by Gateforum Educational Services Pvt. Ltd. No part of this booklet may be reproduced or utilized in any form without the written permission.

10

More Notes Join us Telegram-: http//:t.me/allexammentor


www.engineeringonyourfingertips.ooo
|ME| GATE-2016-PAPER-03 www.gateforum.com
u  150m / s
Vf2


V2 Vw 2
Vr2
Vf1

u2

Vr1
Vw1 V1

u1

65

Absolute velocity at outlet


  
V2  Vf2  Vw 2
Specific work done on the fluid per unit weight (wsp )
 
 wsp   u. Vw2  Vw1
 
Let ĵ unit vector positive toward upward.
()Ve

Vw1  271.89m / sec ()Ve

u  150m / sec

Vw2  80.16

www.
engi
neer
ingonyour
finger
tips.
ooo
()Ve


w  (150 ˆj). (271.89 ˆj)  (80.16 ˆj)  52.8075kJ / kg 
20. Engineering strain of a mild steel sample is recorded as 0.100%. The true strain is
(A) 0.010% (B) 0.055% (C) 0.099% (D) 0.101%
Key: (C)
0.1
Exp: 
100
We know,
T  ln 1    , where T is True strain and  is Engineering strain
 0.1 
 T  ln 1    0.0009995 T %  0.0009995  100  0.099%
 100 

 ICP–Intensive Classroom Program  eGATE-Live Internet Based Classes DLP  TarGATE-All India Test Series
www.engineeringonyourfingertips.ooo
Leaders in GATE Preparation  65+ Centers across India
© All rights reserved by Gateforum Educational Services Pvt. Ltd. No part of this booklet may be reproduced or utilized in any form without the written permission.

11

More Notes Join us Telegram-: http//:t.me/allexammentor


www.engineeringonyourfingertips.ooo
|ME| GATE-2016-PAPER-03 www.gateforum.com

21. Equal amounts of a liquid metal at the same temperature are poured into three moulds made of
steel, copper and aluminum. The shape of the cavity is a cylinder with 15 mm diameter. The
size of the moulds are such that the outside temperature of the moulds do not increase
appreciably beyond the atmospheric temperature during solidification. The sequence of
solidification in the mould from the fastest to slowest is
(Thermal conductivities of steel, copper and aluminum are 60.5, 401 and 237 W/m-K,
respectively. Specific heats of steel, copper and aluminum are 434, 385 and 903 J/kg-K,
respectively.
Densities of steel, copper and aluminum are 7854, 8933 and 2700 kg/m3, respectively.)
(A) Copper - Steel - Aluminum
(B) Aluminum - Steel – Copper
(C) Copper - Aluminum - Steel
(D) Steel - Copper - Aluminum
Key: (C)
Exp: K steel  60.5; steel  7854; Csteel  434 J/kg K
K copper  401; copper  8933; Ccopper  385 J/kg K
K A1  237; A1  2700; CAl  903 J/kg K

Heat capacity c


 c steel  3408.636 kJ/m3K
 c copper  3439.205 kJ/m3K
 c A1  2438.100 kJ/m3K
 k  60.5
  steel     103  0.0177 103 m 2 /s

 steel
c 3408
 k  401
  copper     103  0.1166 103 m 2 /s

 copper
c 3439
 k  w23.7
  A1     ww. e ng in
eer ing on ymo u
/sr
finger
tips.
ooo
3 3
10 0.1119 10 2


 A1
c 2438
So, cooling rates would be copper, Aluminium, steel

22. In a wire-cut EDM process the necessary conditions that have to be met for making a
successful cut are that
(A) wire and sample are electrically non-conducting
(B) wire and sample are electrically conducting
(C) wire is electrically conducting and sample is electrically non-conducting
(D) sample is electrically conducting and wire is electrically non-conducting
Key: (B)
Exp: In this process, a thin metallic wire is fed on to the conducting workpiece, which is submerged
in a tank of dielectric fluid such as de-ionized water. Wire is fed in the programmed path &
material is cut from the workpiece accordingly. Material removal takes place by a series of
discrete discharges between the wire electrode & workpiece in the presence of a dielectric
fluid. The di-electric fluid gets ionized in between the tool electrode gap thereby creating a

 ICP–Intensive Classroom Program  eGATE-Live Internet Based Classes DLP  TarGATE-All India Test Series
www.engineeringonyourfingertips.ooo
Leaders in GATE Preparation  65+ Centers across India
© All rights reserved by Gateforum Educational Services Pvt. Ltd. No part of this booklet may be reproduced or utilized in any form without the written permission.

12

More Notes Join us Telegram-: http//:t.me/allexammentor


www.engineeringonyourfingertips.ooo
|ME| GATE-2016-PAPER-03 www.gateforum.com

path for each discharge. The area wherein discharge takes place gets heated to very high
temperature such that the surface get melted & removed. The cut particles (debris) get flushed
away by continuous flowing dielectric fluid.
Generally, wire-cut EDM is used for cutting Aluminium, brass, etc. & wire material used for
quicker cutting action is zinc coated brass wires.

23. Internal gears are manufactured by


(A) hobbing (B) shaping with pinion cutter
(C) shaping with rack cutter (D) milling
Key: (B)

24. Match the following part programming codes with their respective functions

Part Programming Codes Functions

P. G01 I. Spindle stop

Q. G03 II. Spindle rotation, clockwise

R. M03 III. Circular interpolation,


anticlockwise
S. M05 IV. Linear interpolation

(A) P – II, Q – I, R – IV, S – III


(B) P – IV, Q – II, R – III, S – I
(C) P – IV, Q – III, R – II, S – I
(D) P – III, Q – IV, R – II, S – I
Key: (C)

25. In PERT chart, the activity time distribution is


(A) Normal www.
eng
(B) ineer i
Binomial ngonyour
(C) fi
nger
Poisson t
ips.
ooo
(D) Beta
Key: (D)

Q. No. 26 – 55 carry Two Marks Each


2 1 0
26.
 
The number of linearly independent eigenvectors of matrix A   0 2 0  is ____ .
0 0 3
Key: 2
Exp: Here   2, 2,3

For   2, No. of L.I eigen vectors

 3  rank of  A  2I   3  2  1
For   3, No. of L.I eigen vectors =1
∴ Total L.I eigen vectors = 2

 ICP–Intensive Classroom Program  eGATE-Live Internet Based Classes DLP  TarGATE-All India Test Series
www.engineeringonyourfingertips.ooo
Leaders in GATE Preparation  65+ Centers across India
© All rights reserved by Gateforum Educational Services Pvt. Ltd. No part of this booklet may be reproduced or utilized in any form without the written permission.

13

More Notes Join us Telegram-: http//:t.me/allexammentor


www.engineeringonyourfingertips.ooo
|ME| GATE-2016-PAPER-03 www.gateforum.com

4
27. The value of the line integral  F.r ds, where C is a circle of radius

units is .
C

Here, F(x, y)  yiˆ  2xjˆ and r is the UNIT tangent vector on the curve C at an arc length s
from a reference point on the curve. î and ĵ are the basis vectors in the x-y Cartesian
reference. In evaluating the line integral, the curve has to be traversed in the counter-clockwise
direction.

Key: 16
Exp: By Green’s theorem,

 F.r ' ds   ydx  2xdy    2  1 dxdy


c c
2
 4 
   16
 

28. lim x  x 2  x  1  x is
(A) 0 (B) ∞ (C) 1/2 (D) −∞
Key: (C)

Exp: lim
x 
 x2  x 1  x   x2  x 1  x
x2  x 1  x
x2  x 1  x2
 lim
x2  x 1  x
x 

 1
x 1  
 x 1 0 1
 lim  
x  1 1 1 0  0  1 2
x 1  2 1
x x

29. Three cards were drawn from a pack of 52 cards. The probability that they are a king, a queen,
and a jack is
16 www. eng64ineer ingonyour f
3inger tips. ooo 8
(A) (B) (C) (D)
5525 2197 13 16575

Key: (A)
4
C1  4C1  4C1 64 16
Exp: Required probability  52
 
C3 22100 5525

30. An inextensible massless string goes over a frictionless pulley. Two


weights of 100 N and 200 N are attached to the two ends of the
string. The weights are released from rest, and start moving due to
gravity. The tension in the string (in N) is .
Key: 133.33
200N

100N

 ICP–Intensive Classroom Program  eGATE-Live Internet Based Classes DLP  TarGATE-All India Test Series
www.engineeringonyourfingertips.ooo
Leaders in GATE Preparation  65+ Centers across India
© All rights reserved by Gateforum Educational Services Pvt. Ltd. No part of this booklet may be reproduced or utilized in any form without the written permission.

14

More Notes Join us Telegram-: http//:t.me/allexammentor


www.engineeringonyourfingertips.ooo
|ME| GATE-2016-PAPER-03 www.gateforum.com

Exp: From F.B.D


200
200  T  a …(1)
g

100
T  100  a …(2)
g
Adding equations (1) and (2) T
T
300 a
100  a  a  g/3 200
g
From equation (1) a 100
200 2 200
200  T    F.B.D F.B.D
g 3 3
T T
200
T  200   133.33N
3 a a

200 100

31. A circular disc of radius 100 mm and mass 1 kg, initially at rest at position A, rolls without
slipping down a curved path as shown in figure. The speed v of the disc when it reaches
position B is m/s.

www.
engi
neer
ingonyour
finger
tips.
ooo
Acceleration due to gravity g = 10 m/s2.
Key: 20
Exp: According to energy conservation principle
Total K.E + P.E = constant
 K.E A  P.E A  K.E B  P.E B
1 1
 0  mg.30  m  v 2  I2  0
2 2
1 1 1
 30 10  1 v 2    1  v 2
2 2 2
 mr 2
v
I  and   
 2 r
3 2
 30 10   v  v  20m/s
4

 ICP–Intensive Classroom Program  eGATE-Live Internet Based Classes DLP  TarGATE-All India Test Series
www.engineeringonyourfingertips.ooo
Leaders in GATE Preparation  65+ Centers across India
© All rights reserved by Gateforum Educational Services Pvt. Ltd. No part of this booklet may be reproduced or utilized in any form without the written permission.

15

More Notes Join us Telegram-: http//:t.me/allexammentor


www.engineeringonyourfingertips.ooo
|ME| GATE-2016-PAPER-03 www.gateforum.com

32. A rigid rod (AB) of length L  2 m is undergoing translational as well as rotational motion
in the x-y plane (see the figure). The point A has the velocity V1  ˆi  2ˆjm / s. The end B is
constrained to move only along the x direction.
The magnitude of the velocity V2 (in m/s) at the end B is
Key: 3
Exp: v1  i  2j V 5 1
 v x  1 and v y  2 2
V2 cos 45
v1  1  2  5
2 2 =α+45°
45 V2
1
Let ϕ is angle between v1 and x-axis.
 tan   2   63.43 V1
V2 sin 45
   45  63.43    18.43
V1 sin  V1 cos
Since, the rod is rigid 
 v1 cos   v 2 cos 45 45

 5 cos18.43  v 2 cos 45  v 2  3m/s

33. A square plate of dimension L × L is subjected to a uniform pressure load p = 250 MPa on
its edges as shown in the figure. Assume plane stress conditions. The Young’s modulus E =
200 GPa.

www
The deformed shape is.
ean gi nee
square r
ofin gonyouLr
dimension i
fn e
2g.r
Ift
ip.
Lso moand   0.001 m, the
2o
Poisson’s ratio of the plate material is ______
Key: 0.2
Exp: According to Hooke’s Law
 y
x  x 
E E
Where,  x   y   p and  x 
2

2  p p 
    
 E E 
21 p 250
  1     1       0.2
2000 E 2  105

 ICP–Intensive Classroom Program  eGATE-Live Internet Based Classes DLP  TarGATE-All India Test Series
www.engineeringonyourfingertips.ooo
Leaders in GATE Preparation  65+ Centers across India
© All rights reserved by Gateforum Educational Services Pvt. Ltd. No part of this booklet may be reproduced or utilized in any form without the written permission.

16

More Notes Join us Telegram-: http//:t.me/allexammentor


www.engineeringonyourfingertips.ooo
|ME| GATE-2016-PAPER-03 www.gateforum.com

34. Two circular shafts made of same material, one solid (S) and one hollow (H), have the same
length and polar moment of inertia. Both are subjected to same torque. Here, S is the twist
and S is the maximum shear stress in the solid shaft, whereas H is the twist and H is the
maximum shear stress in the hollow shaft. Which one of the following is TRUE?
(A) s  H and s  H (B) s  H and s  H
(C) s  H and s  H (D) s  H and s  H
Key: (D)
Exp: According to pure torsion equation
T  G
 
J R 
Let ds and d1 are diameter of solid shaft and outer diameter of hollow shaft
Ts s

Js R s  
 s  h
Th h Rs Rh

Jh R h
  d
 s  h  s   h . s
d s d1 d1
Since, J s  J h
 d1 must be greater than ds
ds
 must be less than 1  s  h and
d1
Ts G s s

T G Js s
   s  h
J  T G 
h
 h h
Jh h

35. A beam of length L is carrying a uniformly distributed load w per unit length. The flexural
rigidity of the beam is EI. The reaction at the simple support at the right end is _____ .

www.
engi
neer
ingonyour
finger
tips.
ooo

wL 3wL wL wL
(A) (B) (C) (D)
2 8 4 8
Key: (B)

Exp: 

A B 

RB


2
A B 
1
RB
 ICP–Intensive Classroom Program  eGATE-Live Internet Based Classes DLP  TarGATE-All India Test Series
www.engineeringonyourfingertips.ooo
Leaders in GATE Preparation  65+ Centers across India
© All rights reserved by Gateforum Educational Services Pvt. Ltd. No part of this booklet may be reproduced or utilized in any form without the written permission.

17

More Notes Join us Telegram-: http//:t.me/allexammentor


www.engineeringonyourfingertips.ooo
|ME| GATE-2016-PAPER-03 www.gateforum.com

4 R 3
1  ; 2  B
8EI 3EI
Since, net vertical deflection at B is zero
 1  2
 4 R B3 3
   RB 
8EI 3EI 8

36. Two masses m are attached to opposite sides of a rigid rotating shaft in the vertical plane.
Another pair of equal masses m1 is attached to the opposite sides of the shaft in the vertical
plane as shown in figure. Consider m = 1 kg, e = 50 mm, e1 = 20 mm, b = 0.3 m, a = 2 m and
a1 = 2.5 m. For the system to be dynamically balanced, m1 should be kg.

Key: 2
Exp: Couple due to m = couple due to m1
mea  m1e1a1
 50   2 
m1  1        2kg
 20   2.5 

37. A single degree of freedom spring-mass system is subjected to a harmonic force of constant
3k
amplitude. For an excitation frequency of , the ratio of the amplitude of steady state
m
www
response to the static .engi ne
deflection oferi
n
the gony
spring isour .f
inger tips. ooo

Key: 0.5
1 1 1
M.F      0.5
 
Exp: 2
1 3 2
1 
n

 ICP–Intensive Classroom Program  eGATE-Live Internet Based Classes DLP  TarGATE-All India Test Series
www.engineeringonyourfingertips.ooo
Leaders in GATE Preparation  65+ Centers across India
© All rights reserved by Gateforum Educational Services Pvt. Ltd. No part of this booklet may be reproduced or utilized in any form without the written permission.

18

More Notes Join us Telegram-: http//:t.me/allexammentor


www.engineeringonyourfingertips.ooo
|ME| GATE-2016-PAPER-03 www.gateforum.com

38. A bolted joint has four bolts arranged as shown in figure. The cross sectional area of each bolt
is 25 mm2. A torque T = 200 N-m is acting on the joint. Neglecting friction due to clamping
force, maximum shear stress in a bolt is MPa.

Key: 40
Exp: Let the resisting force in each bolt = F Newton
3
Net resisting torque (TR) = 4F  50 10 N-m
Applied torque (T) = Resisting Torque
T  4F  50103
200
F 1000  1000N
4  50
Let shear stress developed in each bolt =  MPa
It is given that resisting area (AR) = 25mm2
F    AR
1000
  40MPa
25

39. Consider a fully developed steady laminar flow of an incompressible fluid with viscosity µ
through a circular pipe of radius R. Given that the velocity at a radial location of R/2 from the
centerline of the pipe is U1, the shear stress at the wall is kµU1/R, where K is
Key: 2.667
Exp: Given, ,R u at R/2  U1 ,
y wall  kw
u1w
/Rw.
engi
neer
ingonyour
finger
tips.
ooo
K=?
Velocity profile in horizontal pipe flow is
 r2 
u  u max 1  2 
 R 
 R2 
u1  u max 1  2 
 4R 
3 4
u1  u max  u max  u1
4 3

 u
y wall   
 r r  R
2r 2 2 u max 2 4
   max 2
  u max    u1
R R R R 3
8 u1 8
  k   2.667
3 R 3
 ICP–Intensive Classroom Program  eGATE-Live Internet Based Classes DLP  TarGATE-All India Test Series
www.engineeringonyourfingertips.ooo
Leaders in GATE Preparation  65+ Centers across India
© All rights reserved by Gateforum Educational Services Pvt. Ltd. No part of this booklet may be reproduced or utilized in any form without the written permission.

19

More Notes Join us Telegram-: http//:t.me/allexammentor


www.engineeringonyourfingertips.ooo
|ME| GATE-2016-PAPER-03 www.gateforum.com

40. The water jet exiting from a stationary tank


through a circular opening of diameter 300 mm
impinges on a rigid wall as shown in the figure.
Neglect all minor losses and assume the water
level in the tank to remain constant. The
net horizontal force experienced by the
wall is kN.
Density of water is 1000 kg/m3.
Acceleration due to gravity g = 10 m/s2

Key: 8.76
Exp: Force exerted by a jet of water striking fixed wall is av 2 N where a is area of jet.


 103   0.32  v 2  v  2gh  2 10  6.2
4


103   0.32  2  10  62  8.76kN
4

 x ˆ y ˆ
41. For a two-dimensional flow, the velocity field is u  i 2 j, where ˆi and ˆj are the
x y
2 2
x  y2
basis vectors in the x-y Cartesian coordinate system. Identify the CORRECT statements from
below.
(1) The flow is incompressible.
(2) The flow is unsteady.
y
(3) y-component of acceleration, a y 
 
2
x  y2
2

  x  y
(4) x-component of acceleration, a x 
x  y2 
2 2

(A) (2) and (3) (B) (1) and (3) (C) (1) and (2)
www.engi neer ingonyourfinger tips.ooo(D) (3) and (4)
Key: (B)
x y
Exp: u ; v
x 2  y2 x 2  y2
 Given flow is independent of time of flow is steady
 Density ‘ρ’ is not present in velocity components, so flow is incompressible
Acceleration along x direction
u u
ax  u v
x y
x
 u
x  y2
2

u  x  y   x2x
2 2
y2  x 2 u 2xy
    
x x  y   x  y  y  x  y2 
2 2 2 2 2 2 2 2

 ICP–Intensive Classroom Program  eGATE-Live Internet Based Classes DLP  TarGATE-All India Test Series
www.engineeringonyourfingertips.ooo
Leaders in GATE Preparation  65+ Centers across India
© All rights reserved by Gateforum Educational Services Pvt. Ltd. No part of this booklet may be reproduced or utilized in any form without the written permission.

20

More Notes Join us Telegram-: http//:t.me/allexammentor


www.engineeringonyourfingertips.ooo
|ME| GATE-2016-PAPER-03 www.gateforum.com

u
 u  2
x  y  x   xy  x
2 2 2 3
…(a)
x x  y2 x  y  x  y 
2 2 2 2 2 3

u y 2xy 2xy 2
 v  2  …(b)
y  x  y2   x 2  y 2 2  x 2  y 2 3

xy2  x3   2xy2   x  x 2  y2  x
ax  a  b   
x 
2 3
x 
2 3
x  y2 
2
2
y 2
y 2

So, only statement 1 & 3 are correct.

42. Two large parallel plates having a gap of 10 mm in between them are maintained at
temperatures.
T1 = 1000 K and T2 = 400 K. Given emissivity values, 1  0.5, 2  0.25 and Stefan-
Boltzmann constant  = 5.67 × 10−8 W/m2-K4, the heat transfer between the plates (in
kW/m2) is
Key: 11.049
  T14  T24 
Exp: Q1 2  T1  1000K
1 1 T2  400K
 1
1 2
5.67  108 10004  4004 

1

1
1 1  0.5 2  0.25
0.5 0.25
 11049.696 W 2  11.049 kW 2
m m

43. A cylindrical steel rod, 0.01 m in diameter and 0.2 m in length is first heated to 750 0 C and
then immersed in a water bath at 1000C. The heat transfer coefficient is 250 W/m2-K. The
density, specific heat and thermal conductivity of steel are =7801 kg/m3, c = 473
J/kg-K, and k = 43 W/m-K, respectively. The time required for the rod to reach 3000C is
____ seconds.
Key: 43.49 www. engi neer ingonyour f
inger tips. ooo
Exp: d = 0.01 m
L = 0.2m, R=7801 kg/m3.
t i  750o C, C  473J / kgK
t   100o C, k  43 W / mK
 2
d
V 4 d
h  250 W / m K, Lc   2

A d 4
 t  t   hA h
ln     
 t i  t   Vc cLc
 t  t   h  4
ln   
 t i  t   c  d
 300  100  250  4
ln       43.49s
 750  100  7801 473  0.01

 ICP–Intensive Classroom Program  eGATE-Live Internet Based Classes DLP  TarGATE-All India Test Series
www.engineeringonyourfingertips.ooo
Leaders in GATE Preparation  65+ Centers across India
© All rights reserved by Gateforum Educational Services Pvt. Ltd. No part of this booklet may be reproduced or utilized in any form without the written permission.

21

More Notes Join us Telegram-: http//:t.me/allexammentor


www.engineeringonyourfingertips.ooo
|ME| GATE-2016-PAPER-03 www.gateforum.com

44. Steam at an initial enthalpy of 100 kJ/kg and inlet velocity of 100 m/s, enters an
insulated horizontal nozzle. It leaves the nozzle at 200 m/s. The exit enthalpy (in kJ/kg)
is .
Key: 85 1 2
Exp: h1  100kJ / kg
C1  100m / s
C1 C2
C2  200m / s
h1 h2
2 2
C C
h1   h2 
1 2
2000 2000
1002 2002
h 2  100    85kJ / kg
2000 2000

45. In a mixture of dry air and water vapor at a total pressure of 750 mm of Hg, the partial
pressure of water vapor is 20 mm of Hg. The humidity ratio of the air in grams of water vapor
per kg of dry air (gw/kgda) is ____ .
Key: 17
Exp: Pt  750 mm of Hg
Pv  20 mm of Hg
Humidity ratio (or) specific Humidity
PV
w  0.622
Pt  PV
 20 
 0.622   
 750  20 
kg g
 0.017 w.v  17 w.v
kg d.a kg d.a

46. In a 3-stage air compressor, the inlet pressure is p1 , discharge pressure is p4 and the

wwware
intermediate pressures .
enpg
2i
nee
and i
p3rn
p o
2g 3
pny .our
f
The inge
total rti
ps.
pressureo ooof the compressor is 10
ratio
and the pressure ratios of the stages are equal. If p1  100 kPa, the value of the pressure p3 (in
kPa) is .
Key: 464.16
Exp: 3 stage compressor, P1= 100 kPa
Pressure ratios of all stages are equal
P2 P3 P4
 
P1 P2 P3
P4
Overall pressure ratio   10
P1
P2 P3 P4 3
 rp     10  2.154
P1 P2 P3
P3  (rp )2  P1
 P3  2.514  2.514 100  464.16kPa

 ICP–Intensive Classroom Program  eGATE-Live Internet Based Classes DLP  TarGATE-All India Test Series
www.engineeringonyourfingertips.ooo
Leaders in GATE Preparation  65+ Centers across India
© All rights reserved by Gateforum Educational Services Pvt. Ltd. No part of this booklet may be reproduced or utilized in any form without the written permission.

22

More Notes Join us Telegram-: http//:t.me/allexammentor


www.engineeringonyourfingertips.ooo
|ME| GATE-2016-PAPER-03 www.gateforum.com

47. In the vapour compression cycle shown in the figure, the


evaporating and condensing temperatures are 260 K and 310 K,
respectively. The compressor takes in liquid-vapour mixture (state
1) and isentropically compresses it to a dry saturated vapour
condition (state 2). The specific heat of the liquid refrigerant is
4.8kJ/kg-K and may be treated as constant. The enthalpy of
evaporation for the refrigerant at 310 K is 1054 kJ/kg.
The difference between the enthalpies at state points 1 and 0 (in kJ/kg) is
Key: 1103.51
Exp: dQ  CpdT
3
Cp dT h fg T
s 2  s0   
0 T 310
3 2
 310  1054
s1  s0  4.8ln   ( s1  s 2 )
 260  310 0
4 1
h1  h 0  310  1054
 4.8ln   s
260  260  310
 h1  h 0  1103.51kJ kg

48. Spot welding of two steel sheets each 2 mm thick is carried out successfully by passing 4 kA
of current for 0.2 seconds through the electrodes. The resulting weld nugget formed between
the sheets is 5 mm in diameter. Assuming cylindrical shape for the nugget, the thickness of the
nugget is mm.

Latent heat of fusion for steel 1400 kJ/kg


Effective resistance of the weld joint 200 
Density of steel 8000 kg/m3
Key: 2.91
Exp: t s  2mm
I  4kA; t  0.2
d  5mm; t n  ?
www.
engi
neer
ingonyour
finger
tips.
ooo
L.H 1400
R  200
  8000 kg m3
Energy supplied = I2 Rt …(a)

 
2
 4  103  200  106  0.2  640J

Energy required for melting =  V   L.H  … (b)


 1400  103  8000   52  106  t n  219911.4858t n
4
Equating (a) & (b)
640 = 219911.4858t n
t n  2.91mm
 ICP–Intensive Classroom Program  eGATE-Live Internet Based Classes DLP  TarGATE-All India Test Series
www.engineeringonyourfingertips.ooo
Leaders in GATE Preparation  65+ Centers across India
© All rights reserved by Gateforum Educational Services Pvt. Ltd. No part of this booklet may be reproduced or utilized in any form without the written permission.

23

More Notes Join us Telegram-: http//:t.me/allexammentor


www.engineeringonyourfingertips.ooo
|ME| GATE-2016-PAPER-03 www.gateforum.com

49. For an orthogonal cutting operation, tool material is HSS, rake angle is 22°, chip thickness is
0.8 mm, speed is 48 m/min and feed is 0.4 mm/rev. The shear plane angle (in degrees) is
(A) 19.24 (B) 29.70 (C) 56.00 (D) 68.75
Key: (B)
Exp: Given o  22
Thickness of chip = 0.8mm  t 2 
Feed = 0.4 mm/rev
Speed = 48m/min
In orthogonal cutting = feed = thickness of uncut chip.  t1 
t 0.8
K 2 2
t1 0.4
cos o cos 22
tan     0.57
K  sin o 2  sin 22
  tan 1  0.57   29.7

50. In a sheet metal of 2 mm thickness a hole of 10 mm diameter needs to be punched. The yield
strength in tension of the sheet material is 100 MPa and its ultimate shear strength is 80 MPa.
The force required to punch the hole (in kN) is
Key: 5.0265
Exp: Given
t  2mm
d  10mm
s  80MPa

Syt  100MPa
F  dt  s   10  2  80  5026.5N  5.0265kN

51. In a single point turning operation with cemented carbide tool and steel work piece, it is found
that the Taylor’sw exponent
ww. eng isi
n0.25.
eer Ifng
i the
oncutting
your speed
finge isr
treduced
ips. oooby 50% then the tool life
changes by ____ times
Key: 16
Exp: n = 0.25
V
V2  1
2
V1T1n  V2T2n
V
V1T1n  1 T2n
2
0.25
 T2  2
 T
 1

T2  T1  24
T2  16  T1
Tool life changes by 16 times

 ICP–Intensive Classroom Program  eGATE-Live Internet Based Classes DLP  TarGATE-All India Test Series
www.engineeringonyourfingertips.ooo
Leaders in GATE Preparation  65+ Centers across India
© All rights reserved by Gateforum Educational Services Pvt. Ltd. No part of this booklet may be reproduced or utilized in any form without the written permission.

24

More Notes Join us Telegram-: http//:t.me/allexammentor


www.engineeringonyourfingertips.ooo
|ME| GATE-2016-PAPER-03 www.gateforum.com

52. Two optically flat plates of glass are kept at a small angle  as shown in the figure.
Monochromatic light is incident vertically.

If the wavelength of light used to get a fringe spacing of 1 mm is 450 nm, the wavelength of
light (in nm) to get a fringe spacing of 1.5 mm is _______.
Key: 675
Exp: Fringe space by (d1) = 1mm
h
1  450nm t

d2  1.5mm, 2  ?
x
1 
tan    2 L
2d1 2d 2
 1.5 
 2  450     675nm
 1 

53. A point P (1, 3, −5) is translated by 2iˆ  3jˆ  4kˆ and then rotated counter clockwise by 90
about the z-axis. The new position of the point is
(A) (−6, 3, −9) (B) (−6, −3, −9) (C) (6, 3, −9) (D) (6, 3, 9)
Key: (A)
Exp: We can write vector 2iˆ  3jˆ  4kˆ y

as coordinate form as  2, 3,  4  .

OP  ˆi  3jˆ  5kˆ P 1,3, 5 P'  3,6, 9

OP  3iˆ  6ˆj  9kˆ

Now rotate OP' vector about z axis through 90°
www
in counter clockwise .
engi neer
direction. i
ngonyour fi
nger
tips.ooo
Since, it is rotated about z-axis, therefore z-
x
coordinate remains same. 0
x n  x o cos   yo sin 
yn  x o sin   yo cos 
z
Where  x o , yo  are co-ordinates corresponding
to old values
 xn , yn  are co-ordinates corresponding to new values

After rotating vector OP through an angle θ in counter clockwise
x n  3 cos90  6sin 90
x n  6
y n  3sin 90  6cos90
yn  3
Hence the new coordinate is  6, 3,  9

 ICP–Intensive Classroom Program  eGATE-Live Internet Based Classes DLP  TarGATE-All India Test Series
www.engineeringonyourfingertips.ooo
Leaders in GATE Preparation  65+ Centers across India
© All rights reserved by Gateforum Educational Services Pvt. Ltd. No part of this booklet may be reproduced or utilized in any form without the written permission.

25

More Notes Join us Telegram-: http//:t.me/allexammentor


www.engineeringonyourfingertips.ooo
|ME| GATE-2016-PAPER-03 www.gateforum.com

54. The demand for a two-wheeler was 900 units and 1030 units in April 2015 and May
2015, respectively. The forecast for the month of April 2015 was 850 units. Considering a
smoothing constant of 0.6, the forecast for the month of June 2015 is
(A) 850 units (B) 927 units
(C) 965 units (D) 970 units
Key: (D)
Exp:
Month Demand Forecast    D t 1  1    Ft 1 
April 900 850
May 1030  0.6 900  0.4 850  880
June  0.6 1030  0.4 880  970
 FJune  970 units

55. A firm uses a turning center, a milling center and a grinding machine to produce two parts.
The table below provides the machining time required for each part and the maximum
machining time available on each machine. The profit per unit on parts I and II are Rs. 40 and
Rs. 100, respectively. The maximum profit per week of the firm is Rs. .

Machining time required for


the machine part (minutes) Maximum machining time
Type of machine
available per week (minutes)
I II
Turning Center 12 6 6000
Milling Center 4 10 4000
Grinding Machine 2 3 1800

x2
Key: 40,000
Exp: Linear program formulation is
ww2w.
max z  40x1  100x engi
neer
ingonyour f
inger
tips.
ooo
 0, 1000
Constraints are
12x1  6x 2  6000  0, 600
4x1  10x 2  4000
2x1  3x 2  1800  0, 400
x1 , x 2  0
375, 250
At  0,400  , z  40,000
At  375, 250  ,z  40,000 x1
500,0 900,0 1000,0
At  500,0  ,z  20,000
∴ Maximum profit per week is Rs. 40,000.

 ICP–Intensive Classroom Program  eGATE-Live Internet Based Classes DLP  TarGATE-All India Test Series
www.engineeringonyourfingertips.ooo
Leaders in GATE Preparation  65+ Centers across India
© All rights reserved by Gateforum Educational Services Pvt. Ltd. No part of this booklet may be reproduced or utilized in any form without the written permission.

26

More Notes Join us Telegram-: http//:t.me/allexammentor


www.engineeringonyourfingertips.ooo
ME-GATE-2015 PAPER-01| www.gateforum.com

General Aptitude
Q. No. 1 – 5 Carry One Mark Each

1. What is the adverb for the given word below?


Misogynous
(A) Misogynousness (B) Misogynity
(C) Misogynously (D) Misogynous

Answer: (C)

2. Ram and Ramesh appeared in an interview for two vacancies in the same department. The
probability of Ram‟s selection is 1/6 and that of Ramesh is 1/8. What is the probability that
only one of them will be selected?
47 1 13 35
A B C D
48 4 48 48

Answer: (B)
Exp: P Ram 1 ; p Ramesh 1
6 8
1 7 1 5
p only at p Ram p not ramesh p Ramesh p n 0 R am
6 8 8 6
12 1
40 4

3. Choose the appropriate word/phrase, out of the four options given below, to complete the
following sentence:
Dhoni, as well as the other team members of Indian team, _____ present on the occasion.
(A) were (B) was (C) has (D) have
Answer: (B)

www.
engi
neer
ingonyour
finger
tips.
ooo
4. An electric bus has onboard instruments that report the total electricity consumed since the
start of the trip as well as the total distance covered. During a single day of operation, the bus
travels on stretches M, N, O and P, in that order. The cumulative distances travelled and the
corresponding electricity consumption are shown in the table below.

Stretch Comulative Electricity


distance(km) used (kWh)
M 20 12
N 45 25
O 75 45
P 100 57

The stretch where the electricity consumption per km is minimum is


(A) M (B) N (C) O (D) P
Answer: (D)

India’s No.1 institute for GATE Training 1 Lakh+ Students trained till date 65+ Centers
across India
1
www.engineeringonyourfingertips.ooo

More Notes Join us Telegram-: http//:t.me/allexammentor


www.engineeringonyourfingertips.ooo
ME-GATE-2015 PAPER-01| www.gateforum.com

Exp:
Stretch Comulative Electricity Individual(km) Individual
distance(km) used Distance electricity(kWh)
(kWh)
M 20 12 20 12
N 45 25 25 13
O 75 45 30 20
P 100 57 25 12

For M 12 0.6
20
N 13 0.555
25
O 20 0.667
30
P 12 0.48
25

5. Choose the word most similar in meaning to the given word:


Awkward
(A) Inept (B) Graceful (C) Suitable (D) Dreadful

Answer: (A)

6. In the following sentence certain parts are underlined and marked P, Q and R. One of the
parts may contain certain error or may not be acceptable in standard written communication.
Select the part containing an error. Choose D as your Answer: if there is no error.
The student corrected all the errors that the instructor marked on the answer book
P Q R
(A) P (B) Q (C) R (D) No Error

Answer: (B)
ww
Exp: The is not required inw .
engi
„Q‟ neer
ingonyour
finger
tips.
ooo

7. Given below are two statements followed by two conclusions. Assuming these statements to
be true, decide which one logically follows.
Statement:
I. All film stars are playback singers.
II. All film directors are film stars.
Conclusions:
I. All film directors are playback singers.
II. Some film stars are film directors.
(A) Only conclusion I follows
(B) Only conclusion II follows
(C) Neither conclusion I nor II follows
(D) Both conclusions I and II follow

India’s No.1 institute for GATE Training 1 Lakh+ Students trained till date 65+ Centers
across India
2
www.engineeringonyourfingertips.ooo

More Notes Join us Telegram-: http//:t.me/allexammentor


www.engineeringonyourfingertips.ooo
ME-GATE-2015 PAPER-01| www.gateforum.com

Answer: (D)

8. Lamenting the gradual sidelining of the arts in school curricula, a group of prominent artists wrote to
the Chief Minister last year, asking him to allocate more funds to support arts education in
schools. However, no such increase has been announced in this year‟s Budget. The artists
expressed their deep anguish at their request not being approved, but many of them remain
optimistic about funding in the future.
Which of the statement(s) below is/are logically valid and can be inferred from the above
statements?
i. The artists expected funding for the arts to increase this year.
ii. The Chief Minister was receptive to the idea of increasing funding for the arts.
iii. The Chief Minister is a prominent artist.
iv. Schools are giving less importance to arts education nowadays.
(A) iii and iv (B) i and iv
(C) i, ii and iv (D) i and iii
Answer: (B)

2
9. If a b2 c2 1, then ab + bc + ac lies in the interval
2 1 1
A 1, B ,1 C 1, D 2, 4
3 2 2

Answer: (B)

10. A tiger is 50 leaps of its own behind a deer. The tiger takes 5 leaps per minute to the deer‟s 4.
If the tiger and the deer cover 8 metre and 5 metre per leap respectively, what distance in
meters will the tiger have a run before it catches the deer?

Answer: 800
Exp: Tiger 1leap 8 meter
ww40m
Speed 5leap hr w.engi
min neer
ingonyour
finger
tips.
ooo
Deer 1 leap 5meter
speed 4hr 20m min
Let at time „t‟ the tiger catches the deer.
Distance travelled by deer + initial distance between them
50 8 400m = distance covered by tiger.
40 t 400 20t
400
t 20 min
200
total dis tance 400 20 t 800 ms

India’s No.1 institute for GATE Training 1 Lakh+ Students trained till date 65+ Centers
across India
3
www.engineeringonyourfingertips.ooo

More Notes Join us Telegram-: http//:t.me/allexammentor


www.engineeringonyourfingertips.ooo
ME-GATE-2015 PAPER-01| www.gateforum.com

Mechanical engineering

4 7 8
1. If any two columns of a determinant P 3 1 5 are interchanged, which one of the following
9 6 2
statements regarding the value of the determinant is CORRECT?
(A) Absolute value remains unchanged but sign will change
(B) Both absolute value and sign will change
(C) Absolute value will change but sign will not change
(D) Both absolute value and sign will remain unchanged
Answer: (A)
Q
2. A wheel of radius r rolls without slipping on a horizontal surface
shown below. If the velocity of point P is 10 m/s in the
horizontal direction, the magnitude of velocity of point Q (in P
m/s) is ______.
r
Answer: 20
A
Exp: At p t 'A', the velocity v
10 10 20 m s
v

v 0

3. Which one of the following types of stress-strain relationship best describes the behavior of
brittle materials, such as ceramics and thermosetting plastics, (σ = stress and ε = strain)?

A
B
www.
engi
neer
ingonyour
finger
tips.
ooo

C
D

Answer: (D)

India’s No.1 institute for GATE Training 1 Lakh+ Students trained till date 65+ Centers
across India
4
www.engineeringonyourfingertips.ooo

More Notes Join us Telegram-: http//:t.me/allexammentor


www.engineeringonyourfingertips.ooo
ME-GATE-2015 PAPER-01| www.gateforum.com

4. The function of interpolator in a CNC machine controller is to


(A) control spindle speed
(B) coordinate feed rates of axes
(C) control tool rapid approach speed
(D) perform Miscellaneous (M) functions (tool change, coolant control etc.)
Answer: (B)

0.040
5. Holes of diameter 25.0 0.020 mm are assembled interchangeably with the pins of diameter
0.005
25.0 0.008 mm. The minimum clearance in the assembly will be
(A) 0.048 mm (B) 0.015 mm
(C) 0.005 mm (D) 0.008 mm

Answer: (B)
Exp: Minimum clearance
minimu m hole max ium shaft
25 .020 25 .005
0.015 mm

1 3 2 9
6. Simpson‟s rule is used to integrate the function f x x between x = 0 and x = 1
3 5 5
using the least number of equal sub-intervals. The value of the integral is ___________
Answer: 3
1
Exp: x 0 1
2
3 2 9 9 39 12
y f x x
5 5 5 20 5
1
9ww
12w.
engineer
ingonyour
finger
tips.
ooo
1
2 39
ydx 4
0
2 5 5 20
3

7. Consider fully developed flow in a circular pipe with negligible entrance length effects.
Assuming the mass flow rate, density and friction factor to be constant, if the length of the
pipe is doubled and the diameter is halved, the head loss due to friction will increase by a
factor of
(A) 4 (B) 16 (C) 32 (D) 64
Answer: (D)

India’s No.1 institute for GATE Training 1 Lakh+ Students trained till date 65+ Centers
across India
5
www.engineeringonyourfingertips.ooo

More Notes Join us Telegram-: http//:t.me/allexammentor


www.engineeringonyourfingertips.ooo
ME-GATE-2015 PAPER-01| www.gateforum.com

2
Q
fl
fLv 2 A
Exp: head loss
2gd 2gd
L
head loss
d5
L
h1*
d5
.L L
h *2 5
64
d d5
2
h1 1 h2
64
h2 64 h1

8. For an ideal gas with constant values of specific heats, for calculation of the specific enthalpy,
(A) it is sufficient to know only the temperature
(B) both temperature and pressure are required to be known
(C) both temperature and volume are required to be known
(D) both temperature and mass are required to be known
Answer: (A)

9. A Carnot engine (CE-1) works between two temperature reservoirs A and B, where TA = 900
K and TB = 500 K. A second Carnot engine (CE-2) works between temperature reservoirs B
and C, where TC = 300 K. In each cycle of CE-1 and CE-2, all the heat rejected by CE-1 to
reservoir B is used by CE-2. For one cycle of operation, if the net Q absorbed by CE-1 from
reservoir A is 150 MJ, the net heat rejected to reservoir C by CE-2 (in MJ) is _________.
Answer: 50
1 T1 1 500 900
Exp: 1 4.44
T1 900 Q1
Q 1 Q 53.33 MJ HE W
2 1 1
Q2
1 T3 300
1 0.4 500
2
T2 w ww.
500 engi
neer
ingonyour
finger tips.
ooo
Q2
Q3 1 Q 2 50 MJ
2 HE W
Q3
300

kg
10. A stream of moist air (mass flow rate = 10.1 kg/s) with humidity ratio of 0.01
kg dry air
mixes with a second stream of superheated water vapour flowing at 0.1 kg/s. Assuming
proper and uniform mixing with no condensation, the humidity ratio of the final stream
kg
in is ___________.
kg dry air
Answer: 0.0197

India’s No.1 institute for GATE Training 1 Lakh+ Students trained till date 65+ Centers
across India
6
www.engineeringonyourfingertips.ooo

More Notes Join us Telegram-: http//:t.me/allexammentor


www.engineeringonyourfingertips.ooo
ME-GATE-2015 PAPER-01| www.gateforum.com

m1 1 m 2 2
Exp: new
m1 m 2
0.1 10.1 .1 1
.0197 kg kg dry air
10.1 .1
11. Consider a steel (Young‟s modulus E = 200 GPa) column hinged on both sides. Its height is
1.0m and cross-section is 10 mm × 20 mm. The lowest Euler critical buckling load (in N) is
______.
Answer: 3289.86
2 2
EI 200 109 .02 .013
Exp: Euler 's critical load
l2 12
3289.8681 N

12. Air enters a diesel engine with a density of 1.0 kg/m3. The compression ratio is 21. At steady
state, the air intake is 30 × 10–3 kg/s and the net work output is 15 kW. The mean effective
pressure (kPa) is _______.
Answer: 525
work output
Exp: mep
swept volume
15
1
30 10 3 1
21
525kPa

13. Match the following products with preferred manufacturing processes:

Product Process
P Rails (1)Blow
molding
Q Engine crankshaft (2) Extrusion
R www.
Aluminium engi neer
channels i
n
(3)gonyour
Forging fi
nger
tips.
ooo
S PET water bottles (4) Rolling

A P 4, Q 3, R 1, S 2 B P 4, Q 3, R 2, S 1
C P 2, Q 4, R 3, S 1 D P 3, Q 4, R 2, S 1
Answer: (B)

th
1
14. Under certain cutting conditions, doubling the cutting speed reduces the tool life to of
16
the original. Taylor‟s tool life index(n) for this tool-workpiece combination will be _______
Answer: 0.25

India’s No.1 institute for GATE Training 1 Lakh+ Students trained till date 65+ Centers
across India
7
www.engineeringonyourfingertips.ooo

More Notes Join us Telegram-: http//:t.me/allexammentor


www.engineeringonyourfingertips.ooo
ME-GATE-2015 PAPER-01| www.gateforum.com

Exp: VT n C
2
V1T1n T1
2V1
16
on solving we get
n 0.25

15. Consider a slider crank mechanism with nonzero masses and inertia. A constant torque is applied on
the crank as shown in the figure. Which of the following plots best resembles variation of crank angle,
versus time

A B
Time

Time

C D

Time Time

Answer: (D) www.


engi
neer
ingonyour
finger
tips.
ooo
Exp.

Fa
f
f
Ft

India’s No.1 institute for GATE Training 1 Lakh+ Students trained till date 65+ Centers
across India
8
www.engineeringonyourfingertips.ooo

More Notes Join us Telegram-: http//:t.me/allexammentor


www.engineeringonyourfingertips.ooo
ME-GATE-2015 PAPER-01| www.gateforum.com

Net Torque m crank ext (CW) Ft R(CCW)


from Newton‟s second law

ext I
ft R I (CW)
d
Ft R
dt
t t
d dt Ft Rdt
0 0 0
t
t Ft Rdt
0

d t
t Ft Rdt
dt 0

t t t t
d tdt Ft Rdt dt
0 0 0 0

t2 t t
Ft Rdt dt
2 0 0

1 cos x 2
16. The value of limx 0 is
2x 4
1 1
A 0 B C D undefined
2 4
Answer: (C)
1 cos x 2 0
Exp: lim 4
x 0 2x 0
Using L Hospital Rule
sin x 2 2x 0
lim
w0ww.
engi
neer
ingonyour
finger
tips.
ooo
3
x 0 8x
cos x 2 2x2x sin x 2 2
lim
x 0 24x 2

India’s No.1 institute for GATE Training 1 Lakh+ Students trained till date 65+ Centers
across India
9
www.engineeringonyourfingertips.ooo

More Notes Join us Telegram-: http//:t.me/allexammentor


www.engineeringonyourfingertips.ooo
ME-GATE-2015 PAPER-01| www.gateforum.com

cos x 2 4x 2 2sin x 2
lim
x 0 24x 2
sin x 2 2x 4x 2 cos x 2 8x 2 cos x 2 2x
lim
x 0 48x
2 3
8sin x x 8 cos x 2 x 4 cos x 2 x
lim
x 0 48x
2 3
8 cos x 2x x sin x 2 3x 2 8 sin x 2 2x cos x 2 4 sin x 2 2x cosx 2
lim
x 0 48
12 1
48 4

17. Two identical trusses support a load of 100 N as shown in the figure. The length of each truss
is 1.0 m, cross-sectional area is 200 mm2; Young‟s modulus E = 200 GPa. The force in the
truss AB (in N) is ______

A C

B
30O 30O

100 N
2Fsin 30
Answer: 100
Exp: 2f sin30 100 F
F
f 100 N

www.
engi
neer
ingonyour
finger
tips.
ooo
100
IV

18. Among the four normal distributions with


probability density functions as shown
below, which one has the lowest variance? III

A I B II
II
C III D IV
I

Answer: (D)
2 1 0 1 2
India’s No.1 institute for GATE Training 1 Lakh+ Students trained till date 65+ Centers
across India
10
www.engineeringonyourfingertips.ooo

More Notes Join us Telegram-: http//:t.me/allexammentor


www.engineeringonyourfingertips.ooo
ME-GATE-2015 PAPER-01| www.gateforum.com

Exp: We have Probability distribution function of Normal Distribution


x
1 2 2
f x e ________(1)

Variance = σ2 is lowest
σ also lowest
If σ decreases f(x) increases (∵ from (1))
Curve will have highest peak

2 z1
19. Given two complex numbers Z1 5 5 3 i and z 2 2i, the argument of in
3 z2
degrees is
(A) 0 (B) 30 (C) 60 (D) 90

Answer: (A)
Exp: z1 5 5 3 i

1 5 3 1
arg z1 tan tan 3 60
5
2
z2 2i
3

1 2 1
arg z 2 tan tan 3 60
2
3
z1
arg arg z1 arg z 2
z2
60 60 0

20. www
The Blasius equation .engi
related ne
to eringo
boundary nyotheory
layer urfing
ise
ar
tips.
ooo
(A) third-order linear partial differential equation
(B) third-order nonlinear partial differential equation
(C) second-order nonlinear ordinary differential equation
(D) third-order nonlinear ordinary differential equation

Answer: (D)

21. A swimmer can swim 10 km in 2 hours when swimming along the flow of a river. While
swimming against the flow, she takes 5 hours for the same distance. Her speed in still water
(in km/h) is ________.
Answer: 35
Exp: Let Swimmer = x
River = y

India’s No.1 institute for GATE Training 1 Lakh+ Students trained till date 65+ Centers
across India
11
www.engineeringonyourfingertips.ooo

More Notes Join us Telegram-: http//:t.me/allexammentor


www.engineeringonyourfingertips.ooo
ME-GATE-2015 PAPER-01| www.gateforum.com

10
2
x y
10
5
x y
On solving we get x 35 km h

22. For flow of viscous fluid over a flat plate, if the fluid temperature is the same as the plate
temperature, the thermal boundary layer is
(A) thinner than the velocity boundary layer
(B) thicker than the velocity boundary layer
(C) of the same thickness as the velocity boundary layer
(D) not formed at all
Answer: (D)

23. Which one of the following is the most conservative fatigue failure criterion?
(A) Soderberg (B) Modified Goodman
(C) ASME Elliptic (D) Gerber
Answer: (A)

24. In a linear arc welding process, the heat input per unit length is inversely proportional to
(A) welding current (B) welding voltage
(C) welding speed (D) duty cycle of the power source
Answer: (C)

25. Consider a stepped shaft subjected to a twisting moment applied at B as shown in the figure.
Assume shear modulus, G = 77 GPa. The angle of twist at C (in degrees) is _____

10Nm All dimensions


in mm
www.
engi
neer
ingonyour
finger
tips.
ooo
20 10
B C
A 500 500

Answer: 0.236
Exp: Angle of twist at (C) = Angle of twist at (B)
TL
GJ
10 0.5 32
0.236050
77 109 .024

India’s No.1 institute for GATE Training 1 Lakh+ Students trained till date 65+ Centers
across India
12
www.engineeringonyourfingertips.ooo

More Notes Join us Telegram-: http//:t.me/allexammentor


www.engineeringonyourfingertips.ooo
ME-GATE-2015 PAPER-01| www.gateforum.com

26. A 10 mm diameter electrical conductor is covered by an insulation of 2 mm thickness. The


conductivity of the insulation is 0.08 W/m-K and the convection coefficient at the insulation
surface is 10 W/m2-K. Addition of further insulation of the same material will
(A) increase heat loss continuously
(B) decrease heat loss continuously
(C) increase heat loss to a maximum and then decrease heat loss
(D) decrease heat loss to a minimum and then increase heat loss

Answer: (C)
Exp: rc = 8mm
∴ the heat lost increases to maximum and then decreases.

27. A machine element is subjected to the following bi-axial state of stress; σx = 80 MPa; σy = 20
MPa; xy 40MPa. If the shear strength of the material is 100 MPa, the factor of safety as
per Tresca‟s maximum shear stress theory is
(A) 1.0 (B) 2.0 (C) 2.5 (D) 3.3
Answer: (B)

2
80 20 80 20
Exp: 1 402
2 2

50 502
100

2 0
1 2
50
7
100
FOS 2
50

www.
engi
neer
ingonyour
finger
tips.
ooo
28. The probability of obtaining at least two “ SIX” in throwing a fair dice 4 time is
A 425 432 B 19 144 C 13 144 D 125 432
Answer: (B)
1
Exp: n = 4; p
6
1 5
q 1
6 6
p x 2 1 p x 2
1 p x 0 p x 1
0 4 1 3
1 5 1 5 19
1 4C0 4C1
6 6 6 6 144

India’s No.1 institute for GATE Training 1 Lakh+ Students trained till date 65+ Centers
across India
13
www.engineeringonyourfingertips.ooo

More Notes Join us Telegram-: http//:t.me/allexammentor


www.engineeringonyourfingertips.ooo
ME-GATE-2015 PAPER-01| www.gateforum.com

29. A horizontal plate has been joined to a vertical post using four rivets arranged as shown in
figure. The magnitude of the load on the worst loaded rivet (in N) is ________

40mm
500mm

40mm

400 N

Answer: 1839.83
400
Exp: Shear load on all rivets 100N
4
Secondary shear load, due to bending moment
Pe r1
2
r1 r r32
1
2
r42
400 .5 .02 2
2
1767.766953N
.02 2 4

P = 1839.837 N.

30. Temperature of nitrogen in a vessel of volume 2 m3 is 288 K. A U-tube manometer connected


to the vessel shows a reading of 70 cm of mercury (level higher in the end open to
atmosphere). The universal gas constant is 8314 J/kmol-K, atmospheric pressure is 1.01325
bar, acceleration due to gravity is 9.81 m/s2 and density of mercury is 13600 kg/m3. The mass
of nitrogen (in kg) in the vessel is _______.
Answer: 4.55
Exp: p gh 0.7 1360 9.81 9339.12pa
Actual pressure atmospheric pressure 110664.12 pa
pv mrT mwwpvw .
RTengi neer ingonyour f
ingert
ips.
ooo
110664.12 2
4.5539kg
288 8314

2
V
31. The solidification time of a casting is proportional to , where V is the volume of the
A
casting and A is the total casting surface area losing heat. Two cubes of same material and
size are cast using sand casting process. The top face of one of the cubes is completely
insulated. The ratio of the solidification time for the cube with top face insulated to that of the
other cube is
25 36 6
A B C 1 D
36 25 5
Answer: (B)

India’s No.1 institute for GATE Training 1 Lakh+ Students trained till date 65+ Centers
across India
14
www.engineeringonyourfingertips.ooo

More Notes Join us Telegram-: http//:t.me/allexammentor


www.engineeringonyourfingertips.ooo
ME-GATE-2015 PAPER-01| www.gateforum.com

2
Exp: t min k V
A
2
93
t1 k
502
93
t2 k
5
t1 36
t2 25

32. Match the following pairs:

Equation Physical Interpretation


(P) V 0 (I) Incompressible continuity equation

(Q) .V 0 (II) Steady flow

DV (III) Irrotational flow


(R) 0
Dt
V (IV) Zero acceleration of fluid particle
(S) 0
t

A P IV,Q I, R II,S III B P IV,Q III, R I,S II


C P III,Q I, R IV,S II D P III,Q I, R II,S IV
Answer: (C)

33. Steam enters a well insulated turbine and expands isentropically throughout. At an
intermediate pressure, 20 percent of the mass is extracted for process heating and the
remaining steam expands isentropically to 9 kPa.
Inlet to turbine: www. ePn =gine
14 er
MPa,inTgo
=n your
560°C, fhi
n=g er
3486tips. oo
kJ/kg, so= 6.6 kJ/(kg.K)
Intermediate stage: h = 27776 kJ/kg
Exit of turbine: P = 9 kPa, hf = 174 kJ/kg, hg = 2574 kJ/kg,
sf = 0.6 kJ/(kg.K), sg = 8.1 kJ/(kg.K)
If the flow rate of steam entering the turbine is 100 kg/s, then the work output (in MW) is
_______.

Answer: 125.56
1
Exp: h1 = 3486 kJ/kg
5 m
h2 = 2776 kJ/kg
20% 2
s1 = s3 =6.6
6.6 .6 x 0.1 .6 4 1 m 3
x 0.8

India’s No.1 institute for GATE Training 1 Lakh+ Students trained till date 65+ Centers
across India
15
www.engineeringonyourfingertips.ooo

More Notes Join us Telegram-: http//:t.me/allexammentor


www.engineeringonyourfingertips.ooo
ME-GATE-2015 PAPER-01| www.gateforum.com

h 174 .8 2574 174


2094 kJ kg
w 3486 2776 .8 2776 2094
1255.6 kJ kg
125.56 MW

34. Water (ρ = 1000 kg/m3) flows through a venturimeter with inlet diameter 80 mm and throat
diameter 40 mm. The inlet and throat gauge pressures are measured to be 400 kPa and 130
kPa respectively. Assuming the venturimeter to be horizontal and neglecting friction, the inlet
velocity (in m/s) is ________.
Answer: 6
p1 v12 p2 v 22 p1 400000
Exp:
g 2g g 2g p 2 130000
v1 802 v 2 402
v2 4v1
Substituting v 2 and solving for v1 we get
v1 6m s

35. For a canteen, the actual demand for disposable cups was 500 units in January and 600 units
in February. The forecast for the month of January was 400 units. The forecast for the month
of March considering smoothing coefficient as 0.75 is _______.
Answer: 560.75
Exp: Forecast for Feb 400 500 400 400 .25 100 475
Forecast for march 475 600 475 560.75

36. Consider a spatial curve in three-dimensional space given in parametric form by


2
x t cos t, y t sin t, z t t, 0 t
2
The length of the curve is ______
www.
engi
neer
ingonyour
finger
tips.
ooo
Answer: 1.86
Exp: The length of the curve
2 2 2 2
dx dy dz
dt
0
dt dt dt
2 2
2 2 2
sin t cos t dt
0
2 2
2 2 4 4
sin t cos t 2
dt 1 2
dt
0 0

4 2 4
1 2
.t0 1 1.8622
2 2

India’s No.1 institute for GATE Training 1 Lakh+ Students trained till date 65+ Centers
across India
16
www.engineeringonyourfingertips.ooo

More Notes Join us Telegram-: http//:t.me/allexammentor


www.engineeringonyourfingertips.ooo
ME-GATE-2015 PAPER-01| www.gateforum.com

37. Considering massless rigid rod and small oscillations, the natural frequency (in rad/s) of
vibration of the system shown in the figure is

k 400 N m
m 1kg

r 2r

400 400 400 400


A B C D
1 2 3 4
Answer: (D)
Exp: Form dx = 2rθ
2
d x
2r.
dt 2
Tak ing moments
2r .m.2 r. 400 r r 0
4m. 400 0
400
4

38. A triangular facet in a CAD model has vertices: P1(0, 0, 0); P2(1, 1, 0) and P3(1, 1, 1). The area of the
facet is
(A) 0.500 (B) 0.707
(C) 1.414 (D) 1.732

Answer: (B) www.


engi
neer
ingonyour
finger
tips.
ooo
y
1
Exp: Area b h
2
1
a 2 a
2
a 1 x
1
0.7071
2
z

India’s No.1 institute for GATE Training 1 Lakh+ Students trained till date 65+ Centers
across India
17
www.engineeringonyourfingertips.ooo

More Notes Join us Telegram-: http//:t.me/allexammentor


www.engineeringonyourfingertips.ooo
ME-GATE-2015 PAPER-01| www.gateforum.com

39. In a slab rolling operation, the maximum thickness reduction (∆hmax) is given by ∆hmax = µ2R,
where R is the radius of the roll and µ is the coefficient of friction between the roll and the
sheet. If µ = 0.1, the maximum angle subtended by the deformation zone at the centre of the
roll (bite angle in degree) is ______.
Answer: 5.71
1 h 1
Exp: tan tan
R
5.7106

d2 y 3
40. Fine the solution of y which passes through the origin and the point ln 2, ,
dx 2 4
1 x 1 x
A y e ex B y e e x
2 2
1 x 1 x
C y e ex D y e e x
2 2

Answer: (C)
d2 y
Exp: y D2 1 y 0
dx 2
D2-1 = 0
D = ±1
y c1e x c 2e x

3
Passes through (0,0) and 142,
4
(0,0)
0 = C1 + C2 __________(1)
www.
engi
neer
ingonyour
finger
tips.
ooo

India’s No.1 institute for GATE Training 1 Lakh+ Students trained till date 65+ Centers
across India
18
www.engineeringonyourfingertips.ooo

More Notes Join us Telegram-: http//:t.me/allexammentor


www.engineeringonyourfingertips.ooo
ME-GATE-2015 PAPER-01| www.gateforum.com

3
142,
4
3 C2
C1e142 C2 e 142
C1 2
4 2
1 3
2C1 C2 ______ 2
2 4
solving 1 and 2
1
C1
2
1
C2
2
1 x 1 x
y e e
2 2
1 x
e e x
2
41. For the truss shown in figure, the magnitude of the force in member PR and the support reaction at R
are respectively 100kN
(A) 122.47 kN and 50 kN 60O P

(B) 70.71 kN and 100 kN

(C) 70.71 kN and 50 kN Q 45O R

4m
(D) 81.65 kN and 100 kN

Answer: (C)
Exp: MQ 0 100sin 60
100cos60 FPR
100 cos60 4 R a 4
Ra 50kN www.
engi
neer
ingonyour
finger
tips.
ooo
FPR cos 45 100cos 60 FPR 70.71 kN
Q
36.6 50

42. A ball of mass 0.1 kg, initially at rest, is dropped from height of 1 m. Ball hits the ground and
bounces off the ground. Upon impact with the ground, the velocity reduces by 20%. The
height (in m) to which the ball will rise is ______
Answer: 0.64

India’s No.1 institute for GATE Training 1 Lakh+ Students trained till date 65+ Centers
across India
19
www.engineeringonyourfingertips.ooo

More Notes Join us Telegram-: http//:t.me/allexammentor


www.engineeringonyourfingertips.ooo
ME-GATE-2015 PAPER-01| www.gateforum.com

Exp: v 2gh 2 9.01 1 4.4294 m s


v ' 0 v 3.5435m s
v2
h 0.64m
2g

43. A DC welding power source has a linear voltage-current (V-I) characteristic with open circuit
voltage of 80 V and a short circuit current of 300 A. For maximum arc power, the current (in
Amperes) should be set as _____.
Answer: 150
80
Exp: v 80 I
300
80 2
p v7 80I I
300
Differentiating and equating to '0'
I 150A

44. A well insulated rigid container of volume 1 m3 contains 1.0 kg of an ideal gas [Cp = 1000
J/(kg.K) and Cv = 800 J/(kg.K)] at a pressure of 105 Pa. A stirrer is rotated at constant rpm in
the container for 1000 rotations and the applied torque is 100 N-m. The final temperature of
the gas (in K) is _________.
Answer: 1128.31
Exp: Work T.
100 100 2 Cp T
T 628.3105K
P1V1 mRT1
102 1 1 0.2 T1
T1 500, T2 628.31 500 1128.31K
www.
engi
neer
ingonyour
finger
tips.
ooo
45. A pinion with radius r1, and inertia I1 is driving a gear with radius r2 and inertia I2. Torque 1

is applied on pinion. The following are free body diagrams of pinion and gear showing
important forces (F1 and F2) of interaction. Which of the following relations hold true?

2
1
F2
1

1 , 2 Angular
r1 Displacements
F1 r2
Inertia l1
Inertia l2

India’s No.1 institute for GATE Training 1 Lakh+ Students trained till date 65+ Centers
across India
20
www.engineeringonyourfingertips.ooo

More Notes Join us Telegram-: http//:t.me/allexammentor


www.engineeringonyourfingertips.ooo
ME-GATE-2015 PAPER-01| www.gateforum.com

r1
A F1 F2 ; 1 I1 1 :F2 I2 1
r22

2
r1 r1
B F1 F2 ; 1 I1 I2 1 : F2 I2 1
r2 r22

1
C F1 F2 ; 1 I1 1 :F2 I2 2
r2
2
r1 1
D F1 F2 ; 1 I1 I2 1 ;F2 I2 2
r2 r2

Answer: (B)

Exp: velocity of point of contact r


1 1 r
2 2 … (1)

Considering pinion and gear as a system net force is zero on system so ,from Newton‟s third

law; F1 F2

Rewrite equation (1)

r1 1 r2 2 … (2)

Differentiating above equation with respect to Time „t‟

r1 1 r2 2 … (3)

consider gear as a system


F2 r2 I2 2

I2w1w
I2 r1 r w. engi
neer
ingonyour
finger
tips.
ooo
F2 1
r2 r2 r22

consider Pinion as a system


Net torque on pinion ext F1 r1 1

From Newton‟s second law

India’s No.1 institute for GATE Training 1 Lakh+ Students trained till date 65+ Centers
across India
21
www.engineeringonyourfingertips.ooo

More Notes Join us Telegram-: http//:t.me/allexammentor


www.engineeringonyourfingertips.ooo
ME-GATE-2015 PAPER-01| www.gateforum.com

ext I1 1 (taking sign convention anticlockwise as positive)


1 I1 1 F1r1
Put F1 value in above equation
r1 1
I1 1 I2 r
1 1
r22
2
r1
1 I1 I2 1
r2

46. A cantilever beam with flexural rigidity of 200 Nm2 is loaded as shown in the figure. The
deflection (in mm) at the tip of the beam is _____.

500 N

50 mm
100mm

Answer: 0.26
Exp: Deflection = Deflection at load + Slope × Distance
Wl3 Wl2
.05
3EI 2EI
W 500
l .05
EI 200
0.2604 mm
47. In the assembly shown below, the part dimensions are:

L1 22.0 0.01 www.


mm, engi
neer
ingonyour
finger
tips.
ooo
0.005
L2 L3 10.0 mm.

Assuming the normal distribution of part dimensions, the dimension L4 in mm for assembly
condition would be:

L4
L2 L3
L1

India’s No.1 institute for GATE Training 1 Lakh+ Students trained till date 65+ Centers
across India
22
www.engineeringonyourfingertips.ooo

More Notes Join us Telegram-: http//:t.me/allexammentor


www.engineeringonyourfingertips.ooo
ME-GATE-2015 PAPER-01| www.gateforum.com

0.008 0.012 0.016 0.020


A 2.0 B 2.0 C 2.0 D 2.0

Answer: (D)

Exp: L1max L 2 max L3max L1max


L 4 max L1max L 2 max L3max
22.01 505 10.005
2mm
L 4 min L1min L 4 min L3 min
2mm
L4 2 0.00 mm

48. A mobile phone has a small motor with an eccentric mass used for vibrator mode. The
location of the eccentric mass on motor with respect to center of gravity (CG) of the mobile
and the rest of the dimensions of the mobile phone are shown. The mobile is kept on a flat
horizontal surface.
CG Motor
10cm Eccentric
P Q Mass

6cm 3cm

Given in addition that the eccentric mass = 2 grams, eccentricty = 2.19 mm, mass of the
mobile = 90 grams, g = 9.81 m/s2. Uniform speed of the motor in RPM for which the mobile
will get just lifted off the ground at the end Q is approximately
(A) 3000 (B) 3500 (C) 4000 (D) 4500
Answer: (B)
Exp: When lifted from ground at Q
Reaction = 0
ww
∴ taking moments w.e
about ng
„p‟ i
ne
and eringo
equating ny
to 0our
finger
tips.
ooo
2
.09×.06 = mr ×.09
9.01×.09×.06 = .002×2.19×10-3× 2
×.09
= 366.50 rad/s
= 3500 rpm

49. A precision instrument package (m = 1 kg) needs to be mounted on a surface vibrating at 60


Hz. It is desired that only 5% of the base surface vibration amplitude be transmitted to the
instrument. Assuming that the isolation is designed with its natural frequency significantly
lesser than 60 Hz, so that the effect of damping may be ignored. The stiffness (in N/m) of the
required mounting pad is ________.
Answer: 6767.6

India’s No.1 institute for GATE Training 1 Lakh+ Students trained till date 65+ Centers
across India
23
www.engineeringonyourfingertips.ooo

More Notes Join us Telegram-: http//:t.me/allexammentor


www.engineeringonyourfingertips.ooo
ME-GATE-2015 PAPER-01| www.gateforum.com

Exp: 2 N 2 60 376.99 rad second


2 2
1
.05 2
20 1 21
n n
1
n
2
2 21 n

n 82.266 rad s
k
n 82.266
m
l 6767.6005 N m
50. Following data refers to the activities of a project, where, node 1 refers to the start and node 5
refers to the end of the project

Activity Duration (days)


1-2 2
2-3 1
4-3 3
1-4 3
2-5 3
3-5 2
4-5 4

The critical path (CP) in the network is


(A) 1-2-3-5 (B) 1-4-3-5 (C) 1-2-3-4-5 (D) 1-4-5

Answer: (B)

Exp:
2 5
w
0w2w.
en2gi
neer
ingonyour
finger
tips.
ooo
5
2 3 6 8
1 3
0 3 3 6

4 3 7

Critical path-1-4-3-5
Time taken = 8 days

India’s No.1 institute for GATE Training 1 Lakh+ Students trained till date 65+ Centers
across India
24
www.engineeringonyourfingertips.ooo

More Notes Join us Telegram-: http//:t.me/allexammentor


www.engineeringonyourfingertips.ooo
ME-GATE-2015 PAPER-01| www.gateforum.com

51. For flow through a pipe of radius R, the velocity and temperature distribution are as follows:
3
r
u r, x C1 , and T r, x C2 1 , where C1 and C 2 are constants
R

2 R
The bulk mean temperature is given by Tm u r, x T r, x rdr,
umR 2 0

with Um being the mean velocity of flow. The value of Tm is


0.5C2 0.6C2
A B 0.5C2 C 0.6C2 D
Um Um

Answer: (C)
R 3
2 r
Exp: Tm C1C2 1 r dr
u mR 2 0 R
R
2C1C2 r4
r dr
u mR 2 0
R3
2C1C2 3 2 0.6C1C2
R
u m R 2 10 um

Since u(r,x) is constant, um = C1

∴ tm = 0.6C2

52. Consider an ant crawling along the curve (x – 2)2 + y2 = 4, where x and y are in meters. The
ant starts at the point (4, 0) and moves counter-clockwise with a speed of 1.57 meters per
second. The time taken by the ant to reach the point (2, 2) is (in seconds) _______.
Answer: 2

Exp: www.
engi
neer
ingonyour
finger
tips.
ooo
1
circumference
4
1
4
4
time 2sec 0,0
1.5 2,0 4,0

India’s No.1 institute for GATE Training 1 Lakh+ Students trained till date 65+ Centers
across India
25
www.engineeringonyourfingertips.ooo

More Notes Join us Telegram-: http//:t.me/allexammentor


www.engineeringonyourfingertips.ooo
ME-GATE-2015 PAPER-01| www.gateforum.com

53. Air (ρ = 1.2 kg/m3 and kinematic viscosity, v = 2 × 10–5 m2/s) with a velocity of 2 m/s flows
over the top surface of a flat plate of length 2.5 m. If the average value of friction coefficient
1.328
is Cf , the total drag force (in N) per unit width of the plate is ________.
Re x
Answer: 0.0159
1.320
Exp: Cf
Re x
vd vd.
R ex

v 2m / s
l 2.5m
2 10 5 m 2 / s
1
F Cf A 2
2
A 2.5 1
On substituting we get
F = 0.0159N

54. The velocity field of an incompressible flow is given by


V = (a1x + a2y + a3z)i + (b1x + b2y + b3z)j + (c1x + c2y + c3z)k, where a1= 2 and c3 = – 4. The
value of b2 is ________.
Answer: 2
u v w
Exp: 0
x y z
a1 b 2 c3 0
2 4 b2 0
b2 2
55. An orthogonal turning operation is carried out under the following conditions: rake angle =
wwwspeed
5°, spindle rotational .
engi =n eer
400 ingo
rpm; nyofeed
axial ur f
in
=g er
0.4 tips.
m/min oand
ooradial depth of cut = 5
mm. The chip thickness tc, is found to be 3 mm. The shear angle (in degrees) in this turning
process is ________.

Answer: 32.239
3
Exp: Chip thickness ratio(r) = 0.6
5

To find shear angle


r cos 0.6cos5
tan 0.6306
1 r sin 1 0.6sin 5
tan 0.6306 32.24o
1

India’s No.1 institute for GATE Training 1 Lakh+ Students trained till date 65+ Centers
across India
26
www.engineeringonyourfingertips.ooo

More Notes Join us Telegram-: http//:t.me/allexammentor


www.engineeringonyourfingertips.ooo
EE-GATE-2015 PAPER-02| www.gateforum.com

General Aptitude

Q. No. 1 – 5 Carry One Mark Each

1. Find the missing sequence in the letter series below:


A, CD, GHI,?, UVWXY
(A) LMN (B) MNO (C) MNOP (D) NOPQ
Answer: (C)
Exp:
A, C D, G H I, M N O P, UVWXY

B E,F JKL QRST


1 2 3 4

2. Choose the correct verb to fill in the blank below:


Let us ______________.
(A) Introvert (B) alternate (C) atheist (D) altruist
Answer: (B)

3. Choose the most appropriate word from the options given below to complete the following
sentence?
If the athlete had wanted to come first in the race, he ___________several hours every day.
(A) Should practice (B) Should have practiced
(C) Practised (D) Should be practicing
Answer: (B)
Exp: For condition regarding something which already happened, should have practiced is the
correct choice.

4. Choose the most suitable one word substitute for the following expression
www. engi neer ingonyour finger tips. ooo
Connotation of a road or way
(A) Pertinacious (B) Viaticum
(C) Clandestine (D) Ravenous
Answer: (B)
Exp: Viaticum and connotation both are associated with the feeling of spirtuality.

5. If x>y>I, which of the following must be true?


 i  ln x  ln y  ii  ex  ey  iii  yx  x y  iv  cos x  cos y
 A   i  and ii   B i  and iii 
 C iii  and  iv   D  ii  and iv
Answer: (A)

India’s No.1 institute for GATE Training 1Lakh+Studentstrainedtilldate65+Centerscross


India
1


www.engineeringonyourfingertips.ooo

More Notes Join us Telegram-: http//:t.me/allexammentor


www.engineeringonyourfingertips.ooo
EE-GATE-2015 PAPER-02| www.gateforum.com

Exp: For whole numbers, greater the value greater will be its log.
Same logic for power of e.
Q. No. 6 – 10 Carry Two Marks Each
6. From a circular sheet of paper of radius 30cm, a sector of 10% area is removed. If the
remaining part is used to make a conical surface, then the ratio of the radius and height of the
cone is________.
Answer: 2.06
Exp: 90% of area of sheet = Cross sectional area of cone
 0.9   30  30   r1  30
 27 cm  r1

 Height of the cone  302  27 2


 13.08 cm
Then r/h=27/13.08=2.06

7. In the following question, the first and the last sentence of the passage are in order and
numbered 1 and 6. The rest of the passage is split into 4 parts and numbered as 2,3,4, and 5.
These 4 parts are not arranged in proper order. Read the sentences and arrange them in a
logical sequence to make a passage and choose the correct sequence from the given options.
1. One Diwali, the family rises early in the morning.
2. The whole family, including the young and the old enjoy doing this,
3. Children let off fireworks later in the night with their friends.
4. At sunset, the lamps are lit and the family performs various rituals
5. Father, mother, and children visit relatives and exchange gifts and sweets.
6. Houses look so pretty with lighted lamps all around.
(A) 2, 5, 3, 4 (B) 5, 2, 4, 3 (C) 3, 5, 4,2 (D) 4, 5, 2,
3
Answer: (B)

8. Ms X will be inw ww. eng


Bagdogra ine
from er ingony
01/05/2014 toour finger
20/05/2014 tips
and .ooo
from 22/05/2014 to
31/05/2014. On the morning of 21/05/204, she will reach Kochi via Mumbai
Which one of the statements below is logically valid and can be inferred from the above
sentences?
(A) Ms. X will be in Kochi for one day, only in May
(B) Ms. X will be in Kochi for only one day in May
(C) Ms. X will be only in Kochi for one day in May
(D) Only Ms. X will be in Kochi for one day in May.
Answer: (B)

9. log tan1O  log tan 2O  ......  log tan89O is .........


 A 1  B 1 2 C 0  D 1
Answer: (C)

India’s No.1 institute for GATE Training 1Lakh+Studentstrainedtilldate65+Centerscross


India
2


www.engineeringonyourfingertips.ooo 

More Notes Join us Telegram-: http//:t.me/allexammentor


www.engineeringonyourfingertips.ooo
EE-GATE-2015 PAPER-02| www.gateforum.com

Exp: 
log tan1o  log tan 89o  log tan1o  tan 89 o
 log  tan1  cot1 
o o

 log1
0
Using the same logic total sum is ‘ 0’ .

10. Ram and Shyam shared a secret and promised to each other that it would remain between
them. Ram expressed himself in one of the following ways as given in the choices below.
Identify the correct way as per standard English.
(A) It would remain between you and me.
(B) It would remain between I and you
(C) It would remain between you and I
(D) It would remain with me.
Answer: (A)

Mechanical Engineering
Q. No. 1 – 25 Carry One Mark Each

1. The uniaxial yield stress of a material is 300 MPa. According to von Mises criterion, the shear
yield stress (in MPa) of the material is ______
Answer: 173.28
Exp. If there is uniaxial loading yield stress is  y
As per Von Mises failure theory
y2  12  12  2 2
Under pure shear stress loading
1 = -  2 = τ
then
 y 2  2  w
2
w
 w
2
.
engi
neer
ingonyour
finger
tips.
ooo
 32
 
   y hence   y  0.577 y  173.28
3 3
where  isshear yield stress

2. The primary mechanism of material removal in electrochemical machining (ECM) is


(A) Chemical corrosion
(B) etching
(C) ionic dissolution
(D) spark erosion
Answer: (C)

India’s No.1 institute for GATE Training 1Lakh+Studentstrainedtilldate65+Centerscross


India
3


www.engineeringonyourfingertips.ooo 

More Notes Join us Telegram-: http//:t.me/allexammentor


www.engineeringonyourfingertips.ooo
EE-GATE-2015 PAPER-02| www.gateforum.com

3. Curl of vector V(x,y,z) = 2x 2i  3z2 j  y3 k at x  y  z  1 is

A  3i  B 3i  C 3i  4j
 D 3i  6k
Answer: (A)

i j k
  
Exp: Curl of V  x, y, z  
x y z
2x 2 3z 2 y3
 i 3y 2  6z   j 0  0  k 0  0


 3y 2  6z i  x  y  z 1

 3i

4. A small ball of mass 1kg moving with a velocity of 12m/s undergoes a direct central impact
with a stationary ball of mass 2 kg. The impact is perfectly elastic. The speed (in m/s) of 2 kg
mass ball after the impact will be ________
Answer: 8
Exp: For elastic collision
m1u1+m2u2 = m1v1+m2v2 (1) _________ moment conservation
m1 = 1 kg u1 = 12 m/s
m2 = 2 kg u2 = 0 m/s
1 1 1 1
m1u12  m 2u 22  m1v12  m 2 v 22  2  ____ energyconservation
2 2 2 2
From (1) equation
12 = v1+2v2 ________(3)
www.
engi
neer
ingonyour
finger
tips.
ooo
From (2) equation
1 1 1 1
 1 144   2  0   1  v12   2  v 22
2 2 2 2
 144  v1  2v2 ____  4 
2 2

From (3) and (4)


144  144  4v22  48v 2  2v22
 6v 22  48v 2  0
6v 2  v 2  8   0
 v 2  8m s

India’s No.1 institute for GATE Training 1Lakh+Studentstrainedtilldate65+Centerscross


India
4


www.engineeringonyourfingertips.ooo

More Notes Join us Telegram-: http//:t.me/allexammentor


www.engineeringonyourfingertips.ooo
EE-GATE-2015 PAPER-02| www.gateforum.com

5. A rod is subjected to a unit-axial load within linear elastic limit. When the change in the stress
is 200 MPa, the change in the strain is 0.001. If the Poisson’ s ratio of the rod is 0.3, the
modulus of rigidity (in GPa) is _____________
Answer: 77
Exp: Modulus of rigidity (G)
E
G
2 1   
It has given change in stress = 200 MPa
Change in strain = 0.001
Here
200 = E×0.003
200 MPa
E  200  103 MPa
0.002
 200GPa
200 200 100 1000
G     77 GPa
2 1  0.3 2  1.3 1.3 13

6. Within a boundary layer for a steady incompressible flow, the Bernoulli equation
(A) holds because the flow is steady
(B) holds because the flow is incompressible
(C) holds because the flow is transitional
(D) does not hold because the flow is frictional
Answer: (D)
Exp: Bernoulli equation does not hold because it is for non viscous flow

7. The atomic packing factor for a material with body centered cubic structure is _______
Answer: 0.64

8. If a foam insulation is added to a 4cm outer diameter pipe as shown in the figure, the critical
ww(in
radius of insulation w. eng
cm) isi
n eer ingonyour
_____________ finger tips. ooo

 4cm
Foam h 0  2 W m2 .K
kfoam  0.1W m.K h i  15W m2  K

Pipe
k pipe  15W m.K

Answer: 5
India’s No.1 institute for GATE Training 1Lakh+Studentstrainedtilldate65+Centerscross
India
5


www.engineeringonyourfingertips.ooo

More Notes Join us Telegram-: http//:t.me/allexammentor


www.engineeringonyourfingertips.ooo
EE-GATE-2015 PAPER-02| www.gateforum.com

k 0.1
Exp: Critical radius rc    .05m
h0 2
 5cm

9. During the development of a product an entirely new process plan is made based on design
logic, examination of geometry and tolerance information. This type of process planning is
known as
(A) Retrieval (B) Generative
(C) Variant (D) Group technology based
Answer: (B)

10. Annual demand of a product is 50000 units and the ordering cost is Rs. 7000 per order
considering the basic economic order quantity model, the economic order quantity is 10000
units. When the annual inventory cost is minimized, the annual inventory holding cost (in Rs.)
is _______
Answer: 35000
Exp: At optimum total inventory cost (TIC), annual inventory hold is cost is equal to annual
inventory ordering cost
= Number of orders × ordering cost per order
50000
  7000
10000
 5  7000
 35000
11. Sales data of a product is given in the following table:

Month January February March April May


Number of unit 10 11 16 19 25
sold
www.
engi
neer
ingonyour
finger
tips.
ooo
Regarding forecast for the month of June, which one of the following statements is TRUE?
(A) Moving average will forecast a higher value compared to regression
(B) Higher the value of order N, the greater will be the forecast value by moving average.
(C) Exponential smoothing will forecast a higher value compared to regression.
(D) Regression will forecast a higher value compared to moving average
Answer: (D)

 a 
12. The Vander Waals equation of state is  p  2   v  b   RT, where p is pressure, v is
 v 
specific volume, T is temperature and R is characteristic gas constant. The SI unit of a is
 A J kg.K  B m kg  C m5 kg  s2  D Pa kg

India’s No.1 institute for GATE Training 1Lakh+Studentstrainedtilldate65+Centerscross


India
6


www.engineeringonyourfingertips.ooo

More Notes Join us Telegram-: http//:t.me/allexammentor


www.engineeringonyourfingertips.ooo
EE-GATE-2015 PAPER-02| www.gateforum.com

Answer: (C)
a
Exp: p both term should gave same unit since they are getting added
2
2
N  kg 
  a 3 
m 
2
m
m6 m m5 kg m5
 a  unit   .kg.  
kg 2 s2 m 2 kg 2s 2 kgs 2

13. Which of the following statements regarding a Rankine cycle with reheating are TRUE?
(i) increase in average temperature of heat addition
(ii) reduction in thermal efficiency
(iii) drier steam at the turbine exit
(A) only (i) and (ii) are correct (B) only (ii) and (iii) are correct
(C) only (i) and (iii) are correct (D) (i), (ii) and (iii) are correct
Answer: (C)
Exp: With reheat average temperature of heat addition increases. Hence, efficiency of cycle also
increases. Further, the quality of steam is higher at turbine exit.

14. In a spring-mass system, the mass is m and the spring constant is k. The critical damping
coefficient of the system is 0.1kg/s. In another spring-mass system, the mass is 2m and the
spring constant is 8K. The critical damping coefficient (in kg/s) of this system is __________
Answer: 0.4
CC1
Exp: n1 
2m1
CC1  2 K1m1  2 Km
CC2  2 K 2 m 2  2 8K  2m  4CC1  4  0.1  0.4
www.
engi
neer
ingonyour
finger
tips.
ooo
15. The COP of a cannot heat pump operating between 6OC and 37OC is ___________
Answer: 10
T1 310
Exp:  COP c.p.    10
T1  T2 31

India’s No.1 institute for GATE Training 1Lakh+Studentstrainedtilldate65+Centerscross


India
7


www.engineeringonyourfingertips.ooo 

More Notes Join us Telegram-: http//:t.me/allexammentor


www.engineeringonyourfingertips.ooo
EE-GATE-2015 PAPER-02| www.gateforum.com

16. The number of degrees of freedom of the planetary gear train shown in the figure is

20teeth gear

arm

50 teeth
gear

(A) 0 (B) 1 (C) 2 (D) 3

Answer: (C)
Exp: A planetary gear train has 2 DOF and hence requires too input to get desired output.

17. A rope-brake dynamometer attached to the crank shaft of an I.C. engine measures a brake
power of 10kW when the speed of rotation of the shaft is 400 rad/s. The shaft torque (in N-m)
sensed by the dynamometer is _______
Answer: 25
Exp: P = T
10000
T  25N  m
400

18. At least one eigen value of a singular matrix is


(A) Positive (B) Zero (C) Negative (D) Imaginary
Answer: (B)

19. If the fluid velocity for a potential flow is given by V  x, y   u  x, y  i  v  x, y  j with usual
notations, then the slope of the potential line at (x,y) is
v www.
engi
ne
uer
ingonyour
fing
v 2er
tips.
ooo u
A  B   C  D
u v u2 v
Answer: (B)
 
Exp: d  dx  dy
x y
d  udx  vdy
dy u

dx v

India’s No.1 institute for GATE Training 1Lakh+Studentstrainedtilldate65+Centerscross


India
8


www.engineeringonyourfingertips.ooo

More Notes Join us Telegram-: http//:t.me/allexammentor


www.engineeringonyourfingertips.ooo
EE-GATE-2015 PAPER-02| www.gateforum.com

20. Which one of the following statements is TRUE?


(A) The ‘ GO’ gage controls the upper limit of a hole
(B) The ‘ NO’ gage controls the lower limit of a shaft
(C) The ‘ GO’ gage controls the lower limit of a hole
(D) The ‘ NO GO’ gage controls the lower limit of a hole
Answer: (C)
Exp: Go size = maximum material limit of component = Lower limit of hole

21. There vendors were asked to supply a very high precision component. The respective
probabilities of their meeting the strict design specifications are 0.8, 0.7 and 0.5. Each vendor
supplies one component. The probability that out of total three components supplied by the
vendors, at least one will meet the design specification is _________
Answer: 0.97
Exp: Probability (at least one will meet specification) = 1-probability (none will meet specification)
= 1-(1-0.8)×(1-0.7)×(1-0.5)
= 1-0.2×0.3×0.5
= 1-0.03
= 0.97

22. The Laplace transform of ei5t where i  1 , is


s  5i s  5i s  5i s  5i
 A  B C  D
s2  25 s 2  25 s 2  25 s2  25
Answer: (B)
Exp:  
L ei5t  L  cos5t  isin 5t 
s  5i
 L  cos5t   iL  sin 5t  
s 2  25

www.
engi
neer
ingonyour
finger
tips.
ooo
23. A gas is stored in a cylindrical tank of inner radius 7 m and wall thickness 50 mm. The gage
pressure of the gas is 2MPa. The maximum shear stress (in MPa) in the wall is
(A) 35 (B) 70 (C) 140 (D) 280
Answer: (C)
pd 2  14
Exp: 1    280 MPa
2t 2  0.05
pd 2  14
2    140 MPa
4t 4  0.05
3  0
   2 2  3 3  1 
Maximum shear stress  Maximum  1 , ,   140MPa
 2 2 2 

India’s No.1 institute for GATE Training 1Lakh+Studentstrainedtilldate65+Centerscross


India
9


www.engineeringonyourfingertips.ooo

More Notes Join us Telegram-: http//:t.me/allexammentor


www.engineeringonyourfingertips.ooo
EE-GATE-2015 PAPER-02| www.gateforum.com

24. In the laminar flow of air  Pr  0.7  over a heated plate if  and T denote, respectively, the
hydrodynamic and thermal boundary layer thicknesses, then
A   T  B   T  C   T  D   0 but T  0
Answer: (C)
Exp: When Pr < 1 δt > δ
Pr > 1 δt < δ
Pr = 1 δt = δ

25. At x = 0, the function f(x) = x has


(A) A minimum (B) A maximum
(C) A point of inflexion (D) neither a maximum nor minimum
Answer: (A)
Exp: For negative values of x, f(x) will be positive
For positive values of x, f(x) will be positive
∴ minimum value of f(x) will occur at x = 0

26. The total emissive power of a surface is 500 W/m2 at a temperature T1 and 1200 W/m2 at a
temperature T2. Where the temperatures are in Kelvin. Assuming the emissivity of the surface
T
to be constant, the ratio of the temperatures 1 is
T2
(A) 0.308 (B) 0.416 (C) 0.803 (D) 0.874
Answer: (C)
Exp: If temperature is T1 then
Emissive power  T14

500  T14 ___ 1


or
500  k T14 ___ 1w
 ww.
engi
neer
ingonyour
finger
tips.
ooo
similarly
1200  kT24 ___  2 
from 1  2 
4
 T1  500
  
 T2  1200
14
T1  500 
   0.803
T2  1200 

27. A hallow shaft of 1m length is designed to transmit a power of 30 kW at 700 rpm. The
maximum permissible angle of twist in the shaft is 1O. The inner diameter of the shaft is 0.7

India’s No.1 institute for GATE Training 1Lakh+Studentstrainedtilldate65+Centerscross


India
10


www.engineeringonyourfingertips.ooo

More Notes Join us Telegram-: http//:t.me/allexammentor


www.engineeringonyourfingertips.ooo
EE-GATE-2015 PAPER-02| www.gateforum.com

times the outer diameter. The modulus of rigidity is 80 GPa. The outside diameter (in mm) of
the shaft is _______
Answer: 44.52
Exp: P = T
2  700
30  1000  T 
60
T  409.256 N  m
T G

IP l
 l  1m 
409.256 80  109   
  
 180    radians 
32
 
1  0.7 4 d 04 1
 180 

Solving, we get
d0 = 44.5213 mm

28. In a Rankine cycle, the enthalpies at turbine entry and outlet are 3159kJ/kg. and 2187 kJ/kg,
respectively. If the specific pump work is 2kJ/kg the specific steam consumption (in kg/kW-
h) of the cycle based on net output is __________
Answer: 3.71
3600
Exp: Specific steam consumption 
WT  WP

WT = h 2  h1 = 3159-2187 kJ/kg
WT = 972 kJ/kg
WP = 2 kJ/kg
3600
Thus specific steam consumption  kg kW  h
972  2
www.
engi
neer
ingo nyou
3.71kg rf
inger
kW.h t i
ps.
ooo

29. A single point cutting tool with 0O rake angle is used in an orthogonal machining process. At
a cutting speed of 180 m/min, the thrust fore is 490N. If the coefficient of friction between the
tool and the chip is 0.7, then the power consumption (in kW) for the machining operation is
__________
Answer: 2.1
F F sin   FT cos 
Exp:   C
N FC cos   FT sin 
Given :   0
F
  T
FC

India’s No.1 institute for GATE Training 1Lakh+Studentstrainedtilldate65+Centerscross


India
11


www.engineeringonyourfingertips.ooo 

More Notes Join us Telegram-: http//:t.me/allexammentor


www.engineeringonyourfingertips.ooo
EE-GATE-2015 PAPER-02| www.gateforum.com

490
0.7 
FC
FC  700 N
180 1
Power consumption, P  Fc  Vc  700    kW 
60 1000
P  2.1kW

30. The chance of a student passing an exam is 20%. The chance of a student passing the exam
and getting above 90% marks in it is 5% Given that a student passes the examination, the
probability that the student gets above 90% marks is
1 1 2 5
(A) (B) (C) (D)
18 4 9 18
Answer: (B)
Exp: Let Astudent passes the exam
Bstudent gets above 90% marks
Given P(A) = 20%; P(A∩B) = 5%
P  A  B 5% 1
required probabilityis P  B A    
P  A 20% 4

31. A manufacturer has the following data regarding a product:


Fixed cost per month = Rs. 50000
Variable cost per unit = Rs.200
Selling price per unit = Rs.300
Production capacity = 1500 units per month
If the production is carried out at 80% of the rated capacity, that the monthly profit (in Rs.) is
________
Answer: 70000
Exp: Profit per month = 0.8 ×1500 × (300-200) – 50000
= 120000-50000

www=.
e ngineer
70000 ingonyour
finger
tips.
ooo

32. The head loss for a laminar incompressible flow through a horizontal circular pipe is h1. Pipe
length and fluid remaining the same, if the average flow velocity doubles and the pipe
diameter reduces to half its previous value, the head loss is h2. The ratio h2/h1 is
(A) 1 (B) 4 (C) 8 (D) 16

Answer: (C)
u avg
Exp: head loss h 
D2

India’s No.1 institute for GATE Training 1Lakh+Studentstrainedtilldate65+Centerscross


India
12


www.engineeringonyourfingertips.ooo 

More Notes Join us Telegram-: http//:t.me/allexammentor


www.engineeringonyourfingertips.ooo
EE-GATE-2015 PAPER-02| www.gateforum.com

2
h D  u
 2   1  avg,2
h1  D 2  u avg,1
 22  2
h2
8
h1

33. A cube and a sphere made of cat iron (each of volume 1000 cm3) were cast under identical
conditions. The time taken for solidifying the cube was 4s. The solidification time (in s) for
the sphere is _________

Answer: 6.15
2
v
Exp: solidification Time  k  
A
2
 a3  a
2
for cube, t  4  k.  2   k.  
 6a  6
a2
k. 4
36
ka 2  36  4 _____ 1
vol.of cube  vol.of sphere
4
 r3  a 3
3
13
 3 
r   a
 4 
2
4 3
 3 r   r 2
solidification timefor sphere, t '  k.  2 
 
 4r   3 
www. engi neer ingonyour f
inger
tips.
ooo
2
  3 1 3 
 k    .a / 3 
  4  
 
23
 3 
 k.a 2    9
 4 
 6.1573sec

34. One kg of air (R = 287 J/kg.K) undergoes an irreversible process between equilibrium state 1
 20O C, 0.9 m3  and equilibrium state 2 (20 OC, 0.6m3). The change in entropy S2 – S1 (in
J/kg.K) is _________
Answer: -116.36

India’s No.1 institute for GATE Training 1Lakh+Studentstrainedtilldate65+Centerscross


India
13


www.engineeringonyourfingertips.ooo

More Notes Join us Telegram-: http//:t.me/allexammentor


www.engineeringonyourfingertips.ooo
EE-GATE-2015 PAPER-02| www.gateforum.com

V2
Exp: s 2  s1  mR ln
V1
0.6
 287 ln  116.368J / kgK
0.9
35. In a plane stress condition, the components of stress at point are x  20 MPa, y  80MPa
and xy  40MPa. The maximum shear stress ( in MPa) at the point is
(A) 20 (B) 25 (C) 50 (D)
100
Answer: (C)

   y 
2
  2
Exp: max  1   x   xy
2

2  2 

 80  20 
2

    40
2

 2 
 50 MPa

36. Work is done on an adiabatic system due to which its velocity changes from 10 m/s to 20 m/s,
elevation increases by 20 m and temperature increases by 1 K. The mass of the system is 10
kg. CV  100J  kg.K  and gravitational acceleration is 10 m/s2. If there is no change in any
other component of the energy of the system, the magnitude of total work done (in kJ) on the
system is________
Answer: 4.5
Exp: Using SFEE
 v 2 v  2 
m  1  2    z1  z 2  g   h1  h 2    W
 2 2  
www.
engi
neer
ingonyour
finger
tips.
ooo
102 202 
10    (20) 10  100  1   W
 2 2 
W  4.5kJ
 Work done on the system is 4.5 kJ.

37. A hollow shaft do  2di where d o and di are the outer and inner diameters respectively)
needs to transmit 20kW power at 3000 RPM. If the maximum permissible shear stress is 30
MPa, dO is
 A  11.29mm  B 22.58mm  C 33.87mm  D 45.16mm
Answer: (B)
Exp: P  T

India’s No.1 institute for GATE Training 1Lakh+Studentstrainedtilldate65+Centerscross


India
14


www.engineeringonyourfingertips.ooo

More Notes Join us Telegram-: http//:t.me/allexammentor


www.engineeringonyourfingertips.ooo
EE-GATE-2015 PAPER-02| www.gateforum.com

2 3000
20 103  T 
60
 T  63.662 N  m
T 
 ,
Ip r
63.662 30  106
 (r0  d),
 4 d
(15d1 ) 1
32
 d1  11.295 mm
d0  2d1  22.59 mm

38. A cantilever beam OP is connected to another beam PQ with a pin joint as shown in the
figure. A load of 10kN is applied at the mid-point of PQ. The magnitude of bending moment
(in kN-m) at fixed end O is \
10kN
O P Q

2m 1m

(A) 2.5 (B) 5 (C) 10 (D) 25

Answer: (C)
Exp:
10kN

www.
engi
neer
10 kN
i
ngonyour
finger
tips.
ooo

5kN 5kN

5kN

2m

M = 5 × 2 = 10 kN

India’s No.1 institute for GATE Training 1Lakh+Studentstrainedtilldate65+Centerscross


India
15


www.engineeringonyourfingertips.ooo 

More Notes Join us Telegram-: http//:t.me/allexammentor


www.engineeringonyourfingertips.ooo
EE-GATE-2015 PAPER-02| www.gateforum.com

39. The flow stress (in MPa) of a material is given by


  5000.1
Where  is true strain. The Young’ s modulus of elasticity of the material is 200 GPa. A block
of thickness 100 mm made of this material is compressed to 95 mm thickness and then the
load is removed. The final dimension of the block (in mm) is _________

Answer: 95.18
100
Exp: Truestrain  ln  0.5129
95
  500   0.5129   371.5147523
.1

Upto elastic limits using hooks law


l
E
l
371.5147523  106  100
 200  109 
l
 l  0.18575mm  considering this for elastic recovery
∴ This will be added to 95 mm
 Final dimension  95.18575 mm

40. The initial velocity of an object is 40m/s. The acceleration a of the object is given by the
following expression: a= 0.1V
Where V is the instantaneous velocity of the object. The velocity of the object after 3 seconds
will be _______
Answer: 29.632
Exp: a=-0.1V
dv
 0.1V
dt
www.
engi
neer
ingonyour
finger
tips.
ooo
lnv  0.1t  ln k
V  ke 0.1t
at t  0 ; V  40
 k  40
V  40e 0.1t
at t  3 sec onds
V  40 e0.13  29.6327 m / s

41. A balanced counter flow heat exchanger has a surface area of 20m2 and overall heat transfer
coefficient of 20 W m2  K Air  CP  1000J kg  K  entering at 0.4 kg/s and 280 K is to be
preheated by the air leaving the system at 0.4 kg/s and 300 K. The outlet temperature (in K)
of the preheated air is
India’s No.1 institute for GATE Training 1Lakh+Studentstrainedtilldate65+Centerscross
India
16


www.engineeringonyourfingertips.ooo 

More Notes Join us Telegram-: http//:t.me/allexammentor


www.engineeringonyourfingertips.ooo
EE-GATE-2015 PAPER-02| www.gateforum.com

(A) 290 (B) 300 (C) 320 (D) 350


Answer: (A)
Exp: Counter flow heat exchanged
Surface Area A  20m massflowrate  0.4kg s
2

20W TemperatureTci  280K


u 2
m K Tco  ?
J
Cp of air  1000
kgK

Since m is same for both flow = 0.4 kg/s
Assume Cp is same = 1000 J/kg.K
Hence
Thi
Tco Tho
Tci
T1  Ti  T 0  T2  Th 0  Tci

T1  300  Tco  Tho  280

Tm  T1  T2



uATm  mC p  Tco  Tci 

20  20   300  Tco   0.4  1000  Tco  280 

2Tco  300  280

580
Tco   290K
2

42. The values of function f (x) at 5 discrete point are given below:
www.
engi
neer
ingonyour
finger
tips.
ooo
x 0 0.1 0.2 0.3 0.4
f(x) 0 10 40 90 160

0.4

Using Trapezoidal role with step size of 0.1, the value of  f  x  dx


0
is _________

Answer: 22

x 0 0.1 0.2 0.3 0.4


Exp: y  f (x) 0 10 40 90 160
y0 y1 y2 y3 y4

India’s No.1 institute for GATE Training 1Lakh+Studentstrainedtilldate65+Centerscross


India
17


www.engineeringonyourfingertips.ooo

More Notes Join us Telegram-: http//:t.me/allexammentor


www.engineeringonyourfingertips.ooo
EE-GATE-2015 PAPER-02| www.gateforum.com

h
 (y0  y4 )  2(y1  y 2  y3 )
0.4 0.4
 0
f (x)dx   ydx 
0 2
0.1
  (0  160)  2(10  40  90)
2
 22

43. In a two-stage wire drawing operation, the fractional reduction (ratio of change in cross-
sectional area to initial cross-sectional area) in the first stage is 0.4. The fractional reduction
in the second stage is 0.3. The overall fractional reduction is
(A) 0.24 (B) 0.58 (C) 0.60 (D) 1.00
Answer: (A)
Exp: Since only option (A) is less then 0.3. (A) is the correct answer, as overall reduction will be
less then the reduction in the first stage.

44. A single-degree. Freedom spring-mass system is subjected to a sinusoidal force of 10 N


amplitude and frequency  along the axis of the spring. The stiffness of the spring is 150
N/m, damping factor is 0.2 and the undamped natural frequency is 10. At steady state, the
amplitude of vibration (in m) is approximately
 A  0.05  B 0.07  C 0.70  D  0.90
Answer: (B)

f0 / s
Exp: Amplitude of vibration A=
2 2
    2 
1     
 n   n 
10 / 150

2
  1 2   1
2

www.
eng neer
1i i
n n
goyu
2o rf nge
i 10 r
0.2  ti
ps.
ooo
  10   
 0.06605
� 0.07

45. For a fully developed laminar flow of water (dynamic viscosity 0.001 Pa-s) through a pipe of
radius 5cm. the axial pressure gradient is – 10Pa/m. The magnitude of axial velocity (in m/s)
at a radial location of 0.2 cm is ________
Answer: 6.24
1 P 2 2
Exp: G
4 x

R r 
1

4  0.001

 (10) (0.05) 2  (0.002)2 
India’s No.1 institute for GATE Training 1Lakh+Studentstrainedtilldate65+Centerscross
India
18


www.engineeringonyourfingertips.ooo

More Notes Join us Telegram-: http//:t.me/allexammentor


www.engineeringonyourfingertips.ooo
EE-GATE-2015 PAPER-02| www.gateforum.com

u  6.24m / s

46. In a certain slider-crank mechanism, lengths of crank and connecting rod are equal. If the
crank rotates with a uniform angular seed of 14 rad/s and the crank length is 300 mm, the
maximum acceleration of the slider (in m/s2) is ___________
Answer: 117.6
Exp: a max  2r2 (when   0) i.e at Inner dead centre

 2  0.3142
a max  117.6 m / s2

1
   9xi  3yj.ndS over the sphere giveb by x  y2  z 2  9 is ____
2
47. The surface integral
s

Answer: 216
Exp: By Gauss divergence theorem,

 S
F.n ds   divFdV
V

Here F  9x i  3y
div F  9  3  6

 
1

  1 1
9x i  3yj .nds   6 dV  6V
 V 
s

1 4 3
 6  r 
 3 
 8(3)3
 216.
www.
engi
neer
ingonyour
finger
tips.
ooo
48. A cantilever beam with square cross-section of 6mm side is subjected to a load of 2kN normal
to the top surface as shown in the figure. The young’ s modulus of elasticity of the material of
the beam is 210 GPa. The magnitude of slope. (in radian) at Q (20 mm from the fixed end) is
________

20mm 2kN

O
Q P
100mm
Answer: 0.1587
Px
Exp: Slope. Q  (2L  x)
2EI
where x is measured from the fixed end
India’s No.1 institute for GATE Training 1Lakh+Studentstrainedtilldate65+Centerscross
India
19


www.engineeringonyourfingertips.ooo

More Notes Join us Telegram-: http//:t.me/allexammentor


www.engineeringonyourfingertips.ooo
EE-GATE-2015 PAPER-02| www.gateforum.com

2000(0.02)
 Q  (2  0.1  0.02)
(0.006)4
2  210 10 
9

12
=-0.1587
The magnitude of slope is 0.1587 radian

49. A cylindrical uranium fuel rod of radius 5 mm in a nuclear reactor is generating heat at the
rate of 4 107 W m3 . The rod is cooled by a liquid (convective heat transfer coefficient 1000
W m2 .K ) at 25O C. At steady state, the surface temperature (in K) of the rod is
(A) 308 (B) 398 (C) 418 (D)
448
Answer: (B)
q�g
Exp: The surface temp. (Tw )  Ta  R
2h
4 107
 298   5 103
2 1000
Tw  398 K

50. For the same values of peak pressure, peak temperature and heat rejection, the correct order of
efficiencies for Otto, Dual and Diesel cycles is
 A  otto  Dual  Diesel  B Diesel  Dual  otto
 C Dual  Diesel  otto  D Diesel  Otto  Dual
Answer: (B)
Exp: For same values of peak pressure and temperature. Diesel cycle is most efficient and
Otto cycle is least. Efficiency of dual cycle lies in between.
diesel > dual > otto

www.
engi
neer
ingonyour
finger
tips.
ooo
51. During a TIG welding process, the are current and are voltage were 50 A and 60 V,
respectively, when in the welding speed was 150 mm/mi. In another process, the TIG welding
is carried out at a welding speed of 120 mm/min at the same arc voltage and heat input to the
material so that weld quality remains the same. The welding current (in A) for this process is
(A) 40.00 (B) 44.72 (C) 55.90 (D) 62.25
Answer: (A)
Exp: Total heat input = VIt
time t is inversely proportional to weld speed (S)

India’s No.1 institute for GATE Training 1Lakh+Studentstrainedtilldate65+Centerscross


India
20


www.engineeringonyourfingertips.ooo 

More Notes Join us Telegram-: http//:t.me/allexammentor


www.engineeringonyourfingertips.ooo
EE-GATE-2015 PAPER-02| www.gateforum.com

V2 I 2 V1I1
 
S2 S1
120
I2   50
150
I 2  40 A

52. Consider the following differential equation:


dy
 5y; initial condition : y  2 at t  0.
dt
The value of y at t = 3 is
 A  5e10  B 2e10 C 2e 15  D  15e2
Answer: (C)
dy
Exp:  5y
dt
dy
  5dt  variablesseparable form 
y

Integrating,
ln y  5t  c _____ 1

when y  2at t  0  initialconditional, 1 gives 


c  ln 2
 y 
ln y  5t  ln 2  ln    5t  y  2e 5t
2
at t  3, y  2e15
53. For the truss shown in the figure, the magnitude of the force (in kN) in the member SR is

www.
engi
neer
ingonyour
finger
30kN t
ips.
ooo
W V S R

1m

Q
P U T
1m 1m 1m

(A) 10 (B) 14.14 (C) 20 (D) 28.28

Answer: (C)
India’s No.1 institute for GATE Training 1Lakh+Studentstrainedtilldate65+Centerscross
India
21


www.engineeringonyourfingertips.ooo

More Notes Join us Telegram-: http//:t.me/allexammentor


www.engineeringonyourfingertips.ooo
EE-GATE-2015 PAPER-02| www.gateforum.com

Exp: 30kN

W V S R

1m

P X
U T
Rx
Rp 2m 1m
3m
Rp + Rx = 30000
Mp = 0
S R
Rx×3 = 2×3000
 Rx = 20kN
20
and Rp = 10 kN x
T
for balance at ‘ x’  FRx = 20kN 20

at ‘ RT’  FRT cos45 = 20


20
  FRT  ____ (1)
cos 45
Also FSR = FRTcos45 ____ (2)
from (1) and (2)
FSR = 20 kN

54. A project consists of 7 activities. The network along with the time durations (in days) for
various activities is shown in the figure.

12 11 10
1 3 5 6
www.
engi
neer
ingonyour
finger
tips.
ooo
14
12
9

2 7 4

The minimum time (in days) for completion of the project is _____
Answer: 40
Exp: Time taken for 3 paths are as follows
Path 1 = 12+11+10
Path 2 = 14+12+10
Path 3 = 14+7+9+10
India’s No.1 institute for GATE Training 1Lakh+Studentstrainedtilldate65+Centerscross
India
22


www.engineeringonyourfingertips.ooo 

More Notes Join us Telegram-: http//:t.me/allexammentor


www.engineeringonyourfingertips.ooo
EE-GATE-2015 PAPER-02| www.gateforum.com

∴ Path3 is longest i.e. path 3 is critical path


∴ Project duration = 40 days

55. A resistance-capacitance relaxation circuit is used in an electrical discharge machining


process. The discharge voltage is 100 V. At a spark cycle time of s, the average power input
required is 1 kW. The capacitance  in F  in the circuit is
(A) 2.5 (B) 5.0 (C) 7.5 (D)
10.0
Answer: (A)
Exp: Voltage = 100V
Power = 1kW
V2 100  100
Power  R   10
R 1000
 25
  RC  c    2.5 F
R 10

www.
engi
neer
ingonyour
finger
tips.
ooo

India’s No.1 institute for GATE Training 1Lakh+Studentstrainedtilldate65+Centerscross


India
23


www.engineeringonyourfingertips.ooo 

More Notes Join us Telegram-: http//:t.me/allexammentor


www.engineeringonyourfingertips.ooo
ME-GATE-2015 PAPER| www.gateforum.com

General aptitude

Q. No. 1 – 5 Carry One Mark Each

1. Five teams have to compete in a league, with every team playing every other team exactly
once, before going to the next round. How many matches will have to be held complete
the league round of matches?
(A) 20 (B) 10 (C) 8 (D) 5
Answer: (B)
Exp: For a match to be played, we need 2 teams
L No of matches = no. of ways of selections 2 teams out of 5
5C 2 10

2. Tanya is older than Enc.


Cliff is older than Tanya.
Eric is older than Cliff.
If the first two statements are true, then the third statement is
(A) True (B) False (C) Uncertain (D) Data insufficient
Answer: (B)

3. Choose the appropriate word/phase, out of the four options given below, to complete the
following sentence:
Apparent lifelessness ___________ dormant life.
(A) harbours (b) lead to (c) supports (d) affects
Answer: (A)
ww
Exp: Apparent: looks w.
like engi
neer
ingonyour
finger
tips.
ooo
dormant: hidden
Harbour: give shelter
Effect (verb): results in

4. Choose the statement where underlined word is used correctly.


(A) When the teacher eludes to different authors, he is being elusive
(B) When the thief keeps eluding the police, he is being elusive
(C) Matters that are difficult to understand, identify or remember are allusive
(D) Mirages can be allusive, but a better way to express them is illusory
Answer: (B)
Exp: Elusive: Difficult to answer.

© All r ight s reser ved by GATE For um Educat ional Serv ices Pvt . Lt d. No part of t his book let m ay be reproduced or ut ilized
in any for m w it hout t he wr it t en perm ission. Discuss t his quest ions paper at w w w .g a t e m e n t o r . co m .

www.engineeringonyourfingertips.ooo 1

More Notes Join us Telegram-: http//:t.me/allexammentor


www.engineeringonyourfingertips.ooo
ME-GATE-2015 PAPER| www.gateforum.com

5. Fill in the blank with the correct idiom/phrase.


That boy from the town was a __________ in the sleepy village.
(A) Dog out of herd (B) Sheep from the heap
(C) Fish out of water (D) Bird from the flock
Answer: (C)
Exp: From the statement, it appears that boy found it tough to adapt to a very different
situation.
Q. No. 6 – 10 Carry Two Marks Each

6. Right triangle PQR is to be constructed in the xy – plane so that the right angle is at P and
line PR is parallel to the-axis. The x and y coordinates of P, Q, and R are to be integers
that satisfy the inequalities: –4 ≤ x ≤ 5 and 6 ≤ y ≤ 16. How many different triangles
could be constructed with these properties?
(A) 110 (B) 1,100 (C) 9,900 (D) 10,000
Answer: (C)
Exp: X1 4 X1 5
Y1 6 Y1 16
X2 9 chairs X1 X2
Y2 10 chairs Y1 Y2
Total triangles 10 11 9 10 9900

7. Select the appropriate option in place of underlined part of the sentence.


Increased productivity necessary reflects greater efforts made by the employees.
(A) Increase in productivity necessary
(B) Increase productivity is necessary
(C) Increase in productivity necessarily
(D) No improvement required
Answer: (C) www.
engi
neer
ingonyour
finger
tips.
ooo

8. Given below are two statements followed by two conclusions. Assuming these statements
to be true, decide which one logically follows:
Statements:
I. No manager is a leader. II. All leaders are executive.
Conclusions:
I. No manager is a executive. II. All executive is a manager.
(A) Only conclusion I follows. (B) Only conclusion II follows.
(C) Neither conclusion I nor II follows. (D) Both conclusion I and II follow.
Answer: (C)
Exp: Executive

L NL

© All r ight s reser ved by GATE For um Educat ional Serv ices Pvt . Lt d. No part of t his book let m ay be reproduced or ut ilized
in any for m w it hout t he wr it t en perm ission. Discuss t his quest ions paper at w w w .g a t e m e n t o r . co m .

www.engineeringonyourfingertips.ooo 2

More Notes Join us Telegram-: http//:t.me/allexammentor


www.engineeringonyourfingertips.ooo
ME-GATE-2015 PAPER| www.gateforum.com

Therefore concluding diagram can be


It can be manager that is manager can be executive also.
Some executives are also leaders that is not a manager

9. A coin is tossed thrice. Let X be the event that head occurs in each of the first two tosses.
Let Y be the event that a tail occurs on the third toss. Let Z be the event that two tails
occurs in three tosses. Based on the above information, which one of the following
statements is TRUE?
(A) X and Y are not independent
(B) Y and Z are dependent
(C) Y and Z are independent
(D) X and Z independent
Answer: (B)
Exp: Let y as tail occurred in third toss
and z as two tails in third toss which can be {TTH, THT, HTT}
y = {TTH, TTT}
∴ both y and z are dependent.

10. In the given figure angle Q is a right angle, PS:QS = 3:1, RT:QT = 5:2 and PU:UR = 1:1.
If area of triangle QTS is 20 cm2, then the area of triangle PQR in cm2 is ______.
R

U T

P Q
S
Answer: 280
Exp: Let area of triangle PQR be ‘ A’
SQ 1 w 1ww. engi neer i
ngonyour
finger
tips.
ooo
PQ 1 3 4
QT 2 2
QR 2 5 7
1
Area of le QTS SQ QT
2
1 1 2
PQ QR
2 4 7
1 2 1
PQ QR
4 7 2
1
Area of le PQR
14

© All r ight s reser ved by GATE For um Educat ional Serv ices Pvt . Lt d. No part of t his book let m ay be reproduced or ut ilized
in any for m w it hout t he wr it t en perm ission. Discuss t his quest ions paper at w w w .g a t e m e n t o r . co m .

www.engineeringonyourfingertips.ooo 3

More Notes Join us Telegram-: http//:t.me/allexammentor


www.engineeringonyourfingertips.ooo
ME-GATE-2015 PAPER| www.gateforum.com

1
given 20cm 2 A
14
A 14 20 280cm 2

Mechanical Engineering
Q. No. 1 – 25 Carry One Mark Each

1. Three parallel pipes connected at the two ends have flow-rates Q1, Q2 and Q3 respectively,
and the corresponding frictional head losses are hL1, hL2 and hL3 respectively. The correct
expressions for total flow rate (Q) and frictional head loss across the two ends (hL) are
(A) Q = Q1 + Q2 + Q3; hL = hL1 + hL2 + hL3
(B) Q = Q1 + Q2 + Q3; hL = hL1 = hL2 = hL3
(C) Q = Q1 = Q2 = Q3; hL = hL1 + hL2 + hL3
(D) Q = Q1 = Q2 = Q3; hL = hL1 = hL2 = hL3
Answer: (B)
Exp: Total flow rate Q Q1 Q2 Q3
head loss h h L1 h L2 h L3

4 2
2. The lowest eigen value of the 2 × 2 matrix is ________
1 3
Answer: 2
4 2
Exp: Let A
1 3

Characteristic equation of A is A I 0
4 2
0
1 3
2
7 w
10ww
0.engi
n2,5
eer i
ngonyour
finger
tips.
ooo

3. Which two of the following joining processes are autogenous?


i. Diffusion welding
ii. Electroslag welding
iii. Tungsten inert gas welding
iv. Friction welding
(A) i and iv (B) ii and iii (C) ii and iv (D) i and iii
Answer: (A)
Exp: Diffusion welding and friction welding are autogenous welding process as they do not
require any filler material.

© All r ight s reser ved by GATE For um Educat ional Serv ices Pvt . Lt d. No part of t his book let m ay be reproduced or ut ilized
in any for m w it hout t he wr it t en perm ission. Discuss t his quest ions paper at w w w .g a t e m e n t o r . co m .

www.engineeringonyourfingertips.ooo 4

More Notes Join us Telegram-: http//:t.me/allexammentor


www.engineeringonyourfingertips.ooo
ME-GATE-2015 PAPER| www.gateforum.com

4. The strain hardening exponent n of stainless steel SS 304 with distinct yield and UTS
values undergoing plastic deformation is
(A) n < 0 (B) n =0 (C) 0 < n < 1 (D) n = 1
Answer: (C)
Exp: n lies between 0 and 1. 0 means material is a perfectly plastic solid, while 1 represents a
100% elastic solid.
5. In the figure, link 2 rotates with constant angular velocity ω2. A slider link 3 moves
outwards with a constant relative velocity VQ/P, where Q is a point on slider 3 and P is a
point on link 2. The magnitude and direction of Coriolis component of acceleration is
given by
Qon 3 VQP

P on 2
3

2 2

(A) 2ω2 VQ/P; direction of VQ/P rotated by 90° in the direction ω2


(B) ω2 VQ/P; direction of VQ/P rotated by 90° in the direction ω2
(C) 2ω2 VQ/P; direction of VQ/P rotated by 90° opposite to the direction of ω2
(D) ω2 VQ/P; direction of VQ/P rotated by 90° opposite to the direction ω2
Answer: (A)
Exp: direction is obtained by rotating velocity vector through 90o in the direction of rotation of
the link.

6. Couette flow is characterized by


(A) steady, incompressible, laminar flow through a straight circular pipe
(B) fully developed turbulent flow through a straight circular pipe
(C) steady, incompressible, laminar flow between two fixed parallel plates
www.
engi
neer
ingonyour
finger
tips.
ooo
(D) steady, incompressible, laminar flow between one fixed plate and the other moving
with a constant velocity
Answer: (D)
Exp: Coutte flow is steady incompressible, laminar flow between one fixed plate and other
moving with constant velocity.
V

7. If P(X) = 1 4,P Y 1 3, and P X Y 1 12, the value of P Y X is


1 4 1 29
A B C D
4 25 3 50

© All r ight s reser ved by GATE For um Educat ional Serv ices Pvt . Lt d. No part of t his book let m ay be reproduced or ut ilized
in any for m w it hout t he wr it t en perm ission. Discuss t his quest ions paper at w w w .g a t e m e n t o r . co m .

www.engineeringonyourfingertips.ooo 5

More Notes Join us Telegram-: http//:t.me/allexammentor


www.engineeringonyourfingertips.ooo
ME-GATE-2015 PAPER| www.gateforum.com

Answer: (C)
1
P X Y 12 1
Exp: P Y X
P X 1 3
4
8. In a machining operation, if the generatrix and directix both are straight lines, the surface
obtained
(A) cylindrical (B) helical
(C) plane (D) surface of revolution
Answer: (C)
Exp: The surface obtained is plane.

9. A rigid container of volume 0.5 m3 contains 1.0 kg of water at 120°C (vf = 0.00106
m3/kg, vg = 0.8908 m3/kg). The state of water is
(A) Compressed liquid
(B) Saturated liquid
(C) A mixture of saturated liquid and saturated vapor
(D) Superheated vapor
Answer: (C)
0.5 3
Exp: V m / kg 0.5m3 / kg
1
Since Vf V Vg the state of water is mixture of saturated water and saturated vapour.

10. In full mould (cavity-less) casting process, the pattern is made of


(A) expanded polystyrene (B) wax
(C) epoxy (D) plaster of Paris
Answer: (A)
Exp: The pattern is made of expanded polystrene
www.
engi
neer
ingonyour
finger
tips.
ooo
11. A gear train is made up of five spur gears as shown in the figure. Gear 2 is driver and gear
6 is driven member. N2, N3, N4, N5 and N6 represent number of teeth on gears 2, 3, 5 and 6
respectively. The gear(s) which act(s) as idler(s) is/are
N5
N3
N6
N2 2

4 5 6
Driver 3
Driven
N4
(A) only 3 (B) only 4
(C) only 5 (D) Both 3 and 5
Answer: (C)

© All r ight s reser ved by GATE For um Educat ional Serv ices Pvt . Lt d. No part of t his book let m ay be reproduced or ut ilized
in any for m w it hout t he wr it t en perm ission. Discuss t his quest ions paper at w w w .g a t e m e n t o r . co m .

www.engineeringonyourfingertips.ooo 6

More Notes Join us Telegram-: http//:t.me/allexammentor


www.engineeringonyourfingertips.ooo
ME-GATE-2015 PAPER| www.gateforum.com

2 2 3 5 N3 N5 N 6 N3 N 6
Exp:
6 3 5 6 N 2 N 4 N5 N2 N4
only Gear(5) is Idle

12. Let be an arbitrary smooth real valued scalar function and V be an arbitrary smooth
vector valued function in a three-dimensional space. Which one of the following is an
identity?
A Cur l V DivV B Div V 0

C DivCurl V 0 D Div V DivV


Answer: (C)

13. Which of the following statements are TRUE for damped vibrations?
P. For a system having critical damping, the value of damping ratio is unity and system
does not undergo a vibratory motion.
Q. Logarithmic decrement method is used to determine the amount do damping in a
physical system.
R. In case of damping due to dry friction between moving surfaces resisting force of
constant magnitude acts opposite to the relative motion.
S. For the case of viscous damping, drag force is directly proportional to the square of
relative velocity.
(A) P and Q only (B) P and S only
(C) P, Q and R only (D) Q and S only
Answer: (C)

sin x
14. The value of limx 0 is ____
2sin x x cos x
Answer: -0.333 www.
engi
neer
ingonyour
finger
tips.
ooo
sin x 0
Exp: lim form
x 0 2sin x x cos x 0
cos x
lim L Hospital Rule
x 0 2cos x cos x x sin x
1
3

15. The ratio of momentum diffusivity (v) to thermal diffusivity (α), is called
(A) Prandtl number (B) Nusselt number
(C) Biot number (D) Lewis number
Answer: (A)

© All r ight s reser ved by GATE For um Educat ional Serv ices Pvt . Lt d. No part of t his book let m ay be reproduced or ut ilized
in any for m w it hout t he wr it t en perm ission. Discuss t his quest ions paper at w w w .g a t e m e n t o r . co m .

www.engineeringonyourfingertips.ooo 7

More Notes Join us Telegram-: http//:t.me/allexammentor


www.engineeringonyourfingertips.ooo
ME-GATE-2015 PAPER| www.gateforum.com

Cp VCp V V
Exp: Pr
k k k
cp

16. For the given fluctuating fatigue load, the values of stress amplitude and stress ratio are
respectively
MPa
250
200
150
100
50
0 Time

(A) 100 MPa and 5 (B) 250 MPa and 5


(C) 100 MPa and 0.20 (D) 250 MPa and 0.20
Answer: (C)
max min 250 50
Exp: stress amplitude 100 Mpa
2 2
min 50
Stress ratio 0.2
max 250

2
17. Using a unit step size, the value of integral x ln x dx by trapezoidal rule is ______
1

Answer: 0.69
x 1 2
Exp:
y 1hx 0 21h2
www.
engi
neer
ingonyour
finger
tips.
ooo
By Trapezoidal Rule,
2
1
x1nx dx 0 21n2 1n2 0.69
1
2

18. A weight of 500 N is supported by two metallic ropes as shown in the figure. The values
of tensions T1 and T2 are respectively

30O
T1
90O T2

120O

500 N

© All r ight s reser ved by GATE For um Educat ional Serv ices Pvt . Lt d. No part of t his book let m ay be reproduced or ut ilized
in any for m w it hout t he wr it t en perm ission. Discuss t his quest ions paper at w w w .g a t e m e n t o r . co m .

www.engineeringonyourfingertips.ooo 8

More Notes Join us Telegram-: http//:t.me/allexammentor


www.engineeringonyourfingertips.ooo
ME-GATE-2015 PAPER| www.gateforum.com

(A) 433 N and 250 N (B) 250 N and 433 N


(C) 353.5 N and 250 N (D) 250 N and 353.5 N
Answer: (A)
Exp: Using sine rule

T1 T2 500 T1
T2
sin120o sin150o sin 90o 90o
T1 500 sin120o ; T2 500sin150o
T1 433 N T2 250 N 150o 120o

500

19. In the notation (a/b/c) : (d/e/f) for summarizing the characteristics of queueing situation,
the letters ‘ b’ and ‘ d’ stand respectively for
(A) service time distribution and queue discipline
(B) number of servers and size of calling source
(C) number of servers and queue discipline
(D) service time distribution and maximum number allowed in system
Answer: (A)
Exp: b: Service time distribution (usually represented by ‘ m’ )
D: Queuing discipline (usually represented by ‘ GD’ )

20. The thermodynamic cycle shown in figure (T/s diagram) indicates


2
(A) Reversed Cannot cycle T
(B) Reversed Brayton cycle P C

(C) Vapor compression cycle 3 P C 1


www.
engi
neer
ingonyour
finger
tips.
ooo
(D) Vapor absorption cycle 4
s
Answer: (B)
Exp: T-S diagram represent a reversed brayton cycle used in air conditioning of aero planes
where air is used as a refrigerant.

21. A drill is positioned at point P and its has to proceed to point Q. The coordinates of point
Q in the incremental system of defining position of a point in CNC part program will be
Y

P
12
5

3 4
© All r ight s reser ved by GATE For um Educat ional Serv ices Pvt . Lt d. No part of t his book let m ay be reproduced or ut ilized
in any for m w it hout t he wr it t en perm ission. Discuss t his quest ions paper at w w w .g a t e m e n t o r . co m .

www.engineeringonyourfingertips.ooo 9

More Notes Join us Telegram-: http//:t.me/allexammentor


www.engineeringonyourfingertips.ooo
ME-GATE-2015 PAPER| www.gateforum.com

(A) (3, 12) (B) (5, 7) (C) (7, 12) (D) (4, 7)
Answer: (D)
Exp: In incremental system. Co-ordinates of point Q are (4,7).
22. A cylindrical tank with closed ends is filled with compressed air at a pressure of 500 kPa.
The inner radius of the tank is 2m, and it has wall thickness of 10 mm. The magnitude of
maximum in-plane shear stress (in MPa) is ____ .
Answer: 25
pd
Exp: Maximum in-plane shear stress max
8t
500 4
MPa 25MPa
8 10

23. An air-standard Diesel cycle consists of the following processes:


1-2: Air is compressed isentropically.
2-3: Heat is added at cosntant pressure.
3-4: Air expands isentropically to the original volume.
4-1: Heat is rejected at constant volume.
If and T denotes the specific heat ratio and temperature, respectively the efficiency of
the cycle is
T4 T1 T4 T1
(A) 1 (B) 1
T3 T2 T3 T2

T4 T1 T4 T1
(C) 1 (D) 1
T3 T2 1 T3 T2
Answer: (B)
2 3
Exp: heat applied, Qs cp T3 T2 p
heat rejected, Qr cr T4 T1 4
www.
engi
neer
ingonyour
finger
tips.
ooo
Qr 1 (T4 T1 )
1 1
Qs (T3 T2 ) 1

24. Saturated vapor is condensed to saturated liquid in condenser. The heat capacity ratio is
cmin
Cr . The effectiveness ( ) of the condenser is
cmax
1 exp NTU(1 Cr ) 1 exp NTU(1 Cr )
(A) (B)
1 Cr 1 Cr exp NTU(1 Cr )
NTU
(C) (D) 1 exp( NTU)
1 NTU
Answer: (D)
© All r ight s reser ved by GATE For um Educat ional Serv ices Pvt . Lt d. No part of t his book let m ay be reproduced or ut ilized
in any for m w it hout t he wr it t en perm ission. Discuss t his quest ions paper at w w w .g a t e m e n t o r . co m .

www.engineeringonyourfingertips.ooo 10

More Notes Join us Telegram-: http//:t.me/allexammentor


www.engineeringonyourfingertips.ooo
ME-GATE-2015 PAPER| www.gateforum.com

Exp: E of condenser is given by 1-exp(-NTU)


Cmin
because Cr 0. (as Cmax )
Cmax

25. For the same material and the mass, which of the following configurations of flywheel
will have maximum mass moment of inertia about the axis of rotation OO’ passing
through the center of gravity

O O O'
A B

C D O
O'

Answer: (B)
Exp: Rim med wheel has maximum mass located away from the axis of rotation Thus will have
maximum moment of inertia.

www.
engi
neer
ingonyour
finger
tips.
ooo
Q. No. 26 – 55 Carry Two Marks Each

26. For ball bearings, the fatigue life L measured in number of revolutions and the radial load
F are related by FL1/3 = K, where K is a constant. It withstands a radial load of 2 kN for a
life of 540 million revolutions. The load (in kN) for a life of one million revolutions is
_______
Answer: 16.286
1
Exp: FL3 k

© All r ight s reser ved by GATE For um Educat ional Serv ices Pvt . Lt d. No part of t his book let m ay be reproduced or ut ilized
in any for m w it hout t he wr it t en perm ission. Discuss t his quest ions paper at w w w .g a t e m e n t o r . co m .

www.engineeringonyourfingertips.ooo 11

More Notes Join us Telegram-: http//:t.me/allexammentor


www.engineeringonyourfingertips.ooo
ME-GATE-2015 PAPER| www.gateforum.com

1 1
F1L31 F2 L32
1 1
2 540 3 F2 (1) 3
F2 16.286 kN

27. The torque (in N-m) exerted on the crank shaft of a two stroke engine can be described as
T = 10000 + 1000 sin 1θ – 1200 cos 2θ, where θ is the crank angle as measured from
inner dead center position. Assuming the resisting torque to be constant, the power (in
kW) developed by the engine at 100 rpm is __________.
Answer: 104
Exp: Tmean 10000N m
2
mean 100
60
200
P Tmean mean 104 104kW
60

28. The value of moment o inertia of the section shown in the figure about the ais-XX is

60
30

All dimensions 45
are in mm 120
15
X X
15

45

30
www.
engi
neer
ingonyour
finger
tips.
ooo
A 8.5050 106 mm4 B 6.88.50 105 mm4

C 7.7625 106 mm4 D 8.5725 106 mm4


Answer: (B)
1 1
Exp: Moment of Inertia. I xx (120)3 60 2 (30)4 30 30 30
12 12
6.885 106 mm4

29. The value of

C 3x 8y2 dx 4y 6xy dy , (where C is boundary of th region bounded by x = 0,


y = 0 and x + y = 1 is) is _____________

© All r ight s reser ved by GATE For um Educat ional Serv ices Pvt . Lt d. No part of t his book let m ay be reproduced or ut ilized
in any for m w it hout t he wr it t en perm ission. Discuss t his quest ions paper at w w w .g a t e m e n t o r . co m .

www.engineeringonyourfingertips.ooo 12

More Notes Join us Telegram-: http//:t.me/allexammentor


www.engineeringonyourfingertips.ooo
ME-GATE-2015 PAPER| www.gateforum.com

Answer: 1.66
Exp: x 0 to x 1 y
&
y 0 to y 1

By Green’ s theorem, 3x 8y 2 dx 4y 6xy dy


C

N M
dxdy
x y
0,1
1 1 y
6y 16y dxdy x y 1
y 0x 0
1 1 y
10ydx dy
0,0 1,0
y 0 x 0
1
1 y
10 yx 0 dy
y 0

1 1
y2 y3
10 y 1 y 0 dy 10
y 0
2 3
0

1 1 5
10 1.66
3 3 3

W
30. A brick wall k 0.9
of thickness 0.18 m separates the warm air in a room from the
m.k
cold ambient air. On a particular winter day, the outside air temperature is – 5°C and the
room needs to be maintained at 27°C. The heat transfer coefficient associated with
W
outside air is 20 2 . Neglecting the convective resistance of the air inside the room,
m K'
w
the heat loss, in is
wwwm .e
2
ngi neer ingonyour fi
nger ti
ps. ooo
(A) 88 (B) 110 (C) 128 (D) 160
Answer: (C)
Exp:

5o C 27o C
2
h 20w / m K

1 1
Total thermal resistance
R th 1 l
h k

© All r ight s reser ved by GATE For um Educat ional Serv ices Pvt . Lt d. No part of t his book let m ay be reproduced or ut ilized
in any for m w it hout t he wr it t en perm ission. Discuss t his quest ions paper at w w w .g a t e m e n t o r . co m .

www.engineeringonyourfingertips.ooo 13

More Notes Join us Telegram-: http//:t.me/allexammentor


www.engineeringonyourfingertips.ooo
ME-GATE-2015 PAPER| www.gateforum.com

1 1
4 W / m2K
R th 1 0.18
20 0.9
T
Q 27 ( 5) 4 128 W / m 2
R th
31. A bullet spins as the shot is fired from a gun. For this purpose, two helical slots as shown
in the figure are cut in the barrel. Projections A and B on the bullet engage in each of the
slots

Gun Barrel A
Bullet

0.5m B

Helical slots are such that one turn of helix is completed over a distance of 0.5 m. If
velocity of bullet when it exits the barrel is 20 m/s, its spinning speed in rad/s is _______.

Answer: 251.3
0.5
Exp: Time taken force revolution 0.025 sec.
20
2
The spinning speed is rad / sec
0.02s
251.3 rad / sec

32. Which of the following statements are TRUE, when the cavitation parameter σ = 0?
i. the local pressure is reduced to vapor pressure
ii. cavitation starts
iii. boiling ofwliquid
ww. e ngi
starts neer
ingonyour
finger
tips.
ooo
iv cavitation stops
(A) i, ii and iv (B) only ii and iii (C) only i and iii (D) i, ii and iii

Answer: (D)
Exp: σ = 0 implies (i), (ii) and (iii)

33. In a CNC milling operation, the tool has to machine the circular arc from point (20, 20) to
(10, 10) at sequence number 5 of the CNC part program. If the center of the arc is at (20,
10) and the machine has incremental mode of defining position coordinates, the correct
tool path command is (A) N 05 G 90 G01 X–10 Y–10 R10
(B) N 05 G91 G03 X–10 Y–10 R10
(C) N 05 G90 G03 X20 Y20 R10

© All r ight s reser ved by GATE For um Educat ional Serv ices Pvt . Lt d. No part of t his book let m ay be reproduced or ut ilized
in any for m w it hout t he wr it t en perm ission. Discuss t his quest ions paper at w w w .g a t e m e n t o r . co m .

www.engineeringonyourfingertips.ooo 14

More Notes Join us Telegram-: http//:t.me/allexammentor


www.engineeringonyourfingertips.ooo
ME-GATE-2015 PAPER| www.gateforum.com

(D) N 05 G91 G02 X20 Y20 R10

Answer: (B)
Exp: for incremental coordinates (G91) and coordinates of final point are ( 10, 10). The tool
moves CCW (counter clockwise), So G03.

34. Ratio of solidification time of a cylindrical casting (height =radius) to the cubic
casting of side two times the height of cylindrical casting is _____.
Answer: 0.5625
2
V
Exp: ts k
A
for cylindrical cavity (1)
2
d2h 2
d
t1 k 4 k d h
dh 4

for cubic casting (2)


2 2 2
a3 a d
t2 k k k a 2d
6a 2 6 3
2
d
2
t1 4 3
2
0.5625
t2 d 4
3

35. The number of degrees of freedom of the linage shown in the figure is

www.
engi
neer
ingonyour
finger
tips.
ooo

(A) -3 (B) -0 (C) 1 (D) 2

Answer: (C)
Exp: Number of links, N = 6
Total number as binary joints, j = 7
F = 3 (N-1) – 2j
= 15-14 = 1.

© All r ight s reser ved by GATE For um Educat ional Serv ices Pvt . Lt d. No part of t his book let m ay be reproduced or ut ilized
in any for m w it hout t he wr it t en perm ission. Discuss t his quest ions paper at w w w .g a t e m e n t o r . co m .

www.engineeringonyourfingertips.ooo 15

More Notes Join us Telegram-: http//:t.me/allexammentor


www.engineeringonyourfingertips.ooo
ME-GATE-2015 PAPER| www.gateforum.com

36. Figure shows a single degree of freedom system. The system consists of a massless rigid
bar OP hinged at O and a mass m at end P. The natural frequency of vibration of the
system i
1 k 1 k k P
A fn B fn
2 4m 2 2m O Q
m
1 k 1 2k a a
C fn D fn
2 m 2 m
Answer: (A)
Exp: force in the spring F= 2mg [from equilibrium]

Deflection as mass at P,
a a
x P
x1 2a 2x O
a
X
2mg 4mg
2 X1
k k
g g k
wn ;
s x1 4m
1 1 k
fn n
2 2 4m

37. For the linear programming problem:

Maximize Z = 3X1 + 2X2

Subject to

– 2X1 + 3X2 ≤ 9

X1 – 5X2 ≥ – 20
www.
engi
neer
ingonyour
finger
tips.
ooo
X 1, X 2 ≥ 0

The above problem has


(A) unbounded solution
(B) infeasible solution
(C) alternative optimum solution
(D) degenerate solution
Answer: (A)
Exp: Plotting the graph for the given constraints as shown in figure.
From figure we can see that LPP has unbounded solution.

© All r ight s reser ved by GATE For um Educat ional Serv ices Pvt . Lt d. No part of t his book let m ay be reproduced or ut ilized
in any for m w it hout t he wr it t en perm ission. Discuss t his quest ions paper at w w w .g a t e m e n t o r . co m .

www.engineeringonyourfingertips.ooo 16

More Notes Join us Telegram-: http//:t.me/allexammentor


www.engineeringonyourfingertips.ooo
ME-GATE-2015 PAPER| www.gateforum.com

x2

(0,4)

( 20,0) x1
( 45,0)

38. Air in a room is at 35° and 60% relative humidity (RH). The pressure in the room is 0.1
MPa. The saturation pressure of water at 35°C is 5.63 kPa. The humidity ratio of the air
(in gram/kg of dry air) is _____.
Answer: 21.74
Pw Pw
Exp: 0.6
Ps 5.63
Pw 3.378KPa
Pw
humidity Ratio, w 0.622
Pa Pw
3.378
0.622
100 3.378
0.021745kg kg of dryair
or 21.745 g/kg of dry air

www.
engi
neer
ingonyour
finger
tips.
ooo
39. A solid sphere 1 of radius ‘ r’ is placed inside a hollow, closed hemispherical surface 2 of
radius ‘ 4r’ . The shape factor F2-1 is

2r 2
1

8r

1 1
A B C 2 D 12
12 2
Answer: (A)

© All r ight s reser ved by GATE For um Educat ional Serv ices Pvt . Lt d. No part of t his book let m ay be reproduced or ut ilized
in any for m w it hout t he wr it t en perm ission. Discuss t his quest ions paper at w w w .g a t e m e n t o r . co m .

www.engineeringonyourfingertips.ooo 17

More Notes Join us Telegram-: http//:t.me/allexammentor


www.engineeringonyourfingertips.ooo
ME-GATE-2015 PAPER| www.gateforum.com

Exp: f11 f12 1


f12 1
f 21 A 2 f12 A1
f12 A1 1 4 r2 1
f 21
A2 1 2 2 12
4 4r 4r
2

40. Newton-Raphson method is used to find the roots of the equation, x3 + 2x2 + 3x – 1 = 0. If
the initial guess is x0 = 1, then the value of x after 2nd iteration is ___________.
Answer: 0.30
Exp: By Newton-Raphson Method,
f x0
1st iteration, x1 x0
f x0
f 1 5 1
1 1
f 1 10 2

Where f x x3 2x 2 3x 1 f 1 5
2
f x 3x 4x 3 f 1 10

f x1
2nd iteration, x 2 x1
f x1
f 0.5
0.5 0.3043
f 0.5

41. The annual requirement of rivets at a ship manufacturing company is 2000 kg. The rivets
are supplied in units of 1 kg costing Rs. 25 each. If the costs Rs. 100 to place an order and
the annual cost of carrying one unit is 9% of its purchase cost, the cycle length of the
order (in days) will be_____
Answer: 76.94
2DC0
www.
engi
neer
ingonyour
finger
tips.
ooo
Exp: EOQ
Ch

2 2000 100
421.637
0.09 25
365
length of cycle 76.948 days
No. of orders
2000
No. of orders
EOQ

© All r ight s reser ved by GATE For um Educat ional Serv ices Pvt . Lt d. No part of t his book let m ay be reproduced or ut ilized
in any for m w it hout t he wr it t en perm ission. Discuss t his quest ions paper at w w w .g a t e m e n t o r . co m .

www.engineeringonyourfingertips.ooo 18

More Notes Join us Telegram-: http//:t.me/allexammentor


www.engineeringonyourfingertips.ooo
ME-GATE-2015 PAPER| www.gateforum.com

42. Laplace transform of the function f(t) is given by F(s) = L{f(t)} = f t e st dt.
0

Laplace transform of the function shown below is given by

f t

1 t
1 e 2s 1 e s
2 2e s
A B C
s 2s s
s
1 2e
D
s
Answer: (C)
Exp: f t 2; 0 t 1
0; otherwise
1 st 1 s
st e 2 2e
L f t 2e dt 2
0
s 0
s
43. Orthogonal turning of a mild steel tube with a tool of rake angle 10° carried out at a feed
of 0.14 mm/rev. If the thickness of the chip produced is 0.28 mm, the values of shear
angle and shear strain will be respectively
(A) 28°20′ and 2.19 (B) 22°20′ and 3.53
(C) 24°30′ and 3.53 (D) 37°20′ and 5.19
Answer: (A)
0.14
Exp: r 0.5
0.28
r cos
tan www.
engi
neer
ingonyour
finger
tips.
ooo
1 r sin
28.3345o
or 28o 20
Shear strain, Y cot tan( ) 2.1859 2.19

44. Steam enters a turbine at 30 bar, 300°C (u = 2750 kJ/kg, h = 2993 kJ/kg) and exits the
turbine as saturated liquid at 15 kPa (u = 225 kJ/kg, h = 226 kJ/kg). Heat loss to the
surrounding is 50 kJ/kg of steam flowing through the turbine. Neglecting changes in
kinetic energy and potential energy, the work output of the turbine (in kJ/kg of steam) is
_______.
Answer: 2717

© All r ight s reser ved by GATE For um Educat ional Serv ices Pvt . Lt d. No part of t his book let m ay be reproduced or ut ilized
in any for m w it hout t he wr it t en perm ission. Discuss t his quest ions paper at w w w .g a t e m e n t o r . co m .

www.engineeringonyourfingertips.ooo 19

More Notes Join us Telegram-: http//:t.me/allexammentor


www.engineeringonyourfingertips.ooo
ME-GATE-2015 PAPER| www.gateforum.com

Exp: Work output (2993 226.50)kJ / kg =2717 kJ/kg

4 3i i
45. For a given matrix , where is i 1, the inverse of matrix P is
i 4 3i

1 4 3i i 1 i 4 3i
A B
24 i 4 3i 25 4 3i i

1 4 3i i 1 4 3i i
C D
24 i 4 3i 25 i 4 3i
Answer: (A)
Exp: |P| = (4+3i)(4-3i)-(i)(-i) = 16+9-1 = 24
4 3i i
adjP
i 4 3i
1 1 4 3i i
P
24 i 4 3i

46. For the overhanging beam shown in figure, the magnitude of maximum bending moment
(in kN-m) is _____.
20kN
10kN m
B
A C
4m 2m

Answer: 40 kN-m
Exp: BMD:

www.
engi
neer
ingonyour
finger
tips.
ooo
5
M
B
(KN m) A C
X

40
RA 10kN
RB 50kN
Maximum bending momentum occurs at reaction B an has a magnitude of 40 kN-m.

47. Figure shows a wheel rotating about O2. Two points A and B located along the radius of
wheel have speeds of 80 m/s and 140 m/s respectively. The distance between the points A
and B is 300 mm. The diameter of the wheel (in mm) is ________

© All r ight s reser ved by GATE For um Educat ional Serv ices Pvt . Lt d. No part of t his book let m ay be reproduced or ut ilized
in any for m w it hout t he wr it t en perm ission. Discuss t his quest ions paper at w w w .g a t e m e n t o r . co m .

www.engineeringonyourfingertips.ooo 20

More Notes Join us Telegram-: http//:t.me/allexammentor


www.engineeringonyourfingertips.ooo
ME-GATE-2015 PAPER| www.gateforum.com

Answer: 1400
Exp: VA 80 m / s, VB 140 m / s 2

rB rA 300 … (i)
A B
rA 80 O2
rB 140
rB
1.75 ...(2)
rA
Solving (1) & (2), rB=700 mm.
diameter of wheel is 1400 mm.

48. The dimensions of a cylindrical side riser (height = diameter) for a 25 cm × 15 cm × 5 cm


steel casting are to be determined. For the tabulated shape factor values given below,
diameter of the riser (in cm) is ______.
Shape Factor 2 4 6 8 10 12
Riser Volume / Casting Volume 1.0 0.70 0.55 0.50 0.40 0.35

Answer: 10.61
l w 25 15
Exp: Shapefactor 8 then from the table
h 5
Vr
0.5
Vc
Vr 0.5 25 15 5 937.5

d3 937.5cm3
4
d 10.61cm
www.
engi
neer
ingonyour
finger
tips.
ooo

49. A Prandtl tube (Pitot-static tube with C = 1) is used to measure the velocity of water. The
differential manometer reading is 10 mm of liquid column with a relative density of 10.
Assuming g = 9.8 m/s2, the velocity of water (in m/s) is _______.
Answer: 1.32
Exp: Velocity as water Cv 2gh
Cv = 1 (Given)
sg
h x 1 0.01 10 1 0.09m
s0
velocity of flow 2 9.8 0.09 1.328m s

© All r ight s reser ved by GATE For um Educat ional Serv ices Pvt . Lt d. No part of t his book let m ay be reproduced or ut ilized
in any for m w it hout t he wr it t en perm ission. Discuss t his quest ions paper at w w w .g a t e m e n t o r . co m .

www.engineeringonyourfingertips.ooo 21

More Notes Join us Telegram-: http//:t.me/allexammentor


www.engineeringonyourfingertips.ooo
ME-GATE-2015 PAPER| www.gateforum.com

50. In a rolling operation using rolls of diameter 500 mm if a 25 mm thick plate cannot be
reduced to less than 20 mm in one pass, the coefficient of friction between the roll and the
plate is __________
Answer: 0.1414
h max
Exp:
R
h max
25 20 5mm
R 250 mm
5
0.1414
250

51. Refrigerant vapor enters into the compressor of a standard vapor compression cycle at –
10°C (h = 402 kJ/kg) and leaves the compression at 50°C(h = 432 kJ/kg). It leaves the
condenser at 30°C (h = 237 kJ/kg). The COP of the cycle is __________.
Answer: 5.5
Exp: work done=432-402=30 kJ/kg
Refrigerating effect =402-237=165 kJ/kg.
165
COP 5.5
30

52. A cantilever bracket is bolted to a column using three M12 × 1.75 bolts, P, Q and R. The
value of maximum shear stress developed in the bolt P (in MPa) is _______.

9kN

40 P
30
Q
30
ww40
w.engi
Rneer
ingonyour
finger
10mmtips.
thickooo

250 All dimension


Answer: 341 are in mm
Exp: P=9kN, e=250 mm P
Primary shear force Pp''
p q
Pp' Pq' Pr' 3kN
3 3 Pp'
Secondary shear force: q
By symmetry C.G lies at the centre of bolt Q. C.G

Pq'

© All r ight s reser ved by GATE For um Educat ional Serv ices Pvt . Lt d. No part of t his book let m ay be reproduced or ut ilized
in any for m w it hout t he wr it t en perm ission. Discuss t his quest ions paper at w w w .g a t e m e n t o r . co m .

www.engineeringonyourfingertips.ooo 22

More Notes Join us Telegram-: http//:t.me/allexammentor


www.engineeringonyourfingertips.ooo
ME-GATE-2015 PAPER| www.gateforum.com

rp 30 mm
rr 30 mm R
rq 0 Pr''
Pe 9000 250
C 1250
2
r
q rq2 rr2 302 02 302 Pr'

Pp'' Pr'' C
''
P p Crp 37.5 kN
Pr' Crr 37.5 kN
Pq'' 0

Resultant shear force


Due to symmetry stress in P Q R will be equal in magnitude

Pp Pr (3) 2 (37.5) 2 38.5648 kN


P 38.5648
340.987 MPa 341MPa
A (0.012) 2

53. A mixture of ideal gases has the following composition by mass:

N2 O2 CO2
60% 30% 10%

If the Universal gas constant is 8314 J/mol-K, the characteristic gas constant of the mixture
www. engi
(inJ/kg.K) is _________. neer ingonyour finger tips. ooo
Answer: 274.99
Universal gas constant
Exp: gas constant of mixture, R m
Average molar mass

100
Average molar mass 30.233 kg / kmol
60 30 10
28 32 44
8314
Rm 274.996J / kg K
30.233

54. A shaft of length 90 mm has a tapered portion of length 55 mm. The diameter of the taper is
80 mm at one end and 65 mm at the other. If the taper is made by tailstock set over method,
the taper angle and the set over respectively are

© All r ight s reser ved by GATE For um Educat ional Serv ices Pvt . Lt d. No part of t his book let m ay be reproduced or ut ilized
in any for m w it hout t he wr it t en perm ission. Discuss t his quest ions paper at w w w .g a t e m e n t o r . co m .

www.engineeringonyourfingertips.ooo 23

More Notes Join us Telegram-: http//:t.me/allexammentor


www.engineeringonyourfingertips.ooo
ME-GATE-2015 PAPER| www.gateforum.com

(A) 15°32′ and 12.16 mm (B) 15°32′ and 15.66 mm


(C) 11°22′ and 10.26 mm (D) 10°32′ and 14.46 mm
Answer: (A)
80 65
Exp: Rate of taper, T 0.27
55
T L 0.27 90
Set over= 12.15
2 2
Taper angle Tan 1 (0.27) =15.10

55. One side of a wall is maintained at 400 K and the other at 300 K. The rate of heat transfer
through the wall is 1000 W and the surrounding temperature is 25°C. Assuming no
generation of heat within the wall, the irreversibility (in W) due to heat transfer through the
wall is ________.
Answer: 248.33
Exp: Q = 1000W
T 25 273 298
T T
Q 1 Q 1 Qir 0
T1 T2
298 298
1000 1 1000 1 Qir 0
400 300
Qir 248.33W

www.
engi
neer
ingonyour
finger
tips.
ooo

© All r ight s reser ved by GATE For um Educat ional Serv ices Pvt . Lt d. No part of t his book let m ay be reproduced or ut ilized
in any for m w it hout t he wr it t en perm ission. Discuss t his quest ions paper at w w w .g a t e m e n t o r . co m .

www.engineeringonyourfingertips.ooo 24

More Notes Join us Telegram-: http//:t.me/allexammentor


www.engineeringonyourfingertips.ooo
ME-GATE-2014 PAPER-01| www.gateforum.com

Q. No. 1 – 5 Carry One Mark Each

1. Choose the most appropriate phrase from the options given below to complete the
following sentence.
The aircraft_______ take off as soon as its flight plan was filed.
(A) is allowed to (B) will be allowed to
(C) was allowed to (D) has been allowed to
Answer: (C)

2. Read the statements:


All women are entrepreneurs.
Some women are doctors
Which of the following conclusions can be logically inferred from the above statements?
(A) All women are doctors (B) All doctors are entrepreneurs
(C) All entrepreneurs are women (D) Some entrepreneurs are doctors
Answer: (D)

3. Choose the most appropriate word from the options given below to complete the following
sentence.
Many ancient cultures attributed disease to supernatural causes. However, modern science
has largely helped _________ such notions.
(A) impel (B) dispel (C) propel (D) repel
Answer: (B)

4. The statisticsw
ofww.
runse ngi ne
scored ine
aringon
series byyour
four finger
batsmentips
are .ooo in the following table,
provided
Who is the most consistent batsman of these four?

Batsman Average Standard deviation


K 31.2 5.21
L 46.0 6.35
M 54.4 6.22
N 17.9 5.90

(A) K (B) L (C) M (D) N


Answer: (A)

www.engineeringonyourfingertips.ooo
 India’s No.1 institute for GATE Training  1 Lakh+ Students trained till date  65+ Centers across India
1

More Notes Join us Telegram-: http//:t.me/allexammentor


www.engineeringonyourfingertips.ooo
ME-GATE-2014 PAPER-01| www.gateforum.com

Exp: If the standard deviation is less, there will be less deviation or batsman is more consistent

5. What is the next number in the series?


12 35 81 173 357 ____
Answer: 725

Exp: 12 35 81 173 357 ________

23 46 92 184 368
difference

357
⇒ 368
725
Q. No. 6 – 10 Carry One Mark Each

6. Find the odd one from the following group:


W,E,K,O I,Q,W,A F,N,T,X N,V,B,D
(A) W,E,K,O (B) I,Q,W,A (B) F,N,T,X (D) N,V,B,D
Answer: (D)
Exp: 1 Q W A
W E K O F N T X N V B D
8 6 4
8 6 4 8 6 4 8 6 2

Difference of position: D

7. For submitting tax returns, all resident males with annual income below Rs 10 lakh should fill
up Form P andww allwresident
.engi nfemales
eer ing ony
with our
incomefinger
below tips
Rs 8.
o ooshould fill up Form All
lakh
people with incomes above Rs 10 lakh should fill up Form R, except non residents with
income above Rs 15 lakhs, who should fill up Form S. All others should fill Form T. An
example of a person who should fill Form T is
(A) a resident male with annual income Rs 9 lakh
(B) a resident female with annual income Rs 9 lakh
(C) a non-resident male with annual income Rs 16 lakh
(D) a non-resident female with annual income Rs 16 lakh
Answer: (B)
Exp: Resident female in between 8 to 10 lakhs haven’t been mentioned.

www.engineeringonyourfingertips.ooo
 India’s No.1 institute for GATE Training  1 Lakh+ Students trained till date  65+ Centers across India
2

More Notes Join us Telegram-: http//:t.me/allexammentor


www.engineeringonyourfingertips.ooo
ME-GATE-2014 PAPER-01| www.gateforum.com

8. A train that is 280 metres long, travelling at a uniform speed, crosses a platform in 60 seconds
and passes a man standing on the platform in 20 seconds. What is the length of the platform
in metres?
Answer: 560
Exp: For a train to cross a person, it takes 20 seconds for its 280m.
So, for second 60 seconds. Total distance travelled should be 840. Including 280 train length
so length of plates =840-280=560
9. The exports and imports (in crores of Rs.) of a country from 2000 to 2007 are given in the
following bar chart. If the trade deficit is defined as excess of imports over exports, in which
year is the trade deficit 1/5th of the exports?

120
Exports Im ports
110
100
90
80
70
60
50
40
30
20
10
0 2006
2000 2001 2002 2003 2004 2005 2007

(A) 2005 (B) 2004 (C) 2007 (D) 2006


Answer: (D)
imports − exp orts 10 1
Exp: 2004, = =
exp orts 70 7
www.
engi
neer
ingonyour
finger
tips.
ooo
26 2
2 0 0 5, =
76 7
20 1
2 0 0 6, =
100 5
10 1
2007, =
100 11

10. You are given three coins: one has heads on both faces, the second has tails on both faces,
and the third has a head on one face and a tail on the other. You choose a coin at random and
toss it, and it comes up heads. The probability that the other face is tails is
(A) 1/4 (B) 1/3 (C) 1/2 (D) 2/3
Answer: (B)

www.engineeringonyourfingertips.ooo
 India’s No.1 institute for GATE Training  1 Lakh+ Students trained till date  65+ Centers across India
3

More Notes Join us Telegram-: http//:t.me/allexammentor


www.engineeringonyourfingertips.ooo
ME-GATE-2014 PAPER-01| www.gateforum.com

Q. No. 1 – 25 Carry One Mark Each

 1 3 0
1. Given that the determinant of the matrix  2 6 4  is -12, the determinant of the matrix
 −1 0 2 
 2 6 0
 4 12 8  is
 
 −2 0 4 
(A) -96 (B) -24 (C) 24 (D) 96
Answer: (A)
2 6 0 1 3 0
= ( 2) = 8 × ( −12 ) = − 96
3
Exp: 4 12 8 2 6 2
−2 0 4 −1 0 2

x − sin x
2. Lt is
1 − cos x
x→0

(A) 0 (B) 1 (C) 3 (D) not defined


Answer: (A)
x − sin x 0
Exp: lim =  
x →0 1 − cos x 0
1 − cos x  0 
Applying L. Hospital Rule, lim = 
x →0 sin x 0
sin x  0 
Once again, L. Hospital rule lim =  = 0
x →0 cos x  1 

The argumentwofwthe
w. engi ne eringo 1 +n
iy
3. complex number ,ourfi
where n
ig
=er
−t
ip
1, s.
is ooo
1− i
π π
( A) −π ( B) − ( C) ( D) π
2 2
Answer: (C)

Exp: Given z =
1+ i
⇒ z=
(1+ i )(1 + i )
1− i (1 − i )( 1+ i )
(1+ i )
2
1+ 2i + i 2 1 + 2i − 1
= = = = i
12 − i 2 1+1 2
Arg ( z ) = Arg ( i )
y π
= tan −1   = tan −1 ∞ =
x 2

www.engineeringonyourfingertips.ooo
 India’s No.1 institute for GATE Training  1 Lakh+ Students trained till date  65+ Centers across India
4

More Notes Join us Telegram-: http//:t.me/allexammentor


www.engineeringonyourfingertips.ooo
ME-GATE-2014 PAPER-01| www.gateforum.com

dx dy
4. The matrix form of the linear system = 3x − 5y and = 4x + 8y is
dt dt
d  x   3 −5  x  d x   3 8  x 
( A)  =   ( B)  =  
dt  y  4 8   y  dt  y  4 −5  y 

d  x  4 −5  x  d  x  4 8   x 
( C)  =   ( D)  =  
dt  y   3 8   y  dt  y   3 −5  y 
Answer: (A)
dx
Exp: Given that = 3x − 5y
dt
dy
= 4x + 8y
dt
d x   3 −5   x 
Matrix term   =    
dt y  4 8   y 

5. Which one of the following describes the relationship among the three vectors, ˆi + ˆj + kˆ ,
2iˆ + 3jˆ + kˆ and 5iˆ + 6ˆj + 4 kˆ ?
(A) The vectors are mutually perpendicular
(B) The vectors are linearly dependent
(C) The vectors are linearly independent
(D) The vectors are unit vectors
Answer: (B)
Exp: Given vectors are i + j + k, 2i + 3j + k and 5i + 6 j + k
1 1 1
2 3 1 = 0
5 6 1
wlinearly
∴ Vectors are ww. en gineer i
dependent. ngonyour
finger
tips.
ooo
6. A circular rod of length ‘L’ and area of cross-section ‘A’ has a modulus of elasticity ‘E’ and
coefficient of thermal expansion ‘ α ’. One end of the rod is fixed and other end is free. If the
temperature of the rod is increased by ∆T, then

(A) Stress developed in the rod is E α ∆T and strain developed in the rod is α ∆T
(B) Both stress and strain developed in the rod are zero
(C) Stress developed in the rod is zero and strain developed in the rod is α ∆T
(D) Stress developed in the rod is E α ∆T and strain developed in the rod is zero
Answer: (C)
Exp: Since one end of the rod is fixed and other is free to expand. Hence the Temperature stresses
δl α∆Tl
= 0 & ∈= = = α∆T
l l

www.engineeringonyourfingertips.ooo
 India’s No.1 institute for GATE Training  1 Lakh+ Students trained till date  65+ Centers across India
5

More Notes Join us Telegram-: http//:t.me/allexammentor


www.engineeringonyourfingertips.ooo
ME-GATE-2014 PAPER-01| www.gateforum.com

7. A metallic rod of 500mm length and 50mm diameter, when subjected to a tensile force of
100KN at the ends, experiences an increase in its length by 0.5 mm and a reduction in its
diameter by 0.015mm. The Poisson’s ratio of the rod material is _________
Answer: 0.29 to 0.31
Exp: l = 500 mm, d = 50 mm, p = 100 KN
δl = 0.5 mm, δl = 0.015
1 Lateral strain
Poisson 's Ratio   =
 m  Longitudinal strail
δd d 0.015 50
= = = 0.3.
δl l 0.5 500

8. Critical damping is the


(A) Largest amount of damping for which no oscillation occurs in free vibration
(B) Smallest amount of damping for which no oscillation occurs in free vibration
(C) Largest amount of damping for which the motion is simple harmonic in free vibration
(D) Smallest amount of damping for which the motion is simple harmonic in free vibration
Answer: (B)

9. A circular object of radius ‘r’ rolls without slipping on a horizontal level floor with the center
having velocity V. The velocity at the point of contact between the object and the floor is
(A) zero (B) V in the direction of motion
(C) V opposite to the direction of motion (D) V vertically upward from the floor
Answer: (A)
Exp: Velocity at point of contact = Rω
(R = Radius of point from Instantaneous centre)
∵ The instantaneous centre is at intersection of object and floor, hence radius R = 0
∴ Velocity at point is zero.
10. For the givenw ww. engi
statements: neer ingonyour f inger ti
ps. ooo
I. Mating spur gear teeth is an example of higher pair
II. A revolute joint is an example of lower pair
Indicate the correct answer.
(A) Both I and II are false (B) I is true and II is false
(C) I is false and II is true (D) Both I and II are true
Answer: (D)
Exp: Since higher pair has a line or point contact and lower pair has a surface of Area contact.
Hence both are true.
i..e,spur gear has line contact (Higher pair) and Revolute joint has surface contact (lower
pair).

www.engineeringonyourfingertips.ooo
 India’s No.1 institute for GATE Training  1 Lakh+ Students trained till date  65+ Centers across India
6

More Notes Join us Telegram-: http//:t.me/allexammentor


www.engineeringonyourfingertips.ooo
ME-GATE-2014 PAPER-01| www.gateforum.com

11. A rigid link PQ is 2 m long and oriented at 20O to the horizontal as shown in the figure. The
magnitude and direction of velocity VQ, and the direction of velocity VP are given. The
magnitude of VP ( in m / s ) at this instant is
VQ = 1m / s
45O
Q

20O
VP P

(A) 2.14 (B) 1.89 (C) 1.21 (D) 0.96


Answer: (D)
Exp: By Instantaneous center method; Say ω = angular velocity of rod
•I
vq = rq .ω ___ (1)
v p = rp .ω ___ ( 2 )
(1) ÷ ( 2 ) rq
V2 = ln β
vq rq rp
= ; vp = .v q
vp rp rq
45° rp
65°
rp rq •
Now by sine law, = 20°
sin ( 45 + 20 ) sin ( 70° )
Q
20 30°
rp sin ( 65° )
= 20°
rq sin ( 70° )
vp P
sin ( 65 )
so v p = v q = 0.96.
sin ( 70 )
12. Biot number signifies the ratio of
(A) Convective resistance in the fluid to conductive resistance in the sold
ww
(B) Conductive w. engi
resistance ne
in er
the ingo
solid tonyour fing
convective er tips.
resistance oo
in ofluid
the
(C) Inertia force to viscous force in the fluid
(D) Buoyancy force to viscous force in the fluid
Answer: (B)
Exp: Biot-number:
Biot number provides a way to compare the conduction resistance within a solid body to the
convection resistance external to that body (offered by the surrounding fluid) for heat
transfer:
hs Volume of the body
Bi = ; s=
k Surface area
Where‘s’ is a characteristic dimension of the solid
‘h’ is convective heat transfer coefficient
‘k’ is thermal conductivity of the body.

www.engineeringonyourfingertips.ooo
 India’s No.1 institute for GATE Training  1 Lakh+ Students trained till date  65+ Centers across India
7

More Notes Join us Telegram-: http//:t.me/allexammentor


www.engineeringonyourfingertips.ooo
ME-GATE-2014 PAPER-01| www.gateforum.com

13. The maximum theoretical work obtainable, when a system interacts to equilibrium with a
reference environment, is called
(A) Entropy (B) Enthalpy (C) Exergy (D) Rothalpy
Answer: (C)
Exp: Exergy (or) Available Energy:
The maximum portion of energy which could be converted into useful
work by ideal processes which reduce the system to dead state(a state in equilibrium with the
earth and its atmosphere).

14. Consider a two-dimensional laminar flow over a long cylinder as shown in the figure below.

2 3
U∞ 1
T∞ 4
TS

The free stream velocity is U ∞ and the free stream temperature T∞ is lower than the cylinder
surface temperature TS. The local heat transfer coefficient is minimum at point
(A) 1 (B) 2 (C) 3 (D) 4
Answer: (B)
Exp: For laminar flow, the heat transfer coefficient is minimum where the boundary layer
thickness is maximum and vice versa. For turbulent-region boundary layer thickness is
maximum at 3 but for laminar boundary layer thickness is maximum at 2 so minimum heat
transfer coefficient.
15. For a completely submerged body with centre of gravity ‘G’ and centre of buoyancy ‘B’, the
condition of stability will be
(A) G is located below B (B) G is located above B
(C) G and B are coincident (D) independent of the locations of G and B
Answer: (A) www. engi neer i
ngonyour finger tips. ooo
Exp: A body in a liquid is said to be stable, when given small displacement, it returns to its original
position.
Stability of completely submerged Bodies
W

B
G
Fn

The center of gravity ‘G’ is below t he center of Buoyancy ‘B’.

www.engineeringonyourfingertips.ooo
 India’s No.1 institute for GATE Training  1 Lakh+ Students trained till date  65+ Centers across India
8

More Notes Join us Telegram-: http//:t.me/allexammentor


www.engineeringonyourfingertips.ooo
ME-GATE-2014 PAPER-01| www.gateforum.com

16. In a power plant, water (density = 1000 kg/m3) is pumped from 80 KPa to 3 MPa. The pump
has an isentropic efficiency of 0.85. Assuming that the temperature of the water remains the
same, the specific work (in kJ/kg) supplied to the pump is
(A) 0.34 (B) 2.48 (C) 2.92 (D) 3.43
Answer: (D)
volume of fluid 1 1
Exp: Specific volume = = = = 10−3 m 3 kg
Mass of fluid ρ 1000
Isentropic compressor work
η=
Actual compressor work
V ( ∆P )
Actual compressor work =
η
10 × ( 3000 − 80 ) KPa
−3

=
0.85
2.92
= = 3.43 kJ kg.
0.85

17. Which one of the following is a CFC refrigerant?


(A) R744 (B) R290 (C) R502 (D) R718
Answer: (C)
Exp: Among all refrigerants R502 is the only CFC refrigerant.

18. The jobs arrive at a facility, for service, in a random manner. The probability distribution of
number of arrivals of jobs in a fixed time interval is
(A) Normal (B) Poisson (C) Erlang (D) Beta
Answer: (B)
Exp:
λ =1
0.04 λg
=o
www.
engi
neer
in 4nyour
finger
tips.
ooo
0.03
λ = 10 P
p ( x = K) 0.01

0.09
0.05

0.00
0 5 10 15
n
k
Poission distribution
Since arrival rates depends upon the time factor, so accordingly graph can be chosen from
Poisson distribution, but normal distribution expresses same result throughout.

www.engineeringonyourfingertips.ooo
 India’s No.1 institute for GATE Training  1 Lakh+ Students trained till date  65+ Centers across India
9

More Notes Join us Telegram-: http//:t.me/allexammentor


www.engineeringonyourfingertips.ooo
ME-GATE-2014 PAPER-01| www.gateforum.com

19. In exponential smoothening method, which one of the following is true?


(A) 0 ≤ α ≤ 1 and high value of α is used for stable demand
(B) 0 ≤ α ≤ 1 and high value of α is used for unstable demand
(C) α ≥ 1 and high value of α is used for stable demand
(D) α ≤ 0 and high value of α is used for unstable demand
Answer: (B)
Exp: 0≤α≤2
high value of ‘α’ means more weightage for immediate forecast.
Less value of ‘α’ means relatively less weightage for immediate forecast, or almost equal
weightage for all previous forecast.
Hence high value of forecast is only chosen when nature of demand is not reliable rather
unstable.

20. For machining a rectangular island represented by coordinates P(0,0), Q(100,0), R(100,50)
and (0,50) on a casting using CNC milling machine, an end mill with a diameter of 16 mm is
used.
The trajectory of the cutter centre to machine the island PQRS is
(A) (−8, −8), (108, −8), (108,58), (−8,58) , (−8, −8)
(B) (8,8), (94,8), (94,44), (8,44), (8,8)
(C) (−8,8), (94,0), (94,44), (8,44), (−8,8)
(D) (0,0), (100,0), (100,50), (50,0), (0,0)
y
Answer: (A)
Exp: End mill centre ≡ (0,0)
Since Radius of end mill is 8 mm
∴ call point ‘p’ ≡ -8, -8
Call point ‘Q’w≡ww. eng
(100+8, inee
-8+0) ≡r
ingonyour
(108,-8) f
inger
tips.
ooo
→x direction ( 0,50 ) ≡ S
Call point ‘R’ ≡ (108+0, 50+8) ≡ (108,58) R ≡ (100,50 )
→y direction
Call point ‘S’ ≡ (108-100 -2×8,58-0) ≡ (-8,58) 50

→-x direction
Call point ‘P’ ≡ (-8-0, 50-50-8) ≡ (-8,-8) x

→-y direction P ≡ ( 0,0 ) 100 Q ≡ (100,0 )

21. Which one of the following instruments is widely used to check and calibrate geometric
features of machine tools during their assembly?
(A) Ultrasonic probe (B) Coordinate Measuring Machine (CMM)
(C) Laser interferometer (D) Vernier callipers

www.engineeringonyourfingertips.ooo
 India’s No.1 institute for GATE Training  1 Lakh+ Students trained till date  65+ Centers across India
10

More Notes Join us Telegram-: http//:t.me/allexammentor


www.engineeringonyourfingertips.ooo
ME-GATE-2014 PAPER-01| www.gateforum.com

Answer: (C)
Exp: Geometric accuracy of the machine tools are generally checked by Laser interferometer, as it
is very cheap and easy to handle.

22. The major difficulty during welding of aluminium is due to its


(A) High tendency of oxidation (B) high thermal conductivity
(C) Low melting point (D) low density
Answer: (A)

23. The main cutting force acting on a tool during the turning (orthogonal cutting) operation of a
metal is 400 N. The turning was performed using 2 mm depth of cut and 0.1 mm/rev feed
rate. The specific cutting pressure (in N/mm²) is
(A) 1000 (B) 2000 (C) 3000 (D) 4000
Answer: (B)
FC
Exp: specific cutting energy =
b × t1
400
=
2 × 0.1
= 2000 N mm 2 .

24. The process of reheating the martensitic steel to reduce its brittleness without any significant
loss in its hardness is
(A) Normalising (B) annealing (C) quenching (D) tempering
Answer: (A)

25. In solid-state welding, the contamination layers between the surfaces to be welded are
removed by
(A) Alcohol www.
engi
neer
ingonyour
(B) f
inger
plastic ti
ps. ooo
deformation
(C) water jet (D) sand blasting
Answer: (B)

Q. No. 26 – 55 Carry Two Marks Each

1
26. The integral ∫ ( ydx − xdy )
c
is evaluated along the circle x 2 + y 2 =
4
traversed in counter

clockwise direction. The integral is equal to


π π π
( A) 0 ( B) − ( C) − ( D)
4 2 4
Answer: (C)

www.engineeringonyourfingertips.ooo
 India’s No.1 institute for GATE Training  1 Lakh+ Students trained till date  65+ Centers across India
11

More Notes Join us Telegram-: http//:t.me/allexammentor


www.engineeringonyourfingertips.ooo
ME-GATE-2014 PAPER-01| www.gateforum.com

1
Exp: Given integral ∫ ( ydx − xdy )
c
where C is x 2 + y 2 =
4
Applying Green’s theorem
 ∂N ∂M 
∫ Mdx − Ndy = ∫ ∫  ∂x −
c R
 dxdy
∂y 
where R is region included in c

∫ ydx − xdy = ∫ ∫ ( −1 − 1) dxdy


c R

1
= − 2 ∫∫ dxdy = − 2 × Re gion R = − 2 × area of circle with radius
R 2
−π
2
1
= − 2 ×π   =
2 2

d2y dy
27. If y = f(x) is solution of 2
= 0 with the boundary conditions y = 5 at x = 0, and = 2 at
dx dx
x = 10,f (15) = ________
Answer: 34 to 36
d2 y dy
Exp: Given 2
= 0, y = 5 at x = 0, = 2 at x = 10
dx dx
⇒ Auxiliary equation is m 2 = 0
⇒ m = 0, 0
y c = ( c1 + c 2 x ) e x = c1 + c2 x
yp = 0
General solution y = y c + y p ⇒ y = c1 + c 2 x
y = 5 at x = 0 ⇒ c1 = 5
www
dy.
engi
neer
ingonyour
finger
tips.
ooo
= 2 at x = 10 ⇒ 2 = c 2
dx
y = 5 + 2x
y (15 ) = 5 + 30 = 35

28. In the following table, x is a discrete random variable and p(x) is the probability density. The
standard deviation of x is

X 1 2 3
p(x) 0.3 0.6 0.1

(A) 0.18 (B) 0.36 (C) 0.54 (D) 0.6


Answer: (D)

www.engineeringonyourfingertips.ooo
 India’s No.1 institute for GATE Training  1 Lakh+ Students trained till date  65+ Centers across India
12

More Notes Join us Telegram-: http//:t.me/allexammentor


www.engineeringonyourfingertips.ooo
ME-GATE-2014 PAPER-01| www.gateforum.com

x 1 2 3
Exp: Given
p ( x ) 0.3 0.6 0.1
mean ( µ ) = exp ( x ) = 1× 0.3 + 2 × 0.6 + 3× 0.1
= 0.3 + 1.2 + 0.3
=1.8
E ( x 2 ) = Σx 2 P ( x )
=1× 0.3 + 4 × 0.6 + 9 × 0.1
= 0.3 + 2.4 + 0.9
= 3.6
Variance v ( x ) = E ( x 2 ) −µ 2 = 3.6 − (1.8)
2

S.D ( σ ) = + v ( x ) = + 3.6 − (1.8 ) = 0.36 = 0.6


2

29. Using the trapezoidal role, and dividing the interval of integration into three equal
+1
subintervals, the definite integral ∫ x dx is _____
−1

Answer: 1.10 to 1.12


+1
Exp: ∫−1
x dx
Let y = x
n = no. of subintervals = 3
x − no 1 − ( −1) 2
h= n = =
n 3 3
2 2 2
Values of x are, − 1, − 1+ , − 1 + 2   , −1 + 3  
3 3 3
1 1
x −1 1 −
www. engi
n3eer
in
3gony our
finger
tips.
ooo
1 1
y= x 1 1
3 3
xn
h
trape zodial rule = ∫ f ( x ) dx = 2  ( y
x0
o + y n ) + 2 ( y 1 + ....... + y n −1 ) 

1  1 1 
1
= ∫ x dx = 3 (1+1) + 2  3 + 3 
−1

1  4 1 10 10
=  2 + 3  = 3 × 3 = 9 = 1.1111
3  

30. The state of stress at a point is given by σ x = −6 MPa, σ y = 4 MPa, and τ xy = −8 MPa. The
maximum tensile stress (in MPa) at the point is ______________
Answer: 8.4 to 8.5

www.engineeringonyourfingertips.ooo
 India’s No.1 institute for GATE Training  1 Lakh+ Students trained till date  65+ Centers across India
13

More Notes Join us Telegram-: http//:t.me/allexammentor


www.engineeringonyourfingertips.ooo
ME-GATE-2014 PAPER-01| www.gateforum.com

σx + σ y  σx − σ y 
2

 + ( τ xy )
2
Exp: σ1 = + 
2  2 
−6 + 4  −6 − 4 
2

= +   + ( −8 )
2

2  2 
= 8.43.

31. A block R of mass 100 kg is placed on a block S of mass 150kg as shown in the figure. Block
R is tied to the wall by a mass less and inextensible string PQ. If the coefficient of static
friction for all surfaces is 0.4 the minimum force F (in KN) needed to move the block S is

P Q
R

S F

(A) 0.69 (B) 0.88 (C) 0.98 (D) 1.37


Answer: (D)
Exp: R N1 R N2
µR N1

F
µR N1
µR N2
100 kg 250 kg
F = 0.4 × 100 × 9.81 + 0.4 × 250 × 9.81
= 1.37 kN.

www.
engi
neer
ingonyour
finger
tips.
ooo
32. A pair of spur gears with module 5 mm and a centre distance of 450 mm is used for a speed
reduction of 5:1. The number of teeth on pinion is_______
Answer: 29 to 31
Exp: Given speed Ratio = 5:1
5 T2 d 2
= = ⇒ d 2 = 5d1
1 T1 d1
d1 + d 2
centre distance= = 450
2
⇒ d1 + d 2 = 900 ⇒ 5d1 + d1 = 900
⇒ d1 = 150
d1 150
m= ⇒ T1 = = 30.
T1 5

www.engineeringonyourfingertips.ooo
 India’s No.1 institute for GATE Training  1 Lakh+ Students trained till date  65+ Centers across India
14

More Notes Join us Telegram-: http//:t.me/allexammentor


www.engineeringonyourfingertips.ooo
ME-GATE-2014 PAPER-01| www.gateforum.com

33. Consider a cantilever beam, having negligible mass and uniform flexural rigidity, with length
0.01 m. The frequency of vibration of the beam, with a 0.5 kg mass attached at the free tip, is
100 Hz. The flexural rigidity (in N.m²) of the beam is _______
Answer: 0.064 to 0.067
FL3
Exp: S=
3EI
F 3EI
k= = 3
S l
3Ei m
k=
0.013
k = 3,000,000 EI
k 3000,000 EI
ωn = =
m 0.5
ωn = 2449.48 EI
ωn 2449.48 EI
fn = ⇒ 100 =
2π 2π
EI = 0.065 N.m .
2

34. An ideal water jet with volume flow rate of 0.05 m 3 / s strikes a flat plate placed normal to
its path and exerts a force of 1000 N. Considering the density of water as 1000 Kg/ m3 , the
diameter (in mm) of the water jet is _______
Answer: 56 to 57
Exp: V

Nozzle

V Flat plate
www.
engi
neer
ingonyour
finger
tips.
ooo

Jet of water

Given : Q = 0.05 m3 s V
Fx = 1000 N
ρ = 1000 kg m3
D=?
Fx = Rate of change of momentum in the direction of force

www.engineeringonyourfingertips.ooo
 India’s No.1 institute for GATE Training  1 Lakh+ Students trained till date  65+ Centers across India
15

More Notes Join us Telegram-: http//:t.me/allexammentor


www.engineeringonyourfingertips.ooo
ME-GATE-2014 PAPER-01| www.gateforum.com

mass
= × [initial velocity − Final velocity]
time
mass
= ρav, velocity of jet after striking is equal to zero
sec
 Q
∴ρaV ( V − 0 ) = Fx ∵ Q = aV ⇒ v = a 
 
Q2
1000 = 1000 × a ×
a2
Q 2 = a; a = ( 0.05 )
2

π 2
d = 2.5 × 10−3
4
d = 0.05641 m = 56.41 mm.

35. A block weighing 200 N is in contact with a level plane whose coefficients of static and
kinetic friction are 0.4 and 0.2, respectively. The block is acted upon by a horizontal force (in
Newton) P=10t, where t denotes the time in seconds. The velocity (in m/s) of the block
attained after 10 seconds is _______
Answer: 4.8 to 5.0 200 N
Exp: By 2nd law of Newton in x direction
P − µ k R = ma P
0
dv
⇒ P − µk R = m Zf y = 0
dt
2 t µR R = 200 N
1 2
⇒ v 2 − v1 = ∫ dv = ( P − µ k R ) dt
m t∫1
200 N

1
Free Body diagram
10 P
1
(10t − 0.2 × 200 ) dt
 ∫0
=
 200
 
 9.81  µ kR
v 2 − 0 = 4.90 m s.
www.
engi
neer
ingonyour
finger
tips.
o2
oo
36. A slider crank mechanism has slider of mass 10 kg, stroke of 0.2 m and rotates with a
uniform angular velocity of 10 rad/s. The primary inertia forces of the slider are partially
balanced by a revolving mass of 6 kg at the crank, placed at a distance equal to crank radius.
Neglect the mass of connecting rod and crank. When the crank angle (with respect to slider
axis) is 30°, the unbalanced force (in Newton) normal to the slider axis is _______
Answer: 29 to 31
0.2 A
Exp: r= = 0.1 m = 6kg
2
130°
F = mr (102 ) sin θ
m

= 6 × 0.1 × 100 × sin 30°


m F
= 30 N

www.engineeringonyourfingertips.ooo
 India’s No.1 institute for GATE Training  1 Lakh+ Students trained till date  65+ Centers across India
16

More Notes Join us Telegram-: http//:t.me/allexammentor


www.engineeringonyourfingertips.ooo
ME-GATE-2014 PAPER-01| www.gateforum.com

37. An offset slider-crank mechanism is shown in the figure at an instant. Conventionally, the
Quick Return Ratio (QRR) is considered to be greater than one. The value of QRR is
_______
40 mm
20 mm
10 mm

Answer: 1.2 to 1.3


Exp:
AB = stroke length
AO = 40 − 20 = 20 A B
BO = 40 + 20 = 60 C
∠AOB = ∠BOC − ∠AOC 10 mm
O
= 80.41 − 60
= 20.41
180 + φ
QRR = = 1.255.
180 − φ
Q.38 A rigid uniform rod AB of length L and mass m is hinged at C such that AC = L/3, CB = 2L/3.
Ends A and B are supported by springs of spring constant k. The natural frequency of the
system is given by

k k

L/3 2L / 3

k www.
engi
neker
ingonyour
fin2k
gert
ips.
ooo 5k
( A) ( B) ( C) ( D)
2m m m m
Answer: (D)

39. A hydrodynamic journal bearing is subject to 2000 N load at a rotational speed of 2000 rpm.
Both bearing bore diameter and length are 40 mm. If radial clearance is 20 µm and bearing is
lubricated with an oil having viscosity 0.03 Pa.s, the Sommerfeld number of the bearing is
_______
Answer: 0.75 to 0.85
 r  µDS
2

Exp: S= 
c P

www.engineeringonyourfingertips.ooo
 India’s No.1 institute for GATE Training  1 Lakh+ Students trained till date  65+ Centers across India
17

More Notes Join us Telegram-: http//:t.me/allexammentor


www.engineeringonyourfingertips.ooo
ME-GATE-2014 PAPER-01| www.gateforum.com

d = 40 mm ⇒ r = 20 mm

c = 20 µm ⇒ c = 20 × 10−3 mm

µ = 0.03 PaS ⇒ µ = 0.03 × 10−6 MPa.S

2000
DS = 2000 rpm ⇒ DS = rps
60
p 2000
P= = = 1.25 MPa
l × d 40 × 40
2000
2 0.03 × 10−6 ×
 20  60 = 0.8.
S= −3 
×
 20 × 10  1.25

40. A 200 mm long, stress free rod at room temperature is held between two immovable rigid
walls. The temperature of the rod is uniformly raised by 250°C. If the Young’s modulus and
coefficient of thermal expansion are 200 GPa and 1×10-5 /OC, respectively, the magnitude of
the longitudinal stress (in MPa) developed in the rod is _______

Answer: 499 to 501


Exp: l = 200, ∆T = 250°C, α = 1 × 10−5 
C
E = 200 GPa = 200 × 10 MPa3

σ = α∆TE
= 1 × 10−5 × 250 × 200 × 103
= 500 MPa.

41. 1.5 kg of water is in saturated liquid state at 2 bar (vf = 0.001061 m3/kg, uf = 504.0 kJ/kg, hf
= 505 kJ/kg). Heat is added in a constant pressure process till the temperature of water
w(wvw=. engi n
me e rin,g
uo =n your fingehr t=i
p s.ooo
3
reaches 400°C 1.5493 / Kg 2967.0 kJ/kg, 3277.0 kJ/kg). The heat added (in
kJ) in the process is _______
Answer: 4155 to 4160
Exp: Given, m = 1.5 kg
h1 = h f = 505 kJ kg h 2 = 3277.0
From Ist Law,
dQ = du + pdv = dh − vdp
dQ = dh ( as vdp = 0 )
Q add = dQ = m ( h 2 − h1 ) = ( 3277.0 − 505 ) × 1.5
Q added = 4158 kJ.

www.engineeringonyourfingertips.ooo
 India’s No.1 institute for GATE Training  1 Lakh+ Students trained till date  65+ Centers across India
18

More Notes Join us Telegram-: http//:t.me/allexammentor


www.engineeringonyourfingertips.ooo
ME-GATE-2014 PAPER-01| www.gateforum.com

42. Consider one dimensional steady state heat conduction across a wall (as shown in figure
below) of thickness 30 mm and thermal conductivity 15 W/m.K. At x = 0, a constant heat
flux, q" = 1 × 105 W/m2 is applied. On the other side of the wall, heat is removed from the
wall by convection with a fluid at 25OC and heat transfer coefficient of 250 W/m2.K. The
temperature (in OC), at x = 0 is _______

T1
q = 1 × 10 5 W / m 2

T2
T∞ = 25O C
Answer: 620 to 630
x
Τ − T∞ x=0
Exp: Q=
L 1
+
kA hA
Q T − T∞
q= =
A L+1
k h
T − 25
1 × 105 =
30 × 10−3 1
+
15 250
600 = T − 25°C
T = 625°C.

43. Water flows through a pipe having an inner radius of 10 mm at the rate of 36 kg/hr at 25OC.
The viscosity of water at 25OC is 0.001 kg/m.s. The Reynolds number of the flow is _______
Answer: 635 to 638
Exp: given Q = 36 kg hr
www.
engi
neer
ingonyour
finger
tips.
ooo
1m3 hr = 1000 kg hr
so converting kg hr to m3 s
Q = 10 −5 m3 s
ρVD ρD Q
Re = = ×
µ µ A
ρD Q 4ρQ 4 × 1000 × 10−5
= × = =
µ π D 2 µDπ 0.001 × 20 × 10−3 × π
4
R e = 636.62.

44. For a fully developed flow of water in a pipe having diameter 10 cm, velocity 0.1 m/s and
kinematic viscosity 10-5 m2/s, the value of Darcy friction factor is _______
Answer: 0.06 to 0.07

www.engineeringonyourfingertips.ooo
 India’s No.1 institute for GATE Training  1 Lakh+ Students trained till date  65+ Centers across India
19

More Notes Join us Telegram-: http//:t.me/allexammentor


www.engineeringonyourfingertips.ooo
ME-GATE-2014 PAPER-01| www.gateforum.com

Exp: Given ,D = 10 cm = 0.1 m


V = 0.1 m/s
ν = 10-5 m2/s
VD 0.1 × 0.1
Re = =
ν 10−5
R e = 1000
∴ flow is laminar
64 64
Darcy friction factor = ( for laminar flow ) = = 0.064.
Re 1000
45. In a simple concentric shaft-bearing arrangement, the lubricant flows in the 2 mm gap
between the shaft and the bearing. The flow may be assumed to be a plane Couette flow with
zero pressure gradient. The diameter of the shaft is 100 mm and its tangential speed is 10 m/s.
The dynamic viscosity of the lubricant is 0.1 kg/m.s. The frictional resisting force (in
Newton) per 100 mm length of the bearing is _______
Answer: 15 to 16
du
Exp: τ w = τcylinder = µ
dr
du
F = A×µ
dr u (r)
πDµ [ u t − u w ] 100
=
( ∆t ) mm

u t = tangential velocity
u w = velocity at bearing 2 mm
0.1 [10 − 0]
F = π × 0.1 × 0.1×
2 × 10−3
F = 15.707 N.

46.
www.
engi
neer
ingonyour
finger
tips.
ooo
The non-dimensional fluid temperature profile near the surface of a convectively cooled flat
TW − T
2
y  y
plate is given by = a + b + c   , where y is measured perpendicular to the plate,
TW − T∞ L  L
L is the plate length, and a, b and c are arbitrary constants. TW and T∞ are wall and ambient
temperatures, respectively. If the thermal conductivity of the fluid is k and the wall heat flux
q′′ L
is q′′ , the Nusselt number Nu = is equal to
Tw − T∞ k

(A) a (B) b (C) 2c (D) ( b + 2c )


Answer: (B)
Tw − T
2
by y
Exp: =a+ + c 
Tw − T∞ L L

www.engineeringonyourfingertips.ooo
 India’s No.1 institute for GATE Training  1 Lakh+ Students trained till date  65+ Centers across India
20

More Notes Join us Telegram-: http//:t.me/allexammentor


www.engineeringonyourfingertips.ooo
ME-GATE-2014 PAPER-01| www.gateforum.com

 by y 
2

T = Tw + ( T∞ − Tw ) a + + c  
 L  L  
dT  b 2Cy 
q′′ = − k = − k ( T∞ − Tw )  + 2 
dy L L 
at y = 0
q′′ L
⇒ = b = Nu
( Tw − T∞ ) k
Nu = b.

47. In an air-standard Otto cycle, air is supplied at 0.1 MPa and 308 K. The ratio of the specific
heats (γ) and the specific gas constant (R) of air are 1.4 and 288.8 J/kg.K, respectively. If the
compression ratio is 8 and the maximum temperature in the cycle is 2660 K, the heat (in
kJ/kg) supplied to the engine is _______
Answer: 1400 to 1420
Exp: Otto cycle

3
3

Qin 4 T
P
2 Q out 2
4
1
1

V
S
www.
engi
neer
ingonyour
finger
tips.
ooo
(1-2) Isentropic compression
Given :
PV r = constant
P1 = 0.1 MPa
T2
= (r)
γ−1
T1 = 308 K
T1
r = 1.4
T2 = 308 × 80.4
R = 0.2888 kJ kg.k
= 698.40 k
T3 = 2660 K
(2-3) Isochoric Heat addition process

www.engineeringonyourfingertips.ooo
 India’s No.1 institute for GATE Training  1 Lakh+ Students trained till date  65+ Centers across India
21

More Notes Join us Telegram-: http//:t.me/allexammentor


www.engineeringonyourfingertips.ooo
ME-GATE-2014 PAPER-01| www.gateforum.com

CP
= 1.4
CV
R = 0.2888 kJ kg
CP − CV = 0.2888
C 
CV  P − 1 = 0.2888
 CV 
0.2888
CV =
1.4 − 1
0.722 kJ kg
Qin = CV ( T3 − T2 ) = 0.722 ( 2660 − 698.40 ) = 1416.27 kJ kg

48. A reversible heat engine receives 2 kJ of heat from a reservoir at 1000 K and a certain
amount of heat from a reservoir at 800 K. It rejects 1 kJ of heat to a reservoir at 400 K. The
net work output (in kJ) of the cycle is
(A) 0.8 (B) 1.0 (C) 1.4 (D) 2.0
Answer: (C)
Exp:
1000 K 800 K

Q1 = 2 kJ Q2 = ?

H.E W N = ( Q1 + Q 2 ) − Q 3

Q 3 = 1 kJ

www.
engi
neer
ingonyour
finger
tips.
ooo
400 K

We know that for reversible heat engine change in entropy is always zero
That is ∆S = 0
Q3  Q1 Q 2 
− + =0
T3  T1 T2 
1 2 Q
− − 2 =0
400 1000 800
Q 2 = 0.4 kJ
WN = ( Q1 + Q 2 ) − Q3
= ( 2 + 0.4 ) − 1 = 1.4 kJ.

www.engineeringonyourfingertips.ooo
 India’s No.1 institute for GATE Training  1 Lakh+ Students trained till date  65+ Centers across India
22

More Notes Join us Telegram-: http//:t.me/allexammentor


www.engineeringonyourfingertips.ooo
ME-GATE-2014 PAPER-01| www.gateforum.com

49. An ideal reheat Rankine cycle operates between the pressure limits of 10 KPa and 8 MPa,
with reheat being done at 4 MPa. The temperature of steam at the inlets of both turbines is
500°C and the enthalpy of steam is 3185 kJ/kg at the exit of the high pressure turbine and
2247 kJ/kg at the exit of low pressure turbine. The enthalpy of water at the exit from the
pump is 191 kJ/kg .Use the following table for relevant data.

Superheated steam Pressure v h s


temperature (MPa) (m 3
/ kg ) ( kJ / kg ) ( kJ / kg.K )
(OC) 4 0.08644 3446 7.0922
500 4 0.04177 3399 6.7266
Disregarding the pump work, the cycle efficiency (in percentage) is _______
Answer: 40 to 42
Exp: w HP = h 2 − h 3 8 MPa
w LP = h 4 − h 5 4 MPa
2
net work T 10 MPa
w T = w HP + w LP 1 4
= ( h 2 − h3 ) + ( h4 − h5 )
3
Given:
h1 = 191 kJ kg 6 5
h 2 = 3399 kJ kg
h 3 = 3185 kJ kg
h 4 = 3446 kJ kg S
h 5 = 2247 kJ kg
w net
ηcycle =
Qadded
w1 − w pump
ηcycle = www.
engi
neer
ingonyour
finger
tips.
ooo
Qadd

ηcycle =
( h 2 − h3 ) + ( h 4 − h5 )
( h 2 − h1 ) + ( h 4 − h 3 )
ηcycle = 0.407 = 40.7%.

50. Jobs arrive at a facility at an average rate of 5 in an 8 hour shift. The arrival of the jobs
follows Poisson distribution. The average service time of a job on the facility is 40 minutes.
The service time follows exponential distribution. Idle time (in hours) at the facility per shift
will be
5 14 7 10
( A) ( B) ( C) ( D)
7 3 5 3
Answer: (B)

www.engineeringonyourfingertips.ooo
 India’s No.1 institute for GATE Training  1 Lakh+ Students trained till date  65+ Centers across India
23

More Notes Join us Telegram-: http//:t.me/allexammentor


www.engineeringonyourfingertips.ooo
ME-GATE-2014 PAPER-01| www.gateforum.com

Exp: Arrival Rate = 5 jobs in 8 hrs


Service time = 40 min/Job
200 10
∴ Total service time = 40 × 5 = 200 min = = hrs
60 3
10 24 − 10 14
∴ Idle Time shift = 8 − = = hrs.
3 3 3

51. A metal rod of initial length L0 is subjected to a drawing process. The length of the rod at any
( )
instant is given by the expression, L ( t ) = Lo 1 + t 2 where t is the time in minutes. The true
−1
strain rate (in min ) at the end of one minute is _______
Answer: 0.9 to 1.1
l
Exp: ε = ln i
l0
dε l0 1 dli 2t
= × =
dt li l0 dt (1 + t n )
dε 2 × 1
= = 1.
dt 1 + 1

52. During pure orthogonal turning operation of a hollow cylindrical pipe, it is found that the
thickness of the chip produced is 0.5 mm. The feed given to the zero degree rake angle tool is
0.2 mm/rev. The shear strain produced during the operation is _______
Answer: 2.8 to 3.0
0.2 t1
Exp: Chip thickness ratio r = =
0.5 t 2
r cos α
tan φ = = 0.4
1 − r sin α
φ = 21.8°
Shear strain = cot φ + tan ( φ − α ) = cot 21.8 + tan ( 21.8 − α ) = 2.9.
www.
engi
neer
ingonyour
finger
tips.
ooo

53. For the given assembly: 25 H7/g8, match Group A with Group B
Group A Group B
(P) H (I) Shaft Type
(Q) IT8 (II) Hole Type
(R) IT7 (III) Hole Tolerance Grade
(S) g (IV) Shaft Tolerance Grade
(A) P-I, Q-III, R-IV, S-II (B) P-I, Q-IV, R-III, S-II
(C) P-II, Q-III, R-IV, S-I (D) P-II, Q-IV, R-III, S-I
Answer: (D)
Exp: H7 is for hole where 7 indicates its tolerance grade
g8 is for shaft where 8 indicates its tolerance grade

www.engineeringonyourfingertips.ooo
 India’s No.1 institute for GATE Training  1 Lakh+ Students trained till date  65+ Centers across India
24

More Notes Join us Telegram-: http//:t.me/allexammentor


www.engineeringonyourfingertips.ooo
ME-GATE-2014 PAPER-01| www.gateforum.com

54. If the Taylor’s tool life exponent n is 0.2, and the tool changing time is 1.5 min, then the tool
life (in min) for maximum production rate is _______
Answer: 5.9 to 6.1
1 − n  1 − 0.2
Exp: Topt =  × TC  = × 1.5 = 6 min.
 n  0.2

55. An aluminium alloy (density 2600 kg/m3) casting is to be produced. A cylindrical hole of 100
mm diameter and 100 mm length is made in the casting using sand core (density 1600
kg/ m3 ). The net buoyancy force (in Newton) acting on the core is _______
Answer: 7 to 8
Exp: d = 1600 kg m3
ρ = 2600 kg m 3
Net buouancy force = weight of liquid displaced − weight of solid body
= v × ρ × g − v × d × g = vg ( ρ − d )
π 2
= d L × g (ρ − d )
4
π
= 0.12 × 0.1 × 9.81 ( 2600 − 1600 ) = 7.7N.
4

www.
engi
neer
ingonyour
finger
tips.
ooo

www.engineeringonyourfingertips.ooo
 India’s No.1 institute for GATE Training  1 Lakh+ Students trained till date  65+ Centers across India
25

More Notes Join us Telegram-: http//:t.me/allexammentor


www.engineeringonyourfingertips.ooo
ME-GATE-2014 PAPER-02| www.gateforum.com

Q. No. 1 – 5 Carry One Mark Each

1. Choose the most appropriate word from the options given below to complete the following
sentence.
Communication and interpersonal skills are_____ important in their own ways.
(A) each (B) both (C) all (D) either
Answer: (B)

2. Which of the options given below best completes the following sentence?
She will feel much better if she ________________.
(A) will get some rest (B) gets some rest
(C) will be getting some rest (D) is getting some rest
Answer: (B)

3. Choose the most appropriate pair of words from the options given below to complete the
following sentence.
She could not _____ the thought of _________ the election to her bitter rival.
(A) bear, loosing (B) bare, loosing (C) bear, losing (D) bare, losing
Answer: (C)

4. A regular die has six sides with numbers 1 to 6 marked on its sides. If a very large number of
throws show the following frequencies of occurrence: 1 → 0.167; 2 → 0.167; 3 → 0.152; 4 →
0.166; 5 → 0.168; 6 → 0.180. We call this die
(A) irregular (B) biased (C) Gaussian (D) insufficient
Answer: (B) www.
engi
neer
ingonyour
finger
tips.
ooo
Exp: For a very large number of throws, the frequency should be same for unbiased throw. As it
not same, then the die is baised.

5. Fill in the missing number in the series.


2 3 6 15 ___ 157.5 630
Answer: 45
Exp:
2 3 6 15 45 157.5 630

1.5 2 2.5 3 3.5 4

2nd number
is in increa sin g order as shown above
1st number

www.engineeringonyourfingertips.ooo
 India’s No.1 institute for GATE Training  1 Lakh+ Students trained till date  65+ Centers across India
1

More Notes Join us Telegram-: http//:t.me/allexammentor


www.engineeringonyourfingertips.ooo
ME-GATE-2014 PAPER-02| www.gateforum.com

Q. No. 6 – 10 Carry One Mark Each

6. Find the odd one in the following group


Q,W,Z,B B,H,K,M W,C,G,J M,S,V,X
(A) Q,W,Z,B (B) B,H,K,M (C) W,C,G,J (D) M,S,V,X
Answer: (C)
Exp: a W Z B
B H K N
17 23 26 2
W C G J M S V X
6 3 2 6 4 3 6 3 2
6 3 2

7. Lights of four colors (red, blue, green, yellow) are hung on a ladder. On every step of the
ladder there are two lights. If one of the lights is red, the other light on that step will always
be blue. If one of the lights on a step is green, the other light on that step will always be
yellow. Which of the following statements is not necessarily correct?
(A) The number of red lights is equal to the number of blue lights
(B) The number of green lights is equal to the number of yellow lights
(C) The sum of the red and green lights is equal to the sum of the yellow and blue lights
(D) The sum of the red and blue lights is equal to the sum of the green and yellow lights
Answer: (D)

8. The sum of eight consecutive odd numbers is 656. The average of four consecutive even
numbers is 87. What is the sum of the smallest odd number and second largest even number?
Answer: 163
Exp: Eight consecutive odd number =656
a-6, a-1, a-2, a ,a+2 ,a+4, a+6
a+8=656
a=81
www.
Smallest m=75 engi
ne
… er
(1) i
ngonyour
finger
tips.
ooo
Average consecutive even numbers
a −2+a +a +2+a +4
⇒ = 87
4
⇒ a = 86
Second largest number =88
1+2=163

9. The total exports and revenues from the exports of a country are given in the two charts
shown below. The pie chart for exports shows the quantity of each item exported as a
percentage of the total quantity of exports. The pie chart for the revenues shows the
percentage of the total revenue generated through export of each item. The total quantity of
exports of all the items is 500 thousand tonnes and the total revenues are 250 crore rupees.
Which item among the following has generated the maximum revenue per kg?

www.engineeringonyourfingertips.ooo
 India’s No.1 institute for GATE Training  1 Lakh+ Students trained till date  65+ Centers across India
2

More Notes Join us Telegram-: http//:t.me/allexammentor


www.engineeringonyourfingertips.ooo
ME-GATE-2014 PAPER-02| www.gateforum.com
Exports Revenues

Item 6 Item 1
Item 1
Item 6
16% 11%
12%
Item5 Item2 19%
Item 2
12% 20% Item 5
20%
Item3
Item4 20%
19% Item3
22% Item 4
6% 23%

(A) Item 2 (B) Item 3 (C) Item 6 (D) Item 5


Answer: (D)
Exp: Item:2 Item:3
20 23 × 250 × 107
× 250 × 107
100 19 × 500 × 103
20
× 500 × 103
100 1.2 = Item 3
0.5 × 104 = 5 × 103 1 = Item 2
Item: 6 Item:5
19 20 5
= 1.18 = Item 6 = = 1.6 ⇒ 1.6 = Item 5
16 12 3

10. It takes 30 minutes to empty a half-full tank by draining it at a constant rate. It is decided to
simultaneously pump water into the half-full tank while draining it. What is the rate at which
water has to be pumped in so that it gets fully filled in 10 minutes?
(A) 4 times the draining rate (B) 3 times the draining rate
(C) 2.5 times the draining rate (D) 2 times the draining rate
Answer: (A) www. engi neeringonyour f inger tips. ooo
Exp: Vhalf = 30(s) drawing rate = s
Total volume =60 S tank
(s1 )(10) − (s)10 = 30s
s1 (s) − s = 3s
s1 = 4s
s1 = 4drawing rate

www.engineeringonyourfingertips.ooo
 India’s No.1 institute for GATE Training  1 Lakh+ Students trained till date  65+ Centers across India
3

More Notes Join us Telegram-: http//:t.me/allexammentor


www.engineeringonyourfingertips.ooo
ME-GATE-2014 PAPER-02| www.gateforum.com

Q. No. 1 – 25 Carry One Mark Each

 −5 2 
1. One of the eigenvectors of the matrix   is
 −9 6 
 −1  −2  2 1
( A)   ( B)   ( C)   ( D)  
1 9  −1 1
Answer: D
 −5 2 
Exp: Eigen values of the matrix   are 4, -3
 −9 6 
∴ the eigen vector corresponding to eigen vector λ is Ax = λx (verify the options)

1
  is eigen vector corresponding to eigen value λ = 3
1

 e 2x − 1 
2. Lt  is equal to
x → 0 sin ( 4x ) 
 
(A) 0 (B) 0.5 (C) 1 (D) 2
Answer: B
e 2x − 1  0 
Exp: lim is  
x →0 sin 4x  0 
2e 2x
So, Applying L-Hospital Rule, lim
x →0 4cos 4 x
2
= = 0.5
4

Curl of vectorwFw=w
x.e
znig−ine e
zjr+i
n g2o
z3n
kˆyour
finger
tips.
ooo
 2 2ˆ 2 ˆ
3. 2xy 2y is
(A) ( 4yz 3
)
+ 2xy 2 i + 2x 2 zjˆ − 2y 2 zkˆ (B) ( 4yz 3
)
+ 2xy 2 ˆi + 2x 2 zjˆ − 2y 2 zkˆ

(C) 2xz 2 ˆi − 4xyzjˆ + 6y 2 z 2 kˆ (D) 2xz 2 ˆi + 4xyzjˆ + 6y 2 z 2 kˆ


Answer: A

Exp: Given F = x 2 z 2i 2xy 2 zJ + 2y 2 z3 k
i j k
  ∂ ∂ ∂
curl F = ∇× F =
2x 2y 2z
x 2 z2 −2xy z 2y 2 z 3
2

= i ( 4yz3 + 2xy 2 ) − j ( 0 − 2x 2 z ) + k ( −2y 2 z − 0 )


= ( 4yz 2 + 2xy 2 + 2x 2 zj) − 2y 2 z k

www.engineeringonyourfingertips.ooo
 India’s No.1 institute for GATE Training  1 Lakh+ Students trained till date  65+ Centers across India
4

More Notes Join us Telegram-: http//:t.me/allexammentor


www.engineeringonyourfingertips.ooo
ME-GATE-2014 PAPER-02| www.gateforum.com

4. A box contains 25 parts of which 10 are defective. Two parts are being drawn simultaneously
in a random manner from the box. The probability of both the parts being
7 42 25 5
( A) ( B) ( C) ( D)
20 125 29 9
Answer: A
Exp: Two parts can be selected in the following way 15C 2 × 10 C 0
Good Bad
15 10
2 0
1 1
0 2
Total no. of parts = 25.
Two parts can e selected in 25C 2 ways
15C 2 ×10Co 105 7
Required probability = = =
25C 2 300 20

5. The best approximation of the minimum value attained by e − x sin (100x ) for x ≥ 0 is _______
Answer: 1.00 to – 0.94

6. A steel cube, with all faces free to deform, has Young’s modulus, E, Poisson’s ratio v, and
coefficient of thermal expansion, α . The pressure (hydrostatic stress) developed within the
cube, when it is subjected to a uniform increase in temperature, ∆ T, is given by
α ( ∆T ) E α ( ∆T ) E α ( ∆T ) E
(A) 0 ( B) (C) − (D)
1 − 2ν 1 − 2ν 3 (1 − 2ν )
Answer: A
Exp: www
Since all the faces .efree
are ngi ne
to ering
expand on
the your
stressesfing
due er
to ti
ps. ooorise is equal to 0.
temperature

7. A two member truss ABC is shown in the figure. The force (in kN) transmitted in member AB
is _______

A 1m B

10 kN
0.5m

www.engineeringonyourfingertips.ooo
 India’s No.1 institute for GATE Training  1 Lakh+ Students trained till date  65+ Centers across India
5

More Notes Join us Telegram-: http//:t.me/allexammentor


www.engineeringonyourfingertips.ooo
ME-GATE-2014 PAPER-02| www.gateforum.com

Answer: 18 to 22
Exp: FBD of joint B
FAB
 0.5  •B
θ = Tan −1   = 26.5
 1  θ
ΣV = 0
FBC sin θ 10 kN
= FBC
10 kN
FBC cos θ FAB
10
FAB =
Tan 26.2
= 20 kN.

8. A 4-bar mechanism with all revolute pairs has link lengths lf = 20 mm, lin = 40 mm, lco = 50
mm and lout = 60 mm. The suffixes 'f', 'in', 'co' and 'out' denote the fixed link, the input link,
the coupler and output link respectively. Which one of the following statements is true about
the input and output links?
(A) Both links can execute full circular motion
(B) Both links cannot execute full circular motion
(C) Only the output link cannot execute full circular motion
(D) Only the input link cannot execute full circular motion
Answer: A
Exp: S+l<P+Q
20+60<40+50⇒80<90
If smaller link is fixed both input and Output link execute full circular motion.

9. In vibration isolation, which one of the following statements is NOT correct regarding
Transmissibility (T)?
(A) T is nearly unity at small excitation frequencies
(B) T can be always reduced by using higher damping at any excitation frequency
(C) T is unitywatw w.
the engi neratio
frequency er in gon
of 2your f
inger
tips.
ooo
(D) T is infinity at resonance for undamped systems
Answer: B

10. In a structure subjected to fatigue loading, the minimum and maximum stresses developed in
a cycle are 200 MPa and 400 MPa respectively. The value of stress amplitude (in MPa) is
_______
Answer: 99 to 101

Exp:
( σmax − σmin )
Stress Amplitude
2
400 − 200
= = 100.
2

www.engineeringonyourfingertips.ooo
 India’s No.1 institute for GATE Training  1 Lakh+ Students trained till date  65+ Centers across India
6

More Notes Join us Telegram-: http//:t.me/allexammentor


www.engineeringonyourfingertips.ooo
ME-GATE-2014 PAPER-02| www.gateforum.com

11. A thin plate of uniform thickness is subject to pressure as shown in the figure below

y y
x z

Under the assumption of plane stress, which one of the following is correct?
(A) Normal stress is zero in the z-direction
(B) Normal stress is tensile in the z-direction
(C) Normal stress is compressive in the z-direction
(D) Normal stress varies in the z-direction
Answer: A
Exp: For a plane stress criteria.
Normal stress in Z direction = 0.

12. For laminar forced convection over a flat plate, if the free stream velocity increases by a
factor of 2, the average heat transfer coefficient
(A) remains same (B) decreases by a factor of 2
(C) rises by a factor of 2 (D) rises by a factor of 4
Answer: C
Exp: For laminar flow,
Nu = 0.664 ( Re ) ( Pr )
0.5 0.33

0.5
hL  ρVD 
= 0.664  ( Pr )
0.33

k  µ 
www.
engi
neer
ingonyour
finger
tips.
ooo
h ∝ V 0.5 ; h ∝ V.
So when free stream velocity increases by a factor of 2, then the average heat transfer
coefficient rises by a factor of 2.

13. The thermal efficiency of an air-standard Brayton cycle in terms of pressure ratio rp and
( )
γ = c p / c p is given by
1 1 1
( A) ( B) ( C)
1
1− r −1
1− 1− ( D) 1− ( γ −1) / γ
r
p rpr r 1/ r
p rp
Answer: D
1
Exp: Thermal efficiency of air standard efficiency = 1 − γ −1

( rp ) γ

www.engineeringonyourfingertips.ooo
 India’s No.1 institute for GATE Training  1 Lakh+ Students trained till date  65+ Centers across India
7

More Notes Join us Telegram-: http//:t.me/allexammentor


www.engineeringonyourfingertips.ooo
ME-GATE-2014 PAPER-02| www.gateforum.com


14. For an incompressible flow field, v , which one of the following conditions must be satisfied?
 
( A ) ∇.v = 0 ( B) ∇ × v = 0
∂V 
( ) ( )
  
( C) v . ∇ × v = 0 ( D ) + V .∇ V = 0
∂t
Answer: A
Exp: Incompressible flow condition

∇.V = 0

15. A pure substance at 8 MPa and 400 °C is having a specific internal energy of 2864 kJ/kg and
a specific volume of 0.03432 m3 / kg . Its specific enthalpy (in kJ/kg) is _______
Answer: 3135 to 3140
Exp: h = u+pv
h = 2864+ (8000 kPa) × (0.03432)
= 2864 + 274.56 = 3138.56 = 3139

16. In a heat exchanger, it is observed that ∆T1 = ∆T2 , where ∆T1 is the temperature difference
between the two single phase fluid streams at one end and ∆T2 is the temperature difference
at the other end. This heat exchanger is
(A) a condenser (B) an evaporator
(C) a counter flow heat exchanger (D) a parallel flow heat exchanger
Answer: C
Temperature distribution

t h1 t h2 t h1
∆ T2
t c2
∆ T1 ∆ T1
t h2
∆ T2
t c1 www.
engi
neer
ingonyou
tr
c1f
inger
tips.
ooo t c2

(a ) Condenser (∆T1 > ∆T2 ) (b) Evaporator ( ∆T1 > ∆T2 )


Tn1
t h1
∆ T1 Tn 2

t c2 ∆ T1 ∆ T2
t h2
∆ T2 Tc2
t c1
Tc1

(c) Counter flow Heat Exchanger (∆T1 = ∆T2 ) (d) Parallel flow Exchanger (∆T1 > ∆T2 )

From the above temperature distribution, we can say option ‘c’ is the correct answer.

www.engineeringonyourfingertips.ooo
 India’s No.1 institute for GATE Training  1 Lakh+ Students trained till date  65+ Centers across India
8

More Notes Join us Telegram-: http//:t.me/allexammentor


www.engineeringonyourfingertips.ooo
ME-GATE-2014 PAPER-02| www.gateforum.com

17. The difference in pressure (in N/m2) across an air bubble of diameter 0.001 m immersed in
water (surface tension = 0.072 N/m) is _______
Answer: 287 to 289
∆p.r
Exp: Surface tension in a bubble = Pi
4
4T
we know, Pi − Po = Po
r
4 × 0.072
∆p = = 288 N m 2 .
0.001

18. If there are m sources and n destinations in a transportation matrix, the total number of basic
variables in a basic feasible solution is
(A) m + n (B) m + n + 1 (C) m + n - 1 (D) m
Answer: C
Exp:
DESTINATION
TOTAL
O 1 2 3 n

R 1 x11 x12 a1
I
2 x 21 x 22 a2
G
I
N
m x m1 x m2 x mn am
A

T
m n
O
T b1 b2 bn ∑ a i =∑ b J
i =1 J =1
A www.
engi
neer
ingonyour
finger
tips.
ooo
L
n

∑x iJ = a i , ( i = 1,2,....m ) ____ (1)


J =1
m

∑x iJ = b J , ( J = 1,2,....n ) ____ ( 2 )
L =1

x iJ ≥ 0 ___ ( 3)
m m

∑∑ C iJ x iJ = mn ____ ( 4 )
i =1 J =1

It is evident from the above diagram, that m+n equations (1) and (2) constitute m+n-1
independent equations in mn unknowns.

www.engineeringonyourfingertips.ooo
 India’s No.1 institute for GATE Training  1 Lakh+ Students trained till date  65+ Centers across India
9

More Notes Join us Telegram-: http//:t.me/allexammentor


www.engineeringonyourfingertips.ooo
ME-GATE-2014 PAPER-02| www.gateforum.com

19. A component can be produced by any of the four processes I, II, III and IV. The fixed cost
and the variable cost for each of the processes are listed below. The most economical process
for producing a batch of 100 pieces is
Process Fixed cost(in Rs.) Variable cost per piece (in Rs.)
I 20 3
II 50 1
III 40 2
IV 10 4
(A) I (B) II (C) III (D) IV
Answer: B
Exp:
Process Cost of production for 100
I F.c + θ × vc = 20 + 100 × 3 = 320
II 50 + 100 × 1 = 150
III 40 + 100 × 2 = 240
IV 10 + 100 × 4 = 410
Hence process (B) is most economical.

20. The flatness of a machine bed can be measured using


(A) Vernier calipers (B) Auto collimator
(C) Height gauge (D) Tool maker’s microscope
Answer: B
Exp: Flatness of machine bed can be measured from Auto collimator.

21. A robot arm PQ with end coordinates P(0,0) and Q(2,5) rotates counter clockwise about P in
the XY plane by 90°. The new coordinate pair of the end point Q is
www.
engi
neer
ingonyour
finger
tips.
ooo
(A) (-2,5) (B) (-5,2) (C) (-5, -2) (D) (2, -5)
y
Answer: B
Exp:

( 0,5) θ = ( 2,5)

5
tan θ =
θ1 = ( −5, 2 ) 2
5
∴θ = tan −1
90° 2
2 θ = 68.2°
θ = 68°.2°
21.8°
−x 5 P ( 2,0)
x
( 0,0)
2
tan 21.8° =
5
∴θ ' = ( −5,2 ) .

www.engineeringonyourfingertips.ooo
 India’s No.1 institute for GATE Training  1 Lakh+ Students trained till date  65+ Centers across India
10

More Notes Join us Telegram-: http//:t.me/allexammentor


www.engineeringonyourfingertips.ooo
ME-GATE-2014 PAPER-02| www.gateforum.com

22. Match the Machine Tools (Group A) with the probable Operations (Group B):
Group A Group B

(p) Centre lathe (1) Slotting

(q) Milling (2) Counter-boring

(r) Grinding (3) Knurling

(s) Drilling (4) Dressing

(A) P-1, Q-2, R-4, S-3 (B) P-2, Q-1, R-4, S-3
(C) P-3, Q-1, R-4, S-2 (D) P-3, Q-4, R-2, S-1
Answer: C

23. The following four unconventional machining processes are available in a shop floor. The
most appropriate one to drill a hole of square cross section of 6 mm × 6 mm and 25 mm deep
(A) is abrasive Jet Machining (B) is Plasma Arc Machining
(C) is Laser Beam Machining (D) is Electro Discharge Machining
Answer: D

24. The relationship between true strain ( ε T ) and engineering strain ( ε E ) in a uniaxial tension
test is given as
(A) ε g = ln (1 + ε T ) ( B) ε g = ln (1 − ε T )

( C) ε T = ln (1 + ε E ) ( D) ε T = ln (1 − ε E )
Answer: C
 ∆L 
Lf
dL
Exp: ∈True =
Lo

L
= ln 1 +
 Lo 

∆L
But =∈E ⇒∈True = ln (1+ ∈E ) .
Lo www. engi neer i
ngonyour
finger
tips.
ooo

25. With respect to metal working, match Group A with Group B:


Group A Group B
(p) Defect in extrusion I: alligatoring
(q) Defect in rolling II: scab
(r) Product of skew rolling III: Fish tail
(s) Product of rolling through cluster mill IV: Seamless tube
V: thin sheet with tight tolerance
VI: semi-finished balls of ball bearing
(A) P-II, Q-III, R-VI, S-V (B) P-III, Q-I, R-VI, S-V
(C) P-III, Q-I, R-IV, S-VI (D) P-I, Q-II, R-V, S-VI
Answer: B

www.engineeringonyourfingertips.ooo
 India’s No.1 institute for GATE Training  1 Lakh+ Students trained till date  65+ Centers across India
11

More Notes Join us Telegram-: http//:t.me/allexammentor


www.engineeringonyourfingertips.ooo
ME-GATE-2014 PAPER-02| www.gateforum.com

Q. No. 26 – 55 Carry Two Marks Each

26. An analytic function of a complex variable z = x + i y is expressed as f (z) = u(x, y) + i v(x, y),
where i = −1 . If u(x, y) = 2 x y, then v(x, y) must be
(A) x2 + y2 + constant (B) x2 - y2 + constant
2 2
(C) - x + y + constant (D) - x2 - y2 + constant
Answer: C
Exp: Given f ( z ) = u + iv is analytic and u = 2xy
We know that if f(z) is analytic
then CR equations will be satisfied.
∂u ∂v ∂u ∂v
ie., = and = −
∂x ∂y ∂y ∂x
Verify the options which satisfy the above C-R equations.
ν = − x 2 + y2 + cons tan t, satisfies the C-R equation.

dy
27. The general solution of the differential equation = cos ( x + y ) , with c as a constant, is
dx
x + y
( A) y + sin ( x + y ) = x + c ( B) tan   = y+c
2 
 x + y  x + y
( C) cos 
 2 
= x+c ( D) tan 
 2 
= x+c

Answer: D
Exp:
dy
= cos ( x + y )
dx
dy dν dy dν
Put x + y = ν ⇒ 1+ = ⇒ = −1
dx dx dx dx

dν www.
− 1 = cos ν
engi neer ingonyour f
inger
tips.
ooo
dx
1
⇒ dν = dx ( var iable separable )
1+ cos ν
1+ cos 2 θ = 2cos 2 θ 
1
∫ dν = fdx + c  
ν 1+ cos θ = 2cos 2 θ 
2cos 2  2 
2

1 ν
2 ∫ sec 2 dν = x + c
2
ν  x+ y
Tan = x + c ⇒ Tan   = x+c
2  2 

www.engineeringonyourfingertips.ooo
 India’s No.1 institute for GATE Training  1 Lakh+ Students trained till date  65+ Centers across India
12

More Notes Join us Telegram-: http//:t.me/allexammentor


www.engineeringonyourfingertips.ooo
ME-GATE-2014 PAPER-02| www.gateforum.com

28. Consider an unbiased cubic dice with opposite faces coloured identically and each face
coloured red, blue or green such that each colour appears only two times on the dice. If the
dice is thrown thrice, the probability of obtaining red colour on top face of the dice at least
twice is _______
Answer: 0.25 to 0.27
2 1
Exp: p= =
6 3
1 2
q =1− =
3 3
u sin g Binomial distribution
2 1 3 0
1  2 1  2 6 1 7
p ( x ≥ 2 ) = 3C2     + 3C3     = + = .
 3  3 3  3 27 27 27
4
29. The value of ∫ ln ( x ) dx
2.5
calculated using the Trapezoidal rule with five subintervals is

_______
Answer: 1.74 to 1.76
4
Exp: Given ∫ lnx dx
2.5
n =5

4 − 2.5 1.5
Let y = ln x = n=
= 0.30
5 5
x 2.5 2.8 3.1 3.4 3.7 4.0
y = lnx 0.9163 1.0296 1.1314 1.2238 1.3083 1.3863
xn
h
Trapezoidal Rule is ∫ f ( x ) dx
x0
=
2
( y 0 + y n ) + 2y1 + ........y n −1 

4
0.3
∫ lnx dx = ( 0.9163 +1.3863) + (1.0296 +1.314 +1.2238 +1.3083) 
2 
www.
engi neer ingonyour fi
nger t
ips.
ooo
2.5

(
= 0.15 2.3026 + 9.3862 = 1.7533 )

30. The flexural rigidity (EI) of a cantilever beam is assumed to be constant over the length of the
beam shown in figure. If a load P and bending moment PL/2 are applied at the free end of the
beam then the value of the slope at the free end is
P

PL / 2
L

1 PL2 PL2 3 PL2 5 PL2


( A) ( B) ( C) ( D)
2 EI EI 2 EI 2 EI
Answer: B

www.engineeringonyourfingertips.ooo
 India’s No.1 institute for GATE Training  1 Lakh+ Students trained till date  65+ Centers across India
13

More Notes Join us Telegram-: http//:t.me/allexammentor


www.engineeringonyourfingertips.ooo
ME-GATE-2014 PAPER-02| www.gateforum.com

Exp: Slope due to point load


PL2
θ1 =
2EI
ML
θ2 =
EI
PL2 PL L PL2
θ= + =
2EI 2 EI EI

31. A cantilever beam of length, L, with uniform cross-section and flexural rigidity, EI, is loaded
uniformly by a vertical load, w per unit length. The maximum vertical deflection of the beam
is given by
wL4 wL4 wL4 wL4
( A) ( B) ( C) ( D)
8EI 16EI 4EI 24EI
Answer: A
Wl 4
Exp: Maximum deflection for cantilever subjected to UDL is .
8EI

32. For the three bolt system shown in the figure, the bolt material has shear yield strength of 200
MPa. For a factor of safety of 2, the minimum metric specification required for the bolt is

19 k / N

19 k / N

(A) M8 (B) M10 (C) M12 (D) M16


Answer: B
Exp: σ y = 200 MPa
P = 19 × 103 N
P www. engi
neer
ingonyour
finger
tips.
ooo
σ= ⇒ d = 10.9 mm
π 2
d
4
M10

33. Consider a flywheel whose mass M is distributed almost equally between a heavy, ring-like
rim of radius R and a concentric disk-like feature of radius R/2. Other parts of the flywheel,
such as spokes, etc, have negligible mass. The best approximation for α , if the moment of
inertia of the flywheel about its axis of rotation is expressed as α MR2, is _______
Answer: 0.55 to 0.57
Exp: Moment of Inertia of fly wheel
I = mr R 2
R +R 2
2

R = mean Radius of Rim = M   = 0.56 MR ⇒ α = 0.56.


2

 2 

www.engineeringonyourfingertips.ooo
 India’s No.1 institute for GATE Training  1 Lakh+ Students trained till date  65+ Centers across India
14

More Notes Join us Telegram-: http//:t.me/allexammentor


www.engineeringonyourfingertips.ooo
ME-GATE-2014 PAPER-02| www.gateforum.com

34. What is the natural frequency of the spring mass system shown below? The contact between
the block and the inclined plane is frictionless. The mass of the block is denoted by m and
the spring constants are denoted by k1 and k2 as shown below.
k1 + k 2
( A) k1
2m
k1 + k 2
( B)
4m m
k1 − k 2 k2
( C)
m
k1 + k 2
( D) θ
m
Answer: D
Exp: It is parallel
k eq = k1 + k 2
k eq
ωn =
m
k1 + k 2
ωn =
m

35. A disc clutch with a single friction surface has coefficient of friction equal to 0.3. The
maximum pressure which can be imposed on the friction material is 1.5 MPa. The outer
diameter of the clutch plate is 200 mm and its internal diameter is 100 mm. Assuming
uniform wear theory for the clutch plate, the maximum torque (in N.m) that can be
transmitted is _______
Answer: 529 to 532
Exp: As per uniform wear theory
πµPa d
Torque = w  D 2
−g  e
d 2i
8 ww .en n er i
ngonyour
finger
tips. ooo
π × 0.3 × 1.5 × 106
=  0.22 − 0.12 
8
= 530.14N − m

36. A truck accelerates up a 10O incline with a crate of 100 kg. Value of static coefficient of
friction between the crate and the truck surface is 0.3. The maximum value of acceleration (in
m/s2) of the truck such that the crate does not slide down is _______
Answer: 1.0 to 1.3

www.engineeringonyourfingertips.ooo
 India’s No.1 institute for GATE Training  1 Lakh+ Students trained till date  65+ Centers across India
15

More Notes Join us Telegram-: http//:t.me/allexammentor


www.engineeringonyourfingertips.ooo
ME-GATE-2014 PAPER-02| www.gateforum.com
FF ma
Exp: 1

mg sin θ 2

10° mg mg cos θ FBD − 2 ( Truck )

RN
FBD − 1 FF1

mg mg cos10
ΣFalong = ma mg sin10 mg
FF − mg sin θ = ma
⇒ µ mg cos10 − mg sin10 = ma
⇒ a = [ 0.3 cos10 − sin10] 9.81 = 1.19.

37. Maximum fluctuation of kinetic energy in an engine has been calculated to be 2600 J.
Assuming that the engine runs at an average speed of 200 rpm, the polar mass moment of
inertia (in kg.m2) of a flywheel to keep the speed fluctuation within ± 0.5% of the average
speed is _______
Answer: 590 to 595
π
Exp: ∆E = Iω2 Cs
4
 2π × 200   0.5  −0.5  
2

⇒ 2600 = I ×   × − 
 60   100  100  
⇒ I = 595.

38. Consider the w wstates


two w. en gi
of neer
stress i
asn gony
shown inour finger t
ipIs
configurations .oo
and IIo
in the figure below. From
the standpoint of distortion energy (von-Mises) criterion, which one of the following
statements is true? σ σ

τ τ

(A) I yields after II I II


(B) II yields after I
(C) Both yield simultaneously
(D) Nothing can be said about their relative yielding
Answer: C
Exp: Both yields simultaneously.

www.engineeringonyourfingertips.ooo
 India’s No.1 institute for GATE Training  1 Lakh+ Students trained till date  65+ Centers across India
16

More Notes Join us Telegram-: http//:t.me/allexammentor


www.engineeringonyourfingertips.ooo
ME-GATE-2014 PAPER-02| www.gateforum.com

39. A rigid link PQ of length 2 m rotates about the pinned end Q with a constant angular
acceleration of 12 rad/s2. When the angular velocity of the link is 4 rad/s, the magnitude of
the resultant acceleration (in m/s2) of the end P is _______
Answer: 39 to 41
Exp:

2m

ω = 4 rad s
α = 12 rad s 2
a t = Rα = 2 × 12 = 24
a R = ω2 R = 16 × 2 = 32
a = a 2t + a R2
= 242 + 322 = 40 m s 2 .
40. A spur pinion of pitch diameter 50 mm rotates at 200 rad/s and transmits 3 kW power. The
pressure angle of the tooth of the pinion is 20°. Assuming that only one pair of the teeth is in
contact, the total force (in Newton) exerted by a tooth of the pinion on the tooth on a mating
gear is _______
Answer: 638 to 639
Exp: Diameter = 50 mm = 0.05 m

ω× 60
ω = 200 rad s ⇒ N =

α = 20°

P = 3KW

p × 106 × 60 3 × 106 × 60
T= ww =w. engi neer
ingonyour
finger
tips.
ooo
2πN ω× 60
2π ×

T = 15000 N − mm.
Tangential component of force

2T 2 × 1500
Ft = =
d 50
= 600 N
Ft
Total force = = 638.5 N.
cos 20

www.engineeringonyourfingertips.ooo
 India’s No.1 institute for GATE Training  1 Lakh+ Students trained till date  65+ Centers across India
17

More Notes Join us Telegram-: http//:t.me/allexammentor


www.engineeringonyourfingertips.ooo
ME-GATE-2014 PAPER-02| www.gateforum.com

41. A spherical balloon with a diameter of 10 m, shown in the figure below is used for
advertisements. The balloon is filled with helium (RHe = 2.08 kJ/kg.K) at ambient conditions
of 15°C and 100 kPa. Assuming no disturbances due to wind, the maximum allowable weight
(in newton) of balloon material and rope required to avoid the fall of the balloon (Rair = 0.289
kJ/kg.K) is ________

GATE − 2014

Answer: 5300 to 5330


Exp:
F1 F2 F3
( m )by

( m )buyoncy g = ( m )He g + ( m ) ballon g

mass due to mass of helium mass of ballon ( m )he + ( m )ballon


replaced air filled in material and rope

4
100 × 103 × × π × 53 × 9.81
p.vg
F1 = ( m ) buyoncy . g = = 3
R a .T 0.289 × 103 × 288
www.
engi
neer
ingonyour
finger
tips.
ooo
∴ F1 = 6171.3N
4
100 × 103 × × π × 53 × 9.81
p.v.g
F2 = ( m )Helium .g = = 3
R He .T 2.08 × 103 × 288
∴ F2 = 857.45 N
∴ ( F )3 = F1 − F2 = 6171.3 − 857.45 = 5313.85 N.

42. A hemispherical furnace of 1 m radius has the inner surface (emissivity, ε = 1 ) of its roof
maintained at 800 K, while its floor ( ε = 0.5) is kept at 600 K. Stefan-Boltzmann constant is
5.668 × 10-8 W/m2.K4. The net radiative heat transfer (in kW) from the roof to the floor is
_______
Answer: 24.0 to 25.2

www.engineeringonyourfingertips.ooo
 India’s No.1 institute for GATE Training  1 Lakh+ Students trained till date  65+ Centers across India
18

More Notes Join us Telegram-: http//:t.me/allexammentor


www.engineeringonyourfingertips.ooo
ME-GATE-2014 PAPER-02| www.gateforum.com

Exp: Let the base be (1) and hemispherical furnace be (2)


∴ F11 + F12 = 1___ (1)
F21 + F22 = 1___ ( 2 )
∵ F11 = 0 ∴ F12 = 2
A1 πR 2
A1F12 = A 2 F21 ∴ F21 = F12 = F12 = 0.5 F12
A2 2πR 2
∴ F21 = 0.5 × 2 . 0.5
∴ F22 = 0.5
So, F11 = 0, F12 = 1, F21 = 0.5 < F22 = 0.5
Now Radiative heat transfer, ⇒ θ = A1F12 ε 2 × 6 ( 800 4 − 600 4 ) watt
∴θ = π × 12 × 1 × 0.5 × 5.668 × 10−8 ( 800 4 − 600 4 ) watt
or θ = 24.9 kW.

43. Water flows through a 10 mm diameter and 250 m long smooth pipe at an average velocity of
0.1 m/s. The density and the viscosity of water are 997 kg/m3 and 855×10-6 N.s/m2,
respectively. Assuming fully-developed flow, the pressure drop (in Pa) in the pipe is _______
Answer: 6800 to 6900
ρVD
Exp: Re =
µ
997 × 0.1 × 10 × 10−3
= = 1166.081 ( laminar flow )
855 × 10−6
64 64 64
f= = = = 0.054
Re Re 1166.081
flρV 2 0.054 × 250 × 997 × ( 0.1)
2

∆p = = = 6840 pa.
2D 2 × 10 × 10−3

44. A material P w
ofw w. eng
thickness 1in
mmeeis
rin gonyoubetween
sandwiched rfinger ti
psteel
two s.oo o as shown in the figure
slabs,
below. A heat flux 10 kW/m2 is supplied to one of the steel slabs as shown. The boundary
temperatures of the slabs are indicated in the figure. Assume thermal conductivity of this steel
is 10 W/m.K. considering one-dimensional steady state heat conduction for the configuration,
the thermal conductivity (k, in W/m.K) of material P is _______
k=?

q '' = 10kW / m 2
Steelslab Steelslab
T2 = 360 K
T1 = 500 K

20 20 All dim ensions in mm

www.engineeringonyourfingertips.ooo
 India’s No.1 institute for GATE Training  1 Lakh+ Students trained till date  65+ Centers across India
19

More Notes Join us Telegram-: http//:t.me/allexammentor


www.engineeringonyourfingertips.ooo
ME-GATE-2014 PAPER-02| www.gateforum.com

Answer: 0.09 to 0.11


Exp: t1 = 20 mm; k1 = 10 W mk
t 2 = 1 mm; k 2 = ?
t 3 = 20 mm; k 3 = 10 10 W mk
T1 = 500 k; T2 = 360 k; q in = 10 10 kW m 2
Diagram
T1 − T2
qin =
t1 t t
+ 2 + 3
k1A1 k 2 A 2 k 3 A3
Taking unit Area

10 × 103 W m 2 =
( 500 − 360 )
−3
 20 × 10 1 × 10 −3 20 × 10 −3 
 + + 
 10 k2 10 
 1 
10  4 +  = 140
 k2 
1
= 10 ⇒ k 2 = 0.10 W m.k.
k2

45. Consider laminar flow of water over a flat plate of length 1 m. If the boundary layer thickness
at a distance of 0.25 m from the leading edge of the plate is 8 mm, the boundary layer
thickness (in mm), at a distance of 0.75 m, is _______
Answer: 13.5 to 14.2
Exp:
δ k
∴ = , here 'k' is constant
x x
δ δ
∴ 1 = 2
x1 x2
www.
engi
neer
ingonyour
finger
tips.
ooo
8 δ2
∴ = ∴δ 2 = 13.86 mm.
0.25 0.75
δ 0.60
∵ =
x Re x
δ 0.60 0.60 υ
or = =
x xu ∞ υ x. u ∞
∵ υ and u ∞ is constant in both the cases

46. In an ideal Brayton cycle, atmospheric air (ratio of specific heats, cp/cv = 1.4, specific heat at
constant pressure = 1.005 kJ/kg.K) at 1 bar and 300 K is compressed to 8 bar. The maximum
temperature in the cycle is limited to 1280 K. If the heat is supplied at the rate of 80 MW, the
mass flow rate (in kg/s) of air required in the cycle is _______

www.engineeringonyourfingertips.ooo
 India’s No.1 institute for GATE Training  1 Lakh+ Students trained till date  65+ Centers across India
20

More Notes Join us Telegram-: http//:t.me/allexammentor


www.engineeringonyourfingertips.ooo
ME-GATE-2014 PAPER-02| www.gateforum.com

Answer: 105 to 112


T
Exp: 3
Given : γ = 1.4, cp = 1.005 kJ kg.k Qin
P1 = 1 bar, P2 = 8 bar
T1 = 300 k, T3 = 1280 k
2
P
r = 2 =8 4
P1
1
Qin = 80000 kW. Q out
(1-2) isentropic process
1
= ( rp ) r
T2 r− 5
T1
1.4 −1
T2 = 300 × ( 8 ) 1.4 = 543.43 k

Qin = m cp ( T3 − T2 )
i

80,000 = m× 1.005 (1280 − 300 )


i

i
m = 108.07 kg s.

47. Steam at a velocity of 10 m/s enters the impulse turbine stage with symmetrical blading
having
blade angle 30O. The enthalpy drop in the stage is 100 kJ. The nozzle angle is 20O. The
maximum blade efficiency (in percent) is _______
Answer: 85.1 to 89.9
Exp: ( η)max max imum blading efficiency = cos2 α
= cos 2 20° = 0.8830
∴ ( ηb )max = 88.30%
www.
engi
neer
ingonyour
finger
tips.
ooo
48. In a concentric counter flow heat exchanger, water flows through the inner tube at 25OC and
leaves at 42OC. The engine oil enters at 100°C and flows in the annular flow passage. The
exit temperature of the engine oil is 50OC. Mass flow rate of water and the engine oil are 1.5
kg/s and 1 kg/s, respectively. The specific heat of water and oil are 4178 J/kg.K and 2130
J/kg.K, respectively. The effectiveness of this heat exchanger is _______
Answer: 0.65 to 0.67

www.engineeringonyourfingertips.ooo
 India’s No.1 institute for GATE Training  1 Lakh+ Students trained till date  65+ Centers across India
21

More Notes Join us Telegram-: http//:t.me/allexammentor


www.engineeringonyourfingertips.ooo
ME-GATE-2014 PAPER-02| www.gateforum.com

Exp: Tc,i = 25°C; Tc,o = 42°C; m c = 1.5 kg s; Cc = 4.178 kJ kg.k


Th,i = 100°C; Th,o = 50°C; m h = 1 kg s; C h = 2.130 kJ kg.k
m c Cc = 1.5 × 4.178 = 6.267 kW °C = Cmax
m h C h = 1 × 2.130 = 2.130 kW °C = Cmin
C h ( Th,i − Th,o )
effectiveness = E =
Cmin ( Th,i − Tc,i )
100 − 50
= = 0.666.
100 − 25

49. A heat pump with refrigerant R22 is used for space heating between temperature limits of
-20OC and 25OC. The heat required is 200 MJ/h. Assume specific heat of vapour at the time
of discharge as 0.98 kJ/kg.K. Other relevant properties are given below. The enthalpy (in
kJ/kg) of the refrigerant at isentropic compressor discharge is _______
Saturation
Pressure Specific enthalpy Specific entropy
temperature

Tsat ( C)
O P(MN/m2) hf(kJ/kg) hg(kJ/kg) Sf(kJ/kg/K Sg(kJ/kg.K)

-20 0.2448 177.21 397.53 0.9139 1.7841


25 1.048 230.07 413.02 1.1047 1.7183
Answer: 430 to 440
Exp: given
T1 = 253 K
T2 ' = 298 K 2

S1 = S2 ' = S22 TC 2'


T 4 3
T2
∴ S1 = S2 = S2w
'+w
Cw .
P lnengi neer
ingonyour
finger
tips.
ooo
T2 '
TC
T2 5 1
1.7841 = 1.7183 + 0.98 ln
298
∴ T2 = 318.69 K
∴ Enthalpy of discharge of compressor
5
⇒ h 2 = h 2 '+ CP ( T2 '− T1 ')
∴ h 2 = 413.02 + 0.98 ( 318.69 − 298)
kJ
∴ h 2 = 433.3 .
kg

www.engineeringonyourfingertips.ooo
 India’s No.1 institute for GATE Training  1 Lakh+ Students trained till date  65+ Centers across India
22

More Notes Join us Telegram-: http//:t.me/allexammentor


www.engineeringonyourfingertips.ooo
ME-GATE-2014 PAPER-02| www.gateforum.com

50. A project has four activities P, Q, R and S as shown below.


Activity Normal duration (days) Predecessor Cost slope (Rs./day)
P 3 - 500
Q 7 P 100
R 4 P 400
S 5 R 200
The normal cost of the project is Rs. 10,000/- and the overhead cost is Rs. 200/- per day. If
the project duration has to be crashed down to 9 days, the total cost (in Rupees) of the project
is _______
Answer: 12490 to 12510
Exp: 100 day
P Q
1 2 4
7 days
3 days
5 days
500 day S
S 200 day
2 days 3
400 day

Heuristic model to solve the crashing problem:


Step I - Find the normal time
Step II - Determine critical path ≡ P-R-S = 12days.
Step III - Crash the lowest cost slope from the critical path

≡ crash activity 's' by 2 days


Now critical path = P − Q (10days )
∴ Total cost of the project
= 10000 + 200w
×w w.en
(10days )g
+i
ne
cra er i
shinngg on
cos t your
finger
tips.
ooo
= 10,000 + 2000 + 200 × 2 = 10,000 + 2000 + 400 = 12, 400.

51. Consider the following data with reference to elementary deterministic economic order
quantity
model
Annual demand of an item 100000
Unit price of the item (in Rs.) 10
Inventory carrying cost per unit per year (in Rs.) 1.5
Unit order cost (in Rs.) 30
The total number of economic orders per year to meet the annual demand is ______
Answer: 49 to 51

www.engineeringonyourfingertips.ooo
 India’s No.1 institute for GATE Training  1 Lakh+ Students trained till date  65+ Centers across India
23

More Notes Join us Telegram-: http//:t.me/allexammentor


www.engineeringonyourfingertips.ooo
ME-GATE-2014 PAPER-02| www.gateforum.com

Exp: Given D = 100000 year


unit cos t = 10
Cc = 1.5
Co = 30 order
2 × D × Co 2 × 100000 × 30
∴ EOQ = = − 2000
Cc 1.5
Demand 100000
∴ No of order 's / year = = = 50.
EOQ 2000

52. For the CNC part programming, match Group A with Group B:
Group A Group B
(p) circular interpolation, counter clock wise I: G02
(q) dwell II: G03
(r) circular interpolation, clock wise III: G04
(s) point to point countering IV: G00
(A) P-II, Q-III, R-I, S-IV (B) P-I, Q-III, R-II, S-IV
(C) P-I, Q-IV, R-II, S-III (D) P-II, Q-I, R-III, S-IV
Answer: A
Exp: G00 - Point to point movement
G02 - Circular interpolation, cw
G02 - Circular interpolation, ccw
G04 - Dwell

www.
engi
neer
ingonyour
finger
tips.
ooo
53. A mild steel plate has to be rolled in one pass such that the final plate thickness is 2/3rd of the
initial thickness, with the entrance speed of 10 m/min and roll diameter of 500 mm. If the
plate widens by 2% during rolling, the exit velocity (in m/min) is ________
Answer: 14.6 to 14.8

54. A hole of 20 mm diameter is to be drilled in a steel block of 40 mm thickness. The drilling is


performed at rotational speed of 400 rpm and feed rate of 0.1 mm/rev. The required approach
and over run of the drill together is equal to the radius of drill. The drilling time (in minute) is
(A) 1.00 (B) 1.25 (C) 1.50 (D) 1.75
Answer: B

www.engineeringonyourfingertips.ooo
 India’s No.1 institute for GATE Training  1 Lakh+ Students trained till date  65+ Centers across India
24

More Notes Join us Telegram-: http//:t.me/allexammentor


www.engineeringonyourfingertips.ooo
ME-GATE-2014 PAPER-02| www.gateforum.com

L
Exp: T=
fN
L = t + Ap1
Ap1 = 0.5 D(holes diameter)
= 10 mm
t = 40 mm
50
T= = 1.25 min.
0.1 × 400

55. A rectangular hole of size 100 mm × 50 mm is to be made on a 5 mm thick sheet of steel


having ultimate tensile strength and shear strength of 500 MPa and 300 MPa, respectively.
The hole is made by punching process. Neglecting the effect of clearance, the punching force
(in kN) is
(A) 300 (B) 450 (C) 600 (D) 750
Answer: B
Exp: F = τ0 × A s
= 300 × 1500
= 450 kN.
for Rec tan gular hole, A s = 2 ( a + b ) t
= 2 (100 + 50 ) × 5
= 1500 mm 2

www.
engi
neer
ingonyour
finger
tips.
ooo

www.engineeringonyourfingertips.ooo
 India’s No.1 institute for GATE Training  1 Lakh+ Students trained till date  65+ Centers across India
25

More Notes Join us Telegram-: http//:t.me/allexammentor


www.engineeringonyourfingertips.ooo
ME-GATE-2014 PAPER-03| www.gateforum.com

Q. No. 1 – 5 Carry One Mark Each

1. “India is a country of rich heritage and cultural diversity.” Which one of the following facts
best supports the claim made in the above sentence?
(A) India is a union of 28 states and 7 union territories.
(B) India has a population of over 1.1 billion.
(C) India is home to 22 official languages and thousands of dialects.
(D) The Indian cricket team draws players from over ten states.
Answer: C
Exp: Diversity is shown in terms of difference language

2. The value of one U.S. dollar is 65 Indian Rupees today, compared to 60 last year. The Indian
Rupee has ____________.
(A) Depressed (B) Depreciated (C) Appreciated (D) Stabilized
Answer: B

3. 'Advice' is ________________.
(A) a verb (B) a noun
(C) an adjective (D) both a verb and a noun
Answer: B

4. The next term in the series 81, 54, 36, 24 … is ________


Answer: 16
2
Exp: 81 − 54 = 27;27 × = 18
3
2
54 − 36 = 18;18 × = 12
3
ww
36 − 24 = 12;12 × w=.
2
e8ngi
neer
ingonyour
finger
tips.
ooo
3
∴ 24 − 8 = 16

5. In which of the following options will the expression P < M be definitely true?
(A) M < R > P > S (B) M > S < P < F
(C) Q < M < F = P (D) P = A < R < M
Answer: D

Q. No. 6 – 10 Carry Two Marks Each

6. Find the next term in the sequence: 7G, 11K, 13M, ___
(A) 15Q (B) 17Q (C) 15P (D) 17P
Answer: B

www.engineeringonyourfingertips.ooo
 India’s No.1 institute for GATE Training  1 Lakh+ Students trained till date  65+ Centers across India
1

More Notes Join us Telegram-: http//:t.me/allexammentor


www.engineeringonyourfingertips.ooo
ME-GATE-2014 PAPER-03| www.gateforum.com

7. The multi-level hierarchical pie chart shows the population of animals in a reserve forest. The
correct conclusions from this information are:

Beetle Tiger
Re d − ant

Elephant
Mammal
Honey
− bee In sec t
Leopard
Re ptile
Snake
Moth Bird
Crocadile
Hawk Drongo
Bulbul
Butterfly
(i) Butterflies are birds
(ii) There are more tigers in this forest than red ants
(iii) All reptiles in this forest are either snakes or crocodiles
(iv) Elephants are the largest mammals in this forest
(A) (i) and (ii) only (B) (i), (ii), (iii) and (iv)
(C) (i), (iii) and (iv) only (D) (i), (ii) and (iii) only
Answer: D
Exp: It is not mentioned that elephant is the largest animal

8. A man can row at 8 km per hour in still water. If it takes him thrice as long to row upstream,
as to row downstream, then find the stream velocity in km per hour.
Answer: 4
Exp: Speed of man=8; Left distance =d
d
Time taken= www. engi neer i
ngonyour
finger
tips.
ooo
8
Upstream:
Speed of stream=s
⇒ speed upstream = S' = (8 − s)
 d 
t' =  
8−s
Downstream:
d
Given speed downstream = t '' =
8+s
3d d
⇒ 3t ' = t '' ⇒ =
8−s 8+s
3d d
⇒ = ⇒ s = 4km / hr
8−s 8+s

www.engineeringonyourfingertips.ooo
 India’s No.1 institute for GATE Training  1 Lakh+ Students trained till date  65+ Centers across India
2

More Notes Join us Telegram-: http//:t.me/allexammentor


www.engineeringonyourfingertips.ooo
ME-GATE-2014 PAPER-03| www.gateforum.com

9. A firm producing air purifiers sold 200 units in 2012. The following pie chart presents the
share of raw material, labour, energy, plant & machinery, and transportation costs in the total
manufacturing cost of the firm in 2012. The expenditure on labour in 2012 is Rs. 4,50,000. In
2013, the raw material expenses increased by 30% and all other expenses increased by 20%.
If the company registered a profit of Rs. 10 lakhs in 2012, at what price (in Rs.) was each air
purifier sold?

Trans
Labour
portat
ion 15%

10%
Plant and Raw Material
machinery 20%
30%
Energy
25%

Answer: 20,000
15
Exp: Total expenditure= = x = 4,50,000
100
x=3×106
Profit=10 lakhs
So, total selling price =40,00,000 … (1)
Total purifies=200 … (2)
S.P of each purifier=(1)/(2)=20,000

10. A batch of one hundred bulbs is inspected by testing four randomly chosen bulbs. The batch
is rejected if even one of the bulbs is defective. A batch typically has five defective bulbs.
The probability that the current batch is accepted is _________
Answer: 0.8145
www.
engi
neer
ingonyour
fi
95nger
tips.
ooo
Exp: Probability for one bulb to be non defective is
100
4
 95 
∴ Probabilities that none of the bulbs is defectives   = 0.8145
 100 

www.engineeringonyourfingertips.ooo
 India’s No.1 institute for GATE Training  1 Lakh+ Students trained till date  65+ Centers across India
3

More Notes Join us Telegram-: http//:t.me/allexammentor


www.engineeringonyourfingertips.ooo
ME-GATE-2014 PAPER-03| www.gateforum.com

Q. No. 1 – 25 Carry One Mark Each

1. Consider a 3 × 3 real symmetric matrix S such that two of its eigen values are a ≠ 0 , b ≠ 0
 x1   y1 
with respective eigenvectors  x 2  ,  y 2  . If a ≠ b then x1y1 + x2y2 + x2y2 equals
 x 3   y 3 
(A) a (B) b (C) ab (D) 0
Answer: (D)
Exp: We know that the Eigen vectors corresponding to distinct Eigen values of real symmetric
matrix are orthogonal.
 x1   y1 
  
 x 2   y 2  = x1 y1 + x 2 y 2 + x 3 y3 = 0
 x 3   y3 

2. If a function is continuous at a point,


(A) The limit of the function may not exist at the point
(B) The function must be derivable at the point
(C) The limit of the function at the point tends to infinity
(D) The limit must exist at the point and the value of limit should be same as the value of the
function at that point
Answer: (D)
Exp: We know that f ( x ) is continuous at x=a, if lim f ( x ) exists and equal to f ( a )
x →a

3. Divergence of the vector field x 2 ziˆ + xyjˆ − yz 2 kˆ at (1, -1, 1) is


(A) 0 (B) 3 (C) 5 (D) 6
Answer: (C)

Exp: Given F = x 2 ai + xy j − yz 2 k
  www. engi neer i
ngonyour
finger
tips.
ooo
div F = ∇.F
∂ ∂ ∂
= ( x2z ) + ( xy ) + ( − yz 2 )
∂x ∂y ∂z
= 2xz + x − 2yz

div F at (1, − 1,1) = 2 + 1 + 2 = 5

4. A group consists of equal number of men and women. Of this group 20% of the men and
50% of the women are unemployed. If a person is selected at random from this group, the
probability of the selected person being employed is _______
Answer: 0.64 to 0.66

www.engineeringonyourfingertips.ooo
 India’s No.1 institute for GATE Training  1 Lakh+ Students trained till date  65+ Centers across India
4

More Notes Join us Telegram-: http//:t.me/allexammentor


www.engineeringonyourfingertips.ooo
ME-GATE-2014 PAPER-03| www.gateforum.com

Exp: Let M → men


W → women
E → employed
U → unemployed
Given P ( M ) = 0.5
P ( W ) = 0.5

( M ) = 0.20
P U

P ( U ) = 0.50
W
By Total probability,
P ( U) = P ( M) P U ( M )
+ P(W) P U
M ( )
= 0.5 × 0.20 + 0.5 × 0.50 = 0.35
Required probability = P ( E ) = 1 − P ( U ) =1 − 0.35 = 0.65

1 3
5. The definite integral
x ∫
dx is evaluated using Trapezoidal rule with a step size of 1. The
1

correct answer is ________


Answer: 1.1 to 1.2
3
1
Exp: Given, ∫ x dx
1

h = step size =1
3-1
n = no.of .sub intervals = =2
1
Let y = 1
x
x 1 2 3
1 ww 1 w1.engi neer ingonyour f i nger
tips.
ooo
y= 1
x 2 3
By trapezoidal rule
h
∫x0 f ( x ) dx = 2 ( y0 + yn ) + 2 ( y1 + .......... + yn −1 )
xn

31 1  1   1  1 4  1 7
∫1 x dx = 2 1+ 3  + 2  2   = 2  3 +1 = 2 × 3 = 1.1667

6. A rotating steel shaft is supported at the ends. It is subjected to a point load at the centre. The
maximum bending stress developed is 100 MPa. If the yield, ultimate and corrected
endurance strength of the shaft material is 300 MPa, 500 MPa and 200 MPa, respectively,
then the factor of safety for the shaft is _______
Answer: 1.9 to 2.1

www.engineeringonyourfingertips.ooo
 India’s No.1 institute for GATE Training  1 Lakh+ Students trained till date  65+ Centers across India
5

More Notes Join us Telegram-: http//:t.me/allexammentor


www.engineeringonyourfingertips.ooo
ME-GATE-2014 PAPER-03| www.gateforum.com

Exp: σ = min of
(S yt ,Sut ,Se )
FoS
200
FOS = = 2.
100

7. Two solid circular shafts of radii R1 and R2 are subjected to same torque. The maximum
shear stresses developed in the two shafts are τ1 and τ2 . If R1/ R2=2, then τ2 / τ1 is _______
Answer: 7.9 to 8.1
16T
Exp: τ= 3
πd
3 3
τ d  R 
⇒ 2 =  1  =  1  = 23 = 8.
τ1  d 2   R 2 

8. Consider a single degree-of-freedom system with viscous damping excited by a harmonic


force. At resonance, the phase angle (in degree) of the displacement with respect to the
exciting force is
(A) 0 (B) 45 (C) 90 (D) 135
Answer: (C)

9. A mass m1 of 100 kg travelling with a uniform velocity of 5 m/s along a line collides with a
stationary mass m2 of 1000 kg. After the collision, both the masses travel together with the
same velocity. The coefficient of restitution is
(A) 0.6 (B) 0.1 (C) 0.01 (D) 0
Answer: (D)
Velocity of separation
Exp: Coefficient of Restitution=
Velocity of approach
V2 − V1 V − V
= = =0
u1 − u 2 5−0
∵ V2 = V1 = V final velocity is same.
www.
engi
neer
ingonyour
finger
tips.
ooo
10. Which one of following is NOT correct?
(A) Intermediate principal stress is ignored when applying the maximum principal stress
theory
(B) The maximum shear stress theory gives the most accurate results amongst all the failure
theories
(C) As per the maximum strain energy theory, failure occurs when the strain energy per unit
volume exceeds a critical value
(D) As per the maximum distortion energy theory, failure occurs when the distortion energy
per unit volume exceeds a critical value
Answer: (B)

www.engineeringonyourfingertips.ooo
 India’s No.1 institute for GATE Training  1 Lakh+ Students trained till date  65+ Centers across India
6

More Notes Join us Telegram-: http//:t.me/allexammentor


www.engineeringonyourfingertips.ooo
ME-GATE-2014 PAPER-03| www.gateforum.com

11. Gear 2 rotates at 1200 rpm in counter clockwise direction and engages with Gear 3. Gear 3
and Gear 4 are mounted on the same shaft. Gear 5 engages with Gear 4. The numbers of teeth
on Gears 2, 3, 4 and 5 are 20, 40, 15 and 30, respectively. The angular speed of Gear 5 is

20T 2

3
4 5

40T 15T 30T

(A) 300 rpm counter clockwise (B) 300 rpm clockwise


(C) 4800 rpm counter clockwise (D) 4800 rpm clockwise
Answer: (A)
N 5 T2 × T4 20 × 15
Exp: = =
N 2 T3 × T5 40 × 30
⇒ N 5 = 1200 × 0.25 = 300 rpm ccw.

12. Consider a long cylindrical tube of inner and outer radii, ri and ro , respectively, length, L
and thermal conductivity, k. Its inner and outer surfaces are maintained at Ti and T0,
respectively ( Ti > TO ). Assuming one-dimensional steady state heat conduction in the
radial direction, the thermal resistance in the wall of the tube is
1 r  1 r  1 r 
( A) ln  1  ( B)
1
( C) ln  o  ( D) ln  o 
2πkL  r0  2 πri k 2πri k  ri  4πri k  ri 
Answer: (C)
Exp: A r = 2 πrL
From Fourier 's Law
q r = − kA r www. engi
neer
ingonyour f
inger
tips.ooo
dT
dr
dT
q r = −2πkr L
dr
Boundary conditions : roro
T = Ti at r = ri riri
T = To at r = ro L
2πkL ( Ti − To )
q=
ln ( ro ri )
Ti − To T − To
= = i
ln ( ro ri ) R th
2πkL
ln ( ro ri )
R th = .
2πkL

www.engineeringonyourfingertips.ooo
 India’s No.1 institute for GATE Training  1 Lakh+ Students trained till date  65+ Centers across India
7

More Notes Join us Telegram-: http//:t.me/allexammentor


www.engineeringonyourfingertips.ooo
ME-GATE-2014 PAPER-03| www.gateforum.com

13. Which one of the following pairs of equations describes an irreversible heat engine?
δQ δQ
( A ) ∫ δQ > 0 and ∫ <0 ( B) ∫ δQ < 0 and ∫ <0
T T
δQ δQ
( C) ∫ δQ > 0 and ∫ >0 ( D ) ∫ δQ < 0 and ∫ >0
T T
Answer: (A)
δQ
Exp: For clausius theorem, ∫ T
< 0 (for irreversible Heat engine).

14. Consider the turbulent flow of a fluid through a circular pipe of diameter, D. Identify the
correct pair of statements.

I. The fluid is well-mixed


II. The fluid is unmixed
III. ReD < 2300
IV. ReD > 2300
(A) I, III (B) II, IV (C) II, III (D) I, IV
Answer: (D)
Exp: Re D > 2300 means it is a turbulent flow. In turbulent flow, the fluid is well mixed. The fluid
is unmixed, for a very-low Reynolds number laminar flow.

15 For a gas turbine power plant, identify the correct pair of statements.
P. Smaller in size compared to steam power plant for same power output
Starts quickly compared to steam power plant
R. Works on the principle of Rankine cycle
S. Good compatibility with solid fuel
(A) P, Q (B) R, S (C) Q, R (D) P, S
Answer: (A)
Exp: Steam powerw ww.
plants eng
are ineer
bulky i
dueng toon your f
presence i
ng
of er ti
boilerp s.
andocondenser.
oo Gas turbines are
compact, as compressors and turbines are coupled on a common shaft. In steam power plants,
boiler takes lot of time to get started, as compared to Gas Turbines.

16. A source at a temperature of 500 K provides 1000 kJ of heat. The temperature of environment
is 27OC. The maximum useful work (in kJ) that can be obtained from the heat source
is_______
500 K
Answer: 399 to 401
w Tsin k 1000 kJ
Exp: η = net = 1 −
Q in Tsource
300 Wnet HE Wnet
1− =
500 1000
Wnet = 1000 × 0.4
= 400 kJ.
300 K

www.engineeringonyourfingertips.ooo
 India’s No.1 institute for GATE Training  1 Lakh+ Students trained till date  65+ Centers across India
8

More Notes Join us Telegram-: http//:t.me/allexammentor


www.engineeringonyourfingertips.ooo
ME-GATE-2014 PAPER-03| www.gateforum.com

17. A sample of moist air at a total pressure of 85 KPa has a dry bulb temperature of 30°C
(saturation vapour pressure of water = 4.24 KPa). If the air sample has a relative humidity of
65%, the absolute humidity (in gram) of water vapour per kg of dry air is _______
Answer: 19 to 22
Exp: PT = 85 KPa, DBT = 30°C
Pw .s = 4.24 KPa, RH = 65%
RH
∴ Pw = Pw .s × = 4.24 × 0.65 = 2.756 KPa
100
Pw
now ω = 622 = gm ( of water ) mg. d. a
PT
622 × 2.756
∴ω = = 20.17 gm ( of water ) mg. d. a
85

18. The process utilizing mainly thermal energy for removing material is
(A) Ultrasonic Machining (B) Electrochemical Machining
(C) Abrasive Jet Machining (D) Laser Beam Machining
Answer: (D)

19. The actual sales of a product in different months of a particular year are given below:
September October November December January February
180 280 250 190 240 ?
The forecast of the sales, using the 4-month moving average method, for the month of
February is _______
Answer: 239 to 241
Exp: Number of periods = 4, then the past 4 months average sales is fore cast for next 4 months.
280 + 250 + 190 + 240
So, = 240.
4
20. A straight turning operation is carried out using a single point cutting tool on an AISI 1020
www. engi neer ingonyour finger tips. ooo
steel rod. The feed is 0.2 mm/rev and the depth of cut is 0.5 mm. The tool has a side cutting
edge angle of 60°. The uncut chip thickness (in mm) is _______
Answer: 0.08 to 0.12
Exp: t1 = f cos θ
= 0.2 × cos 60
t1 = 0.1 mm
where t1 = uncut chip thickness

21. A minimal spanning tree in network flow models involves


(A) All the nodes with cycle/loop allowed
(B) All the nodes with cycle/loop not allowed
(C) Shortest path between start and end nodes
(D) All the nodes with directed arcs

www.engineeringonyourfingertips.ooo
 India’s No.1 institute for GATE Training  1 Lakh+ Students trained till date  65+ Centers across India
9

More Notes Join us Telegram-: http//:t.me/allexammentor


www.engineeringonyourfingertips.ooo
ME-GATE-2014 PAPER-03| www.gateforum.com

Answer: (B)
Exp: A path forms a loop or cycle if it connects a node itself. A spanning tree links all the nodes of
network with no loops allowed.

22. Match the casting defects (Group A) with the probable causes (Group B):
Group A Group B
(p) Hot tears 1: Improper fusion of two streams of liquid metal
(q) Shrinkage 2: Low permeability of the sand mould
(r) Blow holes 3: Volumetric contraction both in liquid and solid stage
(s) Cold Shut 4: Differential cooling rate
(A) P-1, Q-3, R-2, S-4 (B) P-4, Q-3, R-2, S-1
(C) P-3, Q-4, R-2, S-1 (D) P-1, Q-2, R-4, S-3
Answer: (B)

23. Cutting tool is much harder than the workpiece. Yet the tool wears out during the tool-work
interaction, because
(A) extra hardness is imparted to the workpiece due to coolant used
(B) oxide layers on the workpiece surface impart extra hardness to it
(C) extra hardness is imparted to the workpiece due to severe rate of strain
(D) vibration is induced in the machine tool
Answer: (C)

24. The stress-strain curve for mild steel is shown in the figure given below. Choose the correct
option referring to both figure and table.

T
St U
re
www.
e ngi
ss neer
ingonyoRur
finger
tips.
ooo
Q
0
(N P S

Strain e ( % )

Point on the graph Description of the point


P 1. Upper Yield Point
Q 2. Ultimate Tensile Strength
R 3. Proportionality Limit
S 4. Elastic Limit
T 5. Lower Yield Point
U 6. Failure

www.engineeringonyourfingertips.ooo
 India’s No.1 institute for GATE Training  1 Lakh+ Students trained till date  65+ Centers across India
10

More Notes Join us Telegram-: http//:t.me/allexammentor


www.engineeringonyourfingertips.ooo
ME-GATE-2014 PAPER-03| www.gateforum.com

(A) P-1, Q-2, R-3, S-4, T-5, U-6 (B) P-3, Q-1, R-4, S-2, T-6, U-5
(C) P-3, Q-4, R-1, S-5, T-2, U-6 (D) P-4, Q-1, R-5, S-2, T-3, U-6
Answer: (C)

25. The hot tearing in a metal casting is due to


(A) high fluidity
(B) high melt temperature
(C) wide range of solidification temperature
(D) low coefficient of thermal expansion
Answer: (C)
Q. No. 26 – 55 Carry Two Marks Each

26. An analytic function of a complex variable z = x + iy is expressed as f(z) = w(x, y) + iv(x, y),
where i = −1 . If u(x, y) = x2 – y2, then expression for v(x, y) in terms of x, y and a general
constant c would be

x 2 + y2 ( x − y)2 + c
(A) xy + c (B) +c (C) 2xy + c ( D)
2 2
Answer: (C)
Exp: Given f ( z ) = x ( x , y ) + iv ( x, y ) is analytic and x = x 2 − y 2

We know that if f ( z ) = µ + iv is analytic then C-R equations will be satisfied.


∂µ ∂v ∂µ −∂v
ie., = and =
∂x ∂xy ∂y ∂x
∴ v = 2xy + c is correct answer
www.
engi
neer
ingonyour
finger
tips.
ooo

27. Consider two solutions x ( t ) = x1 ( t ) and x ( t ) and x ( t ) = x 2 ( t ) of the differential equation

d2x ( t ) dx1 ( t ) dx 2 ( t )
2
+ x ( t ) = 0, t > 0, Such that x1(0) = 1, 1, = 0, x 2 ( 0) = 0, = 1.
dt dt t =0
dt t =0

x1 ( t ) x 2 ( t )
dx ( t ) dx 2 ( t )
The Wronskian W ( t ) = 1 at t = π is
dt dt 2

(A) 1 (B) -1 (C) 0 (D) π


2
Answer: (A)

www.engineeringonyourfingertips.ooo
 India’s No.1 institute for GATE Training  1 Lakh+ Students trained till date  65+ Centers across India
11

More Notes Join us Telegram-: http//:t.me/allexammentor


www.engineeringonyourfingertips.ooo
ME-GATE-2014 PAPER-03| www.gateforum.com

Exp: Given Differential equation is


d2x ( t )
+ x (t) = 0
dt 2
Auxiliary equation is m 2 +1 = 0
m = 0± i
Complementary y solution is
x c = c1 cos t + c 2 sin t
Particular solution x p = 0
∴ General solution x = c1 cos t + c 2 sin t
Let x1 ( t ) = cos t x 2 ( t ) = sin t
dx1 dx
clearly x1 ( 0 ) = 1 = 0 and x 2 ( 0 ) = 0, 2 = 1
dt dt
x1 ( t ) x2 (t )
cos t sin t
W = dx1 dx 2 = = cos 2 t + sin 2 t = 1
− sin t cos t t=x
2

dt dt t = x 2

28 A machine produces 0, 1 or 2 defective pieces in a day with associated probability of 1/6, 2/3
and 1/6, respectively. The mean value and the variance of the number of defective pieces
produced by the machine in a day, respectively, are
(A) 1 and 1/3 (B) 1/3 and 1 (C) 1 and 4/3 (D) 1/3 and 4/3
Answer: (A)
Exp: Let ‘x’ be no. of defective pieces.
x 0 1 2
P(x) 1 2 1
6 3 6
mean ( µ ) = E(x) = Σ x P(x)
 1 w e 1i  e 
=  0 ×ww
 + .
1×ng
 +n
e
2×r
1i
ngonyour
finger
tips.
ooo
 6  3  6
2 1
=0+ + =1
3 3
E ( x 2 ) = Σx 2 P ( x )
 1  2  1
=  0 ×  + 1×  +  4 × 
 6  3  6
2 2 4
=0 + + =
3 3 3
Variance V ( x ) = E ( x 2 ) − {E ( x )}
2

4
= −1 = 1
3 3

www.engineeringonyourfingertips.ooo
 India’s No.1 institute for GATE Training  1 Lakh+ Students trained till date  65+ Centers across India
12

More Notes Join us Telegram-: http//:t.me/allexammentor


www.engineeringonyourfingertips.ooo
ME-GATE-2014 PAPER-03| www.gateforum.com

29. The real root of the equation 5x − 2cosx −1 = 0 (up to two decimal accuracy) is _______
Answer: 0.53 to 0.56
Exp: Let f ( x ) = 5x − 2cos x − 1
⇒ f ' ( x ) = 5 + 2sin x
f ( 0 ) = − 3; f (1) = 2.9
By intermediate value theorem roots lie between 0 and 1.
Let x 0 = 1rad = 57.32°
By Newton Raphson method,
f ( xn )
x n +1 = x n −
f '( xn )
2x n sin x n + 2 cos x n + 1
⇒ x n +1 =
5 + 2sin x n
⇒ x1 = 0.5632
⇒ x 2 = 0.5425
⇒ x 3 = 0.5424

30. A drum brake is shown in the figure. The drum is rotating in anticlockwise direction. The
coefficient of friction between drum and shoe is 0.2. The dimensions shown in the figure are
in mm. The braking torque (in N.m) for the brake shoe is _______

1000 N
800
480

100

200
www.
engi
neer
ingonyour
finger
tips.
ooo
Drum

Answer: 63 to 65
Exp: Taking moments about pin joint (∑ M )
⇒ 1000 × 0.8 = R N × 0.48 + µR N × 0.1
800
⇒ RN =
0.5
= 1600
TB = µ R N × r
200
= 0.2 × 1600 × = 64 N − m.
1000

www.engineeringonyourfingertips.ooo
 India’s No.1 institute for GATE Training  1 Lakh+ Students trained till date  65+ Centers across India
13

More Notes Join us Telegram-: http//:t.me/allexammentor


www.engineeringonyourfingertips.ooo
ME-GATE-2014 PAPER-03| www.gateforum.com

31. A body of mass (M) 10 kg is initially stationary on a 45° inclined plane as shown in figure.
The coefficient of dynamic friction between the body and the plane is 0.5. The body slides
down the plane and attains a velocity of 20 m/s. The distance travelled (in meter) by the body
along the plane is _______

45O

Answer: 56 to 59
Exp: FBD of block RN µR N
Net force = ma
mg sin 45 − µmg cos 45 = ma
⇒ a = 3.46 m s 2
⇒ V 2 − u 2 = 2as mg cos 45
V 2
20 2 mg sin 45 mg
⇒S= = = 57.8 m.
2a 2 × 3.46

32. Consider a simply supported beam of length, 50h, with a rectangular cross-section of depth,
h, and width, 2h. The beam carries a vertical point load, P, at its mid-point. The ratio of the
maximum shear stress to the maximum bending stress in the beam is
(A) 0.02 (B) 0.10 (C) 0.05 (D) 0.01
Answer: (D)
Exp: Given, b = 2h, d = h, l = 50h, force = p
3 shear force
shear stress = ×
2 d×b
3 p 3p
= × = 2
2 h × 2h w4h ww. engi neer ingonyour fi
nger t i
ps. ooo
M
Bending stress = ×y
I
p×l 2 d
= ×
bd 3 2
12
h 25
p × 50 h 2 × × 6 ph 2
2= 2 75p
= = 2
( 2h ) h 3 h 2
h
12
3P
Shear stres 2 3
= 4h = = 0.01.
Bending stress 75p 4 × 75
h2

www.engineeringonyourfingertips.ooo
 India’s No.1 institute for GATE Training  1 Lakh+ Students trained till date  65+ Centers across India
14

More Notes Join us Telegram-: http//:t.me/allexammentor


www.engineeringonyourfingertips.ooo
ME-GATE-2014 PAPER-03| www.gateforum.com

33. The damping ratio of a single degree of freedom spring-mass-damper system with mass of 1
kg, stiffness 100 N/m and viscous damping coefficient of 25 N.s/m is _______
Answer: 1.24 to 1.26
Exp: Damping ratio
C C
( ξ) = =
Cc 2 km
C 25
= = = 1.25
2 1 × 100 20

34. An annular disc has a mass m, inner radius R and outer radius 2R. The disc rolls on a flat
surface without slipping. If the velocity of the centre of mass is v, the kinetic energy of the
disc is
9 11 13 15
( A ) mv 2 ( B) mv 2 ( C) mv 2 ( D ) mv 2
16 16 16 16
Answer: (C)
1 1
Exp: ∴ I = m ( 2R ) − mR 2
2

2 2
3
= mR 2
2
1 1 3 3 1
( K.E )Rotation = I.ω2 = . .mR 2 .ω2 = . .m ( 2Rω)
2

2 2 2 4 4
3
∴ ( K.E ) R = .mv 2 2R V R
16
1
( K.E )Translation = mv2
2
( K.E )T = ( K.E )R + ( K.E )T
3 1 11
= mv 2 + mv 2 = mv 2 .
16 2 16

35. A force P is w ww.


applied e
atnagineer
distanceinxg ony
from ou
the rfi
endnofge r
thetips.
beam o asoo
shown in the figure. What
would be the value of x so that the displacement at ‘A’ is equal to zero?

(A) 0.5L L

A
(B) 0.25L

(C) 0.33L X
P

L
(D) 0.66L
Answer: (C)

www.engineeringonyourfingertips.ooo
 India’s No.1 institute for GATE Training  1 Lakh+ Students trained till date  65+ Centers across India
15

More Notes Join us Telegram-: http//:t.me/allexammentor


www.engineeringonyourfingertips.ooo
ME-GATE-2014 PAPER-03| www.gateforum.com

Exp:
P

 P (l − x )
x

Pl3
δ1 = due to po int load ( downward )
3EI
ML2
δ2 = due to moment ( upward )
2EI
Pl3 P ( l − x ) l
2

⇒ δ = δ1 − δ 2 = − =0
3EI 2EI
Pl3 Pl3 P × l2 x
⇒ − + =0
3EI 2EI 2EI
 −1
2
 Pl
⇒  l + x =0
3  2EI
1
⇒ x = l = 0.33L.
3

36. Consider a rotating disk cam and a translating roller follower with zero offset. Which one of
the following pitch curves, parameterized by t, lying in the interval 0 to 2π, is associated with
the maximum translation of the follower during one full rotation of the cam rotating about the
center at (x, y) = (0, 0) ?
( A) x ( t ) = cos t, y ( t ) = sin t ( B) x ( t ) = cos t, y ( t ) = 2sin t
1 1
( C) x ( t) =
w + cos t, y ( t ) = 2sin t
w w .e n g in ee rin gonyo(u
D ) x ( t ) = + cos t, y ( t ) = sin t
rfing e rt
2 2i
ps.ooo
Answer: (C)
Exp: From all the four options the maximum amplitudes is in point ‘C’ as t = 0.
( x )t =0 ( y )t =0
(A) x =1 y=0
( B) x =1 y=0
3
(C) x= y=0
2
3
(D) x= y=0
2
Option ‘C’ has maximum net amplitude.

www.engineeringonyourfingertips.ooo
 India’s No.1 institute for GATE Training  1 Lakh+ Students trained till date  65+ Centers across India
16

More Notes Join us Telegram-: http//:t.me/allexammentor


www.engineeringonyourfingertips.ooo
ME-GATE-2014 PAPER-03| www.gateforum.com

37. A four-wheel vehicle of mass 1000 kg moves uniformly in a straight line with the wheels
revolving at 10 rad/s. The wheels are identical, each with a radius of 0.2 m. Then a constant
braking torque is applied to all the wheels and the vehicle experiences a uniform deceleration.
For the vehicle to stop in 10 s, the braking torque (in N.m) on each wheel is _______
Answer: 9 to 11
Exp:
m = 1000 kg, ω = 10 Rad sec , R = 0.2 m, t = 10 sec when ω = 0
∴ωF = ωi − αt

∴ 0 = 10 − α × 10 ∴α = 1 rad sec 2
Now T = I.α

 1000 
∴ ( T )each wheel =   × 0.2 × 1
2

 4 
∴ ( T )each wheel = 10 N − m

38. A slider-crank mechanism with crank radius 60 mm and connecting rod length 240 mm is
shown in figure. The crank is rotating with a uniform angular speed of 10 rad/s, counter
clockwise. For the given configuration, the speed (in m/s) of the slider is _______

240
60 90 O

Answer: 0.54 to 0.68


www.
engi
neer
ingonyour
finger
tips.
ooo
39. Consider an objective function Z ( x1 ,x 2 ) = 3x1 + 9x 2 and the constraints

x1 + x 2 ≤ 8,
x1 + 2x 2 ≤ 4,
x1 ≥, x 2 ≥ 0,
The maximum value of the objective function is _______
Answer: 17 to 19
Exp: Roots are (12,-4) and (12,-2)
∴ Maximum value of objective function = 3(12)+9(-2) = 18.

www.engineeringonyourfingertips.ooo
 India’s No.1 institute for GATE Training  1 Lakh+ Students trained till date  65+ Centers across India
17

More Notes Join us Telegram-: http//:t.me/allexammentor


www.engineeringonyourfingertips.ooo
ME-GATE-2014 PAPER-03| www.gateforum.com

40. A mass-spring-dashpot system with mass m = 10 kg, spring constant k = 6250 N/m is excited
by a harmonic excitation of 10 cos(25t) N. At the steady state, the vibration amplitude of the
mass is 40 mm. The damping coefficient (c, in N.s/m) of the dashpot is _______

F = 10 cos ( 25t )

k c

Answer: 9 to 11
F
Exp: X=
( k − mω ) + ( Cω)
2 2 2

= 40mm = 0.04
F = 10N
ω = 25
10
⇒ 0.04 =
( 6250 − 10 × 25 ) + ( C × 25)
2 2 2

C = 10Ns / m

41. A certain amount of an ideal gas is initially at a pressure P1 and temperature T1. First, it
undergoes a constant pressure process 1-2 such that T2 = 3T1/4. Then, it undergoes a constant
volume process 2-3 such that T3 = T1/2. The ratio of the final volume to the initial volume of
the ideal gas w
is ww.engi neer i
ngonyour fi
nger t ips. ooo
(A) 0.25 (B) 0.75 (C) 1.0 (D) 1.5
Answer: (B)
Exp: For (1 − 2) process : Cons tan t pressure process {P2 = P1}
T1 T2 T
= ⇒ V2 = 2 × V1
V1 V2 T1
For (2 − 3) process : Cons tan t Volume process {V3 = V2 }

3T1
Given T2 =
4
3T 3 V3 V2 3
V2 = 1 × V1 = V1 ⇒ = = = 0.75
4T1 4 V1 V1 4
.

www.engineeringonyourfingertips.ooo
 India’s No.1 institute for GATE Training  1 Lakh+ Students trained till date  65+ Centers across India
18

More Notes Join us Telegram-: http//:t.me/allexammentor


www.engineeringonyourfingertips.ooo
ME-GATE-2014 PAPER-03| www.gateforum.com

42. An amount of 100 kW of heat is transferred through a wall in steady state. One side of the
wall is maintained at 127OC and the other side at 27OC. The entropy generated (in W/K) due
to the heat transfer through the wall is _______
Answer: 80 to 85
Q
Exp: ∆S1 =
T1 Q
Q
∆S2 =
T2
( ∆S )generated = ∆S1 + ∆S2 Q
Q Q
= −
400 300
100 × 103  1 1  400K 300K
=  − 
100  4 3 
= 103 × −0.0833
= −83.33 W / K
Entropy generated = 83.33 W K.

43. A siphon is used to drain water from a large tank as shown in the figure below. Assume that
the level of water is maintained constant. Ignore frictional effect due to viscosity and losses at
entry and exit. At the exit of the siphon, the velocity of water is
Q

O ZQ
ZP

zO

www.
engi
neer
ingonyour
finger
tips.
ooo
D a tu m ZR R

( A) (
2g ZQ − ZR ) ( B) 2g ( Z P − Z R )

( C) 2g ( ZO − Z R ) ( D) 2g ZQ
Answer: (B)
Exp: Applying Bernoulli’s equation between the points ‘P’ and ‘R’.
Patm VP2 P V2
+ + Zp = atm + R + ZR
ρg 2g ρg 2g
Since the level of liquid in tank remains the same, VP = 0
∴ VR = 2g ( ZP − ZR ) .

www.engineeringonyourfingertips.ooo
 India’s No.1 institute for GATE Training  1 Lakh+ Students trained till date  65+ Centers across India
19

More Notes Join us Telegram-: http//:t.me/allexammentor


www.engineeringonyourfingertips.ooo
ME-GATE-2014 PAPER-03| www.gateforum.com

44. Heat transfer through a composite wall is shown in figure. Both the sections of the wall have
equal thickness (l). The conductivity of one section is k and that of the other is 2k. The left
face of the wall is at 600 K and the right face is at 300 K. The interface temperature Ti (in K)
of the composite wall is _______

600 K T1 300 K

Heat flow
k
2k

l l

Answer: 399 to 401


Exp: R = R1 + R 2
L L L L 3L
= + = + =
KA KA KA 2KA 2KA
T − TR
Q= L
TR
300 KA
= = 200 × →1
3L L
2KA
At interface
T − Ti
Q= L
Ri
600 − Ti KA
= = ( 600 − Ti ) →2
L L
KA www. engi neer
ingonyour
finger
tips.
ooo
Equating 1 and 2
KA KA
200 = ( 600 − Ti )
L L
200 = 600 − Ti
Ti = 400K

45. A fluid of dynamic viscosity 2 × 10-5 kg/m.s and density 1 kg/m3 flows with an average
velocity of 1 m/s through a long duct of rectangular (25 mm × 15 mm) cross-section.
Assuming laminar flow, the pressure drop (in Pa) in the fully developed region per meter
length of the duct is _______
Answer: 1.7 to 2.0
Exp: Given,

www.engineeringonyourfingertips.ooo
 India’s No.1 institute for GATE Training  1 Lakh+ Students trained till date  65+ Centers across India
20

More Notes Join us Telegram-: http//:t.me/allexammentor


www.engineeringonyourfingertips.ooo
ME-GATE-2014 PAPER-03| www.gateforum.com

µ = 2 × 10−5 kg m.s, ρ = 1 kg m3 , u av = 1m sec


Duct area = 25 mm × 15 mm

∆P 4 × λ × ρ × U av
2
∵ = ___ (1)
L 2 × Dn

here λ = Friction factor


D n = Dia

4× A
∴ Dn = ( A = Area, P = parameter )
P
4 × 25 × 15
λ= = 18.75 mm ___ ( 2 )
2 ( 25 + 15 )

16 ρ.u av .D n
λ= ∴ Re =
Re π
1 × 1 × 18.75 × 10−3
∴ Re = = 937.5
2 × 10−5
16
∴λ = = 1.707 × 10−2 ___ ( 3)
937.5
Here from equation ( 3)

∆P 4 × 1.707 × 10 × (1) × 2
−2 2

= = 1.8208 pa m.
L 2 × 18.75 × 10−3

46. At the inlet of an axial impulse turbine rotor, the blade linear speed is 25 m/s, the magnitude
of absolute velocity is 100 m/s and the angle between them is 25°. The relative velocity and
the axial component of velocity remain the same between the inlet and outlet of the blades.
The blade inlet and outlet velocity triangles are shown in the figure. Assuming no losses, the
www. engi neer
specific work (in J/kg) is _______
ingonyour finger tips. ooo

100 m / s 78 m / s
78 m / s 58.6 m / s

25O
25m / s

Answer: 3250 to 3300

www.engineeringonyourfingertips.ooo
 India’s No.1 institute for GATE Training  1 Lakh+ Students trained till date  65+ Centers across India
21

More Notes Join us Telegram-: http//:t.me/allexammentor


www.engineeringonyourfingertips.ooo
ME-GATE-2014 PAPER-03| www.gateforum.com

Exp:
Vr2 = 78 m sec
V1 = 100 m sec

Vr1 = 78 m sec
V2 = 58 m sec
β1 β2 α
Given α = 25°
Vr1 sin β1 = V1 sin α
u = 25 m sec
∴ 78sin β1 = 100 sin 25°
∴β1 = 32.81°
∵ Vr1 = Vr2 ∴β1 = β2 = 32.81°
now ∆Vw = Vr1 cos β1 + Vr2 cos β2 = 2Vr1 cos β1 = 2 × 78 × cos32.81°
∴∆Vw = 131.1136 m sec
∴ Sp. power = ∆Vw .u = 131.1136 × 25 = 3277.84 J kg.

47. A solid sphere of radius r1 = 20 mm is placed concentrically inside a hollow sphere of radius
r2 = 30 mm as shown in the figure.

1
2
r1

r2

ww
The view factor Fw
21.
e ng
for i
neer
radiation i
ngo
heat nyouis
transfer r
finger
tips.
ooo
2 4 8 9
( A) ( B) ( C) ( D)
3 9 27 4
Answer: (B)
Exp: F11 + F12 = 1, here F11 = 0
2
∴ F12 = 1
1 r1
now from Reciprocating law
A1F12 = A 2 F21 r2
2
 10  4
∴ 2π ( 20 ) × 1 = 2π ( 30 ) × F21 ⇒∴ F21 =   = .
2 2

 30  9

www.engineeringonyourfingertips.ooo
 India’s No.1 institute for GATE Training  1 Lakh+ Students trained till date  65+ Centers across India
22

More Notes Join us Telegram-: http//:t.me/allexammentor


www.engineeringonyourfingertips.ooo
ME-GATE-2014 PAPER-03| www.gateforum.com

48. A double-pipe counter-flow heat exchanger transfers heat between two water streams. Tube
side water at 19 liter/s is heated from 10OC to 38OC. Shell side water at 25 liter/s is entering at
46OC. Assume constant properties of water, density is 1000 kg/m3 and specific heat is 4186
J/kg K. The LMTD (in OC) is ________
Answer: 10.8 to 11.2
i
Exp: Given: m h = 25 L S; Th,i = 46°C, Th ,o = ?
i
m c = 19 L S; Tc,i = 10°C, Tc,o = 38°C
ρ = 1000 kg m 3 ,C = 4186 J kg.k
Th ,i
Energy balance

m c C ( Tc,o − Tc,i ) = m n C ( Th ,i − Th ,o )
i

Tc,o
19 ( 38 − 10 ) = 25 ( 46 − Th,o ) Th,o

Th ,o = 24.72°C
Tc,i
θ −θ
LMTD = 1 2
ln θ1 θ2
θ1 = Th,i − Tc,o = 46 − 38 = 8°C
θ2 = Th,o − Tc,i = 24.72 − 10 = 14.72
8 − 14.72
LMTD = = 11.0206°C.
ln ( 8 14.72 )

49. A diesel engine has a compression ratio of 17 and cut-off take place at 10% of the stroke.
Assuming ratio of specific heats ( γ ) as 1.4, the air-standard efficiency (in percent) is_______
Answer: 58 to 62
V1
Exp: Compression ratio = 17 = = V1 = 17V2 V3
V2 P 3
2
V3
ww
where ρ = cut of w .en
ratio =gineer
V2
ingonyour
finger
tips.
ooo
10
V3 − V2 = VS
100
V3 − V2 = 0.1( V1 − V2 ) = 0.1(17V2 − V2 ) = 0.1(16V2 )
4
V3 = 1.6V2 + V2
V3 = 2.6V2
1
V3
= ρ = 2.6
V2
Vs
 ρr − 1 
1 V2
η =1− r −1   V1 V
r (r)  ρ −1 
1  2.61.4 − 1 
=1− ×   = 0.5960 = 59.60%.
1.4 (17 )
1.4 −1
 2.6 − 1 

www.engineeringonyourfingertips.ooo
 India’s No.1 institute for GATE Training  1 Lakh+ Students trained till date  65+ Centers across India
23

More Notes Join us Telegram-: http//:t.me/allexammentor


www.engineeringonyourfingertips.ooo
ME-GATE-2014 PAPER-03| www.gateforum.com

50. Consider the given project network, where numbers along various activities represent the
normal time. The free float on activity 4-6 and the project duration, respectively, are
2
2 5
3
3
2 5
1 3 5 6 7

4
2
4

(A) 2, 13 (B) 0, 13 (C) -2, 13 (D) 2, 12

Answer: (A) 3
2 5
Exp: 2 5
3

0 2 13
2 5 7 8 5
1 3 6 7
6

2 2 4

4
For 4-6
F.F = (E5-Ei) – Ti5 = (8-2) – 4 = 2
∴ 2, 13.

51. A manufacturer can produce 12000 bearings per day. The manufacturer received an order of
8000 bearings per day from a customer. The cost of holding a bearing in stock is Rs.0.20 per
month. Setup cost per production run is Rs.500. Assuming 300 working days in a year, the
frequency of production run should be
(A) 4.5 days
www. e(B)
ngi 4.5 months
neer ingonyo (C) 6.8 days
urfinger t i
ps. ooo(D) 6.8 months
Answer: (C)
52. A cylindrical blind riser with diameter d and height h, is placed on the top of the mold cavity
of a closed type sand mold as shown in the figure. If the riser is of constant volume, then the
rate of solidification in the riser is the least when the ratio h/d is

Sprue ba sin
d

Riser h

Mold cavity

(A) 1:2 (B) 2:1 (C) 1:4 (D) 4:1


Answer: (A)

www.engineeringonyourfingertips.ooo
 India’s No.1 institute for GATE Training  1 Lakh+ Students trained till date  65+ Centers across India
24

More Notes Join us Telegram-: http//:t.me/allexammentor


www.engineeringonyourfingertips.ooo
ME-GATE-2014 PAPER-03| www.gateforum.com

π 2
Exp: v= Dh
4
4v d
h= ____ (1) h
πD 2
π 4v π 4v π 2
A S = πDh + D 2 = πD 2 + D 2 = + D
4 πD 4 D 4
For min A S
dAS
=0
dD
−4v π 4v dπ
2
+ 2D = 0 ⇒ 2 =
D 4 D 2
πD3 π 2
v= = Dh
8 4
h 1
D = 2h ⇒ =
D 2
h : D = 1: 2.

53. The diameter of a recessed ring was measured by using two spherical balls of diameter d2 =60
mm and d1=40 mm as shown in the figure.

H1
H2
d1 Diameter
C

H
A B
R
Re cessed Ring

www.
engi
neer
ingonyour
finger
tips.
ooo
d 2 Diameter

The distance H2 = 35.55 mm and H1 = 20.55 mm. The diameter (D, in mm) of the ring gauge
is _______
Answer: 92 to 94
θ θ
Exp: D ring = d1 sec + 2 ( h1 + r1 ) Tan
2 2
∴θ = 60°
= 4 − sec30 + 2 ( 20.55 + 20 ) tan 30
D ring = 93 mm.

www.engineeringonyourfingertips.ooo
 India’s No.1 institute for GATE Training  1 Lakh+ Students trained till date  65+ Centers across India
25

More Notes Join us Telegram-: http//:t.me/allexammentor


www.engineeringonyourfingertips.ooo
ME-GATE-2014 PAPER-03| www.gateforum.com

54. Which pair of following statements is correct for orthogonal cutting using a single-point
cutting tool?
P. Reduction in friction angle increases cutting force
Reduction in friction angle decreases cutting force
R. Reduction in friction angle increases chip thickness
S. Reduction in friction angle decreases chip thickness
(A) P and R (B) P and S (C) Q and R (D) Q and S
Answer: (D)

55. For spot welding of two steel sheets (base metal) each of 3 mm thickness, welding current of
10000 A is applied for 0.2s. The heat dissipated to the base metal is 1000 J. Assuming that
the heat required for melting 1 mm3 volume of steel is 20 J and interfacial contact resistance
between sheets is 0.000 2 Ω , the volume (in mm3) of weld nugget is _______
Answer: 140 to 160
H.R
Exp. I 2 RZ = p × volume of nugget ×
g

1400
100002 × 0.0002 × 0.2 = 1 × volume of nugget ×
1000
volume of nugget = 2857.1 J.mm3
1 mm3 volume of steel is 20J
2857.1 J.mm3
volume of nugget = . = 142.8 mm3 .
20 J

www.
engi
neer
ingonyour
finger
tips.
ooo

www.engineeringonyourfingertips.ooo
 India’s No.1 institute for GATE Training  1 Lakh+ Students trained till date  65+ Centers across India
26

More Notes Join us Telegram-: http//:t.me/allexammentor


www.engineeringonyourfingertips.ooo
ME-GATE-2014 PAPER-04| www.gateforum.com

Q. No. 1 – 5 Carry One Mark Each

1. Which of the following options is the closest in meaning to the word underlined in the
sentence below?
In a democracy, everybody has the freedom to disagree with the government.
(A) dissent (B) descent (C) decent (D) decadent
Answer: (A)

2. After the discussion, Tom said to me, 'Please revert!’ He expects me to _________.
(A) retract (B) get back to him
(C) move in reverse (D) retreat
Answer: (B)

3. While receiving the award, the scientist said, "I feel vindicated". Which of the following is
closest in meaning to the word ‘vindicated’?
(A) punished (B) substantiated (C) appreciated (D) chastened
Answer: (B)

4. Let f ( x, y ) = x n y m = P. If x is doubled and y is halved, the new value of f is

( A) 2n− m P ( B) 2 m− n P ( C) 2 ( n − m ) P ( D) 2 ( m − n ) P
Answer: (A)
m
1
Exp. P ' = 2n X n   y m
2
= 2n − m X n Y m = 2n − m P
www.
engi
neer
ingonyour
finger
tips.
ooo
5. In a sequence of 12 consecutive odd numbers, the sum of the first 5 numbers is 425. What is
the sum of the last 5 numbers in the sequence?
Answer: 495
Exp. 8th observation is 7×2=14 more than 1st observation
9th observation is 14 more than 2nd observation
10th observation is 14 more than 3rd observation
11th observation 14 more than 4th observation
12th observation 14 more than 5th observation
Total 14×5=70
Sum of the first five numbers =425
Sum of last five numbers =495

www.engineeringonyourfingertips.ooo
 India’s No.1 institute for GATE Training  1 Lakh+ Students trained till date  65+ Centers across India
1

More Notes Join us Telegram-: http//:t.me/allexammentor


www.engineeringonyourfingertips.ooo
ME-GATE-2014 PAPER-04| www.gateforum.com

Q. No. 6 – 10 Carry Two Marks Each

6. Find the next term in the sequence: 13M, 17Q, 19S, ___
(A) 21W (B) 21V (C) 23W (D) 23V
Answer: (C)

7. If ‘KCLFTSB’ stands for ‘best of luck’ and ‘SHSWDG’ stands for ‘good wishes’, which of
the following indicates ‘ace the exam’?
(A) MCHTX (B) MXHTC (C) XMHCT (D) XMHTC
Answer: (B)
Exp. KCLFTSB: BST-Best, F-Of, LCK-Luck (Reverse order)
SHSWDG: GD-Good, WSHS-Wishes (Reverse order)
Similarly “ace the Exam’- C-Ace, T-The, XM-Exam

8. Industrial consumption of power doubled from 2000-2001 to 2010-2011. Find the annual rate
of increase in percent assuming it to be uniform over the years.
(A) 5.6 (B) 7.2 (C) 10.0 (D) 12.2
Answer: (B)

9. A firm producing air purifiers sold 200 units in 2012. The following pie chart presents the
share of raw material, labour, energy, plant & machinery, and transportation costs in the total
manufacturing cost of the firm in 2012. The expenditure on labour in 2012 is Rs. 4,50,000. In
2013, the raw material expenses increased by 30% and all other expenses increased by 20%.
What is the percentage increase in total cost for the company in 2013?

Transp
ortatio Labour
n 10% 15%

Plant and Raw Material


machinery 20%
www.
engi
n eer
30% ingonyour
finger t ips.ooo
Energy
25%
Answer: 22
Exp.
2012 2013
Transport (10%) 300,000 360,000
Labour (15%) 450,000 540,000
Raw material (20%) 750,000 780,000
Energy (25%) 750,000 900,000
Plant and Machinery (30%) 900,000 1,080,000
Total 3,000,000 3,660,000
Percentage increase in total cost =22%

www.engineeringonyourfingertips.ooo
 India’s No.1 institute for GATE Training  1 Lakh+ Students trained till date  65+ Centers across India
2

More Notes Join us Telegram-: http//:t.me/allexammentor


www.engineeringonyourfingertips.ooo
ME-GATE-2014 PAPER-04| www.gateforum.com

10. A five digit number is formed using the digits 1,3,5,7 and 9 without repeating any of them.
What is the sum of all such possible five digit numbers?
(A) 6666660 (B) 6666600 (C) 6666666 (D) 6666606
Answer: (B)
Exp. The digit in unit place is selected in 4! Ways
The digit in tens place is selected in 4! Ways
The digit in hundreds place is selected in 4! Ways
The digit in thousands place is selected in 4! Ways
The digit in ten thousands place is selected in 4! Ways
Sum of all values for 1
4!× 1 × (100 + 101 + 102 + 103 + 104 )
= 4!× 11111 × 1
Similarly for ‘3’ 4 ! × (11111) × 3
Similarly for ‘5’ 4 ! × (11111) × 5
Similarly for ‘7’ 4 ! × (11111) × 7
Similarly for ‘9’ 4 ! × (11111) × 9

∴ sum of all such numbers = 4! × (11111) × (1 + 3 + 5 + 7 + 9 )

= 24 × (11111) × 25 = 6666600

www.
engi
neer
ingonyour
finger
tips.
ooo

www.engineeringonyourfingertips.ooo
 India’s No.1 institute for GATE Training  1 Lakh+ Students trained till date  65+ Centers across India
3

More Notes Join us Telegram-: http//:t.me/allexammentor


www.engineeringonyourfingertips.ooo
ME-GATE-2014 PAPER-04| www.gateforum.com

Q. No. 1 – 25 Carry One Mark Each

1. Which one of the following equations is a correct identity for arbitrary 3×3 real matrices P, Q
and R?

( A) P ( Q + R ) = PQ + RP ( B) ( P − Q)2 = P2 − 2PQ + Q2
(C) det ( P + Q ) = det P + det Q ( D) ( P + Q)
2
= P 2 + PQ + QP + Q 2

Answer: (D)
2
( x − 1) 2 sin ( x − 1) dx
2. The value of the integral ∫0 ( x − 1) 2 + cos ( x − 1) is

(A) 3 (B) 0 (C) -1 (D) -2


Answer: (B)

∫ ( x − 1) sin ( x − 1)
2 2

Exp: Let I = 0
dx
( x − 1) + cos ( x − 1)
2

( 2 − x − 1) sin ( 2 − x − 1)
2
2
We know that , ∫0 ( 2 − x − 1)2 + cos ( 2 − x − 1) dx
(1 − x ) sin (1 − x )
2
2
= ∫0 (1 − x )2 + cos (1 − x ) dx
2 ( x − 1) sin ( x − 1)
2

= −∫ dx = −I
( x − 1) + cos ( x − 1)
0 2

⇒ I + I = 0 ⇒ 2I = 0 ⇒ I = 0

dy
3. The solution w
of w
thew .
engvalue
initial i
neer
ingondx
problem yo =u
−rf
ingye(0)
2xy; rti p
=s
2.
ooo
is

(A) 1 + e− x (B) 2e − x (C) 1 + e x


2 2 2 2
(D) 2ex
Answer: (B)
dy
Exp: Given = − 2xy , y ( 0 ) = 2 is
dx
dy
⇒ + 2xy = 0
dx
dy
comparing with + py = Q
dx
P = 2x; Q = 0

I.F = e ∫ = e∫
pdx 2xdx
= ex
2

www.engineeringonyourfingertips.ooo
 India’s No.1 institute for GATE Training  1 Lakh+ Students trained till date  65+ Centers across India
4

More Notes Join us Telegram-: http//:t.me/allexammentor


www.engineeringonyourfingertips.ooo
ME-GATE-2014 PAPER-04| www.gateforum.com

Solution is y ( IF ) = ∫ Q ( IP ) dx + C

= 0+C
2
y ex
∴ y = Ce − x
2

Given y ( 0 ) = 2 ⇒ 2 = C
∴ y = 2e − x
2

4. A nationalized bank has found that the daily balance available in its savings accounts follows
a normal distribution with a mean of Rs. 500 and a standard deviation of Rs. 50. The
percentage of savings account holders, who maintain an average daily balance more than Rs
500 is _______
Answer: 49 to 51
Exp: Given M = 500
σ = 50
P ( X > 500 ) = ? 0.50 0.50

Where X follows normal distribution


We know that standard normal variable
x = 500
x−M z=0
z=
σ
500 − 500
z= = 0
50
∴ ( X > 500 ) = P ( z > 0 ) = 0.50 (see figure)

s
5. Laplace transform of cos ( ω t ) is . The laplace transform of e-2t cos(4t) is
s +ω
2 2

s−2 s+2
( A) ( B)
( s − 2 ) 2 + 16 ( s − 2 )2 + 16
s − 2www. engi
neer
ingonyour f i
nge sr+t i
2ps .ooo
( C) ( D)
( s + 2 ) + 16
2
( s + 2 ) + 16
2

Answer: (D)

Exp: We know that if L {f ( t )} = F ( s )

Then L {eat f ( t )} = F ( s − a )
s+2
∴ L {e −2t cos 4t} =
(s + 2) + 42
2

s+2
=
(s + 2) +16
2

www.engineeringonyourfingertips.ooo
 India’s No.1 institute for GATE Training  1 Lakh+ Students trained till date  65+ Centers across India
5

More Notes Join us Telegram-: http//:t.me/allexammentor


www.engineeringonyourfingertips.ooo
ME-GATE-2014 PAPER-04| www.gateforum.com

6. In a statically determinate plane truss, the number of joints (j) and the number of members
(m) are related by
(A) j = 2m – 3 (B) m = 2j + 1 (C) m = 2j – 3 (D) m = 2j – 1
Answer: (C)

7. If the Poisson's ratio of an elastic material is 0.4, the ratio of modulus of rigidity to Young's
modulus is _______
Answer: 0.35 to 0.36
Exp: From E=2G(1+ µ )
G 1 1
= = = 0.357
E 2(1 + µ) 2(1 + 0.4)

8. Which one of the following is used to convert a rotational motion into a translational motion?
(A) Bevel gears (B) Double helical gears
(C) Worm gears (D) Rack and pinion gears
Answer: (D)

9. The number of independent elastic constants required to define the stress-strain relationship
for an isotropic elastic solid is _______
Answer: 1.9 to 2.1

10. A point mass is executing simple harmonic motion with an amplitude of 10 mm and
frequency of 4 Hz. The maximum acceleration (m/s2) of the mass is _______
Answer: 6.3 to 6.4
Exp: Given A=10 mm=0.01 m
f =4Hz
x=Acosωt

dx
v= = −Aω
wsin
ww ωt.
engi
neer
ingonyour
finger
tips.
ooo
dt
d2x
= a = −Aω2 cos ωt
dt 2
For maximum, t=0

∴ a = Aω2 = 0.01( 2πf )


2

∵ ω = 2πf = 0.01( 2 × π × 4 ) = 6.32 m / s 2


2

11. Ball bearings are rated by a manufacturer for a life of 106 revolutions. The catalogue rating of
a particular bearing is 16 kN. If the design load is 2 kN, the life of the bearing will be p × 106
revolutions, where p is equal to _______
Answer: 500 to 540

www.engineeringonyourfingertips.ooo
 India’s No.1 institute for GATE Training  1 Lakh+ Students trained till date  65+ Centers across India
6

More Notes Join us Telegram-: http//:t.me/allexammentor


www.engineeringonyourfingertips.ooo
ME-GATE-2014 PAPER-04| www.gateforum.com

k
c
Exp: L =   × 106 k = 3 (Ball bearing)
w
3
 16 
=   × 106 = 83 × 106 = P × 106
 2

P = 512

12. As the temperature increases, the thermal conductivity of a gas


(A) increases
(B) decreases
(C) remains constant
(D) increases up to a certain temperature and then decreases
Answer: (A)
nV λ CV
Exp: K=
3N A
Where K is thermal conductivity
V is mean particle speed
λ is mean free path
CV is molar head capacity
NA is Avogadro’s number
n is particles per unit volume
Gases transfer heat by direct collisions between molecules. As the temperature increases, the
thermal conductivity increases due to increase in speed, movement and collisions in the
molecules. From the above expression, by increasing mean particle speed, the thermal
conductivity increases.

13. A reversed Carnot cycle refrigerator maintains a temperature of -5OC. The ambient air
temperature is 35OC. The heat gained by the refrigerator at a continuous rate is 2.5 kJ/s. The
power (in watt) required to pump this heat out continuously is _______
Answer: 370 to 375
Q1 Q www. engi
neer
ingonyour
finger
tips.
ooo
Exp: = 2
268 308
2.5 × 308
Q2 =
268
Q 2 = 2.873 kw
w = Q 2 − Q1 = 0.373 kw = 373.13 watt.

14. A flow field which has only convective acceleration is


(A) a steady uniform flow (B) an unsteady uniform flow
(C) a steady non-uniform flow (D) an unsteady non-uniform flow
Answer: (C)
Exp: Convective acceleration is the effect of time independent acceleration of fluid with respect to
space that means flow is steady non-uniform flow.

www.engineeringonyourfingertips.ooo
 India’s No.1 institute for GATE Training  1 Lakh+ Students trained till date  65+ Centers across India
7

More Notes Join us Telegram-: http//:t.me/allexammentor


www.engineeringonyourfingertips.ooo
ME-GATE-2014 PAPER-04| www.gateforum.com

15. Match Group A with Group B:


Group A Group B
P: Biot number 1: Ratio of buoyancy to viscous force
Q: Grashof number 2: Ratio of inertia force to viscous force
R: Prandtl number 3: Ratio of momentum to thermal diffusivities
S: Reynolds number 4: Ratio of internal thermal resistance to boundary layer
thermal resistance
(A) P-4, Q-1, R-3, S-2 (B) P-4, Q-3, R-1, S-2
(C) P-3, Q-2, R-1, S-4 (D) P-2, Q-1, R-3, S-4
Answer: (A)

16. Kaplan water turbine is commonly used when the flow through its runner is
(A) axial and the head available is more than 100 m
(B) axial and the head available is less than 10 m
(C) radial and the head available is more than 100 m
(D) mixed and the head available is about 50 m
Answer: (B)
Exp: Kaplan turbine is an axial flow turbine and works at low heads (<50).

17. Moist air at 35OC and 100% relative humidity is entering a psychometric device and leaving
at 25OC and 100% relative humidity. The name of the device is
(A) Humidifier (B) Dehumidifier (C) Sensible heater (D) Sensible cooler
Answer: (B)
Exp:
1
2 ω

www.
engi
neer
ingonyour
finger
tips.
ooo
T

Since water content in moist air is reducing. The device is de-humidifier.

18. The total number of decision variables in the objective function of an assignment problem of
size n × n (n jobs and n machines) is
(A) n2 (B) 2n (C) 2 n - 1 (D) n
Answer: (A)
Exp: A, n×n assignment problem, if it is solved as a LPP it will have n2 variables.

www.engineeringonyourfingertips.ooo
 India’s No.1 institute for GATE Training  1 Lakh+ Students trained till date  65+ Centers across India
8

More Notes Join us Telegram-: http//:t.me/allexammentor


www.engineeringonyourfingertips.ooo
ME-GATE-2014 PAPER-04| www.gateforum.com

19. Demand during lead time with associated probabilities is shown below:
Demand 50 70 75 80 85
Probability 0.15 0.14 0.21 0.20 0.30
Expected demand during lead time is _______
Answer: 74 to 75
Exp: Pr obability
xi fi x i fi
50 0.15 7.5
70 0.14 9.8
75 0.21 15.75
80 0.2 16
85 0.3 25.5
Expected demand during lead time = 74.55.

20. Within the Heat Affected Zone (HAZ) in a fusion welding process, the work material
undergoes
(A) microstructural changes but does not melt
(B) neither melting nor microstructural changes
(C) both melting and microstructural changes after solidification
(D) melting and retains the original microstructure after solidification
Answer: (A)
21. Two separate slab milling operations, 1 and 2, are performed with identical milling cutters.
The depth of cut in operation 2 is twice that in operation 1. The other cutting parameters are
identical.
The ratio of maximum uncut chip thicknesses in operations 1 and 2 is _______
Answer: 0.70 to 0.72
www.
engi
neer
ingonyour
finger
tips.
ooo
d1
Exp: Ratio of 1 and 2 is = ∴ d 2 = 2d1
d2
d1
=
2d1
1
= = 0.707.
2

22. The principle of material removal in Electrochemical machining is


(A) Fick's law (B) Faraday's laws
(C) Kirchhoff’s laws (D) Ohm’s law
Answer: (B)

www.engineeringonyourfingertips.ooo
 India’s No.1 institute for GATE Training  1 Lakh+ Students trained till date  65+ Centers across India
9

More Notes Join us Telegram-: http//:t.me/allexammentor


www.engineeringonyourfingertips.ooo
ME-GATE-2014 PAPER-04| www.gateforum.com

23. Better surface finish is obtained with a large rake angle because
(A) the area of shear plane decreases resulting in the decrease in shear force and cutting force
(B) the tool becomes thinner and the cutting force is reduced
(C) less heat is accumulated in the cutting zone
(D) the friction between the chip and the tool is less
Answer: (A)

24. Match the heat treatment processes (Group A) and their associated effects on properties
(Group B) of medium carbon steel:

Group A Group B

P: Tempering I: Strengthening and grain refinement

Q: Quenching II: Inducing toughness

R: Amealing III: Hardening

S: Normalizing IV: Softening

(A) P-III, Q-IV, R-II, S-I (B) P-II, Q-III, R-IV, S-I
(C) P-III, Q-II, R-IV, S-I (D) P-II, Q-III, R-I, S-IV
Answer: (B)
25. In a rolling process, the maximum possible draft, defined as the difference between the initial
and the final thickness of the metal sheet, mainly depends on which pair of the following
parameters
P: Strain
Q: Strength of the work material
R: Roll diameter
S: Roll velocity
T: Coefficientwof
ww .engbetween
friction ineer i
ngo
roll ny
and our
work finger tips. ooo
(A) Q, S (B) R, T (C) S, T (D) P, R
Answer: (B)

Q. No. 26 – 55 Carry Two Marks Each

3i dz
26. If z is a complex variable, the value of ∫ 5 z
is

(A) - 0.511-1.57i (B) - 0.511+1.57i


(C) 0.511- 1.57i (D) 0.511+1.57i
Answer: (B)
dz 3i
= ln ( z )
3i
Exp: ∫ 5 z 5

www.engineeringonyourfingertips.ooo
 India’s No.1 institute for GATE Training  1 Lakh+ Students trained till date  65+ Centers across India
10

More Notes Join us Telegram-: http//:t.me/allexammentor


www.engineeringonyourfingertips.ooo
ME-GATE-2014 PAPER-04| www.gateforum.com

π
= ln 3i − ln5 = ln3 + i − ( n5 + i0 )
2
∴ lnz = lnr + iθ where r = x 2 + y 2 , θ = argz 
 
22
= ln3 − ln5 + i = − 0.511 +1.57i
14

2
x
27. The value of integral ∫∫
0
0
e x + y dydx is

2
1
( e − 1)
1 2
( ) (
1 2
) 1  1
2
(A) (B) e −1 (C) e −e (D)  e − 
2 2 2 2 e
Answer: (B)
Exp. ∫∫
0
2 x

0
e x + y dy dx = ∫
0
2
ex ( ∫ e dy ) dx
x

0
y

e x ( e y ) dx e x ( e x − 1) dx
2 x 2
= ∫
0 0
= ∫ 0
2
 e 2x x 
= ∫ ( e − e ) dx = 
2
2x x
−e 
0
 2 0
e4 2 1 e4 1
= − e − +1 = − e 2 +
2 2 2 2
= ( e 4 − 2e 2 +1) = ( e 2 − 1)
1 1 2

2 2

28. The number of accidents occurring in a plant in a month follows Poisson distribution with
mean as 5.2. The probability of occurrence of less than 2 accidents in the plant during a
randomly selected month is
(A) 0.029 (B) 0.034 (C) 0.039 (D) 0.044
Answer: (B)
www.
engi
neer
ingonyour
finger
tips.
ooo
Exp: Given λ = 5.2
Let x be random variable which follows Poisson’s distribution
P ( x < 2 ) = P ( x = 0 ) + P ( x = 1)
e −λ λ 0 e −λ 1 −5.2
= + λ = e ( 6.2 ) = 0.0055 × 6.2 = 0.034
0! 1!

dx
29. Consider an ordinary differential equation . = 4t + 4. If x = x0 at t = 0, the increment in x
dt
calculated using Runge-Kutta fourth order multi-step method with a step size of ∆t = 0.2 is
(A) 0.22 (B) 0.44 (C) 0.66 (D) 0.88
Answer: (D)

www.engineeringonyourfingertips.ooo
 India’s No.1 institute for GATE Training  1 Lakh+ Students trained till date  65+ Centers across India
11

More Notes Join us Telegram-: http//:t.me/allexammentor


www.engineeringonyourfingertips.ooo
ME-GATE-2014 PAPER-04| www.gateforum.com

dx
Exp: Given = 4t + 4 x = x o at t = 0
dt
n = 0.2
Calculate x(0.2) value
K1 = f ( t 0 , x 0 ) = f ( 0, x 0 ) = 4

 h 1 
K 2 = f  t 0 + , x 0 + K 1h 
 2 2 
= f ( 0 + 0.1, x 0 + 0.4 ) = f ( 001, x 0 + 0.4 ) = 4 ( 0.1) + 4 = 4.4

 h K h
K3 = f  x0 + , x0 + 2 
 2 2 
= f ( t 0 + 0.1, x 0 + ( 2.2 ) ( 0.2 ) ) = f ( 0.1, x 0 + 0.44 ) = 4 ( 0.1) + 4 = 4.4

K 4 = f ( t 0 + h, x 0 + K 3 h )
= f ( 0 + 0.2, x 0 + 0.88 ) = f ( 0.2, x 0 + 0.88 ) = 4 ( 0.2 ) + 4 = 4.8
h
x ( 0.2 ) = x1 = x 0 + ( K1 + 2K 2 + 2K 3 + K 4 )
6

( 4 + 2 ( 4.4 ) + 2 ( 4.4 ) + ( 4.8) )


0.2
= x0 +
6

( 4 + 2 ( 4.4 ) + 2 ( 4.4 ) + ( 4.8) )


0.2
= x0 +
6
0.2
= x0 + ( 4 + 8.8 + 8.8 + 4.8) = x 0 + 0.88
6
Increment as x = x1 − x 0 = x 0 + 0.88 − x 0 = 0.88

30. A shaft is subjected to pure torsional moment. The maximum shear stress developed in the
shaft is 100 MPa. The yield and ultimate strengths of the shaft material in tension are 300
MPa and 450w ww.
MPa, engi neer
respectively. ingfactor
The onyo ofurfingusing
safety er
tips.ooo distortion energy (von-
maximum
Mises) theory is _______
Answer: 1.7 to 1.8
σu
Exp: τmax =
2FOS
300
⇒ FOS = = 1.5
2 × 100

31. A thin gas cylinder with an internal radius of 100 mm is subject to an internal pressure of 10
MPa. The maximum permissible working stress is restricted to 100 MPa. The minimum
cylinder wall thickness (in mm) for safe design must be _______
Answer: 9.8 to 10.6
pd 10 × 2 × 100
Exp: σ1 = ⇒ 100 = ⇒ t = 10 mm
2t 2× t

www.engineeringonyourfingertips.ooo
 India’s No.1 institute for GATE Training  1 Lakh+ Students trained till date  65+ Centers across India
12

More Notes Join us Telegram-: http//:t.me/allexammentor


www.engineeringonyourfingertips.ooo
ME-GATE-2014 PAPER-04| www.gateforum.com

32. For the truss shown in the figure, the forces F1 and F2 are 9 kN and 3 kN, respectively. The
force (in kN) in the member QS is

F1 F2
3 3
P Q R

All dim ensions are in m

1.5 3

(A) 11.25 tension (B) 11.25 compression


(C) 13.5 tension (D) 13.5 compression
Answer: (A)
Exp: By method of sections

F1

Q R
P

FQS sin 53.1

θ = 53.1
∑V = 0 S T

F1 = FQS sin 53.1

⇒ FQS = 11.25(T)

33. ww
It is desired to w.
avoide ngi neer
interferencein
ing aonyo
pair ofurfi
ng
spur er t
gears i
ps. oo
having ao
20O pressure angle. With
increase in pinion to gear speed ratio, the minimum number of teeth on the pinion
(A) increases (B) decreases
(C) first increases and then decreases (D) remains unchanged
Answer: (A)

34. A uniform slender rod (8 m length and 3 kg mass) rotates in a vertical plane about a
horizontal axis 1 m from its end as shown in the figure. The magnitude of the angular
acceleration (in rad/s2) of the rod at the position shown is _______

1m 7m
Answer: 1.9 to 2.1

www.engineeringonyourfingertips.ooo
 India’s No.1 institute for GATE Training  1 Lakh+ Students trained till date  65+ Centers across India
13

More Notes Join us Telegram-: http//:t.me/allexammentor


www.engineeringonyourfingertips.ooo
ME-GATE-2014 PAPER-04| www.gateforum.com

35. A bolt of major diameter 12 mm is required to clamp two steel plates. Cross sectional area of
the threaded portion of the bolt is 84.3 mm2. The length of the threaded portion in grip is 30
mm, while the length of the unthreaded portion in grip is 8 mm. Young's modulus of material
is 200 GPa. The effective stiffness (in MN/m) of the bolt in the clamped zone is _______
Answer: 460 to 470
Exp: d1 = 12 mm ;
l1 = 8mm
A 2 = 84.33mm2
π
A1 = (d1 )2
4
A 2 = 84.33 mm 2
l2 = 30mm
A1E1 A E 84.33 × 200
K1 = ; K2 = 2 2 = = 562.2
l1 l2 30

π
(12) 2 × 200
= 4 = 2827.4
8
1 1 1 1 1
= + = +
K K1 K 2 2827.4 562.2

⇒ K = 468.9 MN / m

36. A frame is subjected to a load P as shown in the figure. The frame has a constant flexural
rigidity EI. The effect of axial load is neglected. The deflection at point A due to the applied
load P is

www.
engi
neer
ingonyour
finger
tips.
ooo

1 PL3 2 PL3 PL3 4 PL3


( A) ( B) ( C) ( D)
3 EI 3 EI EI 3 EI
Answer: (D)

www.engineeringonyourfingertips.ooo
 India’s No.1 institute for GATE Training  1 Lakh+ Students trained till date  65+ Centers across India
14

More Notes Join us Telegram-: http//:t.me/allexammentor


www.engineeringonyourfingertips.ooo
ME-GATE-2014 PAPER-04| www.gateforum.com

2
M 2x dx L M y dx
Strain energy ( E ) = ∫
L
Exp: +∫
0 2EI 0 2EI
M X = PX M Y = PL
2 2
1 L (PX) dx L (PL) dx
Pδ = ∫ +∫
2 0 2EI 0 2EI
L
 X3 
P2   2 2
 6 0 P L 2 P 2 L3 1 4 PL3
= + [ ]0
L
X = = × P ×
EI 2EI 3 EI 2 3 EI

4 PL3
δ=
3 EI

37. A wardrobe (mass 100 kg, height 4 m, width 2 m, depth 1 m), symmetric about the Y-Y axis,
stands on a rough level floor as shown in the figure. A force P is applied at mid-height on the
wardrobe so as to tip it about point Q without slipping. What are the minimum values of the
force (in Newton) and the static coefficient of friction µ between the floor and the wardrobe,
respectively?

2m
P 4m

Q
Y

(A) 490.5 and 0.5 (B) 981 and 0.5


(C) 1000.5 and 0.15 (D) 1000.5 and 0.25
Answer: (A) www. engi
neer
ingonyour finger tips. ooo
Exp: Taking moments about Q, ∑ MQ = 0
⇒ W × 1m = P × 2m
100 × 9.81
⇒P= = 490.5 N
2
∑H = 0
⇒ FF = P = 490.5 N

∑ V = 0 ⇒ R N = mg = 981 N

Friction Force = µR N

490.5
µ= = 0.5
981

www.engineeringonyourfingertips.ooo
 India’s No.1 institute for GATE Training  1 Lakh+ Students trained till date  65+ Centers across India
15

More Notes Join us Telegram-: http//:t.me/allexammentor


www.engineeringonyourfingertips.ooo
ME-GATE-2014 PAPER-04| www.gateforum.com

38. Torque and angular speed data over one cycle for a shaft carrying a flywheel are shown in the
figures. The moment of inertia (in kg.m2) of the flywheel is _______

Angular speed
Torque

20 rad / s
N−m
10 rad / s
π 3π / 2 2π
π/2 θ
0 θ
−1500 0 π/2 π 3π / 2 2π
N−m

Answer: 30 to 32
ωmax + ωmin 20 + 0
Exp: ω= = = 10
2 2
ω − ωmin 20 − 0
Cs = max = =2
ω 10
π
∆E = Area of T − θ diagram = × 3000 + 1500 × π = 3000π
2
∆E = Iω2 Cs
⇒ 3000π = I(10)2 × 2
⇒ I = 47.12

39. A single degree of freedom system has a mass of 2 kg, stiffness 8 N/m and viscous damping
ratio 0.02. The dynamic magnification factor at an excitation frequency of 1.5 rad/s is
_______
Answer: 2.0 to 2.4
C C
Exp: Damping ratio = 0.02 = =
www. en2m
gin
ωen e r
ingonSyour finger t ips. ooo
2×8×
m
8
⇒ C = 0.02 × 2 × 8 × = 0.32
8
1
Dynamic magnification factor =
2
c 2 ω2  ω2 
+ 1 − 2 
s2  ωn 
1
= = 0.799
(0.32) 2 × (1.5) 2  (1.5) 2 
+ 1 −
(8) 2  (1) 2 

www.engineeringonyourfingertips.ooo
 India’s No.1 institute for GATE Training  1 Lakh+ Students trained till date  65+ Centers across India
16

More Notes Join us Telegram-: http//:t.me/allexammentor


www.engineeringonyourfingertips.ooo
ME-GATE-2014 PAPER-04| www.gateforum.com

40. A ladder AB of length 5 m and weight (W) 600 N is resting against a wall. Assuming
frictionless contact at the floor (B) and the wall (A), the magnitude of the force P (in Newton)
required to maintain equilibrium of the ladder is _______
A
2.5m

3m
2.5m

W
P
Answer: 399 to 401 B
Exp: Taking moments about B 4m
∑ MB = 0
⇒ W × 2 = RA × 3

600 × 2
⇒ RA = = 400
3

∑H = 0 ⇒ R A = P = 400 N

41. A closed system contains 10 kg of saturated liquid ammonia at 10°C. Heat addition required
to convert the entire liquid into saturated vapour at a constant pressure is 16.2 MJ. If the
entropy of the saturated liquid is 0.88 kJ/kg.K, the entropy (in kJ/kg.K) of saturated vapour is
_______
Answer: 6.4 to 6.7
Exp: Tds = du + pdv ( closed system )
283.15 × 10 ( S2 − S1 ) = 16.2 × 103
16.2 × 103
∴S2 = 0.88 + = 6.6013 kJ kg
283.15 × 10
www.
engi
neer
ingonyour
finger
tips.
ooo
42. A plane wall has a thermal conductivity of 1.15 W/m.K. If the inner surface is at 1100OC and
the outer surface is at 350OC, then the design thickness (in meter) of the wall to maintain a
steady heat flux of 2500 W/m2 should be _______
Answer: 0.33 to 0.35
dT
Exp: Q = KA
dx
Q
q = = 2500 W m 2
A
dT
2500 = K
dx
(1100 − 350 )
2500 = 1.15 ×
x
x = 0.345 m.

www.engineeringonyourfingertips.ooo
 India’s No.1 institute for GATE Training  1 Lakh+ Students trained till date  65+ Centers across India
17

More Notes Join us Telegram-: http//:t.me/allexammentor


www.engineeringonyourfingertips.ooo
ME-GATE-2014 PAPER-04| www.gateforum.com

43. Consider the following statements regarding streamline(s):


(i) It is a continuous line such that the tangent at any point on it shows the velocity vector at
that point
(ii) There is no flow across streamlines
dx dy dz
(iii) = = is the differential equation of a streamline, where u, v and w are velocities
u v w
in directions x, y and z, respectively
(iv) In an unsteady flow, the path of a particle is a streamline
Which one of the following combinations of the statements is true?
(A) (i), (ii), (iv) (B) (ii), (iii), (iv) (C) (i), (iii), (iv) (D) (i), (ii), (iii)
Answer: (D)

( )

44. Consider a velocity field V = K yiˆ + xkˆ , where K is a constant. The vorticity, Ω z , is

(A) - K (B) K (C) - K /2 (D) K /2


Answer: (A)
∂v ∂u
Exp: Ωz = −
∂x ∂y
( v = 0)
∂ ( ky )
∴Ω = − = − k.
∂y
z

45 Water flows through a tube of diameter 25 mm at an average velocity of 1.0m/s. The


properties of water ρ = 1000kg / m3 , µ = 7.25 × 10−4 N.s / m2 ,k = 0.625 W/mK, Pr = 4.85.
Using Nu = 0.023Re0.8 Pr0.4, the convective heat transfer coefficient (in W/m2.K) is ______
Answer: 4600 to 4625
ρVD 1000 × 1 × 25 × 10−3
Exp: Re = = −4
= 34482.758
µ ww7.25
w.e×n gi
10 neer ingonyour f
inger
tips.
ooo
Pr = 4.85
hD
Nu = 0.023 Re0.8 Pr 0.4 = 184.5466 =
k
184.5466 × 0.625
∴h = = 4613.6659 W m 2 k.
25 × 10−3

46. Two identical metal blocks L and M (specific heat = 0.4 kJ/kg.K), each having a mass of 5
kg, are initially at 313 K. A reversible refrigerator extracts heat from block L and rejects heat
to block M until the temperature of block L reaches 293 K. The final temperature (in K) of
block M is _______
Answer: 333 to 335
Exp: Given m1 = m 2 = 5Kg

www.engineeringonyourfingertips.ooo
 India’s No.1 institute for GATE Training  1 Lakh+ Students trained till date  65+ Centers across India
18

More Notes Join us Telegram-: http//:t.me/allexammentor


www.engineeringonyourfingertips.ooo
ME-GATE-2014 PAPER-04| www.gateforum.com

c1 = c 2 = 0.4KJ / Kg.K

TL,i = TM,i = 313K

TM,f = 293K,TL,f = ?

m1c1∆T1 = m 2 c 2 ∆T2

∆T1 = ∆T2

TL,i − TL,f = TM,i − TM,f

313 − TL,f = 313 − 293

TL,f = 333k
47. Steam with specific enthalpy (h) 3214 kJ/kg enters an adiabatic turbine operating at steady
state with a flow rate 10 kg/s. As it expands, at a point where h is 2920 kJ/kg, 1.5 kg/s is
extracted for heating purposes. The remaining 8.5 kg/s further expands to the turbine exit,
where h = 2374 kJ/kg. Neglecting changes in kinetic and potential energies, the net power
output (in kW) of the turbine is _______
Answer: 7580 to 7582
i i
 i i

Exp: w = m tot ( h1 − h 2 ) +  m tot − mout  ( h 2 − h 3 )
 
i
w = 10 ( 3214 − 2920 ) + 8.5 ( 2920 − 2374 )
i
w = 7581 kw.

48. Two infinite parallel plates are placed at a certain distance apart. An infinite radiation shield
is inserted between the plates without touching any of them to reduce heat exchange between
the plates. Assume that the emissivities of plates and radiation shield are equal. The ratio of
the net heat exchange between the plates with and without the shield is
(A) 1/2 (B) 1/3 (C) 1/4 (D) 1/8
www.
engi
neer
ingonyour
finger
tips.
ooo
Answer: (A)
q with sheilds 1
Exp: =
q without sheilds n +1
here, n = 1
q 1
∴ = .
q without 2

49. In a compression ignition engine, the inlet air pressure is 1 bar and the pressure at the end of
isentropic compression is 32.42 bar. The expansion ratio is 8. Assuming ratio of specific
heats ( γ ) as 1.4, the air standard efficiency (in percent) is _______
Answer: 59 to 61

www.engineeringonyourfingertips.ooo
 India’s No.1 institute for GATE Training  1 Lakh+ Students trained till date  65+ Centers across India
19

More Notes Join us Telegram-: http//:t.me/allexammentor


www.engineeringonyourfingertips.ooo
ME-GATE-2014 PAPER-04| www.gateforum.com

1
v  p γ 1
Exp: compression ratio, rk = 1 =  2  = ( 32.42 )1.4 = 11.999.9  12
v 2  p1 
compression ratio 12
cutoff ratio, rk = = = 1.5
exp ansion ratio 8

1  reγ − 1 
η =1−  
rkγ−1  γ ( rc − 1) 

1  (1.5 ) − 1 
1.4

=1−  
120.4  1.4 (1.5 − 1) 
η = 59.599%.

50. The precedence relations and duration (in days) of activities of a project network are given in
the table. The total float (in days) of activities e and f , respectively, are
Activity Predecessors Duration (days)
a - 2
b - 4
c a 2
d b 3
e c 2
f c 4
g d,e 5

(A) 0 and 4 (B) 1 and 4 (C) 2 and 3 (D) 3 and 1


8
Answer: (B)
Exp: www.
engi
neer
ingonyour
fin
f
ger
tips.
7ooo
2 4
C 4
2 4
A e
2
6 11
2 2
1 6 8
B 5 12
7
d
4 3 5

4 3 7

( TF )e = ( L j − E i ) − Tij = ( 7 − 4 ) − 2 = 1
( TF )f = (8 − 0 ) − 4 = 4

www.engineeringonyourfingertips.ooo
 India’s No.1 institute for GATE Training  1 Lakh+ Students trained till date  65+ Centers across India
20

More Notes Join us Telegram-: http//:t.me/allexammentor


www.engineeringonyourfingertips.ooo
ME-GATE-2014 PAPER-04| www.gateforum.com

51. At a work station, 5 jobs arrive every minute. The mean time spent on each job in the work
station is 1/8 minute. The mean steady state number of jobs in the system is _______
Answer: 1.62 to 1.70

52. A GO-No GO plug gauge is to be designed for measuring a hole of nominal diameter 25 mm
with a hole tolerance of ± 0.015 mm. Considering 10% of work tolerance to be the gauge
tolerance and no wear condition, the dimension (in mm) of the GO plug gauge as per the
unilateral tolerance system is
(A) 24.985+−0.003
0.003 (B) 25.015+−0.000
0.006 (C) 24.985+−0.003
0.003 (D) 24.985+−0.003
0.000

Answer: (D)
Exp: 25±0.015
25.015
GT
No Go guage
GO-Gauge
U.L = 24.988
L.L = 24.985
G.T
24.985+−0.003
0.000 W.F 24.985
Go guage

53. A cylindrical riser of 6 cm diameter and 6 cm height has to be designed for a sand casting
mould for producing a steel rectangular plate casting of 7 cm × 10 cm × 2 cm dimensions
having the total solidification time of 1.36 minute. The total solidification time (in minute) of
the riser is _______
Answer: 2.5 to 4.5

54. A cast iron block of 200 mm length is being shaped in a shaping machine with a depth of cut
of 4 mm, feed of 0.25 mm/stroke and the tool principal cutting edge angle of 30O. Number of
cutting strokes per minute is 60. Using specific energy for cutting as 1.49 J/mm3, the average
www.
power consumption en
(in gi ne
watt) iser
ingonyour
_______ finger tips. ooo
Answer: 295 to 305
FC FC
Exp: Specific cutting energy = = 1.49 J mm 3 =
b × t1 4 × 0.25
FC = 1.49 J mm. stroke
60
FC = 1.49 J mm ×
min
FC = 89.4 J mm.min
Power = FC .l = 89.4 × 200 mm J mm.min = 17880 J min
Power = 298 J S or ( W ) .

www.engineeringonyourfingertips.ooo
 India’s No.1 institute for GATE Training  1 Lakh+ Students trained till date  65+ Centers across India
21

More Notes Join us Telegram-: http//:t.me/allexammentor


www.engineeringonyourfingertips.ooo
ME-GATE-2014 PAPER-04| www.gateforum.com

55. A butt weld joint is developed on steel plates having yield and ultimate tensile strength of 500
MPa and 700 MPa, respectively. The thickness of the plates is 8 mm and width is 20 mm.
Improper selection of welding parameters caused an undercut of 3 mm depth along the weld.
The maximum transverse tensile load (in kN) carrying capacity of the developed weld joint is
_______
Answer: 68 to 72
Exp: t = 8 mm
b = 20 mm
d = 3 mm
maximum transverse tensile load = yield strength × ( b − d ) t
= 500 N mm 2 × ( 20 − 3) × 5 mm 2
= 68,000 N
= 68 kN.

www.
engi
neer
ingonyour
finger
tips.
ooo

www.engineeringonyourfingertips.ooo
 India’s No.1 institute for GATE Training  1 Lakh+ Students trained till date  65+ Centers across India
22

More Notes Join us Telegram-: http//:t.me/allexammentor


www.engineeringonyourfingertips.ooo
|ME-GATE-2013 PAPER| www.gateforum.com

Q. No. 1 – 25 Carry One Mark Each

∂u ∂u ∂2u
1. The partial differential equation +u = is a
∂t ∂x ∂x2
(A) Linear equation of order 2 (B) Non-linear equation of order 1
(C) Linear equation of order 1 (D) Non-linear equation of order 2
Answer: (D)

2. The eigen values of symmetric matrix are all


(A) Complex with non-zero positive imaginary part
(B) Complex with non-zero negative imaginary part
(C) Real
(D) Pure imaginary
Answer: (C)

3. Match the CORRECT pairs:


Numerical Integration Scheme Order of Fitting Polynomial
P. Simpson’s 3/8 Rule 1. First
Q. Trapezoidal Rule 2. Second
R. Simpson’s 1/3 Rule 3. Third
(A) P-2; Q-1; R-3 (B) P-3; Q-2; R-1 (C) P-1; Q-2; R-3 (D) P-3; Q-1; R-2
Answer: (D)

4. A rod of length L having uniform cross-sectional area A is subjected to a tensile


force P as shown in the figure below. If the Young’s modulus of the material
varies linearly from E1 to E2 along the length of the rod, the normal stress
developed at the section-SS is
S
E1 E2
www
P.engi
neer
ingonyour
finger
tips.
oooP

S
L /2

P P (E1 − E2 ) PE2 PE1


(A) (B) (C) (D)
A A (E1 + E2 ) AE1 AE2
Answer: (A)
Stress depends on area.

5. The threaded bolts A and B of same material and length are subjected to identical
tensile load. If the elastic strain energy stored in bolt A is 4 times that of the bolt B
and the mean diameter of bolt A is 12mm, the mean diameter of bolt B in mm is
(A) 16 (B) 24 (C) 36 (D) 48

www.engineeringonyourfingertips.ooo
GATEFORUM- India’s No.1 institute for GATE training
1

More Notes Join us Telegram-: http//:t.me/allexammentor


www.engineeringonyourfingertips.ooo
|ME-GATE-2013 PAPER| www.gateforum.com

Answer: (B)
Explanation:
 P2l 
 
E1  2AE 1 A2
= =
E2  P2l  A1
 
 2AE 2

d22
4=
d12
d2 = 12 × d1 = 24

6. A link OB is rotating with a constant angular velocity of 2 rad/s in counter


clockwise direction and a block is sliding radially outward on it with an uniform
velocity of 0.75 m/s with respect to the rod, as shown in the figure below. If
OA = 1m, the magnitude of the absolute acceleration of the block at location A in
m / s2 is

(A) 3 (B) 4 (C) 5 (D) 6


Answer: (C)
Explanation:
α t = 2vω = 2 × 0.75 × 2 = 3

αr =
v2
=4
www.
engi
neer
ingonyour
finger
tips.
ooo
r
∴ Re sul tan t α = 32 + 42 = 5

7. For steady, fully developed flow inside a straight pipe of diameter D, neglecting
gravity effects, the pressure drop ∆p over a length L and the wall shear stress τw
are related by
∆pD ∆pD2 ∆pD 4∆pL
(A) τw = (B) τw = (C) τw = (D) τw =
4L 4L2 2L D
Answer: (A)
Explanation:
π 2 ∆pD
τω ( πDL ) = D ⋅ ∆p ⇒ τ ω =
4 4L

www.engineeringonyourfingertips.ooo
GATEFORUM- India’s No.1 institute for GATE training
2

More Notes Join us Telegram-: http//:t.me/allexammentor


www.engineeringonyourfingertips.ooo
|ME-GATE-2013 PAPER| www.gateforum.com

8. The pressure, dry bulb temperature, and relative humidity of air in a room are
1bar, 30ºC and 70% respectively. If the saturated steam pressure at 30ºC is
4.25kPa, the specific humidity of the room air in kg water vapour / kg dry air is
(A) 0.0083 (B) 0.0101 (C) 0.0191 (D) 0.0232
Answer: (C)
Explanation:
P = 1 bar = 105 Pa = 100 KPa

Psat = 4.25KPa

0.622 × 0.7 × 4.25


Sp .humidity = = 0.019
(100 − 4.25)

9. Consider one-dimensional steady state heat conduction, without heat generation,


in a plane wall; with boundary conditions as shown in the figure below. The
conductivity of the wall is given by k = k 0 + bT ; where k 0 and b are positive
constants and T is temperature.

T2 where T2 > T1

T1

As x increases, the temperature gradient ( dT / dx ) will

(A) Remain constant (B) Be zero (C) Increase (D) Decrease


Answer: (D)

10. In a rolling process, the state of stress of the material undergoing deformation is
(A) Pure compression (B) Pure shear
www.
(C) Compression eng
and i
neer
shear i
ngonyour
finge
(D) rti
ps. oo
Tension o shear
and
Answer: (C)

11. Match the CORRECT pairs.


Processes Characteristics / Application
P. Friction Welding 1. Non-consumable electrode
Q. Gas Metal Arc Welding 2. Joining of thick plates
R. Tungsten Inert Gas Welding 3. Consumable electrode wire
S. Electroslag Welding 4. Joining of cylindrical dissimilar materials
(A) P-4;Q-3;R-1;S-2 (B) P-4;Q-2;R-3;S-1
(C) P-2;Q-3;R-4;S-1 (D) P-2;Q-4;R-1;S-3
Answer: (A)

www.engineeringonyourfingertips.ooo
GATEFORUM- India’s No.1 institute for GATE training
3

More Notes Join us Telegram-: http//:t.me/allexammentor


www.engineeringonyourfingertips.ooo
|ME-GATE-2013 PAPER| www.gateforum.com

12. A metric thread of pitch 2mm and thread angle 60º is inspected for its pitch
diameter using 3-wire method. The diameter of the best size wire in mm is
(A) 0.866 (B) 1.000 (C) 1.154 (D) 2.000
Answer: (C)
Explanations:- For 60o thread angle, best wire size =0.57135xP=1.154

13. Customers arrive at a ticket counter at a rate of 50 per hour and tickets are
issued in the order of their arrival. The average time taken for issuing a ticket is
1min. Assuming that customer arrivals form a Poisson process and service times
are exponentially distributed, the average waiting time in queue in minutes is:
(A) 3 (B) 4 (C) 5 (D) 6
Answer: (C)
Explanation:
λ
λ = 50 / hr µ = 60 / hr W.T = = 0.083hr = 5 min
µ (µ − λ )

14. In simple exponential smoothing forecasting, to give higher weightage to recent


demand information, the smoothing constant must be close to
(A) -1 (B) zero (C) 0.5 (D) 1
Answer: (D)
Explanations:- Value of α close to one have less of a smoothing effect and give greater
weight to recent changes in the data.

15. A steel bar 200 mm in diameter is turned at a feed of 0.25 mm/rev with a depth
of cut of 4 mm. The rotational speed of the workpiece is 160 rpm. The material
removal rate in mm3 / s is
(A) 160 (B) 167.6 (C) 1600 (D) 1675.5
Answer: (D)
Explanations:- f × d × v
π × 200 × 160
= ( 0.25 ) ( 4 ) × = 1675.5
www.
engi
60 neer
ingonyour
finger
tips.
ooo

16. A cube shaped casting solidifies in 5 minutes. The solidification time in minutes
for a cube of the same material, which is 8 times heavier than the original casting
will be
(A) 10 (B) 20 (C) 24 (D) 40
Answer: (B)
2 2
V V 
Explanations:- t = C   ; t1 = 5 = C  1 
A  A1 
Now V2 = 8V1
Which implies each ride is getting doubled. So A 2 = 4A1
2 2
V   V 
t 2 = c  2  = c  1  = 4 × t1 = 4 × 5 = 20 min
A
 2  4A 2 

www.engineeringonyourfingertips.ooo
GATEFORUM- India’s No.1 institute for GATE training
4

More Notes Join us Telegram-: http//:t.me/allexammentor


www.engineeringonyourfingertips.ooo
|ME-GATE-2013 PAPER| www.gateforum.com

17. For a ductile material, toughness is a measure of


(A) Resistance to scratching
(B) Ability to absorb energy up to fracture
(C) Ability to absorb energy till elastic limit
(D) Resistance to indentation
Answer: (B)
Explanations:- Since, toughness has ability to absorb energy up to fracture.

18. In order to have maximum power from a Pelton turbine, the bucket speed must
be
(A) Equal to the jet speed (B) Equal to half the jet speed
(C) Equal to twice the jet speed (D) Independent of the jet speed
Answer: (B)
Explanation: Since, velocity of bucket = ½ times the velocity of jet.

19. Consider one-dimensional steady state heat conduction along x-axis ( 0 ≤ x ≤ L ) ,


through a plane wall with the boundary surfaces ( x = 0 and x = L ) maintained at
temperatures 0º C and 100ºC. Heat is generated uniformly throughout the wall.
Choose the CORRECT statement.
(A) The direction of heat transfer will be from the surface at 100ºC to surface at
0ºC.
(B) The maximum temperature inside the wall must be greater than 100ºC
(C) The temperature distribution is linear within the wall
(D) The temperature distribution is symmetric about the mid-plane of the wall
Answer: (B)

A cylinder contains 5m3 of ideal gas at a pressure of 1 bar. This gas is


20.
www. engi neer ingonyour f
inger tips. ooo
compressed in a reversible isothermal process till its pressure increases to 5 bar.
The work in kJ required for this process is
(A) 804.7 (B) 953.2 (C) 981.7 (D) 1012.2
Answer: (A)
P2
Explanations:- P1 V1 ln = wD
P1

5
⇒ wD = 105 × 5 ln   = 804718.95 = 804.71 kJ
1

21. A long thin walled cylindrical shell, closed at both ends, is subjected to an internal
pressure. The ratio of the hoop stress (circumferential stress) to longitudinal
stress developed in the shell is
(A) 0.5 (B) 1.0 (C) 2.0 (D) 4.0

www.engineeringonyourfingertips.ooo
GATEFORUM- India’s No.1 institute for GATE training
5

More Notes Join us Telegram-: http//:t.me/allexammentor


www.engineeringonyourfingertips.ooo
|ME-GATE-2013 PAPER| www.gateforum.com

Answer: (C)
Pd
Explanations:- σhoop =
2t
Pd
σong =
4t
σhoop
=2
σlong

22. If two nodes are observed at a frequency of 1800 rpm during whirling of a simply
supported long slender rotating shaft, the first critical speed of the shaft in rpm is
(A) 200 (B) 450 (C) 600 (D) 900
Answer: (D)
Explanations:- Since it is simply supported critical speed will be half

23. A planar closed kinematic chain is formed with rigid links PQ = 2.0m, QR = 3.0m,
RS = 2.5m and SP = 2.7m with all revolute joints. The link to be fixed to obtain a
double rocker (rocker-rocker) mechanism is
(A) PQ (B) QR (C) RS (D) SP
Answer: (C)
Explanations:- Since for Rocker – Rocker mechanism the link opposite to smaller link
must be fixed

24. Let X be a nominal variable with mean 1 and variance 4. The probability P ( X < 0 ) is
(A) 0.5
(B) Greater than zero and less than 0.5
(C) Greater than 0.5 and less than 1
(D) 1.0
Answer: (B)
x − µ 0 − µ
Explanations:- P ( x < 0 ) = P  < − P ( Z < −0.5)
 σ σ 
www.
engi
neer
ingonyour
finger
tips.
ooo
= P ( Z > 0.5) = 0.5 − P ( 0 < Z < 0.5) ,
which is greater than zero and less than 0.5

25. Choose the CORRECT set of functions, which are linearly dependent.
(A) sin x, sin2 x and cos2 x (B) cosx, sinx and tan x
2 2
(C) cos 2x, sin x and cos x (D) cos2x, sinx and cosx
Answer: (C)

Explanations:- (C)
∵ cos2 x = cos2 x − sin2 x

∴ cos2 x is the linear combination of the functions ∴ cos2 x, sin2 x

∴ The functions cos2 x, sin2 x and cos2 x are linearly dependent

www.engineeringonyourfingertips.ooo
GATEFORUM- India’s No.1 institute for GATE training
6

More Notes Join us Telegram-: http//:t.me/allexammentor


www.engineeringonyourfingertips.ooo
|ME-GATE-2013 PAPER| www.gateforum.com

Q. No. 26 – 55 Carry Two Marks Each

26. The following surface integral is to be evaluated over a sphere for the given
steady velocity vector field, F = xi + yj + zk defined with respect to a Cartesian
coordinate system having i, j, and k as unit base vectors.
1
∫∫ 4 (F.n)dA
s

Where S is the sphere, x 2 + y 2 + z2 = 1 and n is the outward unit normal vector to


the sphere. The value of the surface integral is
π
(A) π (B) 2π (C) 3 (D) 4π
4
Answer: (A)
1
4 ∫∫∫
Explanations:- divF dv (Using divergence theorem)

1 3
4 ∫∫∫
= 3dv = × volume of the sphere

4
3 4
= × × (1)3 = π as radius = 1
4 3

d2 f
27. The function f(t) satisfies the differential equation + f = 0 and the auxiliary
dt 2
df
conditions, f(0) = 0, (0) = 4 . The Laplace transform of f(t) is given by
dt
2 4 4 2
(A) (B) (C) (D)
s +1 s +1 s2 + 1 s4 + 1
Answer: (C)
Explanations:-
Given Eq is f "(t) + f(t) = 0

L f "(t) + L f(t) = L(0)


www.
engi
neer
ingonyour
finger
tips.
ooo
s2F(s) − sf(0) − f '(0) + F(s) = 0
(s 2
)
+ 1 F(s) = 4
4
f(s) =
s2 + 1

28. Specific enthalpy and velocity of steam at inlet and exit of a steam turbine,
running under steady state, are as given below:
Specific enthalpy (kJ/kg) Velocity(m/s)
Inlet steam condition 3250 180
Exit steam condition 2360 5
The rate of heat loss from the turbine per kg of steam flow rate is 5 kW.
Neglecting changes in potential energy of steam, the power developed in kW by
the steam turbine per kg of steam flow rate, is
(A) 901.2 (B) 911.2 (C) 17072.5 (D) 17082.5

www.engineeringonyourfingertips.ooo
GATEFORUM- India’s No.1 institute for GATE training
7

More Notes Join us Telegram-: http//:t.me/allexammentor


www.engineeringonyourfingertips.ooo
|ME-GATE-2013 PAPER| www.gateforum.com

Answer: (C)
Explanation:
V12 V2
h1 + + dQ = h2 + 2 + dw
2 2

dw = (3250 − 2360 ) +
(180 2
− 52 ) +5
2
= 17072.5 kW

29. Water is coming out from a tap and falls vertically downwards. At the tap
opening, the stream diameter is 20mm with uniform velocity of 2 m/s.
Acceleration due to gravity is 9.81 m / s2 . Assuming steady, inviscid flow,
constant atmospheric pressure everywhere and neglecting curvature and surface
tension effects, the diameter is mm of the stream 0.5m below the tap is
approximately
(A) 10 (B) 15 (C) 20 (D) 25
Answer: (B)

30. A steel ball of diameter 60 mm is initially in thermal equilibrium at 1030 °C in a


furnace. It is suddenly removed from the furnace and cooled in ambient air at
30 °C , with convective heat transfer coefficient h=20 W / m2K. The thermo-
physical properties of steel are: density ρ = 7800 kg / m3 , conductivity
W
k = 40 and specific heat c=600 J/kgK. The time required in seconds to cool
mK
the steel ball in air from 1030 °C to 430 °C is
(A) 519 (B) 931 (C) 1195 (D) 2144
Answer: (D)
Explanations:-
 LAt 
T − Tco − 
 SC V  V
=e  p  0.01 =
Ti − Tco A
 20 × L 
430 − 30 −
=wew w. engi neer i
ngonyour
finger
tips.
ooo

 7100 × 0.01 × 600 
1030 − 30

t = 2144

31. A flywheel connected to a punching machine has to supply energy of 400 Nm


while running at a mean angular speed of 20 radians/s. If the total fluctuation of
speed is not to exceed ±2% , the mass moment of inertia of the flywheel in
kg − m2 is
(A) 25 (B) 50 (C) 100 (D) 125
Answer: (A)
Explanations:-
E = Iw2CS
400
⇒ I= = 25kg − m2
20 × 0.04
2

www.engineeringonyourfingertips.ooo
GATEFORUM- India’s No.1 institute for GATE training
8

More Notes Join us Telegram-: http//:t.me/allexammentor


www.engineeringonyourfingertips.ooo
|ME-GATE-2013 PAPER| www.gateforum.com

32. A compound gear train with gears P, Q, R and S has number of teeth 20, 40,
15 and 20, respectively. Gears Q and R are mounted on the same shaft as
shown in the figure below. The diameter of the gear Q is twice that of the gear
R. If the module of the gear R is 2 mm, the center distance in mm between
gears P and S is Q
(A) 40 S
P
(B) 80
+ + +
(C) 120
(D) 160 R
Answer: (B)
Explanations:-
dR = m × tR = 2 × 15 = 30

dQ = 2dR = 60
dP t 20
= P ⇒ dP = × 60 = 30
dQ tQ 40
dR tR t 30 × 20
= ⇒ ds = dR × S = = 40
dS tS tR 15

dP dQ dR dS
centre dis tan ce = + + +
2 2 2 2
= 15 + 30 + 15 + 20 = 80

33. A pin jointed uniform rigid rod of weight W and Length L is supported horizontally
by an external force F as shown in the figure below. The force F is suddenly
removed. At the instant of force removal, the magnitude of vertical reaction
developed at the support is

www.
engi
neer
ingon
L
your
finger
tips.
ooo

W W
(A) zero (B) (C) (D) W
4 2
Answer: (B)
Explanations:-

1W 2 L
I= L; W× = Iα
3 g 2
3g
⇒α=
2L

www.engineeringonyourfingertips.ooo
GATEFORUM- India’s No.1 institute for GATE training
9

More Notes Join us Telegram-: http//:t.me/allexammentor


www.engineeringonyourfingertips.ooo
|ME-GATE-2013 PAPER| www.gateforum.com

L 3g
∴ Linear accleration at centre = α × =
2 4
3
∴ Inertial force at centre = W
4
3W W
∴ Re action at sup port = W − =
4 4

34. Two cutting tools are being compared for a machining operation. The tool life
equations are:
Carbide tool: VT 1.6 = 3000

HSS tool: VT 0.6 = 200

Where V is the cutting speed in m/min and T is the tool life in min. The carbide
toll will provide higher tool life if the cutting speed in m/min exceeds

(A) 15.0 (B) 39.4 (C) 49.3 (D)60.0

Answer: (B)

Explanations:-

VT1.6
= 15
VT 0.6
⇒ T = 15
V × (15)
1.6
= 3000
⇒ V = 39.4

35. In a CAD package, mirror image of a 2D point P(5, 10) is to be obtained about a
line which passes through the origin and makes an angle of 45° counterlockwise
with the X-axis. The coordinates of the transformed point will be

(A) (7.5, 5)
www.
engi
neer i
ngonyour
(B) (10, 5)
finger tips. ooo
(C) (7.5, -5) (D) (10, -5)

Answer: (B)

36. A linear programming problem is shown below:


Maximize 3x + 7y

3x + 7y ≤ 10
Subject to 4x + 6y ≤ 8
x, y ≥ 0

It has

(A) an unbounded objective function (B) exactly one optimal solution

(C) exactly two optimal solutions (D) infinitely many optimal solutions

www.engineeringonyourfingertips.ooo
GATEFORUM- India’s No.1 institute for GATE training
10

More Notes Join us Telegram-: http//:t.me/allexammentor


www.engineeringonyourfingertips.ooo
|ME-GATE-2013 PAPER| www.gateforum.com

y
Answer: (B)
Explanations:- (0,1.42)

(0,1.33)
3x + 7y ≤ 0
4x + 6y ≤ 8

x
(2,0) (3.33,0)

+0.020
37. Cylindrical pins of 25 +0.010 mm diameter are electroplated in a shop. Thickness of
the plating is 30 +2.0 micron . Neglecting gage tolerances, the size of the GO gage
in mm to inspect the plated components is
(A) 25.042 (B) 25.052 (C) 25.074 (D) 25.084
Answer: (B)
+0.020

Explanations:- 25+0.010 + 0.03±0.002


+0.022
+0.012
25.03
Go gauge = max. Limit = 25.052

38. During the electrochemical machining (ECM) of iron (atomic weight=56,


valency=2) at current of 1000 A with 90% current efficiency, the material
removal rate was observed to be 0.26 gm/s. If Titanium (atomic weight = 48,
valency=3) is machined by the ECM process at the current of 2000 A with 90%
current efficiency, the expected material removal rate in gm/s will be
(A) 0.11 (B) 0.23 (C) 0.30 (D)0.52
Answer: (C)
Explanation:
AI 0.9 × 48 × 2000
Q = =
F2 3w
×w w. en
96500 ×g
3i
neer
ingonyour
finger
tips.
ooo
Q = 0.3

39. A single degree of freedom system having mass 1 kg and stiffness 10kN/m
initially at rest is subjected to an impulse force of magnitude 5 kN for
10 −4 seconds. The amplitude in mm of the resulting free vibration is
(A) 0.5 (B) 1.0 (C) 5.0 (D)10.0
Answer: (C)

40. A bar is subjected to fluctuating tensile load from 20 kN to 100 kN. The material
has yield strength of 240 MPa and endurance limit in reversed bending is 160
MPa. According to the Soderberg principle, the area of cross-section in mm2 of
the bar for a factor of safety of 2 is
(A) 400 (B) 600 (C) 750 (D)1000

www.engineeringonyourfingertips.ooo
GATEFORUM- India’s No.1 institute for GATE training
11

More Notes Join us Telegram-: http//:t.me/allexammentor


www.engineeringonyourfingertips.ooo
|ME-GATE-2013 PAPER| www.gateforum.com

Answer: (D)
σm σv 1
+ =
σy σe F.S

 60 40  1
1000  + =
 A × 240 A × 160  2
A = 1000

41. A simply supported beam of length L is subjected to a varying distributed load


 x
sin  3π  Nm −1 , where the distance x is measured from the left support. The
 L 
magnitude of the vertical force in N at the left support is
L L 2L
(A) zero (B) (C) (D)
3π π π
Answer: (B)
 3πx  N
Explanations:- Load = sin  
 L m

 3πx 
L
TotalLoad = ∫ sin   dx
0  L 
L
−L   3πL   2L
= cos   − cos0 = + x
3π   L   3π

Since the load is distributed equally on both supports


Total Load L
R A = RB = =
2 3π

42. Two large diffuse gray parallel plates, separated by a small distance, have
surface temperatures of 400 K and 300 K. If the emissivities of the surfaces are
0.8 and thewww. engi neer
Stefan-Boltzmanni
nconstant
gonyou r
isfi
nger
5.67 ×t
ip−s
10 8
.
oo
W o2K 4 , the net radiation
/m
heat exchange rate in kW / m2 between the two plates is
(A) 0.66 (B) 0.79 (C) 0.99 (D) 3.96
Answer: (A)

dQ =
(
σA T14 − T24 ); ε = ε2 = 0.8
1
1 1
+ −1
ε1 ε2

dQ = 0.66 kW / h2

43. A hinged gate of length 5 m, inclined at 30 ° with the horizontal and with water
mass on its left, is shown in figure below. Density of water is 1000 kg / m2 . The
minimum mass of the gate in kg per unit width (perpendicular to the plane of
paper), required to keep it closed is

www.engineeringonyourfingertips.ooo
GATEFORUM- India’s No.1 institute for GATE training
12

More Notes Join us Telegram-: http//:t.me/allexammentor


www.engineeringonyourfingertips.ooo
|ME-GATE-2013 PAPER| www.gateforum.com

5m

(A) 5000 (B) 6600 (C) 7546 (D) 9623


Answer: (D)
Explanation:
An equilibrium moment of weight at ‘0’= Moment of pressure force at ‘O’
Moment at O, w × x = Fp × y
Fp = pgAx; A = Area of gate = 5 × 1
x = distance of C.G of Gate from free surface
x
θ = 30 °; sin 30 ° = ⇒ x = 1.25
2.5 y
Ig 8m2 θ
h = centre of pressure= x +
Ax 5m
bd3 1 × 53 2.5
(Ia)xx = = = 10.41 x
12 12 h
2 x
1 2.5
10.41 ×  
h = 1.25 +  2  = 1.66
(5 × 1) 1.25
x Fp
cos θ = ⇒ x = 2.165
2.5
h 1.666
sin θ = ⇒ y = = 3.32
y sin 30
w × 2.165 = p × g × Ax × y = 1000 × 9.8 × 5 × 1 × 1.25 × 3.32
⇒ w = 94021.9
w = mg ⇒ 9584  9623

44. The pressure, temperature and velocity of air flowing in a pipe are 5 bar, 500 K
www. engi neer ingonyour finger tips. ooo
and 50 m/s, respectively. The specific heats of air at constant pressure and at
constant volume are 1.005 kJ/kgK and 0.718 kJ/kgK, respectively. Neglect
potential energy. If the pressure and temperature of the surroundings are 1 bar
and 300 K, respectively, the available energy in kJ/kg of the air stream is
(A) 170 (B) 187 (C) 191 (D) 213
Answer: (B)

45. The probability that a student knows the correct answer to a multiple choice
2
question is . If the student does not know the answer, then the student
3
1
guesses the answer. The probability of the guessed answer being correct is .
4
Given that the student has answered the question correctly, the conditional
probability that the student known the correct answer is
2 3 5 8
(A) (B) (C) (D)
3 4 6 9

www.engineeringonyourfingertips.ooo
GATEFORUM- India’s No.1 institute for GATE training
13

More Notes Join us Telegram-: http//:t.me/allexammentor


www.engineeringonyourfingertips.ooo
|ME-GATE-2013 PAPER| www.gateforum.com

Answer: (D)
A = The student answer the question correctly
E1 = Student knows the correct answer

E2 = Student guesses the correct answer

2 1
p(E1) = , p(E2 ) =
3 3
A A
p(A) = p(E1) × p   + p(E2 ) × p  
 E1   E2 
2 1 1 3
= ×1 + × =
3 3 4 4

1 E 
Using Bayes theorem, p  A 
 

A
p (E1 ) × p   2 × 1
 E1  = 3 =
8
p(A) 3 9
4
d2u du
46. The solution to the differential equation 2
−k = 0 where k is a constant,
dx dx
subjected to the boundary conditions u(0)=0 and u(L)=U, is
x  1 − ekx 
(A) u = U (B) u = U  kL 
L 1 − e 

 1 − e −kx   1 + e −kx 
(C) u = U  −kL 
(D) u = U  − kL 
1 − e  1 + e 
Answer: (B)
d2u du
2
−K =0
dx
dx
www.
engi
neer
ingonyour
finger
tips.
ooo
D2 − kD = 0 D(D − K) = 0
D = 0, D = K
u = C1e0 + C2 ekx
u = C1 + C2 ekx
u(0) = 0
∴ C1 + C 2 = 0..............(1)
u(L) = u
u = C1 + C2 ekL = U..............(2)
solving (1) and (2)
U −U
C1 = , C2 ekx
1−e kL
1 − ekL
 1 − ekL 
u = U kx 
1 − e 

www.engineeringonyourfingertips.ooo
GATEFORUM- India’s No.1 institute for GATE training
14

More Notes Join us Telegram-: http//:t.me/allexammentor


www.engineeringonyourfingertips.ooo
|ME-GATE-2013 PAPER| www.gateforum.com

e
47. The value of the definite integral ∫ 1
x ln(x)dx is

4 2 2 4 2 4 4 2
(A) e3 + (B) e3 − (C) e3 + (D) e3 −
9 9 9 9 9 9 9 9
Answer: (C)
e
∫1
x ln(x)dx
e
 3   3 
 x2   1 x2 
= ln(x) ×
 3 
 − ∫  x × 2 dx
 2 1  3 
e
 3
2 4 3

= ln(x) × x 2 × − × x 2 
 3 9 1
2 4
= e3 +
9 9

Common Data Questions: 48 & 49

A single riveted lap joint of two similar plates as shown in the figure below has
the following geometrical and material details:

P w w P

t
t

Width of the plate w=200 mm, thickness of the plate t=5 mm, number of rivets
n=3, diameter of the rivet dr = 10 mm, diameter of the rivet hole dh = 11 mm,
www.
allowable tensile engi
stress nee
of ri
thenplate
gony σo
pu
=rf
ing
200 erti
MPa, ps.ooo shear stress of the
allowable
rivet σ s = 100 MPa and allowable bearing stress of the σ c = 150 MPa

48. If the rivets are to be designed to avoid crushing failure, the maximum
permissible load P in kN is
(A) 7.50 (B) 15.00 (C) 22.50 (D) 30.00
Answer: (C)
p = σ c × n × d × t = 150 × 3 × 10 × 5 = 22.5kN

49. If the plates are to be designed to avoid tearing failure, the maximum
permissible load P in kN is
(A) 83 (B) 125 (C) 167 (D) 501
Answer: (C)
p = σ t × ( w − 3d ) × t = 200 × (200 − 3 × 11) × 5 = 167kN

www.engineeringonyourfingertips.ooo
GATEFORUM- India’s No.1 institute for GATE training
15

More Notes Join us Telegram-: http//:t.me/allexammentor


www.engineeringonyourfingertips.ooo
|ME-GATE-2013 PAPER| www.gateforum.com

Common Data Questions: 50 & 51

Water (specific heat, cp = 4.18 kJ / kgK ) enters a pipe at a rate of 0.01 kg/s and
a temperature of 20 °C . The pipe, of diameter 50 mm and length 3m, is
W
subjected to a wall heat flux q"w in 2 .
m
50. If q"w = 2500x, where x is in m and in the direction of flow (x=0 at the inlet), the
bulk mean temperature of the water leaving the pipe in °C is
(A) 42 (B) 62 (C) 74 (D) 104
Answer: (B)
Explanation: q"w = 2500x
x =0 q"wi = 0
x = 3 q"wi = 2500 × 3 = 7500
0 + 7500
qavg = = 3750
2
qavg × Area = mcp × ∆T
3750 × π × 0.05 × 3 = 0.01 × 4.18 × 103 (T − 20)
⇒ T = 42.2 + 20 = 62.2 °C

51. If q"w = 5000, and the convection heat transfer coefficient at the pipe outlet is
1000 W / m2K , the temperature in °C at the inner surface of the pipe at the outlet
is
(A) 71 (B) 76 (C) 79 (D) 81

Answer: (D)
Explanation: q"w = 5000 consant
⇒ q"w × A = MCP ∆T
Q = 5000 × π × 0.05 × 3 = 0.01 × 4.18 × 103 (To − 20)
⇒ T0 − 20 = 56.3 = To = 76.3°C
www.
Heat flux between engi
any nee
two ri
ngon
sections yo
is urf
i
same nger
tips.
ooo
Q = hA(Tp − To )
Q
But = q"w = 5000 = h(Tp − To )
A
5000 = 1000(Tp − 76.3)
⇒ Tp = 76.3 + 5 = 81.3

Linked Answer Questions: Q.52 to Q.55 Carry Two Marks Each

Statement for Linked Answer Questions: 52 & 53

In orthogonal turning of a bar of 100 mm diameter with a feed of 0.25 mm/rev,


depth of cut of 4 mm and cutting velocity of 90 m/min, it is observed that the
main (tangential) cutting force is perpendicular to the friction force acting at the
chip-tool interface. The main (tangential) cutting force is 1500 N.

www.engineeringonyourfingertips.ooo
GATEFORUM- India’s No.1 institute for GATE training
16

More Notes Join us Telegram-: http//:t.me/allexammentor


www.engineeringonyourfingertips.ooo
|ME-GATE-2013 PAPER| www.gateforum.com

52. The orthogonal rake angle of the cutting tool in degree is


(A) Zero (B) 3.58 (C) 5 (D) 7.16
Answer: (A)
Explanations:- As cutting f is f to F − F
ar

Rake angle α = 0

53. The normal force acting at the chip-tool interface in N is


(A) 1000 (B) 1500 (C) 2000 (D) 2500
Answer: (B)
Explanations:- Normal force N = FH cos α − FV sin α

= 1500 × cos0 − Fv sin0


= 1500

Statement for Linked Answer Questions: 54 & 55

In a simple Brayton cycle, the pressure ration is 8 and temperatures at the


entrance of compressor and turbine are 300 K and 1400 K, respectively. Both
compressor and gas turbine have isentropic efficiencies equal to 0.8. For the
gas, assume a constant value of cp (specific heat at constant pressure) equal to
1 kJ/kgK and ratio of specific heats as 1.4. Neglect changes in kinetic and
potential energies.

54. The power required by the compressor in kW/kg of gas flow rate is
(A) 194.7 (B) 243.4 (C) 304.3 (D) 378.5
Answer: (C)
Explanations:-
rD = 8; T1 = 300K; T3 = 1400K
γ = 1.4
T2 γ −1
www.
engi
neer
0.4 ingonyour
finger
tips.
ooo
= (rP ) = (r )1.4 ⇒ T2 = 543.43K
3
2 − 2'
T1 γ
T2 − T1 543.43 − 3w
ηC = ⇒ 0.8 = 1 4
41
T21 − T1 T21 − 300
γ −1 0.4
T 1 γ  1 1.4
T21 = 604.29 k; 4 =   = 
T3  rP  8
T4 = 772.86k
T3 − T41 14w − T41
ηT = ⇒ 0.8 =
T3 − T4 1400 − 772.86
T41 = 898.29

( )
wC = C P T21 − T1 = 1 × ( 604.29 − 300 ) = 304.3

www.engineeringonyourfingertips.ooo
GATEFORUM- India’s No.1 institute for GATE training
17

More Notes Join us Telegram-: http//:t.me/allexammentor


www.engineeringonyourfingertips.ooo
|ME-GATE-2013 PAPER| www.gateforum.com

55. The thermal efficiency of the cycle in percentage (%) is


(A) 24.8 (B) 38.6 (C) 44.8 (D) 53.1
Answer: (A)
Explanations:-

( )
wT = CP T3 − T41 = 1 × (14w − 898.29 ) = 501.71

wT − wC
Thermal efficiency =
θ1501.71
595.71 − 304.3 501.71 − 304.3
= = = 24.8%
(
CP T3 − T21 ) 1 × (1400 − 604.29 )

Q. No. 56 – 60 Carry One Mark Each

56. Complete the sentence:


Universalism is to particularism as diffuseness is to ________
(A) specificity (B) neutrality (C) generality (D) adaptation
Answer: (A)
The relation is that of antonyms

57. Were you a bird, you ___________ in the sky.


(A) would fly (B) shall fly
(C) should fly (D) shall have flown
Answer: (A)

58. Which one of the following options is the closest in meaning to the word given
below?
Nadir
(A) Highest www.
e(B)
ngi neer i
ngonyo(C)
Lowest ur fi
nger t
Mediumips.
ooo(D) Integration
Answer: (B)
Nadir in the lowest point on a curve

59. Choose the grammatically INCORRECT sentence:


(A) He is of Asian origin
(B) They belonged to Africa
(C) She is an European
(D) They migrated from India to Australia
Answer: (C)

60. What will be the maximum sum of 44, 42, 40, ... ?
(A) 502 (B) 504 (C) 506 (D) 500

www.engineeringonyourfingertips.ooo
GATEFORUM- India’s No.1 institute for GATE training
18

More Notes Join us Telegram-: http//:t.me/allexammentor


www.engineeringonyourfingertips.ooo
|ME-GATE-2013 PAPER| www.gateforum.com

Answer: (C)
The maximum sum is the sum of 44, 42,- - - - -2.
The sum of ‘n’ terms of an AP
n
= 2a + (n − 1) d
2 
In this case, n = 22, a = 2 and d = 2
∴ Sum = 11  4 + 21 × 2 = 11 × 46 = 506

Q. No. 61 – 65 Carry Two Marks Each

61. Out of all the 2-digit integers between 1 and 100, a 2-digit number has to be
selected at random. What is the probability that the selected number is not
divisible by 7?
(A) 13/90 (B) 12/90 (C) 78/90 (D) 77/90
Answer: (D)
The number of 2 digit multiples of 7 = 13
∴ Probability of choosing a number
90 − 13 77
Not divisible by 7 = =
90 90

62. A tourist covers half of his journey by train at 60 km/h, half of the remainder by
bus at 30 km/h and the rest by cycle at 10 km/h. The average of the tourist in
km/h during his entire journey is
(A) 36 (B) 30 (C) 24 (D) 18
Answer: (C)
Let the totalwww. eng
distance ineer
coveredingo
be nyour
‘D’ finger
tips.
ooo
D
Now, average speed =
Total time taken
D 1 120
= = = = 24 km / hr
D D D 1 1 1 5
2 + +
 120 120 40
 + 4 + 4 
 6 30 10 

 

63. Find the sum of the expression


1 1 1 1
+ + + ..... +
1+ 2 2+ 3 3+ 4 80 + 81

(A) 7 (B) 8 (C) 9 (D) 10

www.engineeringonyourfingertips.ooo
GATEFORUM- India’s No.1 institute for GATE training
19

More Notes Join us Telegram-: http//:t.me/allexammentor


www.engineeringonyourfingertips.ooo
|ME-GATE-2013 PAPER| www.gateforum.com

Answer: (B)
The expression can be written as

( 2 ) − ( 1) + ( 3 ) − ( 2 ) ( ) −( )
2 2 2 2 2 2
81 80
+−−−−−
1+ 2 2+ 3 80 + 81

=
( 2 − 1 )( 1+ 2 )+−−−−−−+( 81 − 80 )( 81 + 80 )
( 1+ 2 ) 80 + 81

64. The current erection cost of a structure is Rs. 13,200. If the labour wages per
day increase by 1/5 of the current wages and the working hours decrease by
1/24 of the current period, then the new cost of erection in Rs. is
(A) 16,500 (B) 15,180 (C) 11,000 (D) 10,120
Answer: (B)
Let ‘W’ be the labour wages, and ‘T’ be the working hours.
Now, total cost is a function of W × T
Increase in wages = 20%
∴ Revised wages = 1.2 W
 100 
Decrease in labour time =  %
 24 
 1  23
∴ Re vised time = 1 −  T = 24 T
 24 
23
∴ Re vised Total cos t = 1.2 × WT = 1.15 WT
24
= 1.15 × 13200 = 15180

65. After several defeats in wars, Robert Bruce went in exile and wanted to commit
suicide. Just before committing suicide, he came across a spider attempting
tirelessly to have its net. Time and again the spider failed but that did not deter it
wwwmaking
to refrain from .engi ne eringony
attempts. our
Such finger ti
attemptsps .
byoo o spider made Bruce
the
curious. Thus, Bruce started observing the near-impossible goal of the spider to
have the net. Ultimately, the spider succeeded in having its net despite several
failures. Such act of the spider encouraged Bruce not to commit suicide. And
then, Bruce went back again and won many a battle, and the rest is history.
Which one of the following assertions is best supported by the above information?
(A) Failure is the pillar of success
(B) Honesty is the best policy
(C) Life begins and ends with adventures
(D) No adversity justifies giving up hope
Answer: (D)

www.engineeringonyourfingertips.ooo
GATEFORUM- India’s No.1 institute for GATE training
20

More Notes Join us Telegram-: http//:t.me/allexammentor


www.engineeringonyourfingertips.ooo

! !
" #
$ % & ' !
()

* " "& + *
!' + ! +,+ ++, ++
-. /+ 0

*+ 1+ " 2+ $ *3+
()

4 ' ++ /+ !
*/
+ * +1 " *+ 1 $ 3 3
() $
56 () 7 ! '
* *
π = π * *

* = ×
*

/+
* = ++ × = 3 3*
*/

8 www.
engi
neer
ing
9o ×f
nyouringe
++ =r
t
3ips.ooo
3*: *

3 & ' 6 !
; ! & ' , &
& ' , < ! & ' ! *
& ' % ;* 9 ; 9 *,

;*

*
;

<

+ " +/ $ + */
() "

© All rights reserved by Gateforum Educational Services Pvt. Ltd. No part of this booklet may be reproduced or utilized in any form without the
www.engineeringonyourfingertips.ooo
written permission.

More Notes Join us Telegram-: http//:t.me/allexammentor


www.engineeringonyourfingertips.ooo
/ &
, ' , '
" ' $ , '
()

= & !
8 8*
> ' ∆
8* 8 8* >
# # " # − $ ?
8 8* 8
()

1 % & ! , & '


0, & ' 0

*/

3/ /

+ +/ + " /+ $ ++
()

2 *+ , *++ *+ ,
' % =++ , ' ' ,
@
4A/ 1A+ " /2+ $ 4 =+
() www.
engi
neer
ingonyour
finger
tips.
ooo
*
56 () B5 ' = %
*
* *
# *+ × + * *
%= = = +3 −
* *
*π * × π × =++
= = = =* 24 0
=+ =+
B 5= × + 3 × =* 24* = 1A+ @
*

A & ! C !D E
! ! 6 %
. 5 6
EC* EC* *EC* 3EC*
" $
*5% 5% 5% 5%

© All rights reserved by Gateforum Educational Services Pvt. Ltd. No part of this booklet may be reproduced or utilized in any form without the
written permission.
www.engineeringonyourfingertips.ooo

More Notes Join us Telegram-: http//:t.me/allexammentor


www.engineeringonyourfingertips.ooo
()
*
'
56 () 5% = E
6*
,
'
5% = E6 + "
6
,
*
E6
5% ' 9 + " 6 + "*
*
'
7 & ! 6 9 +, =+ F6 =+ ' =+
6
7 " = " = +, G

E6* EC*
'= H 6 ' 6 =
*5% *5%

+ 7 , !
! !
* " 3 $ 2
() "
3π*5%
56 () " 6 ! 9
C*
π*5%
" !
C*
8
G , =3
8 *

69+, ( 6 ) = 64 +
www.
engi neer ingonyour
finger
tips.
ooo
6 & &
" ' $
() $
56 ( ) ( 6 ) = 64 + 6 9 +,
& , &'
'

© All rights reserved by Gateforum Educational Services Pvt. Ltd. No part of this booklet may be reproduced or utilized in any form without the
www.engineeringonyourfingertips.ooo
written permission.

More Notes Join us Telegram-: http//:t.me/allexammentor


www.engineeringonyourfingertips.ooo
* 7 , 6* + '* + I* − , &

, ,+ & !'
* *

+ D − D " $ + D+
* * * * 4 4 4
()

56 ( ) J & 6* + '* + I* = , ,+
* *

& 6* + '* + I* − = + , ,+
* *

+ D+ + = + D
* * * *

+
G & = * * = + D
* *
* *
+
* *

4 E

8 "

> $ *

# 4 &

$ 3 " &

8)3, >) , #)*, )4 8)3, >) , #)4, )*


" w w w .eng
8) , >)*, #)3, )4 i
neer
ingonyourf
ingert
$ 8)ip
,s .
oo#)*,
>)4, o )3
()

3 % ! & D , ! II
, &
% '
$ '
" $ ,! !
$ % ,! !
() $

++ +3+
/ % ! ! ', I */ +++−+ + +

++ +4+
I */ +++++ +*+ 6 !'
3+ 4+ " *+ $ +

© All rights reserved by Gateforum Educational Services Pvt. Ltd. No part of this booklet may be reproduced or utilized in any form without the
written permission.
www.engineeringonyourfingertips.ooo

More Notes Join us Telegram-: http//:t.me/allexammentor


www.engineeringonyourfingertips.ooo
() "
E 6 9 6 I K I
9 */ +3+)*/ +*+ 9*+µ

= $ I ,

! &
" ! &
$
() "

1 *++ I ,
4
9+ +**/ /++ & +* 0
*
9A 2 0
= 2 + = " 2 ** $ *4* 4=
()
+*
+ +**/ × /++ × *

* Π × + **
C; 3
56 () G = = = = 2
* $ *×A 2 ×+ 3

2 7 L , ! & ' (α) , & ' ( τ) & ' (ρ)


!' L
α+ρ = τ ρ+α+τ=+ " α+ρ = $ α+ρ =+
() "

A ! ! ' '
4*/ + @0 & 6 / 8 L ' '
+A L & / 8
9**/ A3 w
0ww.
@ eng9*/A2
ineer
in
4go
0n
@ your
fin&ge
'r
tips.ooo
+ 0 B ' !
! E (
=/ 2A " A $ *1 +
()

56 () 8 = × − * = + × 4*/ − **/ A3 + + A × */A2 4 − **/ A3
= 2A++ B@0 = 2 A E

*+ (
J %( 8 2+ 6 4+M
J %%( 8 *+ 6
** /M
% 33+ ,
*++ A++ " 21/ $ 1*+
© All rights reserved by Gateforum Educational Services Pvt. Ltd. No part of this booklet may be reproduced or utilized in any form without the
written permission.
www.engineeringonyourfingertips.ooo

More Notes Join us Telegram-: http//:t.me/allexammentor


www.engineeringonyourfingertips.ooo
()
56 ( ) 7
ϕ 2+ 4+
; ' = = = *

* ϕ *+" ** /

* = 33+ × + =*/ = A++

* I
& ' ω α

( α) (ω)
* *
? α " + *
$ ω*

()

** !D %
!' : !' :,
,

+ " +2 $ * +*
() $
8
56 () G σ 9
*
8
' ' ! , , σ = + −
*

$ ! & L , ( )= & =++

= −+ +

σ
< ! , = + +*, ∴ ! *:
σ
www.
engi
neer
ingonyour
finger
tips.
ooo
*4 ! '96 ! ' = 6*
6)'
0
= 0
3 " 0
4 $ 0
*
()
56 () & & '96 ' 9 6* & * ,
G & 6 9 +, 6 9

=
+
(6 − 6 ) 6 *

6* 64
= − = −
* 4 * 4
+
+
= L
=

© All rights reserved by Gateforum Educational Services Pvt. Ltd. No part of this booklet may be reproduced or utilized in any form without the
written permission.
www.engineeringonyourfingertips.ooo

More Notes Join us Telegram-: http//:t.me/allexammentor


www.engineeringonyourfingertips.ooo
*3 " (6) = 6 & ) ≤6≤
69+, (6)
" ! ) !
" " ) ! $ !
() "
56 () J & 6 9N
6NO
N
6N 69+! !

− 6
*/ *
6 →+ 6
0
3 0
* " $ *
()
* 6 * 6
6 *
− * *
56 () = =* = *× =
6 →+ 6* 6 →+ 6* 6 →+ 6* 3 *
×3
*

*= " I , !
& ! (
I ! */
*
# ! + +=
$ + +/
*3 24 *3 2A " */ + $ */ 1
()
56 () $ 9$ K*6 )
= */ − * × + += − + +/ = *3 24

www.
engi
neer
ingonyour
finger
tips.
ooo
*1 % 3 + ,
3+ 2 +:
!
+ ++= + +4 " + +=* $ + =++
() "
56 () ∆G = $ ( − α)
∆G + ×2
α= − = −
$ 3 +
α = 4 /1
π
α = 4 /1 ×
2+
α = + +=*

© All rights reserved by Gateforum Educational Services Pvt. Ltd. No part of this booklet may be reproduced or utilized in any form without the
written permission.
www.engineeringonyourfingertips.ooo

More Notes Join us Telegram-: http//:t.me/allexammentor


www.engineeringonyourfingertips.ooo
*2 % $" , & ) !
; = *+ + / & ! / 1 G
; & ; & 3++
/++ , &
(
3/;, 3/+ 1/;, //+ " A/;, A/+ $ /+;, /++
() "
56 () ; = *+ + /
=/ , ; = 3/ ;

=1 , ; = // ;

;+
; = ;+ − % ;+ &
%
%

;+
3/ = ;+ − × /++
%
;+
// = ;+ − × 3++
%

< & *
; = A/ &

% = A/+

*A II ! &
, * 4 ! ,
& ' (

www.
engi
neer
ingony
ρour
finger
tips.
ooo

* ρ*

4 ρ4

ρ ρ*
* + + *
* ( + + )
ρ4 ρ4
4 * 4
4 4

ρ + ρ* * + ρ4 4 ρ * 4 + ρ* 4 + ρ4 *
" * $ *
ρ + ρ* + ρ4 ρ + ρ* * + ρ4 4

© All rights reserved by Gateforum Educational Services Pvt. Ltd. No part of this booklet may be reproduced or utilized in any form without the
written permission.
www.engineeringonyourfingertips.ooo !

More Notes Join us Telegram-: http//:t.me/allexammentor


www.engineeringonyourfingertips.ooo
()
56 () ' P L , 6 ,
8 ;* 8 ;**
+ I+ = * + I* +
ρ4 * ρ4 *

? = ?* , ; = + F 8* = 8

G
8 ;*
= *
ρ4 *

*8
;* =
ρ4

8 = ρ + ρ* * + ρ4 4

< ! ,

ρ ρ*
;* = * 4 + + *
ρ* 4 ρ4 4

4+ (" =3 2 @0 ⋅ B ) 2+M" 6
+/ 0 (" = @0 ⋅ B ) 4+M"
* +A 0 % & 6 + 2, CE $
M"
3+ *+ " + $ /
() "
" −
56 () ε= × *

" −

3 2 × + / 2+ − *
+2= ×
* +A × 2+ − 4+
www.
engi
neer
ingonyour
finger
tips.
ooo
*
= 3+ "
− *
= *

+ /× 3 2 × 3+ = * +A × × *
− 4+

*
= 1+ "
θ = θ* = + " ∴ CE $ = + "

4 ! 6
' !' ∆ % α, . P
5 8 P &, &
!
α (∆ )5 *α ( ∆ )5 4α ( ∆ )5 α (∆ )5
− − " − $ −
( − *&) ( − *&) ( − *&) 4 ( − *&)

© All rights reserved by Gateforum Educational Services Pvt. Ltd. No part of this booklet may be reproduced or utilized in any form without the
www.engineeringonyourfingertips.ooo
written permission. "

More Notes Join us Telegram-: http//:t.me/allexammentor


www.engineeringonyourfingertips.ooo
()
4* ! L /+ %
! 3+E8 , '
*, !
2 = " *3 $ 4*
()
56 () 9 /+ K Hτ ! = 3+E8

τ
= *H τ = !
= 1+E8

, = τ
?8
4
π 4 =
?8 = =
= πτ

= × /+ × +4
=
π × 1+
= / 42 ≈ =

44 3++ ! +/ ! !
' , 2+M %
! ! + */, ! L
,
3++

++ = /3 3 " ** $ /1
()
56 () www.
engi
neer
ingonyour
finger
ti
,ps.
ooo! F *
!

F = µθ
= + */×π
=* A = 3++
*

* = 2* 41/

L 9 ( − * ) *

= /3 3 −

43 ! 6 3 ! = ! ! ! '
! 6 ! ! '
! ! !
4
" $
*+ * + *

© All rights reserved by Gateforum Educational Services Pvt. Ltd. No part of this booklet may be reproduced or utilized in any form without the
written permission.
www.engineeringonyourfingertips.ooo #

More Notes Join us Telegram-: http//:t.me/allexammentor


www.engineeringonyourfingertips.ooo
() $
#
56 () J & ,
3 =
! # # #
3 = /
8 ! ! ' # 9 × ×
+ A 2
= / 3
8 ! ! ' #9 × ×
+ A 2
= 3 /
8 ! ! ' # 9 × ×
+ A 2
8 # 9 9 ! & ! ! 9+/

*
' '
4/ " L 6* +6 − 3' = + ! '
6* 6
' (+ ) = + '( )=
L
π6 6 π6 −6 π6
6* " $
* * *
()
*
' '
56 () 6* *
+6 − 3' = + ' (+ ) = +
6 6
'( )=
" 'P $ 5
*
' '
' = 6* = *6 =*
6 6*
' = 6 −* ' = − *6 −4 ' = +=6 −3
6 6*
' = 6 −* ' = − *6 −4 ' = +=6 −3
6 6*
www.
engi
neer
ingonyour
finger
tips.
ooo
6* = ( 6) (
3 + −*
64 ) −3 ( 6 )=+ *

* *
3=+ =3
= ±*
∴ L " 6* + "* 6* + "* 6*
G 6*

4= ' ! L & !
6 + *' + I= 3
*6 + ' + *I= /
6 − ' + I=
L 69 , '9 I9
' 69 , '9 I9 69*, '9 I9+
" % !

© All rights reserved by Gateforum Educational Services Pvt. Ltd. No part of this booklet may be reproduced or utilized in any form without the
written permission.
www.engineeringonyourfingertips.ooo

More Notes Join us Telegram-: http//:t.me/allexammentor


www.engineeringonyourfingertips.ooo
$ !
() "
56 () & L
6 + *' + I= 3 −( )
*6 + ' + *I= / − (* )
6 + ' + I= − (4 )

5 F * , &
'9 ) 3
5 6 * F 4 , &
'9 ) /
C 3 /
G & L

41

+ / +
= / *+ +
+ + − +

& E8 < & E ' ,


& ' , E8
A /+ = +1 " *2 /* $ 3A 3
()

56 () σ L=
*
{( σ − σ** ) + ( σ
*
− σ** ) + = σ** + σ*4
* *
+ σ*4 }
, σ = +, σ** = *+,

σ44 = − +Hσ * = /Hσ*4 = σ 4 =+

∴ σ
www.
engi
neer
ingonyour
finger
tips.
ooo
L= *1 24A E8

σ L
' τ' = = = +1 E8
4

42 $ & !
" +3
< +=
# Q +M
" */ 0
E ' I */
)

+ 12 × +/ + 11/3 × +/

© All rights reserved by Gateforum Educational Services Pvt. Ltd. No part of this booklet may be reproduced or utilized in any form without the
www.engineeringonyourfingertips.ooo
written permission.

More Notes Join us Telegram-: http//:t.me/allexammentor


www.engineeringonyourfingertips.ooo
" + +3 × +/ $ 3 4A1 × +/
() "
56 () =+3
=+=
α= +
;" = * / 0
= */µ
α
φ= = + 3*44 φ = ** A
− α
;" α * /× +
9 = 9 + +3 × +/ 0
(φ − α) × * A × */ × +−=

4A % , / !
/+ $ /++ ,
+* 0 & 2M *
6 ,
4/ 4* 3 " 4 * $ 4+
()
C + + +"
56 () =

$ β
C = /+ H = =* H" = = 1 /× /A = 3 /
* *
= /++ H =+* 0 &
= + /2/ 4/

3+ " ' !
% $ $* !
& ', & 7** & !'

www.
engi
neer
ingonyour
finger
tips.
ooo

$* $ $
− ? " $ −
$ $* $*
() $
$
56 () 7** = − 7* = − = −
$*

3 ! & I
! ' ' # ' !
+++ % & ' !' 3,
! ' ' , !

© All rights reserved by Gateforum Educational Services Pvt. Ltd. No part of this booklet may be reproduced or utilized in any form without the
written permission.
www.engineeringonyourfingertips.ooo

More Notes Join us Telegram-: http//:t.me/allexammentor


www.engineeringonyourfingertips.ooo
3 * " +/ $ + */
() "

56 () ' ' (δ) α


#

# α ; δ α
;
% & ' !' 3 ! ' '
!'

3* 4/ ' +/ &
++ 8 */M" & ' J&
*/M" 4 1 8 , & '

=1: //: " 24: $ 1 :


() $

34 ) D !D & 7
+ I 4+ %
! A3E8 ,
, 6 ! &

7 7

3 33 1 A* " A A4 $ ** =
() "
56 () 8 = + 1+1 τ !

8 = + 1+1 × + × 4+ × A3 = AA41 3
8= A A43 w
Eww.
engi
neer
ingonyour
finger
tips.
ooo

33 *C
I L ! !'
7 & ' &! ,
, L ' ' (

"
*
3
$
C C

© All rights reserved by Gateforum Educational Services Pvt. Ltd. No part of this booklet may be reproduced or utilized in any form without the
www.engineeringonyourfingertips.ooo
written permission.

More Notes Join us Telegram-: http//:t.me/allexammentor


www.engineeringonyourfingertips.ooo
() $
3/
σ66 = 3+E8 Hσ'' = ++E8 τ6' = 3+E8
E P & E8
3+ /+ " =+ $ ++
()
56 () σ66 = 3+ E8 H σ'' = ++ E8 , τ6' = 3+E8

*
σ66 − σ''
# E P 9 + τ*6' = 4+* + 3+* = /+ E8
*

3= & C 7( )= & !'


( + )
( )= ( )= −
" ( )= −
$ ( )= − −

() $

56 () C− = −
( + ) +

C− − = − −
+

/ 4
31 7 6 = , 5 I & &
4

4
* * + /
" $
4 − − *
* * + /
www.
engi
neer
ingonyour
finger
tips.
ooo
()
/−λ 4
56 () − λ% = + =+ (/ − λ ) ( 4 − λ ) = 4 = +
4−λ

λ* − 2λ + / − 4 = + λ* − 2λ + * = + λ = *, λ = =

( − *%) × - = +
, λ=*
4 4 6 + 6 + 6* = +
= H
6* + 6 = 6*

© All rights reserved by Gateforum Educational Services Pvt. Ltd. No part of this booklet may be reproduced or utilized in any form without the
www.engineeringonyourfingertips.ooo
written permission.

More Notes Join us Telegram-: http//:t.me/allexammentor


www.engineeringonyourfingertips.ooo

G L & *

*
$$ % & ' !( "

! " ", &


*
++ , !D I 7
D

3/M "
7
=+M

32 6 7 ! " 6
' ! $ 5 6 ++E8
2 1 / " 3 3 $ ** 4+
()
56 ( ) 7 C P ,
7 7 7
= =
+/ *+ 4/
7 = + 2A=/7 H7 = + 14* 7
7
∴ 6 7 = = ++E8

+ 2A=/7 www.
= ++E8eng7i
n=eer
in
/3go
32nyour
finger
/B t
ips.
ooo
++

3A % 7= , & &
B
+ =4 + 4* " *= $ 3=
()
& # =7 4+ = + =447
7 = B ∴# = + =44B

$$ % & ' #(

! *+ 8 2++ 8 &
' #) 43

© All rights reserved by Gateforum Educational Services Pvt. Ltd. No part of this booklet may be reproduced or utilized in any form without the
www.engineeringonyourfingertips.ooo
written permission.

More Notes Join us Telegram-: http//:t.me/allexammentor


www.engineeringonyourfingertips.ooo
& & L
+* 0 8 #) 43
(

#) 43

8( 8 ) M" ( @0 ) ( @0 ) ( @0 ⋅ B )
( @0 ⋅ B )

*+ )** 4* ** / *41 + +A4 + A/

2++ 4 4 A/ / *=1 4 + 4/3 +A 2

#) 43

8( 8 ) M" ( @0 ) ( @0 ⋅ B )

2++ 3+ *1= 3/ + A/

/+ L

/ A3 24 " 1A $ 4A /

() "
56 ()
*

4
www.
engi
neer
ingonyour
finger
tips.
ooo

8 L = ( * − )
= + * × (*1= 3/ − *41 ) = 1 A

/ 6 @
0

*2 4 3* A " 43 3 $ 3=

© All rights reserved by Gateforum Educational Services Pvt. Ltd. No part of this booklet may be reproduced or utilized in any form without the
www.engineeringonyourfingertips.ooo
written permission.

More Notes Join us Telegram-: http//:t.me/allexammentor


www.engineeringonyourfingertips.ooo
()
56 () # 6 = ( − 3 )
= + * ( *41 − A/ /) = *2 4

) $ * +, & ' (

7 D , & !
& 6 ! !
&' 8 $ '
) 4
3
" /
$ 3
5 ! *
7 A
J =
G , *

/* D
)!) ) ) ) ) ) " ) )) $ )!) ) )
() "
56 () *
5
3 = =
4
" J
2 A
* / 3 =
4 G
www.
engi
neer
ingonyour
finger
tips.
ooo +
$
; / 1
3 7
5 J G− 1 '
A
" JG − = '
$7 G − 2 ' H
∴ $7 G

/4 % &' A + ' ,
" D
A '
" D
" " ! D

© All rights reserved by Gateforum Educational Services Pvt. Ltd. No part of this booklet may be reproduced or utilized in any form without the
written permission.
www.engineeringonyourfingertips.ooo !

More Notes Join us Telegram-: http//:t.me/allexammentor


www.engineeringonyourfingertips.ooo
$ " D
1 '
()
56 () % &' 7 + ' ,
D A '
) $ * +, & ' (

! II 4++ 8 , /++B & ' + 0 % &


*
II ++ 8 & ' 2+ 0 2+
" ++2@
0 B

/3 6
/ =B /4*B " 323B $ 3=2B
() "
56 () Q &* 0* 9 * Q &** 0*

&** )&*
= − * =" − *
*
2+* − +*
= ++2 × /++ − *
*
* = 323B

*
// 6 II
A+ /= 4 " 33 $ *A
() $
56 () 7 ' L ,
ρ ; = ρ* * ;*
8 8* 8 ;
× ; = × * ;* * = *
× ×
# # * 8* ;*
www.
4++ × 323 e+ngi neeringon your
finger
tips.
ooo
= × × 2+ = * A *

/++ × ++
*
2+

/= & !' /L* , L


# /+
! ! !'
6 I
/ + " / $ */
(
56 () 8 = /+L− /L*

© All rights reserved by Gateforum Educational Services Pvt. Ltd. No part of this booklet may be reproduced or utilized in any form without the
written permission.
www.engineeringonyourfingertips.ooo "

More Notes Join us Telegram-: http//:t.me/allexammentor


www.engineeringonyourfingertips.ooo
*
= /+ − +L
H <+
L L*
∴ 6 /+ − +L= + , L= /
5

/1 " & & !


(
P RRRRRRRR
" $
(

/2 " ' -- (
' & ' ! & G

'S 6
" ' 7 ' C , !

$ 'S 6 & '


( $

/A &
!
.
$ $& " $ $ $
(

=+ " & & !


(
$ & RRRRRRRRRR &
www.
engi
neer
ingonyour
finger
tips.
ooo
! " $
(

= ', 8 ) 7 % &
& # L ( G
) , ! & ! $ ', 5& '
, 6 !
! ! & &
J ) '
- !
" &

© All rights reserved by Gateforum Educational Services Pvt. Ltd. No part of this booklet may be reproduced or utilized in any form without the
www.engineeringonyourfingertips.ooo
written permission. #

More Notes Join us Telegram-: http//:t.me/allexammentor


www.engineeringonyourfingertips.ooo
$ ) '
( "
56 () J & '

=* J& L , $ "J 7B @
8, 6
; " 8; $ 8
(
56 () +* +4 +3 +/

$ , "J , 7B , @
8 , ;

+4 +3 +/ +=

=4 " ##5"
8( 1 ' 1
>( 1 ' 1 &
#( $ ! ' !
( $ ! ' & &
8, > >, # " 8, # $ #,
( "
56 () 8 # '
5 T , *, 4, 3U '

=3 ! ! ' ! ! - .
- =+: . 3+: ! ! ! !
!D L ' L ' !
-S ! ! , A=: ! .S ! ! , 1*: !
! ! ' ' ! ! , ! ! ,
!' .
+ *22 + 443 " + ==1 $ + 1*+
(
56 () www.
e6ngi
neer
ingonyo'ur
finger
tips.
ooo
' =+: 3+:
# ! A=: 1*:
& + /1= + *22
+ *22
∴ 8 (6) = = + 443
+ /1= + + *22

=/ '
& ! L ' = *6 − + 6*
' 6 !

2 + " * $ 3
(
56 () ' = *6 − + 6*

© All rights reserved by Gateforum Educational Services Pvt. Ltd. No part of this booklet may be reproduced or utilized in any form without the
www.engineeringonyourfingertips.ooo
written permission.

More Notes Join us Telegram-: http//:t.me/allexammentor


www.engineeringonyourfingertips.ooo
'
= * − + *6
6
*
'
<+∴' 6 * − + *6 = +
6*
6= +
∴ ' = *+ − + = +

www.
engi
neer
ingonyour
finger
tips.
ooo

© All rights reserved by Gateforum Educational Services Pvt. Ltd. No part of this booklet may be reproduced or utilized in any form without the
www.engineeringonyourfingertips.ooo
written permission.

More Notes Join us Telegram-: http//:t.me/allexammentor


ME-Paper Code-A GATE 2011
www.engineeringonyourfingertips.ooowww.gateforum.com

Q. No. 1 – 25 Carry One Mark Each

1. A streamline and an equipotential line in a flow field


(A) Are parallel to each other (B) Are perpendicular to each other
(C) Intersect at an acute angle (D) Are identical
Answer: - (B)
 dy   dy 
Explanation:-   ×  dx  = −1
 dx φ  ψ
Slope of equipotential Line × slope of stream function= -1
They are orthogonal to each line other.

2. If a mass of moist air in an airtight vessel is heated to a higher temperature,


then
(A) Specific humidity of the air increases
(B) Specific humidity of the air decreases
(C) Relative humidity of the air increases
(D) Relative humidity of the air decreases
Answer: - (D)
Explanation:- R.H. Decreases

φ = 100% ω

Heating

DBT OC

3. In a condenser of a power plant, the steam condenses at a temperature of 600 C .


The coolingwwater
ww. en ginee
enters r
ating
300o
Cn your
and finger
leaves t
atips.
450o
Co
.oThe logarithmic mean
temperature difference (LMTD) of the condenser is
(A) 16.20 C (B) 21.60 C (C) 300 C (D) 37.50 C
Answer: - (B)
Explanation: - Flow configuration in condenser as shown below.

60O C 60O C
45O C

30O C
∆T1 − ∆T2 30 − 15
∆T1 = 30O C , ∆T2 = 15O C , LMTD = = = 21.6O C
 ∆T1   30 
ln   ln  
 ∆T2   15 

www.engineeringonyourfingertips.ooo
© All rights reserved by Gateforum Educational Services Pvt. Ltd. No part of this document may be reproduced or utilized in any form without the
written permission. Discuss GATE 2011 question paper at www.gatementor.com. 1

More Notes Join us Telegram-: http//:t.me/allexammentor


ME-Paper Code-A GATE 2011
www.engineeringonyourfingertips.ooowww.gateforum.com

4. A simply supported beam PQ is loaded by a moment of 1kN-m at the mid-span of


the beam as shown in the figure. The reaction forces R P and R Q at supports P
and Q respectively are
1 kN − m
P Q

1m

(A) 1kN downward, 1kN upward (B) 0.5kN upward, 0.5kN downward
(C) 0.5kN downward, 0.5kNupward (D) 1kN upward, 1kN upward
Answer: - (A)
Explanation: - Take moments about ‘Q’
R Q × 1 − 1 = 0 ⇒ R Q = 1kN ↑

But RP + R Q = 0 ⇒ RP = −RQ = − lkN or RP = lkN ↓

5. A double – parallelogram mechanism is shown in the figure. Note that PQ is a


single link. The mobility of the mechanism is

P Q

(A) -1 (B) 0 (C) 1 (D) 2


Answer: - (C)

6. The maximum possible draft in cold rolling of sheet increases with the
(A) Increase in coefficient of friction (B) Decrease in coefficient of friction
w w w .
e n g ine
(C) Decrease in roll radius
e rin g on your finger tips. ooo
(D) Increase in roll velocity
Answer: - (A)

7. The operation in which oil is permeated into the pores of a powder metallurgy
product is known as
(A) Mixing (B) Sintering (C) Impregnation (D) Infiltration
Answer: - (C)

+0.015
8. A hole is dimension φ9+0 mm. The corresponding shaft is of dimension
+0.010
+0.001
φ9 mm. The resulting assembly has
(A) Loose running fit (B) Close running fit
(C) Transition fit (D) Interference fit
Answer: - (C)

www.engineeringonyourfingertips.ooo
© All rights reserved by Gateforum Educational Services Pvt. Ltd. No part of this document may be reproduced or utilized in any form without the
written permission. Discuss GATE 2011 question paper at www.gatementor.com. 2

More Notes Join us Telegram-: http//:t.me/allexammentor


ME-Paper Code-A GATE 2011
www.engineeringonyourfingertips.ooowww.gateforum.com

9. Heat and work are


(A) Intensive properties (B) Extensive properties
(C) Point functions (D) Path functions
Answer: - (D)
Explanation: - Heat and work are path functions.
Since δQ and δW are dependent on path followed between two given end states
of a thermodynamic process undergone by system.

10. A column has a rectangular cross-section of 10mm x 20mm and a length of 1m.
The slenderness ratio of the column is close to
(A) 200 (B) 346 (C) 477 (D) 1000
Answer: - (B)
Explanation:-
length of column L
Slenderness ratio = =
least radius of gyration K
Imin
But K =
A
Where Imin is minimum area moment of inertia i.e. Ixx or Iyy , whicever is less.

20 × 103
For the given section Imin = = 1667mm3
12
1667 1000
∴K = = 2.89 and ratio = = 346
20 × 10 2.89

11. A series expansion for the function sin θ is


θ2 θ4 θ3 θ5
(A) 1 − + − .... (B) θ − + − ....
2! 4! 3! 5!
θ2 θ3 θ3 θ5
(C) 1 + θ + + + .... (D) θ + + + ....
2! 3! 3! 5!
Answer:- (B)
www.
engi
3 5
neer
ingonyour
finger
tips.
ooo
x x
Explanation:- Sinx=x − + − ....
3! 5!

12. Green sand mould indicates that


(A) Polymeric mould has been cured (B) Mould has been totally dried
(C) Mould is green in colour (D) Mould contains moisture
Answer: - (D)

sin θ
13. What is lim equal to?
θ→0 θ
(A) θ (B) sin θ (C) 0 (D) 1
Answer: - (D)
Cosθ
Explanation: - Applying L’ Hospitals rule, we have lim = Cos0 = 1
θ→0 1

www.engineeringonyourfingertips.ooo
© All rights reserved by Gateforum Educational Services Pvt. Ltd. No part of this document may be reproduced or utilized in any form without the
written permission. Discuss GATE 2011 question paper at www.gatementor.com. 3

More Notes Join us Telegram-: http//:t.me/allexammentor


ME-Paper Code-A GATE 2011
www.engineeringonyourfingertips.ooowww.gateforum.com

14. Eigen values of a real symmetric matrix are always


(A) Positive (B) Negative (C) Real (D) Complex
Answer: - (C)
Explanation: - Eigen values of a real symmetric matrix are always real

15. A pipe of 25mm outer diameter carries steam. The heat transfer coefficient
between the cylinder and surroundings is 5W / m2K . It is proposed to reduce the
heat loss from the pipe by adding insulation having a thermal conductivity of
0.05W/mK. Which one of the following statements is TRUE?
(A) The outer radius of the pipe is equal to the critical radius
(B) The outer radius of the pipe is less than the critical radius
(C) Adding the insulation will reduce the heat loss
(D) Adding the insulation will increase the heat loss
Answer: - (C)
k   O.05 
Explanation: -Critical Radius of Insulation =   =   m = 10mm
h  5 

(router ) > rcritical ⇒ Adding insulation shall decrease H.T. Rate.

16. The contents of a well-insulated tank are heated by a resistor of 23Ω in which
10A current is flowing. Consider the tank along with its contents as a
thermodynamic system. The work done by the system and the heat transfer to
the system are positive. The rates of heat (Q), work (W) and change in internal
energy ( ∆U) during the process in kW are

(A) Q = 0, W = −2.3, ∆U = +2.3 (B) Q = +2.3, W = 0, ∆U = +2.3


(C) Q = −2.3, W = 0, ∆U = −2.3 (D) Q = 0, W = +2.3, ∆U = −2.3
Answer: - (A)
Explanation: - www.
engi
neer
ingonyour
finger
tips.
ooo

23Ω Q = 0 adiabatic

i = 10A

( )
Welectric = i2R = 102 × 20 watts = −2.3 kw ( on system)

Ilaw : − φ − w = ∆U

O − ( −Welect ) = ∆u

www.engineeringonyourfingertips.ooo
© All rights reserved by Gateforum Educational Services Pvt. Ltd. No part of this document may be reproduced or utilized in any form without the
written permission. Discuss GATE 2011 question paper at www.gatementor.com. 4

More Notes Join us Telegram-: http//:t.me/allexammentor


ME-Paper Code-A GATE 2011
www.engineeringonyourfingertips.ooowww.gateforum.com

17. Match the following criteria of material failure, under biaxial stresses σ1 and σ2
and yield stress σy , with their corresponding graphic representations :

P. Maximum-normal-stress criterion L. σ2
σy

−σ y σ1
σy

−σ y
σ2
Q. Maximum-distortion-energy criterion M. σy

−σ y σ1
σy

−σ y

R. Maximum-shear–stress criterion N. σ2
σy

σ1
−σ y σy

−σ y

(A) P-M, Q-L, R-N (B) P-N, Q-M, R-L (C) P-M, Q-N, R-L (D) P-N, Q-L,R-M

Answer: - (C)

18. The product of two complex numbers 1 + i and 2-5i is


(A) 7-3i (B) 3-4i (C) -3-4i (D) 7+3i
Answer: - (A)
Explanation: - (1 + i) ( 2 − 5 i) = 2 − 5 i + 2 i + 5 = 7 − 3 i
www.
engi
neer
ingonyour
finger
tips.
ooo
19. Cars arrive at a service station according to Poisson’s distribution with a mean
rate of 5 per hour. The service time per car is exponential with a mean of
10minutes. At steady state, the average waiting time in the queue is
(A) 10 minutes (B) 20 minutes (C) 25 minutes (D) 50 minutes
Answer: - (D)
5
ρ λ 5 5
Wq = ; Where λ = 5/hr, µ = 6/hr, ρ = = ; ∴ Wq = 6 = hr = 50 min
µ−λ µ 6 6−5 6

20. The word kanban is most appropriately associated with


(A) Economic order quantity (B) Just–in–time production
(C) Capacity planning (D) Product design
Answer: - (B)

www.engineeringonyourfingertips.ooo
© All rights reserved by Gateforum Educational Services Pvt. Ltd. No part of this document may be reproduced or utilized in any form without the
written permission. Discuss GATE 2011 question paper at www.gatementor.com. 5

More Notes Join us Telegram-: http//:t.me/allexammentor


ME-Paper Code-A GATE 2011
www.engineeringonyourfingertips.ooowww.gateforum.com

∫ f ( x ) dx
a
21. If f(x) is an even function and a is a positive real number, then equals
−a

(D) 2∫ f ( x ) dx
a
(A) 0 (B) a (C) 2a
0

Answer: - (D)
2 a f ( x ) dx; f ( x ) is even
 ∫0
∫−a ( )
a
Explanation: - f x dx = 
 0 ; f ( x ) is odd

22. The coefficient of restitution of a perfectly plastic impact is


(A) 0 (B) 1 (C) 2 (D) ∞
Answer: - (A)
Re lative velocity of separation
Explanation: - Coefficient of Re stitution =
Re lative velocity of approach
= O for perfectly plastic impact

since both bodies clinge together after impact.

23. A thin cylinder of inner radius 500mm and thickness 10mm is subjected to an
internal pressure of 5MPa. The average circumferential (hoop) stress in MPa is
(A) 100 (B) 250 (C) 500 (D) 1000
Answer: - (B)
Explanation: - Given Data: p = 5MPa ; d = 1000 mm ; t = 10mm

pd
Hoop stress σHoop = = 250 MPa
2t

24. Which one among the following welding processes uses non-consumable
electrode?
ww
(A) Gas metal w.
arc engi neer
welding ingonyour
(B)fingerti
ps.
Submerged ooo
arc welding
(C) Gas tungsten arc welding (D) Flux coated arc welding
Answer: - (C)

25. The crystal structure of austenite is


(A) Body centered cubic (B) Face centered cubic
(C) Hexagonal closed packed (D) Body centered tetragonal
Answer: - (B)
Q. No. 26 – 51 Carry Two Marks Each

26. A torque T is applied at the free end of a stepped rod of circular cross-sections as
shown in the figure. The shear modulus of the material of the rod is G. The
expression for d to produce an angular twist θ at the free end is

www.engineeringonyourfingertips.ooo
© All rights reserved by Gateforum Educational Services Pvt. Ltd. No part of this document may be reproduced or utilized in any form without the
written permission. Discuss GATE 2011 question paper at www.gatementor.com. 6

More Notes Join us Telegram-: http//:t.me/allexammentor


ME-Paper Code-A GATE 2011
www.engineeringonyourfingertips.ooowww.gateforum.com

1
 32TL  4
(A)  
 πθG  L L /2
1
T
 18TL  4
(B)  
 πθG  2d d
1
 16TL  4
(C)  
 πθG 
1
 2TL  4
(D)  
 πθG 
Answer: - (B)
Explanation: -Angular twist at the free end
 
L
 Tl   Tl   Tl  T  l  l  T  l 2 
θ = Σ  =  +  =   +    =  + 
 GJ   GJ 1  GJ 2 G  J 1  J 2  G  π ( 2d) 4 π 4
d
 31 32 
TL  32 32  18TL
= 16 + 2  = L L /2
πGd 4
  πGd4
T
1
18TL  4
⇒ d=   2d d
 πGθ 
(1) (2)

27. Figure shows the schematic for the measurement of velocity of air (density =
1.2kg / m3 ) through a constant–area duct using a pitot tube and a water-tube
manometer. The differential head of water (density = 1000 kg / m3 ) in the two
columns of the manometer is 10mm. Take acceleration due to gravity as
9.8m / s2 . The velocity of air in m/s is
www.
engi
neer
ingonyour
finger
tips.
ooo
(A) 6.4
Flow

(B) 9.0

(C) 12.8 10 mm

(D) 25.6

www.engineeringonyourfingertips.ooo
© All rights reserved by Gateforum Educational Services Pvt. Ltd. No part of this document may be reproduced or utilized in any form without the
written permission. Discuss GATE 2011 question paper at www.gatementor.com. 7

More Notes Join us Telegram-: http//:t.me/allexammentor


ME-Paper Code-A GATE 2011
www.engineeringonyourfingertips.ooowww.gateforum.com

Answer: - (C)
Explanation: - From Bernoulli’s equation
V12 − V22 p − p1 2 (p2 − p1 )
= 2 ⇒ 1 V1 =
2g ρ ag ρa

2 × 98.1
But P2 − P1 = ( ρgh)water = 9810 × 10 × 10−3 = 98.1 N / m2 ∴ V1 = = 1.2 m
1.2 s

28. The values of enthalpy of steam at the inlet and outlet of a steam turbine in a
Rankine cycle are 2800kJ/kg and 1800kJ/kg respectively. Neglecting pump work,
the specific steam consumption in kg/kW-hour is
(A) 3.60 (B) 0.36 (C) 0.06 (D) 0.01
Answer: - (A)
Explanation: - Work done by the turbine
W = 2800 – 1800 = 1000 kJ/kg = 1000 kW − s
kg
1
Specific fuel consumption = × 3600 = 3.6 kg / kw − hr
1000

1 3
29. The integral ∫
dx , when evaluated by using Simpson’s 1/3 rule on two equal
x 1

subintervals each of length 1, equals


(A) 1.000 (B) 1.098 (C) 1.111 (D) 1.120
Answer: - (C)
31
Explanation: - Given ∫ dx
1 x

b−a
Here, a =1, b=3, n=2 and h = =1
n
x0 = 1 x1 = 2 x2 = 3
y0 = 1 1 1
y = = 0.5 y = = 0.33
ww
2
w.
2engi
neer
3
ing
3onyour
finger
tips.
ooo
By Simpson’s rule
31 1 1
∫1 x dx = 3 h ( y1 + y3 ) + 4 ( y2 ) = 3 (1) (1 + 0.33) + 4 (0.51) = 1.11

30. Two identical ball bearings P and Q are operating at loads 30kN and 45kN
respectively. The ratio of the life of bearing P to the life of bearing Q is
(A) 81/16 (B) 27/8 (C) 9/4 (D) 3/2
Answer: - (B)

Explanation: - For ball bearing P. (L )


1
3 =C
Given PP = 30kN and P2 = 45kN
3
LP  P2 
3 3
 45  3 27
=  =  = 2 = 8
L2  Pp   30   

www.engineeringonyourfingertips.ooo
© All rights reserved by Gateforum Educational Services Pvt. Ltd. No part of this document may be reproduced or utilized in any form without the
written permission. Discuss GATE 2011 question paper at www.gatementor.com. 8

More Notes Join us Telegram-: http//:t.me/allexammentor


ME-Paper Code-A GATE 2011
www.engineeringonyourfingertips.ooowww.gateforum.com

31. For the four-bar linkage shown in the figure, the angular velocity of link AB is 1
rad/s. the length of link CD is 1.5 times the length of link AB. In the configuration
shown, the angular velocity of link CD in rad/s is

1 rad / s

A
D

3 2
(A) 3 (B) (C) 1 (D)
2 3
Answer: - (D)
Explanation: -For the given configuration
VAB = VCD ⇒ ωAB AB = ωCD CD
AB 1 2
⇒ ωCD = ωAB =1 × = rad / s
CD 1.5 3

32. A stone with mass of 0.1kg is catapulted as shown in the figure. The total force
Fx (in N) exerted by the rubber band as a function of distance x (in m) is given by
Fx = 300x2 . If the stone is displaced by 0.1m from the un-stretched position
(x=0) of the rubber band, the energy stored in the rubber band is

www.
engi
neer
ingonyour
finger
tips.
ooo
x

Fx

Stone of
mass 0.1kg

(A) 0.01J (B) 0.1J (C) 1J (D) 10J

www.engineeringonyourfingertips.ooo
© All rights reserved by Gateforum Educational Services Pvt. Ltd. No part of this document may be reproduced or utilized in any form without the
written permission. Discuss GATE 2011 question paper at www.gatementor.com. 9

More Notes Join us Telegram-: http//:t.me/allexammentor


ME-Paper Code-A GATE 2011
www.engineeringonyourfingertips.ooowww.gateforum.com

Answer: - (B)
Explanation: -Energy stored in the bar = W.D. by the stone
0.1 0.1 x3
= ∫0 Fx dx == ∫0 300.x2dx = 300.
3
= 100 × 0.13 = 0.1J

33. Consider the differential equation


dy
dx
( )
= 1 + y2 x . The general solution with

constant c is
x2 x 
(A) y = tan + tan c (B) y = tan2  + c 
2 2 
x  x2 
(C) y = tan2   + c (D) y = tan  + c
2  2 
Answer: - (D)
Explanation: - Given differential equation is
dy
dx
(
= 1 + y2 x ⇒ )
dy
1 + y2
= xdx

Integrating on both sides, we have


x2  x2 
⇒ tan−1 y = + c ⇒ y = tan  + c
2  2 

34. An unbiased coin is tossed five times. The outcome of each toss is either a head
or a tail. The probability of getting at least one head is
1 13 16 31
(A) (B) (C) (D)
32 32 32 32
Answer: - (D)
1 31
Explanation: - P(at least one head) = 1- P (no heads) = 1 − 5
=
2 32

www.
engi
neer
ingonyour
finger
tips.
ooo
35. A mass of 1kg is attached to two identical springs each with stiffness k = 20kN/m
as shown in the figure. Under frictionless condition, the natural frequency of the
system in Hz is close to

1kg

(A) 32 (B) 23 (C) 16 (D) 11

www.engineeringonyourfingertips.ooo
© All rights reserved by Gateforum Educational Services Pvt. Ltd. No part of this document may be reproduced or utilized in any form without the
written permission. Discuss GATE 2011 question paper at www.gatementor.com. 10

More Notes Join us Telegram-: http//:t.me/allexammentor


ME-Paper Code-A GATE 2011
www.engineeringonyourfingertips.ooowww.gateforum.com

Answer: - (A)
Explanation: - Natural frequency of the system
ke
ωn =
m
Where k e = k + k = 2k = 2 × 20 = 40 kN / m

40 × 1000
ωn = = 200 rad / s = 32Hz
1

36. The shear strength of a sheet metal is 300MPa. The blanking force required to
produce a blank of 100mm diameter from a 1.5 mm thick sheet is close to
(A) 45kN (B) 70kN (C) 141kN (D) 3500kN
Answer: - (C)
Explanation: - Blanking force = τ . As = 300 × πdt = 300 × π × 100 × 1.5 = 141 kN

37. The ratios of the laminar hydrodynamic boundary layer thickness to thermal
1
boundary layer thickness of flows of two fluids P and Q on a flat plate are and
2
2 respectively. The Reynolds number based on the plate length for both the flows
1
is 104 . The Prandtl and Nusselt numbers for P are and 35 respectively. The
8
Prandtl and Nusselt numbers for Q are respectively
(A) 8 and 140 (B) 8 and 70 (C) 4 and 70 (D) 4 and 35
Answer: - (A)
δt 1 −1
Explanation: - = × Pr 3
δ 1.026
1 δ 1 −1
For fluid Q: − = t = Pr 3 ⇒ Pr = 8
2 δ 1.026
For fluid P :w
-w w.
fromenginee
Laminar r ing
flow onyflat
over ourfinger
plate t
ips.
ooo
1 1
Nu = 0.664 ReL 2 Pr 3 ⇒ Nu = 35
Similarly for fluid Q :

( )
1 1 1 1
Nu = 0.664 ReL 2 Pr 3 = 0.664 104 2
8 3 ≃ 140

38. The crank radius of a single–cylinder I. C. engine is 60mm and the diameter of
the cylinder is 80mm. The swept volume of the cylinder in cm3 is
(A) 48 (B) 96 (C) 302 (D) 603
Answer: - (D)
Explanation: - Stroke of the cylinder l = 2r = 2 × 60 = 120mm
π 2 π
Swept volume = d ×l = × 802 × 120 = 603 cm3
4 4

www.engineeringonyourfingertips.ooo
© All rights reserved by Gateforum Educational Services Pvt. Ltd. No part of this document may be reproduced or utilized in any form without the
written permission. Discuss GATE 2011 question paper at www.gatementor.com. 11

More Notes Join us Telegram-: http//:t.me/allexammentor


ME-Paper Code-A GATE 2011
www.engineeringonyourfingertips.ooowww.gateforum.com

39. A pump handling a liquid raises its pressure from 1 bar to 30 bar. Take the
density of the liquid as 990 kg / m3 . The isentropic specific work done by the
pump in kJ/kg is
(A) 0.10 (B) 0.30 (C) 2.50 (D) 2.93
Answer: - (D)

Explanation: - Work done by the pump = υ (p2 − p1 ) =


(30 − 1) × 100 = 2.93kJ / kg
990

40. A spherical steel ball of 12mm diameter is initially at 1000K. It is slowly cooled in
a surrounding of 300K. The heat transfer coefficient between the steel ball and
the surrounding is 5W / m2K . The thermal conductivity of steel is 20W / mK . The
temperature difference between the centre and the surface of the steel ball is
(A) Large because conduction resistance is far higher than the convective
resistance
(B) Large because conduction resistance is far less than the convective resistance
(C) Small because conduction resistance is far higher than the convective
resistance
(D) Small because conduction resistance is far less than the convective resistance
Answer: - (D)
hL 5 × 0.002
Explanation: - Bi = = = 0.0005
K 20
For the given condition the Biot number tends to zero, that means conduction
resistance is far less than convection resistance. Therefore temperature between
the centre and surface is very small.

41. An ideal Brayton cycle, operating between the pressure limits of 1 bar and 6 bar,
has minimum and maximum temperatures of 300K and 1500K. The ratio of
ww
specific heats ofwthe
.eng ineer
workingin gon
fluid isyour
1.4. f
inge
The rti
ps. ooofinal temperatures in
approximate
Kelvin at the end of the compression and expansion processes are respectively
(A) 500 and 900 (B) 900 and 500 (C) 500 and 500 (D) 900 and 900
Answer: - (A)
Explanation: - Ideal Brayton cycle
At the end of compression, temperature.
T
γ−1 3
P  γ
0.4 1500k
T2 = T1  3  = 300 × 61.4 = 500K 6 bar
 P4  2
1 bar 4
At the end of expansion; temperature.
T3 300k
1500 1
T4 = γ −1
= 0.4
= 900K
 P3  γ 6 1.4 S
 
 P4 

www.engineeringonyourfingertips.ooo
© All rights reserved by Gateforum Educational Services Pvt. Ltd. No part of this document may be reproduced or utilized in any form without the
written permission. Discuss GATE 2011 question paper at www.gatementor.com. 12

More Notes Join us Telegram-: http//:t.me/allexammentor


ME-Paper Code-A GATE 2011
www.engineeringonyourfingertips.ooowww.gateforum.com

42. A disc of mass m is attached to a spring of stiffness k as shown in the figure. The
disc rolls without slipping on a horizontal surface. The natural frequency of
vibration of the system is
1 k
(A)
2π m

1 2k k
(B) m
2π m

1 2k
(C)
2π 3m

1 3k
(D)
2π 2m
Answer: - (C)
Explanation: -

k
o
x
θ

Taking moments about instantaneous centre ‘A’


θ + (kx ) r = 0
I ɺɺ
a

( )
θ + kx ( θr ) r = 0
⇒ IO + mr 2 ɺɺ

1 
⇒  mr 2 + mr 2  θ + k θr 2 = 0
2 
( )
www.
engi
neer
ingonyour
finger
tips.
ooo
kr 2 2k 1 2k
θ +
⇒ ɺɺ θ = 0 ⇒ ɺɺ
θ+ θ = 0; ∴ωn =
3 3m 2π 3m
mr 2
2

43. A 1 kg block is resting on a surface with coefficient of friction µ = 0.1 . A force of


0.8N is applied to the block as shown in figure. The friction force is

0.8N
1 kg

(A) 0 (B) 0.8N (C) 0.98N (D) 1.2N

www.engineeringonyourfingertips.ooo
© All rights reserved by Gateforum Educational Services Pvt. Ltd. No part of this document may be reproduced or utilized in any form without the
written permission. Discuss GATE 2011 question paper at www.gatementor.com. 13

More Notes Join us Telegram-: http//:t.me/allexammentor


ME-Paper Code-A GATE 2011
www.engineeringonyourfingertips.ooowww.gateforum.com

Answer: - (B)
Explanation: -Limiting friction force between the block and the surface is 0.98N. But the
applied force is 0.8N which is less than the limiting friction force. Therefore the
friction force for the given case is 0.8N.

44. Consider the following system of equations


2x1 + x2 + x3 = 0,
x2 − x3 = 0,
x1 + x2 = 0.
This system has
(A) A unique solution (B) No solution
(C) Infinite number of solutions (D) Five solutions
Answer: - (C)
Explanation: - Given equations are
2x1 + x2 + x3 = 0....................(1) x2 − x3 = 0.........................(2)
and x1 + x2 = 0..................(3)
E lim inating x3 from (1) & (2), we have x1 + x2 = 0..................(4)
Clearly (3) & (4) are coincident i.e. they will meet at inf inite po int s
Hence the given equations have inf inite solutions

45. A single–point cutting tool with 120 rake angle is used to machine a steel work–
piece. The depth of cut, i.e. uncut thickness is 0.81mm. The chip thickness under
orthogonal machining condition is 1.8mm. The shear angle is approximately
(A) 220 (B) 260 (C) 560 (D) 760
Answer: - (B)
Explanation: Relation between shear angle ( φ ) , chip thickness ratio (r) and rake angle
(α) is given by
r cos α
Tanφ = www.
1 − r sin α
engi
neer
ingonyour
finger
tips.
ooo
0.81
Where r = = 0.45
1.8
0.45 cos 12
Tanφ = ⇒ φ = 26O
1 − 0.45 sin12

46. Match the following non-traditional machining processes with the corresponding
material removal mechanisms :
Machining process Mechanism of material removal
P. Chemical machining 1. Erosion
Q. Electro-chemical machining 2. Corrosive reaction
R. Electro – discharge machining 3. Ion displacement
S. Ultrasonic machining 4. Fusion and vaporization

www.engineeringonyourfingertips.ooo
© All rights reserved by Gateforum Educational Services Pvt. Ltd. No part of this document may be reproduced or utilized in any form without the
written permission. Discuss GATE 2011 question paper at www.gatementor.com. 14

More Notes Join us Telegram-: http//:t.me/allexammentor


ME-Paper Code-A GATE 2011
www.engineeringonyourfingertips.ooowww.gateforum.com

(A) P-2, Q-3, R-4,S-1 (B) P-2,Q-4,R-3,S-1


(C) P-3, Q-2,R-4,S-1 (D) P-2,Q-3,R-1,S-4
Answer: - (A)

47. A cubic casting of 50mm side undergoes volumetric solidification shrinkage and
volumetric solid contraction of 4% and 6% respectively. No. riser is used.
Assume uniform cooling in all directions. The side of the cube after solidification
and contraction is
(A) 48.32mm (B) 49.90mm (C) 49.94mm (D) 49.96mm
Answer: - (A)
Explanation: - Volumetric solidification shrinkage and volumetric solid contraction cause
decrease in dimensions.
Volume of cube = 503 = 125000 mm3
After considering both the allowances
V = 125000 × 0.96 × 0.94 = 112800 mm3

Side of cube = 3
112800 = 48.32mm

Common Data Questions: 48 & 49

In an experimental set-up, air flows between two stations P and Q adiabatically.


The direction of flow depends on the pressure and temperature conditions
maintained at P and Q. The conditions at station P are 150kPa and 350K. The
temperature at station Q is 300K.
The following are the properties and relations pertaining to air:
Specific heat at constant pressure, Cp = 1.005kJ / kgK;
Specific heat at constant volume, Cv = 0.718kJ / kgK;
Characteristic gas constant, R = 0.287kJ / kgK
Enthalpy, h = cp T
wwwu.
Internal energy, e
=n
cgi
vT
neer
ingonyour
finger
tips.
ooo

48. If the air has to flow from station P to station Q, the maximum possible value of
pressure in kPa at station Q is close to
(A) 50 (B) 87 (C) 128 (D) 150
Answer: - (B)
Explanation: - To cause the flow from P to Q, change in entropy SP − SQ ( ) should be
greater than zero
i.e SP − SQ > O , or let us say S1 − S2 > 0

T  v 
⇒ Cv ln  2  + R ln  2  > 0
 T1   v1 
 300  v 
⇒ 0.718 ln   + 0.287 ln  2  > 0
 350   v1 

www.engineeringonyourfingertips.ooo
© All rights reserved by Gateforum Educational Services Pvt. Ltd. No part of this document may be reproduced or utilized in any form without the
written permission. Discuss GATE 2011 question paper at www.gatementor.com. 15

More Notes Join us Telegram-: http//:t.me/allexammentor


ME-Paper Code-A GATE 2011
www.engineeringonyourfingertips.ooowww.gateforum.com

v  v  0.1107
⇒ − 0.1107 + 0.287 ln  2  > 0 ⇒ ln  2  >
v
 1 v
 1 0.287

v2
⇒ > 1.47
v1
P1v1 P v
From Perfect gas law = 2 2
T1 T2
v2 P T
⇒ = 1 2
v1 P2 T1
P1 T2 150 × 300
∴ > 1.47 ⇒ P2 <
P2 T1 350 × 1.47
⇒ P2 < 87.4 kPa
∴ The maximum value of pressure at Q = 87 kPa

49. If the pressure at station Q is 50kPa, the change in entropy sQ − sP ( ) in kJ/kgK is

(A) -0.155 (B) 0 (C) 0.160 (D) 0.355


Answer: - (C)
T  v 
Explanation: - SQ − SP = Cv ln  2  + R ln  2 
 T1   v1 
From the perfect gas law
v2 P T 150 × 300
= 1 2 = = 2.57
v1 P2 T1 300 × 350
∴ Sa – SP = -0. 1107 + 0.287 ln 2.57= 0.16 kJ/kgK

Common Data Questions: 50 & 51

www. engi neer i


ngonyour finger tips.ooo
One unit of product P1 requires 3 kg of resource R1 and 1kg of resource R 2 . One
unit of product P2 requires 2kg of resource R1 and 2kg of resource R 2 . The profits
per unit by selling product P1 and P2 are Rs.2000 and Rs.3000 respectively. The
manufacturer has 90kg of resource R1 and 100kg of resource R 2 .

50. The unit worth of resource R 2 i.e., dual price of resource R2 in Rs. Per kg is
(A) 0 (B) 1350 (C) 1500 (D) 2000
Answer: - (A)
Explanation: -Because the constraint on resource 2 has no effect on the feasible region.

51. The manufacturer can make a maximum profit of Rs.


(A) 60000 (B) 135000 (C) 150000 (D) 200000
Answer: - (B)
Explanation: -Optimum solution is: 0, 45 and maximum profit = Rs.135000

www.engineeringonyourfingertips.ooo
© All rights reserved by Gateforum Educational Services Pvt. Ltd. No part of this document may be reproduced or utilized in any form without the
written permission. Discuss GATE 2011 question paper at www.gatementor.com. 16

More Notes Join us Telegram-: http//:t.me/allexammentor


ME-Paper Code-A GATE 2011
www.engineeringonyourfingertips.ooowww.gateforum.com

Linked Answer Questions: Q.52 to Q.55 Carry Two Marks Each

Statement for Linked Answer Questions: 52 & 53

A triangular–shaped cantilever beam of uniform–thickness is shown in the figure.


The Young’s modulus of the material of the beam is E. A concentrated load P is
applied at the free end of the beam.

t P

x
α
b α

52. The area moment of inertia about the neutral axis of a cross-section at a distance
x measured from the free end is
bxt3 bxt3 bxt3 xt3
(A) (B) (C) (D)
6l 12l 24l 12
www.
engi
neer
ingonyour
finger
tips.
ooo
Answer: - (B)
Explanation: -
bx bxt3
At a distance of x from the free end width b′ = ; ∴ Moment of Inertia Ix =
l 12l

53. The maximum deflection of the beam is


24Pl3 12Pl3 8Pl3 6Pl3
(A) (B) (C) (D)
Ebt3 Ebt3 Ebt3 Ebt3
Answer: - (D)
Explanation: -
Pl3 6Pl3 bt3
The maximum deflection of the beam ymax = = ; Where I =
3EI Ebt3 18l

www.engineeringonyourfingertips.ooo
© All rights reserved by Gateforum Educational Services Pvt. Ltd. No part of this document may be reproduced or utilized in any form without the
written permission. Discuss GATE 2011 question paper at www.gatementor.com. 17

More Notes Join us Telegram-: http//:t.me/allexammentor


ME-Paper Code-A GATE 2011
www.engineeringonyourfingertips.ooowww.gateforum.com

Statement for Linked Answer Questions: 54 & 55

The temperature and pressure of air in a large reservoir are 400K and 3 bar
respectively. A converging–diverging nozzle of exit area 0.005 m2 is fitted to the
wall of the reservoir as shown in the figure. The static pressure of air at the exit
section for isentropic flow through the nozzle is 50kPa. The characteristic gas
constant and the ratio of specific heats of air are 0.287kJ/kgK and 1.4
respectively.

Flow from the


reservoir

Nozzle
exit

54. The density of air in kg/ m3 at the nozzle exit is


(A) 0.560 (B) 0.600 (C) 0.727 (D) 0.800
Answer: - (C)
Explanation: - Given data:
T1 = 400k, P1 = 300 kPa, P2 = 50kPa, R = 0.289 kJ / kgK

v = 1.4, A2 = 0.005m2
The happened process from entrance to exit is isentropic process, therefore
γ−1 0.4
T2 P  γ  50 1.4
T1
=  2 w⇒
wwT2.
e
=n gi
ne
400 erin
g o=nyour
239.5 f
ki
nger
tips.
ooo
 P1  300 
From the perfect gas equation
P P 50
ρ= or ρ2 = 2 = ⇒ ρ2 = 0.727 kg / m3
RT RT2 0.287 × 3.39.5

55. The mass flow rate of air through the nozzle in kg/s is
(A) 1.30 (B) 1.77 (C) 1.85 (D) 2.06
Answer: - (D)
Explanation: - Mass flow rate m = ρ Q

Where Q = A2 V2 ; But V2 = 2CP T1 − T2 = 2 × 1.005 × ( 400 − 239.5 )

V2 = 568 m
s
∴ m = 0.727 × 0.005 × 568 = 2.06 kg / s

www.engineeringonyourfingertips.ooo
© All rights reserved by Gateforum Educational Services Pvt. Ltd. No part of this document may be reproduced or utilized in any form without the
written permission. Discuss GATE 2011 question paper at www.gatementor.com. 18

More Notes Join us Telegram-: http//:t.me/allexammentor


ME-Paper Code-A GATE 2011
www.engineeringonyourfingertips.ooowww.gateforum.com

Q. No. 56 – 60 Carry One Mark Each

56. Choose the word from the options given below that is most nearly opposite in
meaning to the given word: Amalgamate
(A) Merge (B) Split (C) Collect (D) Separate
Answer: - (B)
Exp: - Amalgamate means combine or unite to form one organization or structure. So
the best option here is split. Separate on the other hand, although a close
synonym, it is too general to be the best antonym in the given question while
Merge is the synonym; Collect is not related.

57. Which of the following options is the closest in the meaning to the word below:
Inexplicable
(A) Incomprehensible (B) Indelible
(C) Inextricable (D) Infallible
Answer: - (A)
Exp: - Inexplicable means not explicable; that cannot be explained, understood, or
accounted for. So the best synonym here is incomprehensible.

58. If Log (P) = (1/2)Log (Q) = (1/3) Log (R), then which of the following options is
TRUE?
(A) P2 = Q3R 2 (B) Q2 = PR (C) Q2 = R 3P (D) R = P2Q2
Answer: - (B)
1 1
Exp:- logP = log Q = log (R ) = k
2 3
∴ P = bk , Q = b2k ,R = b3k
Now, Q2 = b4k = b3k bk = PR

59. Choose thew ww.


most engi neer
appropriateingonyo
word(s) urf
frominger
the t
ips. oo
options o below to complete
given
the following sentence.
I contemplated________Singapore for my vacation but decided against
it.
(A) To visit (B) having to visit (C) visiting (D)for a visit
Answer: - (C)
Exp: - Contemplate is a transitive verb and hence is followed by a gerund Hence the
correct usage of contemplate is verb+ ing form.

60. Choose the most appropriate word from the options given below to complete the
following sentence.
If you are trying to make a strong impression on your audience, you
cannot do so by being understated, tentative or_____________.
(A) Hyperbolic (B) Restrained
(C) Argumentative (D) Indifferent

www.engineeringonyourfingertips.ooo
© All rights reserved by Gateforum Educational Services Pvt. Ltd. No part of this document may be reproduced or utilized in any form without the
written permission. Discuss GATE 2011 question paper at www.gatementor.com. 19

More Notes Join us Telegram-: http//:t.me/allexammentor


ME-Paper Code-A GATE 2011
www.engineeringonyourfingertips.ooowww.gateforum.com

Answer: - (B)
Exp: - The tone of the sentence clearly indicates a word that is similar to understated is
needed for the blank. Alternatively, the word should be antonym of strong (fail to
make strong impression). Therefore, the best choice is restrained which means
controlled/reserved/timid.

Q. No. 61 – 65 Carry Two Marks Each

61. A container originally contains 10 litres of pure spirit. From this container 1 litre
of spirit is replaced with 1 litre of water. Subsequently, 1 litre of the mixture is
again replaced with 1 litre of water and this process is repeated one more time.
How much spirit is now left in the container?
(A) 7.58 litres (B) 7.84 litres (C) 7 litres (D) 7.29 litres
Answer: - (D)
3 3
 10 − 1   9  729
Exp:- 10   = 10   =
 10   10  1000
729
∴ × 1 = 7.29 litres
1000

62. Few school curricula include a unit on how to deal with bereavement and
grief, and yet all students at some point in their lives suffer from losses
through death and parting.

Based on the above passage which topic would not be included in a unit on
bereavement?

(A) how to write a letter of condolence

(B) what emotional stages are passed through in the healing process

(C) what the leading causes of death are

(D) how to give support to a grieving friend

Answer: - (C)
www.
engi
neer
ingonyour
finger
tips.
ooo

Exp: - The given passage clearly deals with how to deal with bereavement and grief and
so after the tragedy occurs and not about precautions. Therefore, irrespective of
the causes of death, a school student rarely gets into details of causes—which is
beyond the scope of the context. Rest all are important in dealing with grief.

63. P, Q, R and S are four types of dangerous microbes recently found in a human
habitat. The area of each circle with its diameter printed in brackets represents
the growth of a single microbe surviving human immunity system within 24 hours
of entering the body. The danger to human beings varies proportionately with the
toxicity, potency and growth attributed to a microbe shown in the figure
below

www.engineeringonyourfingertips.ooo
© All rights reserved by Gateforum Educational Services Pvt. Ltd. No part of this document may be reproduced or utilized in any form without the
written permission. Discuss GATE 2011 question paper at www.gatementor.com. 20

More Notes Join us Telegram-: http//:t.me/allexammentor


ME-Paper Code-A GATE 2011
www.engineeringonyourfingertips.ooowww.gateforum.com

(milligrams of microbe required to destroy half of the body


1000
P (50 mm )
800

mass in kilograms) 600


Toxicity

Q ( 40 mm )
400
S (20 mm )
R ( 30 mm )
200

0
0.2 0.4 0.6 0.8 1
Potency
(Probability that microbe will overcome human immunity system)

A pharmaceutical company is contemplating the development of a vaccine


against the most dangerous microbe. Which microbe should the company target
in its first attempt?
(A) P (B) Q (C) R (D) S
Answer: - (D)
Exp: - By observation of the table, we can say S

P Q R S

Requirement 800 600 300 200

Potency 0.4 0.5 0.4 0.8

64. The variable cost (V) of manufacturing a product varies according to the equation
V= 4q, where q is the quantity produced. The fixed cost (F) of production of same
www. engi neer i
ngonyour fi
nger ti
ps. ooo
product reduces with q according to the equation F = 100/q. How many units
should be produced to minimize the total cost (V+F)?
(A) 5 (B) 4 (C) 7 (D) 6
Answer: (A)
Exp: - Checking with all options in formula: (4q+100/q) i.e. (V+F). Option A gives the
minimum cost.

65. A transporter receives the same number of orders each day. Currently, he has
some pending orders (backlog) to be shipped. If he uses 7 trucks, then at the
end of the 4th day he can clear all the orders. Alternatively, if he uses only 3
trucks, then all the orders are cleared at the end of the 10th day. What is the
minimum number of trucks required so that there will be no pending order at the
end of the 5th day?
(A) 4 (B) 5 (C) 6 (D) 7

www.engineeringonyourfingertips.ooo
© All rights reserved by Gateforum Educational Services Pvt. Ltd. No part of this document may be reproduced or utilized in any form without the
written permission. Discuss GATE 2011 question paper at www.gatementor.com. 21

More Notes Join us Telegram-: http//:t.me/allexammentor


ME-Paper Code-A GATE 2011
www.engineeringonyourfingertips.ooowww.gateforum.com

Answer: - (C)
Exp: - Let each truck carry 100 units.
2800 = 4n + e n = normal
3000 = 10n + e e = excess/pending
100 8000
∴n = ,e =
3 3
5.100 8000
5days ⇒ 500x = +
3 3
8500
⇒ 500x = 17 ⇒ x > 5
3
Minimum possible = 6

www.
engi
neer
ingonyour
finger
tips.
ooo

www.engineeringonyourfingertips.ooo
© All rights reserved by Gateforum Educational Services Pvt. Ltd. No part of this document may be reproduced or utilized in any form without the
written permission. Discuss GATE 2011 question paper at www.gatementor.com. 22

More Notes Join us Telegram-: http//:t.me/allexammentor

You might also like